You are on page 1of 202

Bab Pengukuran

1 Measurement
Tema: Asas Fizik

Revisi Pantas
6 Kuantiti terbitan ialah kuantiti fizik yang diterbitkan
1.1 Kuantiti Fizik
dengan menggabungkan kuantiti-kuantiti asas melalui
Physical Quantities pendaraban atau pembahagian atau kedua-duanya.
1 Kuantiti fizik ialah kuantiti yang boleh diukur. Derived quantities are physical quantities derived by combining
Physical quantity is a quantity that can be measured. base quantities through multiplication or division or both.

2 Kuantiti fizik boleh diukur menggunakan unit imperial 7 Jadual berikut menunjukkan beberapa kuantiti terbitan
atau unit metrik. Unit imperial jarang digunakan pada yang diungkapkan dalam sebutan kuantiti asas dan
masa kini. unitnya.
Physical quantities can be measured by using imperial units or The following table shows some derived quantities expressed in
metric units. Imperial units are seldom used nowadays. terms of base quantities and their units.
Contohnya, unit imperial bagi panjang ialah inci,
manakala unit metrik bagi panjang ialah sentimeter. Dinyatakan
Kuantiti
For example, the imperial unit for length is inches, while the metric dalam Unit
terbitan Simbol Rumus
unit for length is centimetres. kuantiti asas terbitan
Derived Symbol Formula
Expressed in Derived unit
3 Kuantiti fizik terdiri daripada quantity
base quantities
Physical quantities consist of
(a) kuantiti asas/base quantities Isi padu
V V = l3 l×l×l m3
(b) kuantiti terbitan/derived quantities Volume

4 Kuantiti asas ialah kuantiti fizik yang tidak boleh Halaju s l


ditakrifkan dalam sebutan kuantiti fizik yang lain. v v= m s–1
Velocity t t
Base quantities are physical quantities that cannot be defined in
terms of other physical quantities. Ketumpatan m m
ρ ρ= kg m–3
5 Jadual berikut menunjukkan tujuh kuantiti asas dan Density V l×l×l
unit S.I. masing-masing.
The following table shows the seven base quantities and their Pecutan v l
a a= m s–2
respective S.I. units. Acceleration t t×t

Kuantiti asas Unit S.I. Daya m×l


F F = ma kg m s–2
Base quantity S.I. unit Force t×t

Nama Simbol Nama Simbol Tekanan F m×l


Name Symbol Name Symbol P P= kg m–1 s–2
Pressure A l×l×t×t
Panjang meter
l m 8 Kuantiti fizik boleh dibahagikan kepada dua kategori:
Length metre
Physical quantities can be divided into two categories:
Jisim (a) Kuantiti skalar/Scalar quantities
m kilogram kg
Mass (b) Kuantiti vektor/Vector quantities
Masa saat 9 Kuantiti skalar ialah kuantiti fizik yang mempunyai
t s
Time second
magnitud sahaja.
Suhu Scalar quantities are physical quantities which have magnitude
termodinamik only.
T kelvin K
Thermodynamic 10 Contoh kuantiti skalar ialah panjang, suhu, masa, laju
temperature dan jisim.
Arus elektrik Examples of scalar quantities are length, temperature, time, speed
I ampere A and mass.
Electric current
Keamatan 11 Kuantiti vektor ialah kuantiti fizik yang mempunyai
berluminositi Iv candela cd kedua-dua magnitud dan arah.
Luminous intensity Vector quantities are physical quantities which have both
magnitude and direction.
Kuantiti bahan
mol 12 Contoh kuantiti vektor ialah sesaran, halaju, pecutan,
Amount of n mol
mole momentum dan daya.
substance
Examples of vector quantities are displacement, velocity,
acceleration, momentum and force.

01 Strategi A+ SPM Fizik Tg 4 (B01) 5pp.indd 1 18/11/2023 11:04 AM


1.2 Penyiasatan Saintifik
Scientific Investigation
1 Jadual berikut menunjukkan tafsiran bentuk graf.
The following table shows the interpretation of shape of graphs.

Graf Tafsiran Graf Tafsiran


Graph Interpretation Graph Interpretation

y
(x2, y2)
y y
y berkadar terus y bertambah
dengan x dengan x
∆y = y2 – y1
y is directly y increases
(x1, y1) proportional to x with x
∆x = x2 – x1 0 x 0 x
0 x

y
(x2, y2) y y
y bertambah
y berkurang
∆y = y2 – y1 secara linear
(x1, y1) dengan x
dengan x y decreases
∆x = x2 – x1 y increases linearly
(0, c) with x
with x
0 x 0 x
0 x

(0, c) y y
(x1, y1) y berkurang y berkadar
secara linear songsang
∆y = y1 – y2 dengan x dengan x
y decreases linearly y is inversely
(x2, y2)
with x 1 proportional to x
∆x = x1 – x2 0 x 0 –
x
0 x

2 Penyiasatan saintifik merangkumi:


Scientific investigation includes: i-THINK Peta Alir

Pernyataan masalah Inferens Hipotesis


Problem statement Inference Hypothesis

Pemboleh ubah Senarai bahan dan radas Prosedur


Variables List of materials and apparatus Procedure

Pemerhatian dan
Analisis data Perbincangan
pengumpulan data Data analysis Discussion
Observation and data collection

Kesimpulan Laporan lengkap


Conclusion Complete report

01 Strategi A+ SPM Fizik Tg 4 (B01) 5pp.indd 2 18/11/2023 11:04 AM


Praktis PBD
1.1 Kuantiti Fizik/ Physical Quantities
Latihan 1 Kuantiti asas dan kuantiti terbitan/ Base quantities and derived quantities
TP 1 Mengingat kembali pengetahuan dan kemahiran sains mengenai Pengukuran.

TP 2 Memahami Pengukuran serta dapat menjelaskan kefahaman tersebut.

TP 3 Mengaplikasikan pengetahuan mengenai Pengukuran untuk menerangkan kejadian atau fenomena alam dan melaksanakan tugasan mudah.

1 Isi tempat kosong dengan perkataan yang betul. TP 1


Fill in the blanks with the correct words. i-THINK Peta Pokok

Kuantiti fizik
Physical quantities

Video
Kuantiti asas Kuantiti terbitan
Base quantity Derived quantity

Kuantiti yang (a) tidak boleh ditakrifkan Kuantiti yang (c) diterbitkan
dalam sebutan (b) kuantiti asas yang lain. daripada (d) kuantiti asas melalui
The quantity which (a) cannot be (e) pendaraban atau
defined in terms of other (b) base quantities . (f) pembahagian atau kedua-duanya.
The quantity which is (c) derived

from (d) base quantities through


Video
Untuk tujuan pembelajaran (e) multiplication or (f) division
Imbas kod QR atau layari https://www.
youtube.com/watch?v=yIdFcm5oVma untuk or both.
menonton video tentang kuantiti fizik.

2 Nyatakan simbol, unit S.I. dan simbol unit S.I. bagi tujuh kuantiti asas fizik dalam jadual berikut.
State the symbol, S.I. unit and symbol for S.I. unit for the seven base quantities of physics in the following table. TP 1

Simbol kuantiti asas


Kuantiti asas Unit S.I. Simbol unit S.I.
Symbol for base
Base quantity S.I. unit Symbol for S.I. unit
quantity
(a) Panjang meter
l m
Length metre

(b) Jisim
m kilogram kg
Mass

(c) Masa saat


t s
Time second

(d) Arus elektrik


I ampere A
Electric current

(e) Suhu termodinamik


T kelvin K
Thermodynamic temperature

(f) Kuantiti bahan mol


n mol
Amount of substance mole

(g) Keamatan berluminositi


Iv candela cd
Luminous intensity

01 Strategi A+ SPM Fizik Tg 4 (B01) 5pp.indd 3 18/11/2023 11:04 AM


3 Tentukan rumus dan unit terbitan bagi kuantiti terbitan berikut. TP 2
Determine the formula and derived unit for the following derived quantities.

Dalam sebutan Simbol


Kuantiti terbitan Rumus kuantiti asas unit S.I.
Derived quantity Formula In terms of Symbol for
base quantities S.I. unit

(a) Luas, A Luas = Panjang × Lebar


l × l = l2 m × m = m2
Area Area = Length × Width

(b) Isi padu, V Isi padu = Panjang × Lebar × Tinggi


l × l × l = l3 m × m × m = m3
Volume Volume = Length × Width × Height

Jisim
Ketumpatan = m m kg
(c) Ketumpatan, ρ Panjang × Lebar × Tinggi = 3 = kg m–3
Density Mass l×l×l l m3
Density =
Length × Width × Height

Sesaran
Halaju = l m
(d) Halaju, v Masa = m s–1
Velocity Displacement t s
Velocity =
Time

Perubahan halaju
Pecutan = l l m
(e) Pecutan, a Masa = 2 = m s–2
Acceleration Change in velocity t×t t s2
Acceleration =
Time

(f) Daya, F Daya = Jisim × Pecutan l m kg m s–2 atau/or


m× = 2
Force Force = Mass × Acceleration t2 t N (newton)

Momentum = Jisim × Halaju l ml


(g) Momentum, p m× = kg m s–1
Momentum = Mass × Velocity t t

kg m–1 s–2
Daya atau/or
Tekanan = ml m
(h) Tekanan, P Luas = 2 N
Pressure Force t2 × l × l tl = N m–2
Pressure =
m2
Area atau/or
Pa (pascal)

(i) Berat, W Berat = Jisim × Pecutan graviti l ml kg m s–2 atau/or


m× = 2
Weight Weight = Mass × Gravitational acceleration t 2
t N (newton)

kg m2 s–2 atau/or
(j) Kerja, W Kerja = Daya × Sesaran ml ml2 N×m=Nm
×l= 2
Work Work = Mass × Displacement t 2
t atau/or
J (joule)
kg m2 s–3
Kerja atau/or
Kuasa = ml2 ml2 J
(k) Kuasa, P Masa = = J s–1
Power Work t2 × t t3 s
Power = atau/or
Time
W (watt)

01 Strategi A+ SPM Fizik Tg 4 (B01) 5pp.indd 4 18/11/2023 11:04 AM


4 Maklumat berikut menerangkan cara menghasilkan slime. TP 3
Kelaskan kuantiti fizik dan nilainya yang terkandung dalam maklumat tersebut.
The following information explains the method to produce slime.
Classify the physical quantities and their numerical values that are stated in the information.

Letakkan 130 g syampu dan 80 g tepung jagung ke dalam sebuah mangkuk. Kacau sehingga
sebati. Tambahkan 1 m gel pewarna makanan. Masukkan 5 m air dan kacau. Tambahkan
25 m lagi air secara perlahan-lahan dan kacau rata. Uli adunan selama kira-kira 5 minit.
Slime sedia untuk digunakan.
Put 130 g of shampoo and 80 g of cornstarch in a bowl. Stir to combine well. Add 1 m of food colouring gel.
Add 5 m of water and stir. Slowly add 25 m of water and stir evenly. Knead the slime for about 5 minutes.
Slime is ready for use.

Kuantiti asas Kuantiti terbitan


Base quantity Derived quantity

Jisim (130 g, 80 g)
Mass (130 g, 80 g) Isi padu (1 m, 5 m, 25 m)
Volume (1 m, 5 m, 25 m)
Masa (5 minit)
Time (5 minutes)

Latihan 2 Kuantiti skalar dan kuantiti vektor/Scalar quantities and vector quantities
TP 1 Mengingat kembali pengetahuan dan kemahiran sains mengenai Pengukuran.

TP 2 Memahami Pengukuran serta dapat menjelaskan kefahaman tersebut.

TP 3 Mengaplikasikan pengetahuan mengenai Pengukuran untuk menerangkan kejadian atau fenomena alam dan melaksanakan tugasan mudah.

1 Isi tempat kosong dengan perkataan yang betul. TP 1


Fill in the blanks with the correct words. i-THINK Peta Pokok

Kuantiti fizik
Physical quantities

Kuantiti skalar Kuantiti vektor


Scalar quantity Vector quantity

Kuantiti fizik yang mempunyai Kuantiti fizik yang mempunyai


(a) magnitud sahaja. (b) magnitud dan
Physical quantity which has (c) arah .
(a) magnitude only. Physical quantity which has
(b) magnitude and (c) direction .

2 Senarai berikut mengandungi kuantiti skalar dan vektor. TP 2


The following list contains scalar and vector quantities.

Jarak, sesaran, halaju, laju, pecutan, tekanan, daya, jisim, berat, tenaga, kerja,
kuasa, momentum, suhu, isi padu, ketumpatan
Distance, displacement, velocity, speed, acceleration, pressure, force, mass, weight, energy, work,
power, momentum, temperature, volume, density

01 Strategi A+ SPM Fizik Tg 4 (B01) 5pp.indd 5 18/11/2023 11:04 AM


Kelaskan semua kuantiti tersebut kepada kuantiti skalar atau kuantiti vektor.
Classify all the quantities as scalar quantity or vector quantity.

Kuantiti skalar Kuantiti vektor


Scalar quantity Vector quantity

Jarak, laju, tekanan, jisim, tenaga, kerja, kuasa, Sesaran, halaju, pecutan, daya, berat, momentum
suhu, isi padu, ketumpatan Displacement, velocity, acceleration, force, weight,
Distance, speed, pressure, mass, energy, work, power, momentum
temperature, volume, density

3 Tentukan sama ada setiap kuantiti berikut mempunyai magnitud, arah atau kedua-duanya.
Tandakan (3) dalam jadual dan kemudian tulis jenis kuantiti yang terlibat. TP 3
Determine whether each of the following quantities has magnitude, direction or both. Tick (3) in the table and then write
the type of quantity involved.

Situasi Magnitud Arah Jenis kuantiti


Situation Magnitude Direction Type of quantity

(a) Sekolah menengah terletak 15 km ke


utara rumah saya 3 3
Kuantiti vektor
The secondary school is located 15 km to the north Vector quantity
of my house

(b) Faizal mengambil masa 12.4 s untuk


berlari sejauh 100 m semasa hari sukan
Kuantiti skalar
sekolah 3
Scalar quantity
Faizal takes 12.4 s to run at a distance of 100 m
during the school sports day

(c) Isi padu cecair di dalam sebuah bikar


Kuantiti skalar
ialah 400 m 3
Scalar quantity
The volume of liquid in a beaker is 400 m

(d) Sebuah kereta bergerak daripada


keadaan rehat dan menambah halajunya
Kuantiti vektor
sehingga mencapai 140 m s–1 3 3
Vector quantity
A car moves from rest and increases its velocity
until it reaches 140 m s–1

(e) Semangkuk kari ikan dipanaskan


daripada 27 ºC kepada 100 ºC 3
Kuantiti skalar
A bowl of fish curry is heated from 27 °C to Scalar quantity
100 °C

01 Strategi A+ SPM Fizik Tg 4 (B01) 5pp.indd 6 18/11/2023 11:04 AM


1.2 Penyiasatan Saintifik/ Scientific Investigation
Latihan 3 Mentafsir bentuk-bentuk graf/ Interpreting shape of graphs
TP 2 Memahami Pengukuran serta dapat menjelaskan kefahaman tersebut.

Padankan bentuk graf berikut dengan hubungan antara x dengan y yang betul. TP 2
Match the following graphs with the correct relationships between x and y.

1 y 2 y 3 y

x 0 x 0 x
0
• • •

• • • • • •
y berkadar y berkadar
y bertambah y berkurang
songsang y bertambah terus y berkurang
secara linear secara linear
dengan x dengan x dengan x dengan x
dengan x dengan x
y is inversely y increases y is directly y decreases
y increases y decreases
proportional with x proportional with x
linearly with x linearly with x
to x to x
• • • • • •

• • •
4 y 5 y 6 y

0 x 0 x 0 x

Latihan 4 Mengumpul dan menganalisis data/Collecting and analysing data


TP 3 Mengaplikasikan pengetahuan mengenai Pengukuran untuk menerangkan kejadian atau fenomena alam dan melaksanakan tugasan mudah.

TP 4 Menganalisis pengetahuan mengenai Pengukuran dalam konteks penyelesaian masalah mengenai kejadian atau fenomena alam.

1 Rajah berikut menunjukkan bacaan pada sebuah voltmeter yang disambungkan pada suatu litar
lengkap. Arus yang mengalir dilaraskan kepada 0.1 A, 0.2 A, 0.3 A, 0.4 A dan 0.5 A. TP 3 TP 4
The following diagrams show the readings on a voltmeter connected to a complete circuit. The current flowing is adjusted
to 0.1 A, 0.2 A, 0.3 A, 0.4 A and 0.5 A.

1 2 1 2 1 2 1 2 1 2
0 3 0 3 0 3 3 3
0 0
V V V V V

Arus, I = 0.1 A Arus, I = 0.2 A Arus, I = 0.3 A Arus, I = 0.4 A Arus, I = 0.5 A
Current, I = 0.1 A Current, I = 0.2 A Current, I = 0.3 A Current, I = 0.4 A Current, I = 0.5 A

(a) Berdasarkan rajah yang ditunjukkan, isi jadual dengan nilai arus dan voltan yang betul.
Based on the diagrams shown, fill in the table with the correct current and voltage values.

Arus, I (A)
0.1 0.2 0.3 0.4 0.5
Current

Voltan, V (V)
0.2 0.4 0.6 0.8 1.0
Voltage

01 Strategi A+ SPM Fizik Tg 4 (B01) 5pp.indd 7 18/11/2023 11:04 AM


(b) Plot graf V melawan I pada kertas graf di bawah.
Plot the graph of V against I on the graph paper below.

Voltan, V (V) Graf V melawan I


Voltage Graph of V against I

(f) 1.2

(0.5, 1.0)
1.0

0.8

0.6
(d)

0.4

0.2

Arus, I (A)
0 0.1 0.2 0.3 0.4 0.5 0.6 Current

(c) Berdasarkan graf, nyatakan hubungan antara voltan, V dengan arus, I.


Based on the graph, state the relationship between voltage, V and current, I.
V berkadar terus dengan I./V is directly proportional to I.

(d) Hitung kecerunan graf.


Calculate the gradient of the graph.
y2 – y1 1.0 – 0
m= = = 2.0 V A–1
x2 – x1 0.5 – 0

(e) Voltan, V, arus, I dan rintangan, R dihubungkan dengan persamaan


The voltage, V, current, I and resistance, R are related by the equation

V = IR
Berdasarkan jawapan anda di 1(d), tentukan nilai rintangan.
Based on your answer in 1(d), determine the value of resistance.
2.0 Ω

(f) Ramalkan nilai voltan apabila arus, I = 0.6 A.


Predict the value of voltage when the current, I = 0.6 A.
1.2 V (Jawapan boleh diperoleh dengan mengekstrapolasi graf.)
(The answer can be obtained by extrapolating the graph.)
2 Seorang murid menjalankan eksperimen untuk mengkaji hubungan antara kehilangan berat, W
bagi objek yang terendam di dalam air dengan isi padu cecair yang disesarkan, V. Murid tersebut

01 Strategi A+ SPM Fizik Tg 4 (B01) 5pp.indd 8 18/11/2023 11:04 AM


merendamkan sebuah objek yang tergantung pada sebuah neraca spring ke dalam air di dalam
sebuah tin Eureka. Keputusan eksperimen ditunjukkan oleh graf W melawan V di bawah.
A student carried out an experiment to investigate the relationship between the loss in weight, W of an object immersed
in water and the volume of water displaced, V. The student immersed the object which is suspended from a spring balance
into the water in a Eureka can. The results of the experiment are shown by the graph W against V below. TP 3 TP 4
Graf W melawan V
Graph of W against V
W (N)
1.6

(70, 1.4)
1.4

1.3

1.2

1.0

0.8

0.6
(b)

0.4

0.2

V (cm3)
0 10 20 30 40 50 60 70
(a)(ii) 65

(a) Berdasarkan graf,


Based on the graph,
(i) nyatakan hubungan antara W dengan V,
state the relationship between W and V,
W berkadar terus dengan V./ W is directly proportional to V.

(ii) tentukan nilai V apabila W = 1.3 N.


Tunjukkan pada graf, bagaimana anda menentukan nilai V.
determine the value of V when W = 1.3 N.
Show on the graph, how you determine the value of V.
65 cm3

(b) Hitung kecerunan, m, bagi graf W melawan V. Tunjukkan pada graf, bagaimana anda
menentukan nilai m.
Calculate the gradient, m, of the graph of W against V. Show on the graph, how you determine the value of m.
1.4 – 0
m =
70.0 – 0
= 0.02 N cm–3

01 Strategi A+ SPM Fizik Tg 4 (B01) 5pp.indd 9 18/11/2023 11:04 AM


0.5m
(c) Hitung ketumpatan air, ρ, dengan menggunakan rumus ρ = g , dengan keadaan m ialah
kecerunan graf dan g = 9.81 m s . Beri jawapan anda dalam g cm .
–2 –3

0.5m
Calculate the density of water, ρ, using the formula ρ = g , where m is the gradient of graph and g = 9.81 m s .
–2

Give your answer in g cm–3.


0.5m
ρ = g
0.5(0.02 N cm–3)
=
9.81 m s–2
0.01 kg m s–2 cm–3
=
9.81 m s–2
= 0.001 kg cm–3

= 1 g cm–3

(d) Jika air digantikan dengan suatu cecair yang lebih tumpat, apakah yang akan berlaku kepada
kecerunan graf? Terangkan.
If the water is replaced by a denser liquid, what will happen to the gradient of the graph? Explain.
Bertambah kerana kecerunan graf berkadar terus dengan ketumpatan cecair.
Increases because the gradient of the graph is directly proportional to the density of liquid.

(e) Nyatakan satu langkah berjaga-jaga yang perlu diambil untuk memperbaiki keputusan
eksperimen ini.
State one precaution that should be taken to improve the result of this experiment.
Mata mesti berserenjang dengan skala bacaan neraca spring bagi mengelakkan ralat paralaks.
Eyes must be perpendicular to the scale reading of the spring balance to avoid parallax error.

Praktis Berformat SPM


2 Kuantiti yang manakah merupakan kuantiti
Kertas 1 asas?
Which quantity is a base quantity?
1 Berikut ialah ukuran bagi sebuah meja di A Kerja C Momentum
dalam sebuah makmal. Work Momentum
The following is the measurement of a table in a laboratory. B Kuasa D Arus elektrik
Panjang meja = 1.5 meter Power Electric current
Length of table = 1.5 metres
3 Kuantiti fizik yang manakah mempunyai unit
Antara yang berikut, yang manakah adalah S.I. yang betul?
betul? Which physical quantity has the correct S.I. unit?
Which of the following is correct?
Kuantiti fizik Unit S.I.
Kuantiti fizik Magnitud Unit Physical quantity S.I. unit
Physical quantity Magnitude Unit A Daya
kg m s–1
A meter Panjang Force
1.5
metre Length
B Halaju
B meter Panjang m s–2
1.5 Velocity
metre Length
C Tekanan
C Panjang meter kg m–1 s–2
1.5 Pressure
Length metre
D Ketumpatan
D Panjang meter kg m–2
1.5 Density
Length metre

10

01 Strategi A+ SPM Fizik Tg 4 (B01) 5pp.indd 10 18/11/2023 11:04 AM


4 Berikut ialah rumus untuk Hukum Newton A C
Kedua, dengan keadaan F mewakili daya
yang dinyatakan dalam Newton, m mewakili
jisim dan a mewakili pecutan.
The following is the formula for Newton’s Second Law,
where F represents the force in Newton, m represents B D
mass and a represents acceleration.

F = ma
Tentukan unit yang setara dengan Newton.
Determine the unit that is equivalent to Newton.
9 Graf dalam Rajah 1 menunjukkan hubungan
A g m s–1 C kg m s–1
antara beza keupayaan dengan arus dalam
B kg m s2 D kg m s–2
suatu litar. Nilai pintasan-y mewakili daya
5 Antara yang berikut, yang manakah ialah gerak elektrik (d.g.e.) bateri.
kuantiti skalar? The graph in Diagram 1 shows the relationship between
Which of the following is a scalar quantity? potential difference and current in a circuit. The value
I Daya III Jarak of y-intercept represents the electromotive force (e.m.f.)
Force Distance of the battery.
II Kerja IV Sesaran V (V)
Work Displacement
A I dan II C II dan III
I and II II and III
B I dan IV D II, III dan IV
I and IV II, III and IV I (A)
0
6 Antara yang berikut, yang manakah merupakan Rajah 1/ Diagram 1
senarai kuantiti terbitan? Apakah yang perlu dilakukan untuk
Which of the following is a list of derived quantities? mendapatkan nilai d.g.e. tersebut?
A Luas, isi padu, berat What should be done to get the value of e.m.f.?
Area, volume, weight KBAT Mengaplikasi
B Cas, momentum, jisim A Interpolasikan graf
Charge, momentum, mass Interpolate the graph
C Halaju, daya, suhu termodinamik B Cari kecerunan graf
Velocity, force, thermodynamic temperature Find the gradient of the graph
D Kuantiti bahan, keamatan berluminositi, C Ekstrapolasikan graf
arus elektrik Extrapolate the graph
Amount of substance, luminous intensity, electric D Cari luas di bawah graf
current Find the area under the graph

7 Antara pasangan kuantiti skalar dan kuantiti 10 Tujuan eksperimen yang ditunjukkan dalam
vektor yang berikut, yang manakah adalah Rajah 2 ialah untuk mengkaji kesan perubahan
betul? suhu terhadap tekanan. Udara di dalam kelalang
Which of the following pairs of scalar quantity and dipanaskan dan bacaan tolok Bourdon dicatat.
vector quantity is correct? The aim of the experiment shown in Diagram 2 is to
investigate the effect of change in temperature on
Kuantiti skalar Kuantiti vektor
pressure. The air in the flask is heated and the reading of
Scalar quantity Vector quantity
Bourdon gauge is recorded.
A Jisim/ Mass Berat/ Weight Kelalang Termometer
B Halaju/ Velocity Laju/ Speed Tolok Bourdon Flask Thermometer
Bourdon gauge Pengacau
C Isi padu/ Volume Tekanan/ Pressure Stirrer

D Sesaran/ Displacement Jarak/ Distance Air


Water

8 Antara alat pengukuran yang berikut, yang


manakah digunakan untuk mengukur kuantiti
terbitan?
Which of the following measuring instruments is used
to measure derived quantities? Rajah 2/ Diagram 2

11

01 Strategi A+ SPM Fizik Tg 4 (B01) 5pp.indd 11 18/11/2023 11:04 AM


Antara yang berikut, yang manakah merupakan 11 Graf dalam Rajah 3 menunjukkan hubungan
padanan yang betul untuk semua pemboleh antara isi padu, V dengan suhu, T untuk satu
ubah yang terlibat? hukum gas.
Which of the following is the correct match for all the The graph in Diagram 3 shows the relationship between
variables involved? volume, V and temperature, T of a gas law.
V (cm3)
Pemboleh Pemboleh
Pemboleh ubah ubah bergerak ubah
dimanipulasikan balas dimalarkan
Manipulated
Responding Constant
variable
variable variable
A Suhu Tekanan Isi padu –273 0
T (°C)

Temperature Pressure Volume Rajah 3/ Diagram 3

B Berdasarkan graf tersebut, apakah hubungan


Isi padu Suhu Tekanan antara isi padu dengan suhu?
Volume Temperature Pressure
Based on the graph, what is the relationship between
C volume and temperature?
Isi padu Tekanan Suhu
Volume Pressure Temperature
A V berkadar terus dengan T
V is directly proportional to T
D B V berkadar songsang dengan T
Tekanan Suhu Isi padu
Pressure Temperature Volume V is inversely proportional to T
C V berkurang secara linear dengan T
V decreases linearly with T
D V bertambah secara linear dengan T
V increases linearly with T

Kertas 2

Bahagian B
1 Rajah 1 menunjukkan alat untuk mengukur diameter suatu objek.
Diagram 1 shows the instrument for measuring the diameter of an object.

cm 0 1 2 3

0 5 10

Wayar
Wire

Rajah 1/ Diagram 1
(a) Namakan alat pengukuran itu.
Name the measuring instrument.
Angkup Vernier/Vernier calliper
[1 markah/mark]
(b) Nyatakan jenis kuantiti fizik bagi panjang.
State the type of physical quantity of length.
Kuantiti skalar/Kuantiti asas/Scalar quantity/ Basic quantity
[1 markah/mark]
(c) Apakah bacaan terkecil yang dapat diukur oleh alat pengukuran itu?
What is the smallest reading that can be measured by the measuring instrument?
0.01 cm
[1 markah/mark]

12

01 Strategi A+ SPM Fizik Tg 4 (B01) 5pp.indd 12 18/11/2023 11:04 AM


(d) Apakah bacaan yang ditunjukkan oleh alat pengukuran itu?
What is the reading shown by the measuring instrument?
0.77 cm
[1 markah/mark]
(e) Jika bacaan yang ditunjukkan merupakan diameter bagi 10 lilitan wayar, berapakah
diameter wayar itu?
If the reading shown is the diameter of 10 turns of wire, what is the diameter of the wire?
0.77
d = = 0.077 cm
10
[2 markah/marks]

2 Graf dalam Rajah 2.1 menunjukkan hubungan TT TT22


antara panjang bandul, l dengan tempoh ayunan, T.
Manakala, graf dalam Rajah 2.2 pula menunjukkan
hubungan antara panjang bandul, l dengan kuasa
dua tempoh ayunan, T2.
The graph in Diagram 2.1 shows the relationship between
length of pendulum, l and period of oscillation, T. Meanwhile,
the graph in Diagram 2.2 shows the relationship between length 00
ll
00
ll
of pendulum, l and square of period of oscillation, T2. Rajah 2.1/ Diagram 2.1 Rajah 2.2/ Diagram 2.2
(a) Nyatakan
State
(i) pemboleh ubah dimanipulasikan : Panjang bandul, l
manipulated variable : Length of pendulum, l
(ii) pemboleh ubah bergerak balas : Tempoh ayunan, T
responding variable : Period of oscillation, T
[2 markah/marks]
(b) Tulis hubungan yang ditunjukkan oleh setiap graf:
Write the relationship shown by each graph:
(i) Rajah 2.1/ Diagram 2.1 : T bertambah dengan l/ T increases with l
(ii) Rajah 2.2/ Diagram 2.2 : T berkadar terus dengan dengan l/ T is directly proportional to l
2 2

[2 markah/marks]
(c) Hubungan antara panjang bandul dengan tempoh ayunan diberi sebagai
The relationship between length of pendulum and period of oscillation is given as
4π2
T2 = � g � l

Jika panjang bandul ialah 0.5 m, hitung tempoh ayunan.


If the length of pendulum is 0.5 m, calculate the period of oscillation.
[Pecutan graviti/Gravitational acceleration, g = 9.81 m s–2]
4π2
T2 = � �l
g
4π2
T2 = � �(0.5)
9.81
T 2 = 2.01215
T = √ 2.01215
T = 1.419 s

[2 markah/marks]

13

01 Strategi A+ SPM Fizik Tg 4 (B01) 5pp.indd 13 18/11/2023 11:04 AM


Bab Daya dan Gerakan I
2 Force and Motion I
Tema: Mekanik Newton

Revisi Pantas
When the position of an object to its surroundings does not change
2.1 Gerakan Linear with time, it is said to be at rest. Stationary object has zero velocity.
Linear Motion
4 Apabila suatu objek bergerak dengan jarak yang sama
1 Gerakan ialah perubahan kedudukan bagi suatu objek pada selang masa yang sama, gerakan itu disebut
dari semasa ke semasa. gerakan seragam.
Motion is the change in the position of an object over time. When an object moves equal distance at equal time interval, the
motion is called uniform motion.
2 Gerakan linear ialah pergerakan dalam satu lintasan
yang lurus atau pergerakan dalam satu dimensi. 5 Apabila suatu objek bergerak dengan jarak yang tidak
Linear motion is a motion along a straight line or motion in one sama pada selang masa yang sama atau sebaliknya,
dimension. gerakan itu disebut gerakan tidak seragam.
3 Apabila kedudukan suatu objek dengan When an object moves unequal distance at equal time interval or vice
versa, the motion is called non-uniform motion.
persekitarannya tidak berubah dengan masa, objek itu
dikatakan berada dalam keadaan pegun. Objek pegun 6 Jadual berikut menunjukkan kuantiti fizik yang
mempunyai halaju sifar. menghuraikan gerakan linear.
The following table shows the physical quantities that describe linear
motion.

Kuantiti
fizik Definisi Kuantiti Simbol Unit Rumus berkaitan
Physical Definition Quantity Symbol Unit Related formula
quantity
Jarak Jumlah panjang yang dilalui Kuantiti s meter Jarak = Laju × Masa
Distance oleh suatu objek skalar metre Distance = Speed × Time
The total length travelled by an object Scalar quantity (m)

Sesaran Jarak terpendek di antara dua Kuantiti s meter Sesaran = Halaju × Masa
Displacement kedudukan dalam suatu arah vektor metre Displacement = Velocity × Time
tertentu Vector quantity (m)
The shortest distance between two
positions in a particular direction

Laju Kadar perubahan jarak yang Kuantiti v m s–1 Jarak


Speed dilalui skalar Laju =
Masa
Rate of change of distance travelled Scalar quantity Distance
Speed =
Time

Halaju Kadar perubahan sesaran Kuantiti v m s–1 Sesaran


Velocity Rate of change of displacement vektor Halaju =
Masa
Vector quantity Displacement
Velocity =
Time

Pecutan Kadar perubahan halaju Kuantiti a m s–2 Halaju akhir – Halaju awal
Acceleration Rate of change of velocity vektor Pecutan =
Masa
Vector quantity
Final velocity – Initial velocity
Acceleration =
Time

7 Suatu objek dikatakan bergerak dengan halaju change of displacement remains the same, whereas an object is said
seragam apabila kadar perubahan sesaran adalah sama, to be moving with non-uniform velocity if its rate of change of
manakala suatu objek dikatakan bergerak dengan displacement increases or decreases.
halaju tidak seragam apabila kadar perubahan 8 Jadual berikut menunjukkan perbandingan antara
sesarannya semakin bertambah atau semakin halaju seragam dengan halaju tidak seragam.
berkurang. The following table shows the comparison between uniform velocity
An object is said to be moving with uniform velocity if its rate of and non-uniform velocity.

14

02 Strategi A+ SPM Fizik Tg 4 (B02) 5pp.indd 14 18/11/2023 9:44 AM


Halaju seragam Halaju tidak seragam 13 Persamaan gerakan linear:
Uniform velocity Non-uniform velocity Equations of linear motion: i-THINK Peta Buih

Apabila suatu objek Apabila suatu objek v = u + at s = ut + 1 at2


bergerak dalam satu bergerak dengan 2
lintasan lurus dengan perubahan kelajuan atau
laju seragam arah atau kedua-duanya Persamaan gerakan linear
When an object moves in a When an object moves with a Equations of linear motion
straight line with a uniform change in speed or direction
speed or both
s = 1 (u + v)t v2 = u2 + 2as
2
Bergerak dengan sesaran Bergerak dengan sesaran
yang sama dalam sela yang tidak sama dalam dengan keadaan/ where
masa yang sama sela masa yang sama s = Sesaran (m)
Moves with the same Moves with uneven Displacement
displacement in equal time displacement in equal time u = Halaju awal (m s–1)
intervals intervals Initial velocity
v = Halaju akhir (m s–1)
Final velocity
a = Pecutan (m s–2)
Pecutan sifar Pecutan bukan
Acceleration
Zero acceleration sifar (pecutan atau
t = Masa (s)
nyahpecutan)
Time
Non-zero acceleration
(accelerate or decelerate)

2.2 Graf Gerakan Linear


Linear Motion Graphs
9 Jangka masa detik digunakan untuk mengkaji gerakan 1 Terdapat beberapa cara untuk mewakilkan gerakan
linear suatu objek di dalam makmal. linear. Salah satu cara yang paling berguna ialah
The ticker-timer is used to study the linear motion of an object in dengan melukis graf sesaran, halaju dan pecutan
the laboratory.
melawan masa.
There are a number of ways of representing linear motion. The
Bersambung ke bekalan Ditarik oleh sebuah most useful one is graphing displacement, velocity and acceleration
Magnet
kuasa arus ulang-alik objek bergerak against time.
Connected to alternating
kekal Pulled by a
current power supply
Permanent
moving object 2 Jadual berikut menunjukkan senarai maklumat yang
magnet
Plat bergetar
terdapat dalam graf sesaran, halaju dan pecutan
Vibrating plate melawan masa.
The following table shows a list of the information available in the
Kertas karbon displacement, velocity and acceleration against time graphs.
Carbon paper

Graf Graf Graf


Pita detik
Solenoid Jalur logam Ticker tape
Ciri-ciri sesaran-masa halaju-masa pecutan-masa
Metal strip Characteristic Displacement-time Velocity-time Acceleration-time
graph graph graph
Jangka masa detik dengan pita detik Paksi-x
A ticker-timer with ticker tape diwakili oleh Sesaran, s Halaju, v Pecutan, a
The x-axis is Displacement Velocity Acceleration
10 Jangka masa detik disambungkan dengan arus ulang- represented by
alik berfrekuensi 50 Hz dan menghasilkan 50 titik
setiap saat pada pita detik. Paksi-y
The ticker-timer is connected to an alternating current with diwakili oleh Masa, t Masa, t Masa, t
a frequency of 50 Hz and produces 50 dots every second on the The y-axis is Time Time Time
ticker tape. represented by
11 Sela masa antara dua titik dinamakan satu detik. Kecerunan
The time interval between two dots is called one tick. graf adalah
Satu detik/ One tick = 0.02 s sama dengan Halaju Pecutan

The gradient Velocity Acceleration
12 Sesaran bagi objek yang bergerak boleh ditentukan
of the graph is
dengan mengukur jarak di antara titik-titik pada
equal to
pita detik itu menggunakan pembaris. Dengan
menggunakan sesaran yang diperoleh dan masa yang Luas di
diambil, halaju dan pecutan boleh dihitung. bawah graf
The displacement of a moving object can be determined by measuring adalah sama Sesaran
the distance between the dots on the ticker tape using a ruler. Using dengan – –
Displacement
the displacement and time recorded, the velocity and acceleration can The area under
be calculated. the graph is
equal to

15

02 Strategi A+ SPM Fizik Tg 4 (B02) 5pp.indd 15 18/11/2023 9:44 AM


3 Jadual berikut menunjukkan graf sesaran, halaju dan pecutan melawan masa bagi pelbagai jenis gerakan.
The following table shows the displacement, velocity and acceleration graphs against time for various types of motion.

Jenis graf Bentuk graf dan jenis gerakan


Type of graph Shape of graph and type of motion

Sesaran, s (m) Sesaran, s (m) Sesaran, s (m)


Displacement Displacement Displacement

Graf
sesaran-masa
Displacement-
time graph
0 Masa, t (s) 0 Masa, t (s) 0 Masa, t (s)
Time Time Time
Halaju = 0 Halaju seragam Halaju seragam negatif
Velocity = 0 Uniform velocity Negative uniform velocity

Halaju, v (m s–1) Halaju, v (m s–1) Halaju, v (m s–1)


Velocity Velocity Velocity

Graf
halaju-masa
Velocity-time
graph
0 Masa, t (s) 0 Masa, t (s) 0 Masa, t (s)
Time Time Time
Halaju seragam Pecutan seragam Nyahpecutan seragam
Uniform velocity Uniform acceleration Uniform deceleration

Pecutan, a (m s–2) Pecutan, a (m s–2) Pecutan, a (m s–2)


Acceleration Acceleration Acceleration

Graf
pecutan-
masa
Acceleration-
time graph
0 Masa, t (s) 0 Masa, t (s) 0 Masa, t (s)
Time Time Time
Pecutan = 0 Pecutan seragam Pecutan bertambah
Acceleration = 0 Uniform acceleration Increasing acceleration

2.3 Gerakan Jatuh Bebas


Free Fall Motion

1 Medan graviti Bumi mengenakan daya graviti ke atas


sebarang objek yang berada dalam medan itu.
The gravitational field of the Earth exerts gravitational force on any
object located in the field.
Daya graviti
Gravitational force
2 Suatu objek berada dalam keadaan jatuh bebas apabila
satu-satunya daya yang bertindak ke atasnya ialah
daya graviti.
An object is in free fall when the only force acting on it is the
gravitational force.

16

02 Strategi A+ SPM Fizik Tg 4 (B02) 5pp.indd 16 18/11/2023 9:44 AM


3 Secara praktikal, jatuh bebas hanya boleh berlaku 3 Suatu objek boleh mempunyai momentum yang besar
dalam keadaan vakum. jika objek itu mempunyai sama ada jisim yang kecil
Practically, free fall can only take place in vacuum. tetapi halaju yang tinggi atau mempunyai halaju yang
4 Pecutan graviti ialah pecutan objek yang jatuh bebas rendah tetapi jisim yang besar.
An object can have a large momentum if it has either a small mass
disebabkan oleh daya tarikan graviti. Simbol bagi
but a high velocity or a low velocity but a large mass.
pecutan graviti ialah g.
The gravitational acceleration is the acceleration of an object due 4 Prinsip Keabadian Momentum menyatakan bahawa
to the gravitational pull. The symbol of gravitational acceleration is jumlah momentum suatu sistem adalah malar jika
g. tiada daya luar yang bertindak ke atas sistem itu.
5 Nilai bagi pecutan graviti ialah lebih kurang 9.81 m s–2. The Principle of Conservation of Momentum states that the total
The magnitude of gravitational acceleration is approximately momentum of a system is constant if no external force is acted upon
9.81 m s–2. the system.

6 Pecutan graviti tidak bergantung pada jisim suatu 5 Hal ini bermaksud jumlah momentum sebelum
objek yang bergerak. perlanggaran adalah sama dengan jumlah momentum
Gravitational acceleration does not depend on the mass of a moving selepas perlanggaran.
object. This means that the total momentum before collision is equal to the
total momentum after collision.
7 Oleh sebab gerakan jatuh bebas ialah gerakan
pecutan yang seragam, maka persamaan linear 6 Prinsip Keabadian Momentum diaplikasikan dalam
The Principle of Conservation of Momentum is applied in
yang menerangkan gerakan pecutan seragam boleh
(a) perlanggaran kenyal
digunakan.
Since free fall motion is uniformly accelerated motion, the linear elastic collision
equations that describe uniformly accelerated motion can be used. (b) perlanggaran tak kenyal
inelastic collision
v = u + gt (c) letupan
explosion
v2 = u2 + 2gs
s = ut + 1 gt2
2
2.6 Daya
Force

2.4 Inersia 1 Daya boleh menyebabkan suatu objek berjisim


Inertia mengubah halajunya.
Force can cause an object with a mass to change its velocity.
1 Inersia ialah kecenderungan suatu objek untuk kekal
pegun atau terus bergerak jika sedang bergerak. 2 Oleh itu, hubungan antara daya, F, jisim, m, dan
Inertia is the tendency of an object to remain at rest or continue its pecutan, a ialah F ∝ ma.
motion if in motion. Therefore, the relationship between force, F, mass, m and acceleration,
a is F ∝ ma.
2 Konsep inersia dijelaskan dalam Hukum Gerakan
Newton Pertama. 3 Oleh sebab pecutan ialah perubahan halaju dengan
The concept of inertia is explained in Newton’s First Law of masa, maka
Motion. Since acceleration is the change of velocity by time, then

3 Inersia suatu objek diukur dengan jisimnya. Semakin v–u (mv – mu)
besar jisim objek, semakin besar inersia objek tersebut. F ∝ m� t � = F ∝ t
The inertia of an object is measured by its mass. The larger the mass
of an object, the larger the inertia of the object.
4 Hubungan ini dinyatakan dalam Hukum Gerakan
Newton Kedua.
Momentum This relationship is described in Newton’s Second Law of Motion.
2.5
Momentum 5 Daripada hubungan F ∝ ma, F = kma, k ialah pemalar.
From the relationship F ∝ ma, F = kma, k is a constant.
1 Momentum ialah suatu kuantiti yang bergantung pada
6 Dalam unit S.I., 1 N ialah daya yang menghasilkan
jisim dan halaju suatu objek.
Momentum is a quantity that depends on the mass and velocity of pecutan 1 m s–2 apabila bertindak ke atas jisim 1 kg.
an object. Gantikan dalam persamaan, F = kma, kita akan dapat
k = 1.
2 Rumus momentum diberi sebagai In S.I. units, 1 N is the force that gives an acceleration of 1 m s–2 to
The formula of momentum is given as a mass of 1 kg. Substituting in equation, F = kma, we get k = 1.
Maka/ Therefore,
p = mv
F = ma
dengan keadaan/where p = Momentum
m = Jisim/ Mass
dengan keadaan/ where F = Daya/ Force
v = Halaju/ Velocity
m = Jisim/ Mass
a = Pecutan/ Acceleration

17

02 Strategi A+ SPM Fizik Tg 4 (B02) 5pp.indd 17 18/11/2023 9:44 AM


2.7 Impuls dan Daya Impuls
Impulse and Impulsive Force

1 Jadual berikut menunjukkan perbandingan antara impuls dengan daya impuls.


The following table shows the comparison between impulse and impulsive force.

Ciri-ciri Impuls Daya impuls


Characteristic Impulse Impulsive force

Hasil darab bagi daya yang dikenakan ke atas Daya yang bertindak ke atas suatu objek
suatu objek dan masa impak untuk sela masa yang singkat ketika
Definisi The product of the force acting on an object and the time perlanggaran atau hentaman
Definition of action A force which acts on an object for a very short interval
during a collision or impact

Definisi lain Perubahan momentum Kadar perubahan momentum


Another definition Change of momentum Rate of change of momentum

Impuls/ Impulse, J = mv – mu mv – mu
Daya impuls/ Impulsive force, F =
= Ft t

Rumus dengan keadaan/where dengan keadaan/where


Formula F = Daya yang dikenakan mv – mu = Perubahan momentum
Applied force Change of momentum
t = Masa tindakan t = Masa impak
Time of action Time of impact

2 Daya impuls boleh dikurangkan dengan 4 Jadual berikut menunjukkan perbandingan antara
mengurangkan perubahan momentum atau berat dengan jisim.
meningkatkan masa hentaman. The following table shows the comparison between weight and mass.
Impulsive force can be reduced by reducing the change of momentum
or increasing the time of impact. Berat Jisim
Weight Mass
3 Daya impuls boleh ditingkatkan dengan meningkatkan
perubahan momentum atau mengurangkan masa Daya disebabkan oleh Suatu ukuran bagi
hentaman. graviti yang bertindak ke jumlah bahan dalam
Impulsive force can be increased by increasing the change of atas suatu objek suatu objek
momentum or decreasing the time of impact. The force due to gravity acting A measure of the amount of
on an object substance in an object
4 Kebanyakan situasi yang melibatkan daya impuls
melibatkan sepasang daya, iaitu daya tindakan dan Bergantung pada Tidak bergantung pada
daya tindak balas. Hubungan antara daya tindakan kekuatan medan graviti kekuatan medan graviti
dengan daya tindak balas dijelaskan dalam Hukum Depends on the gravitational Independent from the
field strength gravitational field strength
Gerakan Newton Ketiga.
Most situations involving impulsive forces involve a pair of forces, Kuantiti vektor Kuantiti skalar
which are action and reaction. The relationship between action and Vector quantity Scalar quantity
reaction is explained in Newton’s Third Law of Motion.
Unit: Newton (N) Unit: kilogram (kg)

5 Medan graviti ialah suatu kawasan yang menyebabkan


2.8 Berat suatu jisim mengalami satu daya tarikan graviti.
Weight A gravitational field is a region in which a mass experiences
a force due to the gravitational pull.
1 Berat suatu objek, W ialah daya graviti yang bertindak
6 Kekuatan medan graviti, g ialah daya graviti yang
ke atas objek itu.
bertindak ke atas suatu jisim 1 kg.
The weight of an object, W is the gravitational force acting on the
The gravitational field strength, g is the gravitational force acting
object.
on a mass of 1 kg.
W = mg
W
g=
dengan keadaan/ where m = Jisim objek/ Mass of object m
g = Kekuatan medan graviti
Gravitational field strength dengan keadaan/ where W = Berat atau daya graviti
Weight or gravitational force
2 Bagi kekuatan medan graviti suatu objek, semakin
m = Jisim/ Mass
besar jisim objek, semakin besar berat objeknya.
For the gravitational field strength of an object, the greater the mass 7 Unit bagi kekuatan medan graviti ialah N kg–1.
of an object, the greater its weight. The unit of gravitational field strength is N kg–1.

3 Berat tidak sama dengan jisim. 8 Kekuatan medan graviti di Bumi ialah 9.81 N kg–1.
Weight is not the same as mass. The gravitational field strength on the Earth is 9.81 N kg–1.

18

02 Strategi A+ SPM Fizik Tg 4 (B02) 5pp.indd 18 18/11/2023 9:44 AM


Praktis PBD
2.1 Gerakan Linear/ Linear Motion
Latihan 1 Penerangan bagi gerakan linear/ Description of linear motion
TP 1 Mengingat kembali pengetahuan dan kemahiran sains mengenai Daya dan Gerakan I.
TP 2 Memahami Daya dan Gerakan I serta dapat menjelaskan kefahaman tersebut.

1 Tandakan (3) bagi rajah yang menunjukkan gerakan linear dan (7) bagi yang tidak. TP 1
Mark (3) for the diagrams that show linear motion and (7) for those that do not.

(a)

A θ = 60° B

(b)

(c)

2 Lengkapkan peta pokok berikut tentang gerakan linear. TP 2


Complete the following tree map about linear motion. i-THINK Peta Pokok

Gerakan linear
Linear motion

Gerakan linear seragam Gerakan linear tidak seragam


Uniform linear motion Non-uniform linear motion

Objek dalam keadaan (a) Objek bergerak dengan halaju


pegun atau bergerak dengan (d) tidak seragam , maka ada
(b) halaju seragam , maka pecutan (e) pecutan atau (f) nyahpecutan .
ialah (c) sifar . Object is moving with (d) non-uniform velocity ,
Object in (a) stationary state or moving so there is (e) acceleration or
with (b) uniform velocity , so the acceleration (f) deceleration .
is (c) zero .

19

02 Strategi A+ SPM Fizik Tg 4 (B02) 5pp.indd 19 18/11/2023 9:44 AM


Latihan 2 Pengukuran bagi gerakan linear/ Measurement of linear motion
TP 1 Mengingat kembali pengetahuan dan kemahiran sains mengenai Daya dan Gerakan I.

1 Lengkapkan peta buih berganda berikut tentang perbandingan antara jarak dengan sesaran. TP 1
Complete the following double bubble map about the comparison between distance and displacement.
i-THINK Peta Buih Berganda

(a) (e)
Jumlah panjang Jarak terpendek
lintasan yang dilalui objek di antara dua titik dalam arah tertentu
Total length of the path travelled Shortest distance
by an object between two points in a
specific direction

(b) (d) Unit S.I.: (f)


Simbol: S.I. unit: Simbol:
Jarak Sesaran
Symbol: Distance meter (m) Displacement Symbol:
d metre (m) s

(c) (g)
Kuantiti Kuantiti
skalar vektor
Scalar Vector
quantity quantity

2 Lengkapkan peta buih berganda berikut tentang perbandingan antara laju dengan halaju. TP 1
Complete the following double bubble map about the comparison between speed and velocity.
i-THINK Peta Buih Berganda

(a) (f)
Kadar Kadar
perubahan jarak perubahan sesaran
Rate of change Rate of change
of distance (d) of displacement
Simbol:
Symbol:
v
(b) (g)
Kuantiti Kuantiti
skalar Laju Halaju vektor
Speed Velocity
Scalar Vector
quantity quantity
(e)
Unit S.I.:
(c) S.I. unit: (h)
Rumus/Formula: meter per saat (m s–1)
Rumus/Formula:
Jarak, d (m) Sesaran, s (m)
Masa, t (s) metre per second Masa, t (s)
Distance, d (m) Displacement, s (m)
Time, t (s) Time, t (s)

20

02 Strategi A+ SPM Fizik Tg 4 (B02) 5pp.indd 20 18/11/2023 9:44 AM


3 Lengkapkan peta buih berganda berikut tentang perbandingan antara pecutan dengan
nyahpecutan. TP 1
Complete the following double bubble map about the comparison between acceleration and deceleration.
i-THINK Peta Buih Berganda

(d)
Simbol:
Symbol:
a
(a) (h)
Kadar Kadar
penambahan halaju pengurangan halaju
Rate of increase Rate of decrease

of velocity (e) of velocity


Unit S.I.:
S.I. unit:
m s–2
(b) (i)
Halaju Halaju
bertambah dalam berkurang dalam
arah tertentu Pecutan Nyahpecutan arah tertentu
Velocity Acceleration Deceleration Velocity
increases in a decreases in a
specific direction specific direction
(f)
Kuantiti
vektor
(c) Vector (j)
Halaju awal, u quantity Halaju awal, u
lebih kecil berbanding lebih besar berbanding
dengan halaju akhir, v dengan halaju akhir, v
Initial velocity, u is less Initial velocity, u is more
than final velocity, v than final velocity, v
(g) Rumus/Formula:
Halaju akhir, v – Halaju awal, u (m s–1)
Masa, t (s)
Final velocity, v – Initial velocity, u (m s–1)
Time, t (s)

Latihan 3 Menentukan jarak, sesaran, laju, halaju, pecutan dan nyahpecutan


Determining distance, displacement, speed, velocity, acceleration and deceleration
TP 3 Mengaplikasikan pengetahuan mengenai Daya dan Gerakan I untuk menerangkan kejadian atau fenomena alam dan melaksanakan tugasan mudah.

Selesaikan masalah yang berikut. Sekolah


Solve the following problems. School

1 Setiap pagi, Syazwan akan berjalan melepasi rumah


Ahmad yang terletak 60 m dari rumahnya untuk pergi
ke sekolah yang terletak 80 m dari rumah Ahmad
seperti yang ditunjukkan dalam rajah di sebelah. TP 3 80 m
Every morning, Syazwan will walk past Ahmad’s house located Rumah Ahmad
60 m from his house to go to school which is located 80 m Ahmad’s house

from Ahmad’s house as shown in the diagram on the right.


Syazwan 60 m

21

02 Strategi A+ SPM Fizik Tg 4 (B02) 5pp.indd 21 18/11/2023 9:44 AM


(a) Kira
Calculate
(i) jarak yang dilalui oleh Syazwan,
the distance travelled by Syazwan,
d = 60 + 80 = 140 m

(ii) sesaran Syazwan.


the displacement of Syazwan.
s = 602 + 802
= 3 600 + 6 400
= 10 000
= 100 m

(b) Jika Syazwan mengambil masa 5 minit untuk sampai ke sekolah, tentukan
If Syazwan takes 5 minutes to reach the school, determine his
(i) lajunya, (ii) halajunya.
speed, velocity.
140 100
v = v =
5 × 60 5 × 60
140 100
= =
300 300
= 0.467 m s–1 = 0.333 m s–1

2 Sebuah lori bergerak sejauh 10 km ke arah utara sebelum mengubah haluan ke barat sejauh 4 km.
Selepas itu, lori itu bergerak menuju ke selatan sejauh 7 km. TP 3
A lorry moves at a distance of 10 km to the north before turning west for 4 km. After that, the lorry moves southwards
for 7 km.
(a) Lukis gambar rajah yang menunjukkan pergerakan lori tersebut dan label sesarannya
dengan simbol s.
Draw a diagram showing the movement of the lorry and label its displacement with the symbol of s.
4 km

7 km
10 km

s 3 km

O
(b) Kira jarak dan sesaran lori tersebut.
Calculate the distance and displacement of the lorry.
Jarak/ Distance, d = 10 + 4 + 7 = 21 km
Sesaran/ Displacement, s = 42 + 32 = 16 + 9 = 25 = 5 km

(c) Jika lori tersebut mengambil masa 24 minit untuk perjalanan tersebut, kira laju dan halaju
lori tersebut dalam km j–1.
If the lorry takes 24 minutes for the trip, calculate the speed and velocity of the lorry in km ℎ–1.
24
Masa/Time = = 0.4 j/ ℎ
60
d 21
Laju/Speed = v = = = 52.5 km j–1/ km ℎ–1
t 0.4
s 5
Halaju/Velocity = v = = = 12.5 km j–1/ km ℎ–1
t 0.4

22

02 Strategi A+ SPM Fizik Tg 4 (B02) 5pp.indd 22 18/11/2023 9:44 AM


3 Sebuah van memecut secara seragam daripada halaju 20 m s–1 kepada 25 m s–1 dalam masa
2.5 s. Berapakah pecutan van itu? TP 3
A van accelerates uniformly from a velocity of 20 m s–1 to 25 m s–1 in 2.5 s. What is the acceleration of the van?
u = 20 m s–1, v = 25 m s–1, t = 2.5 s
v–u
a=
t
25 – 20
=
2.5
= 2 m s–2

4 Sebuah kereta yang bergerak dengan halaju 16 m s–1 menjadi semakin perlahan apabila lampu
isyarat bertukar merah. Selepas 4 saat, kereta tersebut berhenti di hadapan lampu isyarat. Kira
pecutan kereta itu. TP 3
A car moving at a velocity of 16 m s–1 becomes slower when the traffic light turns red. After 4 seconds, the car stops in
front of the traffic light. Calculate the acceleration of the car.
u = 16 m s–1, v = 0 m s–1, t=4s
v–u
a=
t
0 – 16
=
4
= –4 m s–2
Kereta mengalami nyahpecutan kerana v < u.
The car experiences deceleration because v < u.

Latihan 4 Menghuraikan gerakan linear menggunakan jangka masa detik dan pita detik
Describing linear motion using ticker-timer and ticker tape
TP 1 Mengingat kembali pengetahuan dan kemahiran sains mengenai Daya dan Gerakan I.
TP 2 Memahami Daya dan Gerakan I serta dapat menjelaskan kefahaman tersebut.
TP 3 Mengaplikasikan pengetahuan mengenai Daya dan Gerakan I untuk menerangkan kejadian atau fenomena alam dan melaksanakan tugasan mudah.
TP 4 Menganalisis pengetahuan mengenai Daya dan Gerakan I dalam konteks penyelesaian masalah mengenai kejadian atau fenomena alam.

1 Isi tempat kosong dengan jawapan yang betul. TP 1 TP 2


Fill in the blanks with the correct answers. Pita detik
Ticker tape
Rajah di sebelah menunjukkan sebuah
jangka masa detik dan pita detik. Jangka masa detik
The diagram on the right shows a ticker-timer and ticker tape. Ticker-timer

1 detik
1 tick

(a) Jangka masa detik digunakan untuk menentukan (i) halaju dan (ii) pecutan

suatu objek dengan memplotkan titik-titik di atas pita detik.


The ticker-timer is used to determine the (i) velocity and (ii) acceleration of an object by plotting
points on the ticker tape.
(b) Jangka masa detik itu terdiri daripada penggetar elektrik yang bergetar 50 kali sesaat (50 Hz).
The ticker-timer consists of an electric vibrator
which vibrates 50 times per second (50 Hz).
(c) Sela masa antara dua titik berturutan pada pita detik dikenali sebagai (i) satu detik .
1 –1 0.02 s
Oleh itu, 1 detik adalah sama dengan s atau (ii) .
50
The time interval between two adjacent dots on the ticker tape is known as (i) one tick .
1 –1 0.02 s
Therefore, 1 tick is equal to s or (ii) .
50
23

02 Strategi A+ SPM Fizik Tg 4 (B02) 5pp.indd 23 18/11/2023 9:44 AM


2 Jangka masa detik di 1 digunakan untuk memplot titik-titik pada pita detik yang ditunjukkan.
Nyatakan jenis gerakan yang diwakili bagi setiap pita detik berikut dan jawab soalan yang
berikut. TP 3 TP 4
The ticker-timer in 1 is used to plot the dots on the ticker tapes shown. State the type of motion represented by each of the
following ticker tapes and answer the following questions.

(a) (i) Jenis gerakan/ Type of motion =

Arah gerakan/ Direction of motion


Halaju seragam/ Pecutan sifar

30 cm Uniform velocity/ Zero acceleration


A
(ii) Pita detik manakah yang menunjukkan halaju
paling rendah dan paling tinggi?
B Which ticker tapes show the lowest and highest velocities?
Paling rendah/Lowest: A

C Paling tinggi/Highest: C

(iii) Kira halaju pita detik A, B dan C.


Calculate the velocity of ticker tapes A, B and C.
30 30
vA = = = 125 cm s–1
0.02 × 12 0.24
30 30
vB = = = 250 cm s–1
0.02 × 6 0.12
30 30
vC = = = 375 cm s–1
0.02 × 4 0.08
(b) (i) Jenis gerakan/ Type of motion =

Arah gerakan/ Direction of motion Halaju bertambah/ Memecut


Increasing velocity/ Accelerates

0.25 cm 1.5 cm (ii) Hitung halaju awal./ Calculate the initial velocity.
0.25
u= = 12.5 cm s–1
0.02
(iii) Hitung halaju akhir/ Calculate the final velocity.
1.5
v = = 75 cm s–1
0.02
(iv) Hitung pecutan./ Calculate the acceleration.
75 – 12.5 62.5
a = = = 625 cm s–1
(6 – 1) × 0.02 0.1

(c) (i) Jenis gerakan/ Type of motion =

Arah gerakan/ Direction of motion Halaju berkurang/ Menyahpecutan


Decreasing velocity/ Decelerates

0.5 cm 2.5 cm (ii) Hitung halaju awal./ Calculate the initial velocity.
2.5
u= = 125 cm s–1
0.02
(iii) Hitung halaju akhir./ Calculate the final velocity.
0.5
v= = 25 cm s–1
0.02
(iv) Hitung pecutan./ Calculate the acceleration.
25 – 125 100
a = =– = –1 000 cm s–1
(6 – 1) × 0.02 0.1

24

02 Strategi A+ SPM Fizik Tg 4 (B02) 5pp.indd 24 18/11/2023 9:44 AM


(d) (i) Jenis gerakan/ Type of motion =
Panjang (cm) Halaju bertambah secara seragam/ Pecutan seragam
Length
Velocity increases uniformly/ Uniform acceleration
20

16 (ii) Cari masa yang diambil untuk satu jalur.
Find the time taken for one strip.
12
t = 5 × 0.02 = 0.1 s

8

4
(iii) Hitung halaju awal.
Calculate the initial velocity.
0
Masa (s) 4
Time u= = 40 cm s–1
0.1
(iv) Hitung halaju akhir.
Calculate the final velocity.
20
v= = 200 cm s–1
0.1

(v) Kira jumlah masa yang diambil.



Calculate the total time taken.
t = (5 – 1) × 0.1 = 0.4 s

(vi) Hitung pecutan.


Calculate the acceleration.
200 – 40 160

a= = = 400 cm s–2
0.4 0.4

(e) (i) Jenis gerakan/ Type of motion =


Panjang (cm) Halaju berkurang secara seragam/ Nyahpecutan
Length
seragam
50 Velocity decreases uniformly/ Uniform deceleration
45
40
35 (ii) Cari masa yang diambil untuk satu jalur.
30 Find the time taken for one strip.
25
20 t = 10 × 0.02 = 0.2 s

15
10
5
(iii) Hitung halaju awal.
0 Masa (s) Calculate the initial velocity.
Time
50
u= = 250 cm s–1
0.2

(iv) Hitung halaju akhir.



Calculate the final velocity.
30
v = = 150 cm s–1
0.2

(v) Kira jumlah masa yang diambil.



Calculate the total time taken.
t = (5 – 1) × 0.2 = 0.8 s

(vi) Hitung pecutan.
Calculate the acceleration.
150 – 250 100
a= =– = –125 cm s–2
0.8 0.8

25

02 Strategi A+ SPM Fizik Tg 4 (B02) 5pp.indd 25 18/11/2023 9:44 AM


Latihan 5 Persamaan gerakan linear dengan pecutan seragam
Equations of linear motion with uniform acceleration
TP 1 Mengingat kembali pengetahuan dan kemahiran sains mengenai Daya dan Gerakan I.
TP 2 Memahami Daya dan Gerakan I serta dapat menjelaskan kefahaman tersebut.

1 Lengkapkan peta buih berikut dengan persamaan gerakan linear. TP 2


Complete the following bubbles map with the equations of linear motion.
i-THINK Peta Buih

(a) (b)

1
v = u + at s = ut + 2 at2

Persamaan
gerakan linear
Equations of
linear motion

(d) (c)

1
s = 2 (u + v)t v2 = u2 + 2as

2 Tulis kuantiti yang diwakili oleh setiap simbol dalam persamaan di 1. TP 1


Write the quantity represented by each symbol in the equations in 1.
i-THINK Peta Titi
Simbol
Symbol s u v a t
Kuantiti Sesaran Halaju awal Halaju akhir Pecutan Masa
Quantity Displacement Initial velocity Final velocity Acceleration Time

Latihan 6 Menyelesaikan masalah gerakan linear menggunakan persamaan gerakan linear


Solving the linear motion problems using the equations of linear motion
TP 3 Mengaplikasikan pengetahuan mengenai Daya dan Gerakan I untuk menerangkan kejadian atau fenomena alam dan melaksanakan tugasan mudah.
TP 4 Menganalisis pengetahuan mengenai Daya dan Gerakan I dalam konteks penyelesaian masalah mengenai kejadian atau fenomena alam.
TP 5 Menilai pengetahuan mengenai Daya dan Gerakan I dalam konteks penyelesaian masalah dan membuat keputusan untuk melaksanakan satu tugasan.

Selesaikan masalah yang berikut.


Solve the following problems.
1 Sebuah motosikal memecut dengan kadar 12 m s–2 a

daripada keadaan rehat sepanjang suatu laluan lurus.


Berapakah sesaran motosikal itu selepas 4 saat? TP 3
A motorcycle accelerates at a rate of 12 m s–2 from rest along
a straight path. What is the displacement of the motorcycle
after 4 seconds?
Diberi/Given:
u = 0 m s–1
a = 12 m s–2
t = 4s
1
s = ut + at2
2 1
s = (0)(4) + (12)(4)2 = 96 m
2

26

02 Strategi A+ SPM Fizik Tg 4 (B02) 5pp.indd 26 18/11/2023 9:44 AM


2 Sebuah kereta bergerak dengan halaju 22 m s–1 sepanjang jalan yang lurus. Pemandu menekan brek
selama 5 s dan mengalami nyahpecutan 4 m s–2. Berapakah halaju akhir kereta itu? TP 3
A car moves with a velocity of 22 m s–1 along a straight path. The driver steps on the brake for 5 s and experiences
the deceleration of 4 m s–2. What is the final velocity of the car?
Diberi/Given:
u = 22 m s–1
a = –4 m s–2
t = 5s
v = u + at
= 22 + (–4)(5) = 2 m s–1

3 Zafira memandu keretanya daripada keadaan pegun dengan pecutan seragam sehingga mencapai
halaju 20 m s–1 dalam masa 5.0 s. Tentukan TP 4
Zafira drives her car from rest with uniform acceleration until reaching a velocity of 20 m s–1 in 5.0 s. Determine
(a) pecutan keretanya,
the acceleration of her car,
(b) sesaran kereta dalam 5.0 saat itu,
the displacement of the car in the 5.0 seconds,
(c) halaju kereta pada masa, t = 3.0 s,
the velocity of the car at time, t = 3.0 s,
(d) halaju kereta selepas bergerak sejauh 40 m dari titik mula. KBAT Menganalisis
the velocity of the car after moving 40 m from the starting point.
Diberi/Given:
u = 0 m s–1
v = 20 m s–1
t = 5.0 s
v–u 20 – 0
(a) a = = = 4 m s–2
t 5
1 1
(b) s = (u + v)t = (0 + 20)(5) = 50 m
2 2
atau/or
1 1
s = ut + at2 = (0)(5) + (4)(5)2 = 50 m
2 2
(c) v = u + at = 0 + 4(3.0) = 12 m s–1
(d) v2 = u2 + 2as
v = 02 + 2(4)(40) = 320 = 17.889 m s–1

4 Sebuah kereta sedang bergerak dengan halaju malar 90 km j–1 apabila pemandunya ternampak
sebatang pokok tumbang di hadapan dan segera menekan brek. Kereta berhenti dalam masa 4 s.
Jarak di antara pokok tumbang dengan kereta apabila pemandu ternampak pokok tumbang itu ialah
60 m. Berapakah jarak kereta dari pokok tumbang selepas kereta berhenti? TP 5 KBAT Menganalisis
A car is moving with a constant velocity of 90 km h–1 when the driver sees a fallen tree in front and immediately steps
on the brakes. The car stops in 4 seconds. The distance between the fallen tree and the car when the driver sees the fallen
tree is 60 m. What is the distance of the car from the fallen tree after the car stops?
Diberi/Given: 90 × 1 000
u = 90 km j–1/ km h–1 = = 25 m s–1
3 600
v = 0ms –1

t = 4s
Menggunakan/Using v = u + at,
0 = 25 + a(4)
a = –6.25 m s–2
Maka/Thus,
1 1
s = ut + at2 = (25)(4) + (–6.25)(4)2 = 50 m
2 2
Maka, jarak kereta dari pokok tumbang selepas kereta berhenti = 60 – 50 = 10 m
Thus, the distance of car from fallen tree after the car stops = 60 − 50 = 10 m

27

02 Strategi A+ SPM Fizik Tg 4 (B02) 5pp.indd 27 18/11/2023 9:44 AM


2.2 Graf Gerakan Linear/ Linear Motion Graphs
Latihan 7 Mentafsir graf sesaran-masa/ Interpreting displacement-time graph
TP 1 Mengingat kembali pengetahuan dan kemahiran sains mengenai Daya dan Gerakan I.
TP 2 Memahami Daya dan Gerakan I serta dapat menjelaskan kefahaman tersebut.
TP 3 Mengaplikasikan pengetahuan mengenai Daya dan Gerakan I untuk menerangkan kejadian atau fenomena alam dan melaksanakan tugasan mudah.

Jadual berikut menunjukkan sesaran sebuah kereta pada masa yang berbeza.
The following table shows the displacement of a car at different times.

Masa, t (s)
0 5 10 15 25 30
Time
Sesaran, s (m)
0 100 100 150 0 –50
Displacement

1 Lakarkan graf sesaran-masa bagi data yang diberi. TP 1 TP 3


Sketch the displacement-time graph for the data given.

Sesaran (m)
Displacement

150

100

50

Masa (s)
0 5 10 15 20 25 30 Time

–50

2 Berdasarkan graf sesaran-masa yang dilukis di 1, lengkapkan jadual berikut untuk menerangkan
gerakan kereta tersebut. TP 2 TP 3
Based on the displacement-time graph drawn in 1, complete the following table to explain the motion of the car.

Sela masa Huraian


Time interval Description

(a) 0 s – 5 s (i) Kereta bermula daripada keadaan pegun pada kedudukan 0m .


The car starts from rest at position 0m .
(ii) Kemudian, kereta bergerak dengan halaju seragam dan tiba di kedudukan
100 m pada t = 5 s.
Then, the car moves with uniform velocity and reaches its position at 100 m
at t = 5 s.
100 – 0 100
(iii) Halaju (kecerunan graf) = = = 20 m s–1
5–0 5
Velocity (gradient of the graph)

28

02 Strategi A+ SPM Fizik Tg 4 (B02) 5pp.indd 28 18/11/2023 9:44 AM


Sela masa Huraian
Time interval Description

(b) 5 s – 10 s (i) Kereta berada dalam keadaan pegun .


The car at rest .
(ii) Halaju (kecerunan graf) = 0 m s–1
Velocity (gradient of the graph)

(c) 10 s – 15 s (i) Kereta bergerak dengan halaju seragam dan tiba di kedudukan 150 m
pada t = 15 s.
The car moves with uniform velocity and reaches its position at 150 m at t = 15 s.
150 – 100 50
(ii) Halaju (kecerunan graf) = = = 10 m s–1
15 – 10 5
Velocity (gradient of the graph)

(d) 15 s – 25 s (i) Kereta berpatah balik ke kedudukan asalnya dengan halaju seragam .

The car turns back to its initial position with uniform velocity.
0 – 150 150
(ii) Halaju (kecerunan graf) = =– = –15 m s–1
25 – 15 10
Velocity (gradient of the graph)

(iii) Tanda negatif bagi halaju bermaksud kereta bergerak dalam arah
bertentangan .

The negative sign for velocity means the car moves in the opposite direction.

(e) 25 s – 30 s (i) Kereta terus bergerak dengan halaju seragam dan tiba di kedudukan
50 m di belakang kedudukan asal pada t = 30 s.

The car continues to move with uniform velocity and reaches its position at 50 m
behind the initial position at t = 30 s.
–50 – 0 50
(ii) Halaju (kecerunan graf) = =– = –10 m s–1
30 – 25 5
Velocity (gradient of the graph)

(iii) Tanda negatif bagi halaju bermaksud kereta bergerak dalam arah
bertentangan .

The negative sign for velocity means the car moves in the opposite direction.

29

02 Strategi A+ SPM Fizik Tg 4 (B02) 5pp.indd 29 18/11/2023 9:44 AM


Latihan 8 Mentafsir graf halaju-masa
Interpreting velocity-time graph
TP 1 Mengingat kembali pengetahuan dan kemahiran sains mengenai Daya dan Gerakan I.
TP 2 Memahami Daya dan Gerakan I serta dapat menjelaskan kefahaman tersebut.
TP 3 Mengaplikasikan pengetahuan mengenai Daya dan Gerakan I untuk menerangkan kejadian atau fenomena alam dan melaksanakan tugasan mudah.

Jadual berikut menunjukkan halaju sebuah kereta pada masa yang berbeza.
The following table shows the velocity of a car at different times.

Masa, t (s)
0 5 10 15 20 25 30 35
Time
Halaju, s (m s–1)
0 10 10 10 5 0 0 –10
Velocity

1 Lakarkan graf halaju-masa bagi data yang diberi. TP 1 TP 3


Sketch the velocity-time graph for the data given.

Halaju (m s–1)
Velocity

10

Masa (s)
0 5 10 15 20 25 30 35 Time

–5

–10

2 Berdasarkan graf halaju-masa yang dilukis di 1, lengkapkan jadual berikut untuk menerangkan
gerakan kereta tersebut. TP 2 TP 3
Based on the velocity-time graph drawn in 1, complete the following table to explain the motion of the car.

Sela masa Huraian


Time interval Description

(a) 0 s – 5 s (i) Kereta bermula daripada keadaan pegun pada t = 0 s.


The car starts from rest at t = 0 s.

(ii) Kemudian, kereta bergerak dengan halaju yang bertambah secara seragam
(pecutan seragam ) dan mencapai halaju 10 m s
–1
pada t = 5 s.
Then, the car moves with uniformly increasing velocity ( uniform acceleration) and
reaches a velocity of 10 m s –1
at t = 5 s.
10 – 0 10
(iii) Pecutan (kecerunan graf) = = = 2 m s–1
5–0 5
Acceleration (gradient of the graph)

1
(iv) Sesaran (luas di bawah graf) = × 5 × 10 = 25 m
2
Displacement (area under the graph)

30

02 Strategi A+ SPM Fizik Tg 4 (B02) 5pp.indd 30 18/11/2023 9:44 AM


Sela masa Huraian
Time interval Description

(b) 5 s – 15 s (i) Kereta bergerak dengan halaju seragam (pecutan sifar ).


The car moves with uniform velocity ( zero acceleration).
(ii) Pecutan (kecerunan graf) = 0 m s –2

Acceleration (gradient of the graph)


(iii) Sesaran (luas di bawah graf) = 10 × 10 = 100 m
Displacement (area under the graph)

(c) 15 s – 25 s (i) Kereta bergerak dengan halaju yang berkurang secara seragam
(nyahpecutan seragam ) dan berhenti pada t = 25 s.
The car moves with uniformly decreasing velocity ( uniform deceleration) and
stops at t = 25 s.
0 – 10 10
(ii) Pecutan (kecerunan graf) = =– = –1 m s–2
25 – 15 10
Acceleration (gradient of the graph)

1
(iii) Sesaran (luas di bawah graf) = × 10 × 10 = 50 m
2
Displacement (area under the graph)

(d) 25 s – 30 s Kereta berada dalam keadaan pegun kerana halaju ialah 0 m s–1 .
The car at rest because the velocity is 0 m s–1 .

(e) 30 s – 35 s (i) Kereta berpatah balik dengan halaju meningkat secara seragam
(pecutan seragam ).
The car moves in opposite direction with uniformly increasing velocity
( uniform acceleration).
–10 – 0 10
(ii) Pecutan (kecerunan graf) = =– = –2 m s–2
35 – 30 5
Acceleration (gradient of the graph)

(iii) Tanda negatif bagi pecutan tidak bermaksud kereta mengalami


nyahpecutan. Tanda negatif itu bermaksud kereta memecut dalam arah
bertentangan .
The negative sign for acceleration does not mean the car decelerates. The negative sign means
the car accelerates in the opposite direction.
1
(iv) Sesaran (luas di bawah graf) = × (–10) × 5 = –25 m
2
Displacement (area under the graph)

(v) Tanda negatif pada sesaran bermaksud kereta bergerak dalam arah
bertentangan .

The negative sign for displacement means the car moves in the opposite direction.

31

02 Strategi A+ SPM Fizik Tg 4 (B02) 5pp.indd 31 18/11/2023 9:44 AM


3 Kira jumlah jarak dan sesaran kereta itu.
Calculate the total distance and displacement of the car.
Jarak/Distance = 25 + 100 + 50 + 25 = 200 m
Sesaran/Displacement = 25 + 100 + 50 + (–25) = 150 m

Latihan 9 Bentuk graf gerakan linear berdasarkan jenis gerakan


Shape of linear motion graphs based on types of motion
TP 3 Mengaplikasikan pengetahuan mengenai Daya dan Gerakan I untuk menerangkan kejadian atau fenomena alam dan melaksanakan tugasan mudah.

1 Lakarkan graf yang relevan bagi setiap jenis gerakan yang disenaraikan. TP 3
Sketch the relevant graphs for each type of motion listed.

Jenis gerakan (i) s melawan t (ii) v melawan t (iii) a melawan t


Type of motion s against t v against t a against t
(a) Sesaran tetap s s s v v v a a a
(Halaju sifar)
Constant displacement
(Zero velocity)

0 0 0 t t t
0 0 0 t t t
0 0 0 t t t

(b) Sesaran bertambah s s s v v v a a a


secara seragam dalam
arah positif
(halaju seragam)
Displacement increases
uniformly in positive direction
t t 0t 0 t t t t t t
(uniform velocity) 0 0 0 0 0 0 0

(c) Sesaran berkurang s s s v v v a a a


secara seragam dalam
arah positif
(halaju seragam) 0 0 0 t t t
Displacement decreases
uniformly in positive direction t t t t t t
(uniform velocity) 0 0 0 0 0 0

(d) Sesaran bertambah s s s v v v a a a


secara seragam dalam
arah negatif
0 0 0 t t 0t 0 0 t t t
(halaju seragam)
Displacement increases
uniformly in negative t t t
0 0 0
direction (uniform velocity)
(e) Halaju bertambah secara s s s v v v a a a
seragam dalam arah
positif (pecutan seragam)
Velocity increases uniformly
in positive direction
(uniform acceleration) t t 0t t t t t t t
0 0 0 0 0 0 0 0

(f) Halaju berkurang secara s s s v v v a a a


seragam dalam arah
positif
(nyahpecutan seragam) 0 0 0 t t t
Velocity decreases uniformly
in positive direction t t 0t t t t
0 0 0 0 0
(uniform deceleration)

32

02 Strategi A+ SPM Fizik Tg 4 (B02) 5pp.indd 32 18/11/2023 9:44 AM


Latihan 10 Penyelesaian masalah melibatkan graf gerakan linear
Problem solving involving linear motion graphs
TP 3 Mengaplikasikan pengetahuan mengenai Daya dan Gerakan I untuk menerangkan kejadian atau fenomena alam dan melaksanakan tugasan mudah.

Selesaikan masalah yang berikut.


Solve the following problems.

1 Rajah di sebelah menunjukkan graf sesaran-masa bagi Sesaran (m)


suatu objek yang bergerak daripada keadaan rehat Displacement

menuju ke arah timur, dan kemudian ke barat. TP 3 A B


60
The diagram on the right shows the displacement-time graph of an
object that moves from rest towards the east, and then towards the 40
west.
20
O C
(a) Hitung halaju objek sepanjang Masa (s)
0 2 4 6 8 10 12 Time
Calculate the velocity of the object along
–20 D
(i) OA
v = Kecerunan graf sesaran-masa
Gradient of the displacement-time graph
60 – 0 60
v= = = 30 m s–1
2–0 2

(ii) AB
60 – 60 0
v= = = 0 m s–1
6–2 4
Objek kekal pegun. (Objek berhenti)
The object remains stationary. (The object stops)

(iii) BC
0 – 60 60
v= = – = –15 m s–1
10 – 6 4
Objek bergerak dalam arah bertentangan.
The object moves in the opposite direction.

(iv) CD
–20 – 0 20
v= = – = –10 m s–1
12 – 10 2
Objek bergerak dalam arah bertentangan melepasi kedudukan awal.
The object moves in the opposite direction, passing through the initial position.

(b) Hitung jarak dan sesaran objek.


Calculate the distance and displacement of the object.
Jarak/Distance = 60 + 60 + 20 = 140 m
Sesaran/Displacement = –20 m (ke arah barat/ towards the west)

(c) Hitung laju purata dan halaju purata objek.


Calculate the average speed and average velocity of the object.
140
Laju/Speed = = 11.667 m s–1
12
20
Halaju/Velocity = = 1.667 m s–1
12

33

02 Strategi A+ SPM Fizik Tg 4 (B02) 5pp.indd 33 18/11/2023 9:44 AM


2 Rajah berikut menunjukkan graf halaju-masa bagi suatu objek. TP 3
The following diagram shows the velocity-time graph of an object.

Halaju (m s–1)
Velocity

P
30
Q R
15
O S U Masa (s)
0 10 20 30 40 50 60 70 Time
–15

–30
T

(a) Hitung pecutan objek sepanjang


Calculate the acceleration of the object along
(i) OP
a = Kecerunan graf halaju-masa
Gradient of the velocity-time graph
30 – 0 30
a= = = 3 m s–2
10 – 0 10

(ii) PQ
15 – 30 15
a= = – = –1.5 m s–2
20 – 10 10
Objek mengalami nyahpecutan.
The object decelerates.

(iii) QR
15 – 15 0
a= = = 0 m s–2
30 – 20 10
Objek bergerak dengan halaju seragam. Maka, tiada pecutan.
The object moves with uniform velocity. Thus, there is no acceleration.

(iv) RS
0 – 15 15
a= = – = –0.75 m s–2
50 – 30 20
Objek mengalami nyahpecutan.
The object decelerates.

(v) ST
–30 – 0 30
a= = – = –3 m s–2
60 – 50 10
Objek memecut dalam arah yang bertentangan dengan arah asal.
The object accelerates in the opposite direction of the original direction.

(vi) TU
0 – (–30) 30
a= = = 3 m s–2
70 – 60 10
Objek menyahpecut dalam arah yang bertentangan dengan arah asal.
The object decelerates in the opposite direction of the original direction.

34

02 Strategi A+ SPM Fizik Tg 4 (B02) 5pp.indd 34 18/11/2023 9:44 AM


(b) Hitung sesaran objek dari masa
Calculate the displacement of the object from time
(i) t = 0 s hingga/ until 10 s, (iii) t = 20 s hingga/ until 50 s,
1 1
s= × 30 × 10 = 150 m s= × (10 + 30) × 15 = 300 m
2 2

(ii) t = 10 s hingga/ until 20 s, (iv) t = 50 s hingga/ until 70 s.


1 1
s= × (30 + 15) × 10 = 225 m × 20 × (–30) = –300 m
2 2

(c) Hitung jumlah jarak dan sesaran objek.


Calculate the total distance and displacement of the object.
Jumlah jarak/Total distance = 150 + 225 + 300 + 300 = 975 m
Sesaran/Displacement = 150 + 225 + 300 – 300 = 375 m

2.3 Gerakan Jatuh Bebas/ Free Fall Motion


Latihan 11 Pecutan graviti/Gravitational acceleration
TP 1 Mengingat kembali pengetahuan dan kemahiran sains mengenai Daya dan Gerakan I.
TP 2 Memahami Daya dan Gerakan I serta dapat menjelaskan kefahaman tersebut.

Isi tempat kosong dengan perkataan yang betul tentang pecutan graviti. TP 1 TP 2
Fill in the blanks with the correct words about gravitational acceleration.

1 Jatuh bebas ialah pergerakan bagi suatu objek yang dipengaruhi oleh daya graviti sahaja.
Free fall is a motion of an object that is affected only by gravitational force .

2 Rajah di sebelah menunjukkan seorang budak lelaki melepaskan sebiji batu dari suatu ketinggian.
The diagram on the right shows a boy dropping a stone from a certain height.

(a) Halaju batu meningkat secara seragam .


The velocity of the stone increases uniformly .

(b) Hal ini bermaksud bahawa pecutan adalah v = 20 m s−1


seragam . v = 40 m s−1
This means that the acceleration is uniform . v = 60 m s−1

(c) Jika batu dilepaskan daripada keadaan rehat


dan halajunya, v = 80 m s–1 pada t = 8 s, kira v = 80 m s−1
pecutan, a, batu itu.
If the stone is dropped from rest and its velocity,
v = 80 m s–1 at t = 8 s, calculate the acceleration, a,
of the stone.
v = 100 m s−1
v–u 80 – 0 80
a = = = = 10 m s–2
t 8 8
(d) a bersamaan dengan pecutan graviti, g, iaitu pecutan suatu objek yang disebabkan oleh
daya tarikan graviti .
a is equal to gravitational acceleration, g, which is acceleration of an object due to the gravitational pull .

3 Nilai piawai bagi pecutan graviti, g ialah 9.81 m s–2 . Nilai sebenar bergantung pada
kekuatan medan graviti .
The standard value of gravitational acceleration, g is 9.81 m s–2 . The actual value depends on the
strength of gravitational field .

35

02 Strategi A+ SPM Fizik Tg 4 (B02) 5pp.indd 35 18/11/2023 9:44 AM


Latihan 12 Aktiviti melibatkan gerakan jatuh bebas
Activities involving free fall motion
TP 3 Mengaplikasikan pengetahuan mengenai Daya dan Gerakan I untuk menerangkan kejadian atau fenomena alam dan melaksanakan tugasan mudah.
TP 4 Menganalisis pengetahuan mengenai Daya dan Gerakan I dalam konteks penyelesaian masalah mengenai kejadian atau fenomena alam.

1 Lengkapkan jadual yang diberi berdasarkan aktiviti gerakan jatuh bebas berikut. TP 3 TP 4
Complete the given table based on free fall activities.

Aktiviti A/Activity A

Situasi/ Situation I : Sebiji batu dan sehelai kertas dilepaskan serentak dari ketinggian yang sama.
A stone and a piece of paper are dropped simultaneously from the same height.
Situasi/ Situation II : Kertas direnyukkan menjadi bola. Batu dan kertas sekali lagi dilepaskan
serentak dari ketinggian yang sama.
The piece of paper is crumpled into a ball. The stone and paper are again dropped simultaneously
from the same height.

Ketinggian sama
Same height

Pemerhatian/ Observation
Situasi/ Situation I Situasi/ Situation II
Masa bagi kedua-dua (a) (b)
objek mencecah lantai Tidak sama Sama
Time for both objects to Not same Same
reach the floor

Batu (e) Kedua-dua batu dan kertas


(c) mencecah lantai yang direnyukkan mempunyai
terlebih dahulu kerana luas permukaan yang hampir
mempunyai luas permukaan sama . Oleh itu, rintangan
yang kecil . Oleh itu, udara yang bertindak ke atas
rintangan udara yang bertindak kedua-duanya adalah hampir
ke atasnya adalah kecil .
sama . Rintangan udara itu
Rintangan udara itu boleh
diabaikan. sangat kecil . Jadi, kita boleh
stone
mengabaikan rintangan udara
The reaches the floor first
itu.
because it has small surface area. The stone and crumpled paper
Therefore, the air resistance acting on same
Penerangan have almost the surface area.
Explanation it is small . The air resistance can Therefore, the air resistance acting
be ignored. same
on both is almost the . The
air resistance is very small
. Thus,
(d) Kertas mencecah lantai selepas
we can ignore the air resistance.
batu kerana ia mempunyai
luas permukaan yang besar .
Oleh itu, rintangan udara yang
lebih besar bertindak ke atasnya.
The paper reaches the floor after
the stone because it has large

surface area. Therefore, more air


resistance acts on it.

36

02 Strategi A+ SPM Fizik Tg 4 (B02) 5pp.indd 36 18/11/2023 9:44 AM


(h) Kedua-dua objek jatuh tanpa
(f) Batu jatuh tanpa dipengaruhi rintangan
dipengaruhi oleh
oleh rintangan udara, manakala udara .
kertas dipengaruhi oleh
Both objects fall unaffected by
rintangan udara. air resistance .
The stone’s fall is unaffected by
Kesimpulan air resistance, while the paper is (i) Kedua-dua objek dikatakan
Conclusion affected by air resistance. jatuh bebas .
mengalami
Both objects are said to be in
(g) Batu dikatakan mengalami free fall
jatuh bebas , manakala kertas .

tidak.
The stone is said to be in free fall ,
while the paper is not.

Aktiviti B/Activity B

Sehelai bulu pelepah dan sebiji batu


dilepaskan serentak dari ketinggian Dalam udara
In air
Dalam vakum
In vacuum
yang sama dalam udara. Aktiviti diulang
dalam vakum.
A feather and a stone are dropped simultaneously Bulu pelepah
Feather
from the same height in the air. The activity is
repeated in a vacuum.

Batu
Stone

Pemerhatian/ Observation
Dalam udara/ In air Dalam vakum/ In vacuum

Batu (c) Batu dan bulu pelepah akan jatuh pada kelajuan
(a) akan jatuh lebih cepat
bulu pelepah . yang sama .
daripada
stone
The stone and the feather will fall at the same speed.
The will fall faster than the
feather . (d) Hal ini adalah kerana tiada rintangan udara yang
bertindak ke atas kedua-dua objek semasa berada
(b) Luas permukaan bulu pelepah adalah
besar . Hal ini menyebabkan dalam vakum.
rintangan udara yang bertindak ke This is because there is no air resistance on both objects while
in a vacuum.
atasnya juga menjadi besar.
The surface area of the feather is large . (e) Kedua-dua objek yang jatuh hanya dipengaruhi
This causes the air resistance acting on it daya graviti . Kedua-dua objek dikatakan
oleh
to also become large. mengalami jatuh bebas walaupun mempunyai
jisim berbeza .
Both falling objects are only affected by the
gravitational force . Both objects are said to be in
free fall even though they have different masses .

37

02 Strategi A+ SPM Fizik Tg 4 (B02) 5pp.indd 37 18/11/2023 9:44 AM


Latihan 13 Persamaan gerakan linear bagi gerakan jatuh bebas
Equations of linear motion for free fall motion
TP 2 Memahami Daya dan Gerakan I serta dapat menjelaskan kefahaman tersebut.
TP 3 Mengaplikasikan pengetahuan mengenai Daya dan Gerakan I untuk menerangkan kejadian atau fenomena alam dan melaksanakan tugasan mudah.

1 Tulis persamaan gerakan linear bagi gerakan jatuh bebas. TP 2 TP 3


Write the equations of linear motion for free fall motion. i-THINK Peta Buih

(a) (b)

1
v = u + gt s = ut + 2 gt2
Persamaan
gerakan linear
bagi gerakan jatuh
bebas
Equations of
linear motion for
free fall motion
(d) (c)

1
s = 2 (u + v)t v2 = u2 + 2gs

Latihan 14 Penyelesaian masalah melibatkan gerakan jatuh bebas


Problem solving involving free fall motion
TP 3 Mengaplikasikan pengetahuan mengenai Daya dan Gerakan I untuk menerangkan kejadian atau fenomena alam dan melaksanakan tugasan mudah.
TP 4 Menganalisis pengetahuan mengenai Daya dan Gerakan I dalam konteks penyelesaian masalah mengenai kejadian atau fenomena alam.
TP 5 Menilai pengetahuan mengenai Daya dan Gerakan I dalam konteks penyelesaian masalah dan membuat keputusan untuk melaksanakan satu tugasan.

Selesaikan masalah yang berikut.


Solve the following problems.

1 Sebuah kotak jatuh dari ketinggian 80 m dan menghentam tanah. TP 3


A box falls from a height of 80 m and hits the ground.
[g = 9.81 m s–2]
(a) Kira masa yang diambil oleh kotak untuk menghentam tanah.
Calculate the time taken by the box to hit the ground.
(b) Tentukan halaju akhir kotak apabila ia menghentam tanah.
Determine the final velocity of the box when it hits the ground.

(a) u = 0 m s–1, s = 80 m, g = 9.81 m s–2, t=?


1
s = ut + gt2
2
1
–80 = (0)t + (–9.81)t2
2
t2 = 16.31
t = 4.039 s

(b) u = 0 m s–1, g = 9.81 m s–2, t = 4.039 s, v = ?


v = u + gt
v = 0 + (–9.81)(4.039)
v = –39.623 m s–1

Halaju negatif kerana kotak bergerak ke bawah.


Negative velocity because the box moves downwards.

38

02 Strategi A+ SPM Fizik Tg 4 (B02) 5pp.indd 38 18/11/2023 9:44 AM


2 Sebiji batu dilontar menegak ke atas dari tanah dengan halaju 40 m s–1 dan menghentam tanah
selepas beberapa saat. TP 4
A stone is thrown vertically upwards from the ground with a velocity of 40 m s–1 and hits the ground after a few seconds.
(a) Kira masa yang diambil oleh batu untuk menghentam tanah.
Calculate the time taken by the stone to hit the ground.
(b) Tentukan halaju akhir batu apabila ia menghentam tanah.
Determine the final velocity of the stone when it hits the ground.
(a) Gerakan ke atas
Upward motion
• u = 40 m s–1
• Pada ketinggian maksimum, v = 0 m s–1
At maximum height, v = 0 m s–1
• g = 9.81 m s–2
v = u + gt
0 = 40 + (–9.81)t
9.81t = 40
t = 4.077 s
Maka, masa yang diambil oleh batu untuk menghentam tanah = 4.077 × 2 = 8.154 s.
Thus, the time taken by the stone to hit the ground = 4.077 × 2 = 8.154 s.
(b) Gerakan ke bawah
Downward motion
• Pada ketinggian maksimum, u = 0 m s–1
At maximum height, u = 0 m s–1
• g = 9.81 m s–2
• t = 4.077 s
Halaju akhir/Final velocity,
v = u + gt
= 0 + (–9.81)(4.077)
= –39.995 m s–1

Halaju negatif kerana batu bergerak ke bawah.


Negative velocity because the stone moves downwards.

3 Amber melontar bola ke atas dan selepas 2 saat, ia mencapai


ketinggian maksimum. Bola itu kemudiannya mengambil masa
3 saat untuk jatuh ke lantai seperti yang ditunjukkan dalam rajah
di sebelah. TP 5
Amber throws a ball upwards and after 2 seconds, it reaches the maximum
height. The ball then takes 3 seconds to fall to the ground as shown in the
diagram on the right.
Kira halaju awal, ketinggian maksimum dari kedudukan awal,
halaju sebelum menghentam tanah dan ketinggian maksimum
bola dari tanah. KBAT Menganalisis
[Pecutan graviti, g = 9.81 m s–2]
Calculate the initial velocity, maximum height from initial position, velocity
before hitting the ground and maximum height of the ball from the ground.
[Gravitational acceleration, g = 9.81 m s–2]
Gerakan ke atas:
Upward motion:
• Pada ketinggian maksimum, v = 0 m s–1
At maximum height, v = 0 m s–1
• g = 9.81 m s–2 t=2s
• t=2s
Halaju awal/Initial velocity:
t=3s
v–u
a=
t
0–u
–9.81 =
2
u = 19.62 m s–1

39

02 Strategi A+ SPM Fizik Tg 4 (B02) 5pp.indd 39 18/11/2023 9:44 AM


Ketinggian maksimum dari kedudukan awal:
Maximum height from initial position:
1 2
s = ut + gt
2
1
= 19.62(2) + (–9.81)(2)2
2
= 39.24 – 19.62

= 19.62 m

atau/ or atau/ or
1
s = (u + v)t v2 = u2 + 2gs s
2
02 = 19.622 + 2(–9.81)(s)
1 19.62s = 384.944
= (19.62 + 0)(2)
2 s = 19.62 m
= 19.62 m

Gerakan ke bawah:
Downward motion:
• Pada ketinggian maksimum, u = 0 m s–1
At maximum height, u = 0 m s–1
• g = 9.81 m s–2
• t = 3 s
Halaju akhir/Final velocity:
v = u + gt
= 0 + (–9.81)(3)
= –29.43 m s–1
Ketinggian maksimum dari tanah:
Maximum height from the ground:
1 2
s = ut + gt
2
1
= (0)(3) + (–9.81)(3)2
2
= –44.145 m

atau/ or atau/ or
1
s = (u + v)t v2 = u2 + 2gs
2
(–29.43)2 = 02 + 2(–9.81)(s)
1 –19.62s = 866.1249 s
= (0 – 29.43)(3)
2 s = –44.145 m
= –44.145 m
Maka, ketinggian maksimum bola dari tanah ialah 44.145 m.
Thus, the maximum height of the ball from the ground is 44.145 m.

2.4 Inersia/ Inertia


Latihan 15 Inersia dan aplikasinya dalam kehidupan seharian/ Inertia and its applications in daily life
TP 1 Mengingat kembali pengetahuan dan kemahiran sains mengenai Daya dan Gerakan I.
TP 2 Memahami Daya dan Gerakan I serta dapat menjelaskan kefahaman tersebut.
TP 3 Mengaplikasikan pengetahuan mengenai Daya dan Gerakan I untuk menerangkan kejadian atau fenomena alam dan melaksanakan tugasan mudah.

1 Apakah inersia? TP 1
What is inertia?
Inersia ialah kecenderungan objek untuk kekal pegun atau jika ia sedang bergerak, ia kekal bergerak.
Inertia is the tendency of an object to remain stationary, or if it is moving, to continue moving.

40

02 Strategi A+ SPM Fizik Tg 4 (B02) 5pp.indd 40 18/11/2023 9:44 AM


2 Nyatakan hukum fizik yang menjelaskan konsep inersia. TP 2
State the law of physics that explains the concept of inertia.
Hukum Gerakan Newton Pertama/Newton’s First Law of Motion

3 Huraikan hukum fizik yang dinyatakan di 2. TP 2


Describe the law of physics stated in 2.
Hukum Gerakan Newton Pertama menyatakan bahawa sesuatu objek yang pegun akan kekal dalam keadaan
pegun dan sesuatu objek bergerak akan kekal bergerak dengan halaju seragam dalam satu garis lurus jika
tiada daya luar yang bertindak ke atasnya.
Newton’s First Law of Motion states that an object at rest remains at rest and an object in motion continues in motion with a uniform
velocity in a straight line if no external forces acted on it.

4 Kaji situasi berikut tentang aplikasi inersia dalam kehidupan seharian dan isi tempat kosong dengan
perkataan yang betul. TP 3
Study the following situations about the applications of inertia in daily life and fill in the blanks with the correct words.

Situasi/ Situation Penjelasan/ Explanation


(a)
(i) Penumpang di dalam sebuah bas terhumban ke belakang
apabila bas yang pegun bergerak secara mendadak.
The passengers in a bus are thrown backwards when the bus which
is stationary moves suddenly.

(ii) Inersia penumpang cenderung untuk mengekalkan


keadaan rehatnya/ pegunnya .

Inertia of the passengers tends to maintain the state of rest .

(b)
(i) Penumpang di dalam sebuah bas terhumban ke hadapan
apabila bas yang sedang bergerak membrek secara mendadak.
The passengers in a bus are thrown forwards when the bus which
is moving stops abruptly.

(ii) Inersia penumpang cenderung untuk mengekalkan


gerakannya .

Inertia of the passengers tends to maintain its motion .

(c) (i) Apabila sebuah buku di bahagian bawah satu longgokan


buku ditarik keluar dengan laju , buku di atasnya akan
jatuh secara menegak ke bawah .

When a book placed at the lower position of a stack of books is pulled


IXXX
out quickly , the books above it will drop vertically
IXXX downwards .
IXXX
(ii) Inersia buku di atas cenderung untuk mengekalkan
Tarik
Pull
keadaan rehatnya/ pegunnya .

Inertia of the books above it tends to maintain at rest .

41

02 Strategi A+ SPM Fizik Tg 4 (B02) 5pp.indd 41 18/11/2023 9:44 AM


Latihan 16 Hubungan antara jisim dengan inersia
Relationship between mass and inertia
TP 3 Mengaplikasikan pengetahuan mengenai Daya dan Gerakan I untuk menerangkan kejadian atau fenomena alam dan melaksanakan tugasan mudah.

1 Jalankan aktiviti berikut untuk menyiasat hubungan antara jisim dengan inersia. Kemudian,
lengkapkan pemerhatian dan kesimpulan berdasarkan aktiviti berikut. TP 3
Carry out the following activity to investigate the relationship between mass and inertia. Then, complete your observations
and conclusions based on the following activity.

Dua buah baldi, A dan B, digantung dan diayun pada masa yang sama oleh seorang murid.
Panjang tali dan jisim awal kedua-dua baldi adalah sama. Baldi A dibiarkan kosong, manakala baldi
B diisi dengan pasir. Pemerhatian dicatatkan.
Pails A and B are hung and swung at the same time by a student.
The length of rope and the initial mass of the two pails are the same. Pail A is left empty, whereas pail B is filled with sand.
The observations are recorded.

Kosong
Empty
Pasir
Sand

A B

Diayun pada masa yang sama


Swung at the same time

Pemerhatian/ Observations:
(a) Baldi B mempunyai jisim yang lebih besar berbanding dengan baldi A.
Pail B has greater mass compared to pail A.

(b) Pada awalnya, baldi B lebih sukar ditolak berbanding dengan baldi A.
Initially, pail B is more difficult to push compared to pail A.

(c) Baldi B berayun lebih lama berbanding dengan baldi A.


Pail B swings longer compared to pail A.

Kesimpulan/ Conclusions:
(a) Baldi B lebih (i) sukar untuk berayun dan lebih (ii) sukar untuk diberhentikan
berbanding dengan baldi A. Hal ini adalah kerana ia mempunyai (iii) lebih
kecenderungan untuk mengekalkan keadaan asalnya. Dengan kata lain, ia mempunyai lebih
(iv) inersia .
Pail B is more (i) difficult to swing and more (ii) difficult to stop compared to pail A. This is because
it has (iii) greater tendency to maintain its initial state. In other words, it has more (iv) inertia .

(b) Inersia suatu objek meningkat apabila jisimnya meningkat.


The inertia of an object increases when its mass increases.

42

02 Strategi A+ SPM Fizik Tg 4 (B02) 5pp.indd 42 18/11/2023 9:44 AM


Latihan 17 Kesan positif dan negatif inersia dalam kehidupan seharian
Positive and negative effects of inertia in daily life
TP 3 Mengaplikasikan pengetahuan mengenai Daya dan Gerakan I untuk menerangkan kejadian atau fenomena alam dan melaksanakan tugasan mudah.

1 Lengkapkan peta pelbagai alir berikut untuk membincangkan kesan positif inersia dalam kehidupan
harian. TP 3
Complete the following multi-flow map to discuss the positive effects of inertia in daily life.
i-THINK Peta Pelbagai Alir

(a) Untuk mengeringkan air daripada (b) Inersia titisan air pada payung
payung, putarkan payung dengan cenderung untuk mengekalkan
laju gerakan putarannya dan terus
. Kemudian, hentikan
gerakan secara mendadak . terserak daripada payung.
To shed water from umbrella, rotate the Inertia of water droplets tends to
maintain its spinning motion and are
umbrella at high speed . Then, stop the
suddenly thrown off from the umbrella.
motion .

(c) Sos cili mudah keluar daripada (d) Inersia sos cili di dalam botol
digoncangkan cenderung mengekalkan
botol jika botol
gerakannya ke bawah
dengan kuat dan gerakan
menyebabkan ia terus mengalir
diberhentikan secara mendadak.
keluar daripada botol.
Chilli sauce flows out more easily from the
shaken vigorously
Inertia of chilli sauce in the bottle tends to
bottle if the bottle is maintain its downward motion ,
and the motion is stopped suddenly.
causing it to flow out from the bottle.

Kesan
(e) Seorang budak lelaki berlari dalam (f) Inersia lembu yang besar
positif
corak zig-zag apabila dikejar inersia disebabkan jisim yang besar.
seekor lembu. Positive Hal ini menyebabkan lembu itu
zig-zag effects sukar mengubah arah gerakan
A boy runs in a pattern of semasa mengejar budak lelaki itu.
when pursued by a cow. inertia
Maka, budak lelaki itu berjaya
melarikan diri.
Inertia of the cow is large because its
mass is large . This makes it difficult
to change the direction of motion

while pursuing the boy. Thus, the boy is


able to escape.

(g) Kepala penukul yang longgar dapat


(h) Inersia kepala penukul cenderung
diketatkan dengan menghentak
pemegangnya dengan kuat pada
mengekalkan keadaan gerakannya
ke bawah walaupun penukul
lantai. berhenti apabila menghentam
The loose head of a hammer can be lantai.
tightened by hitting the handle hard Inertia of head of the hammer tends
on the floor. to maintain its downward motion even
though the hammer stops when
hitting the floor.

43

02 Strategi A+ SPM Fizik Tg 4 (B02) 5pp.indd 43 18/11/2023 9:44 AM


2 Lengkapkan peta pelbagai alir berikut untuk membincangkan kesan negatif inersia dalam kehidupan
harian. TP 3
Complete the following multi-flow map to discuss the negative effects of inertia in daily life.
i-THINK Peta Pelbagai Alir

(b) Pemandu kereta terhumban


(a) Pemandu tidak memakai tali ke hadapan
pinggang keselamatan apabila kereta
.
seat belt .
berhenti secara tiba-tiba.
The driver does not wear forwards
The driver is thrown
when the car stops suddenly.

(c) Tangki lori minyak yang besar (d) Tangki lori minyak dengan
tidak dibahagikan kepada Kesan inersia yang besar akan terus
bahagian-bahagian yang kecil . negatif terhumban ke hadapan apabila
The big tank of the tanker is not divided
inersia lori berhenti secara tiba-tiba.
Negative
into small compartments . The tank of the tanker with great
effects
inertia will crash forwards
of inertia
when the lorry stops suddenly.

(e) Muatan berat seperti kayu (f) Kayu balak akan terhumban
balak tidak diikat dengan ke arah pemandu apabila
kuat apabila diangkut oleh lori berhenti secara tiba-tiba.
lori. Pemandu akan tercedera .
Heavy loads such as logs are not tied The logs will crash towards the
tightly when transported by driver when the lorry stops
lorry. suddenly. The driver will be injured .

2.5 Momentum/ Momentum


Latihan 18 Definisi momentum/ Definition of momentum
TP 1 Mengingat kembali pengetahuan dan kemahiran sains mengenai Daya dan Gerakan I.
TP 2 Memahami Daya dan Gerakan I serta dapat menjelaskan kefahaman tersebut.

Isi tempat kosong dengan perkataan yang betul.


Fill in the blanks with the correct words.

1 Momentum ditakrifkan sebagai hasil darab jisim dan halaju .


Momentum can be defined as the product of mass and velocity .
2 Momentum dapat dihitung dengan rumus
Momentum can be calculated by formula
p = mv

dengan keadaan/where p = Momentum/ Momentum

m = Jisim/ Mass

v = Halaju/ Velocity

3 Unit S.I. bagi momentum ialah kg m s–1 .


The S.I. unit for momentum is kg m s–1 .

4 Momentum ialah kuantiti vektor .


Momentum is a vector quantity.

44

02 Strategi A+ SPM Fizik Tg 4 (B02) 5pp.indd 44 18/11/2023 9:44 AM


Latihan 19 Penyelesaian masalah melibatkan momentum
Solving problems involving momentum
TP 3 Mengaplikasikan pengetahuan mengenai Daya dan Gerakan I untuk menerangkan kejadian atau fenomena alam dan melaksanakan tugasan mudah.

Selesaikan masalah yang berikut.


Solve the following problems.

1 Sarah yang berjisim 55 kg berlari dengan 2 Sebuah lori berjisim 1 050 kg bergerak ke arah
halaju 2 m s–1. Kira momentum Sarah. TP 3 barat dengan halaju 18 m s–1.
Sarah who has a mass of 55 kg runs with a velocity Kira momentum lori itu. TP 3
of 2 m s–1. Calculate the momentum of Sarah. A lorry with a mass of 1 050 kg moves to the west with
a velocity of 18 m s–1. Calculate the momentum of the
2 m s−1
lorry.


18 m s–1

Momentum, p = m × v
= 55 × 2
= 110 kg m s–1 Momentum, p = m × v
= 1 050 × (–18)
= –18 900 kg m s–1

3 Kira halaju sebiji bola berjisim 200 g yang 4 Kira jisim sebuah objek yang mempunyai
mempunyai momentum 1.5 kg m s–1. TP 3 halaju 0.3 m s–1 dan momentum 12 kg m s–1.
Calculate the velocity of a ball with a mass of 200 g Calculate the mass of an object that has a velocity
that has a momentum of 1.5 kg m s–1. of 0.3 m s–1 and a momentum of 12 kg m s–1. TP 3
200 12 = m × 0.3
1.5 = ×v 12
1 000
1.5 m = = 40 kg
v = = 7.5 m s–1 0.3
0.2

Latihan 20 Prinsip Keabadian Momentum


Principle of Conservation of Momentum
TP 1 Mengingat kembali pengetahuan dan kemahiran sains mengenai Daya dan Gerakan I.
TP 2 Memahami Daya dan Gerakan I serta dapat menjelaskan kefahaman tersebut.
TP 3 Mengaplikasikan pengetahuan mengenai Daya dan Gerakan I untuk menerangkan kejadian atau fenomena alam dan melaksanakan tugasan mudah.

1 Lengkapkan Prinsip Keabadian Momentum berikut. TP 1 TP 2


Complete the following Principle of Conservation of Momentum.

(a) Prinsip Keabadian Momentum menyatakan bahawa jumlah momentum sebelum


perlanggaran adalah sama dengan jumlah momentum selepas perlanggaran.
Principle of Conservation of Momentum states that the total momentum before collision is the
same as the total momentum after the collision.
(b) Merujuk Prinsip Keabadian Momentum, nyatakan terbitan rumus bagi suatu sistem yang
melibatkan dua objek sebelum dan selepas perlanggaran.
Referring to the Principle of Conservation of Momentum, state the derived formula for a system of two objects
before and after collision.
m1u1 + m2u2 = m1v1 + m2v2
dengan keadaan/ where
m1 = Jisim objek 1/ Mass of object 1
m2 = Jisim objek 2/ Mass of object 2
u1 = Halaju awal objek 1/ Initial velocity of object 1
u2 = Halaju awal objek 2/ Initial velocity of object 2
v1 = Halaju akhir objek 1/ Final velocity of object 1
v2 = Halaju akhir objek 2/ Final velocity of object 2

45

02 Strategi A+ SPM Fizik Tg 4 (B02) 5pp.indd 45 18/11/2023 9:44 AM


2 Lengkapkan jadual berikut tentang perlanggaran kenyal, perlanggaran tak kenyal dan
letupan. TP 3
Complete the following table about elastic collision, inelastic collision and explosion.

(a) Perlanggaran kenyal (b) Perlanggaran tak kenyal (c) Letupan


Elastic collision Inelastic collision Explosion

u=0 u=0 u=0


m1 u 1 m2 u 2 m 1 u 1 m2u2 m1um u m2u2
11 1
m12u12 mmuu
2 12 1
m2 u 2
1 1 2 1 12 12 2 21 1 22 1 2 1 2

m2v2 m2v2m1v1 m2m


v21v1 m1vv1 mv1v1 m1vv1m2v2 m1m
v12v2 m2v2
2 1 2 1 2 1 12 1 2 11 2 2 1 2 1 2

(i) Objek bergerak (i) Objek bergerak (i) Dua objek yang asalnya
berasingan dengan bersama-sama (halaju dalam keadaan rehat
halaju berbeza yang sama ) selepas berpisah dan bergerak
selepas perlanggaran. perlanggaran. dalam arah bertentangan
Objects move separately Objects move together selepas letupan.
with different ( same velocity) after Two objects initially at rest

velocities after collision. collision. separate and move in opposite


directions after the explosion.

(ii) Jumlah momentum (ii) Jumlah momentum (ii) Jumlah momentum


diabadikan . diabadikan . diabadikan .
Total momentum is Total momentum is Total momentum is
conserved . conserved . conserved .

(iii) Tenaga kinetik (iii) Tenaga kinetik (iii) Tenaga kinetik


diabadikan . tidak diabadikan . tidak diabadikan .
Kinetic energy is Kinetic energy is Kinetic energy is
conserved . not conserved . not conserved .

(iv) Jumlah tenaga diabadikan . (iv) Jumlah tenaga diabadikan . (iv) Jumlah tenaga diabadikan .
Total energy is conserved . Total energy is conserved . Total energy is conserved .

(v) Rumus/Formula: (v) Rumus/Formula: (v) Rumus/Formula:


m1u1 + m2u2 = m1v1 + m2v2 m1u1 + m2u2 = (m1 + m2)v 0 = m1v1 + m2v2

Latihan 21 Penyelesaian masalah melibatkan momentum linear


Solving problems involving linear momentum
TP 3 Mengaplikasikan pengetahuan mengenai Daya dan Gerakan I untuk menerangkan kejadian atau fenomena alam dan melaksanakan tugasan mudah.

Selesaikan masalah yang berikut.


Solve the following problems.

1 Seorang murid melepaskan sebiji bola berjisim 1.5 kg dari ketinggian 5 m. Berapakah momentum
bola itu sejurus sebelum menghentam lantai? TP 3
A student releases a ball with a mass of 1.5 kg from a height of 5 m. What is the momentum of the ball just before it hits
the floor?
u = 0, a = g = 9.81 m s–2, s = 5 m, v = ?
v2 = u2 + 2as
v2 = (0)2 + 2(–9.81)(–5)
v = 9.905 m s–1
p = mv = (1.5)(9.905) = 14.858 kg m s–1

46

02 Strategi A+ SPM Fizik Tg 4 (B02) 5pp.indd 46 18/11/2023 9:44 AM


2 Sebuah troli berjisim 1 kg bergerak dengan halaju 0.90 m s-1 dan berlanggar dengan sebuah lagi troli
pegun berjisim 2 kg. Selepas perlanggaran, troli-troli itu bergerak bersama-sama. Berapakah halaju
troli-troli selepas perlanggaran? TP 3
A trolley with a mass of 1 kg moves at a velocity of 0.90 m s–1 and collides with a stationary trolley with a mass of 2 kg.
After the collision, the trolleys move together. What is the speed of the trolleys after the collision?
m1u1 + m2u2 = (m1 + m2)v
1(0.9) + 2(0) = (1 + 2)v
0.9
v =
3
= 0.3 m s–1

3 Satu objek berjisim 3 kg, R bergerak pada halaju 5 m s–1 dan melanggar objek berjisim 2 kg, S yang
bergerak pada halaju 1 m s–1 dari arah bertentangan. Selepas perlanggaran, R bergerak pada halaju
3 m s–1 dalam arah awal. Kira halaju S. TP 3
A 3 kg object, R moves at a velocity of 5 m s–1 and collides with a 2 kg object, S that moves at a velocity of 1 m s–1
from the opposite direction. After the collision, R moves at a velocity of 3 m s–1 in the initial direction. Calculate the
velocity of S.
mRuR + mSuS = mRvR + mSvS
3(5) + 2(–1) = 3(3) + 2(vS)
2vS = 15 – 2 – 9
2vS = 4
vS = 2 m s–1 (Sama arah dengan R/Same direction as R)

4 Sepucuk pistol berjisim 1.2 kg melepaskan sebutir peluru


berjisim 1.6 g. Peluru itu bergerak dengan halaju 300 m s–1.
A pistol with a mass of 1.2 kg fires a bullet with a mass of 1.6 g. The
bullet moves with a velocity of 300 m s–1. TP 3
Cari halaju sentakan pistol.
Find the velocity of the pistol’s recoil.
0 = m1v1 + m2v2
1.6
0 = 1.2(v1) + � �(300)
1 000
1.2v1 = –0.48
v1 = 0.4 m s–1 (Arah bertentangan dengan peluru/Opposite direction to the bullet)

Latihan 22 Aplikasi Prinsip Keabadian Momentum


Applications of the Principle of Conservation of Momentum
TP 3 Mengaplikasikan pengetahuan mengenai Daya dan Gerakan I untuk menerangkan kejadian atau fenomena alam dan melaksanakan tugasan mudah.

Isi tempat kosong dengan jawapan yang betul tentang aplikasi Prinsip Keabadian Momentum. TP 3
Fill in the blanks with the correct answers about the applications of the Principle of Conservation of Momentum.

1 Roket
Rocket
Roket menghasilkan momentum
Rocket gains momentum
Gas ekzos
Exhaust gases

47

02 Strategi A+ SPM Fizik Tg 4 (B02) 5pp.indd 47 18/11/2023 9:44 AM


(a) Campuran bahan api hidrogen dan oksigen terbakar dalam kebuk pembakaran .
The mixture of hydrogen and oxygen fuels burn in the combustion chamber .

(b) Gas panas keluar melalui ekzos pada halaju yang sangat tinggi .
Hot gases are expelled through the exhaust at a very high velocity.

(c) Gas panas dengan halaju tinggi menghasilkan momentum yang tinggi ke belakang .
The high velocity hot gases produce a high backwards momentum .

(d) Menurut Prinsip Keabadian Momentum, momentum yang sama dan berlawanan
dihasilkan dan bertindak pada roket, menolak roket ke hadapan.
By the Principle of Conservation of Momentum, an equal and opposite momentums are produced
and act on the rocket, pushing the rocket forwards.

2 Enjin jet
Jet engine

Pengambilan Mampatan Pembakaran Ekzos


Intake Compression Combustion Exhaust

Pengambilan udara Kebuk pembakaran Turbin


Air intake Combustion chamber Turbine

(a) Udara diambil melalui bahagian hadapan dan dimampatkan oleh kompresor.
Air is taken in through the front and compressed by the compressor.

(b) Bahan api disuntik dan dibakar dengan udara termampat di kebuk pembakaran .
Fuel is injected and burnt with the compressed air in the combustion chamber .

(c) Gas panas dipaksa melalui enjin untuk memutarkan bilah turbin , yang memutarkan
kompresor.
The hot gas is forced through the engine to turn the turbine blades , which turns the compressor.

(d) Gas panas berhalaju tinggi memancut keluar daripada ekzos dengan momentum
yang tinggi .
High velocity hot gases are ejected from the exhaust with high momentum.

(e) Hal ini menghasilkan momentum yang sama dan bertentangan untuk menolak
pesawat jet ke hadapan .
This produces an equal and opposite momentums to push the jet plane forwards .

48

02 Strategi A+ SPM Fizik Tg 4 (B02) 5pp.indd 48 18/11/2023 9:44 AM


2.6 Daya/ Force
Latihan 23 Definisi daya
Definition of force
TP 1 Mengingat kembali pengetahuan dan kemahiran sains mengenai Daya dan Gerakan I.
TP 2 Memahami Daya dan Gerakan I serta dapat menjelaskan kefahaman tersebut.

1 Lengkapkan pernyataan dan ungkapan/persamaan berikut. TP 1 TP 2


Complete the following statements and expressions/equations.

(a) Hukum Gerakan Newton Kedua menyatakan bahawa daya yang bertindak ke atas suatu
objek adalah berkadar langsung dengan kadar perubahan momentum objek tersebut
dalam arah yang sama dengan arah perubahan momentum.
Newton’s Second Law of Motion states that the force acting on an object is directly propotional to the
rate of change of momentum of the object in the same direction as the direction of change of momentum.

(b) Dalam bentuk persamaan/ In the equation form,


Daya Kadar perubahan momentum

Force Rate of change of momentum

Momentum akhir – Momentum awal


Final momentum – Initial momentum
Daya ∝
Force Masa/ Time

mv – mu
F ∝
t
v–u
F ∝ m� �
t

F ∝ ma
F = kma
(k ialah pemalar ./ k is a constant .)
(c) Daya 1 N ditakrifkan sebagai daya yang menyebabkan pecutan 1 m s–2 ke atas objek
berjisim 1 kg .
A force of 1 N is defined as the force that causes an acceleration of 1 m s–2 on an object of mass 1 kg .
(d) Maka/ Thus, F = kma
(1) = k(1)(1)
k = 1
F = ma

dengan keadaan/where
F = Daya/Force
m= Jisim/ Mass

a = Pecutan/ Acceleration

(e) Unit S.I. bagi daya ialah Newton (N) atau kg m s–2 .
The S.I. unit of force is Newton (N) or kg m s–2 .
(f) Daya paduan atau daya bersih ialah hasil tambah semua daya yang bertindak ke atas
suatu objek.
Resultant force or net force is the sum of all forces acting on an object.

49

02 Strategi A+ SPM Fizik Tg 4 (B02) 5pp.indd 49 18/11/2023 9:44 AM


Latihan 24 Hubungan antara daya, jisim dan pecutan
Relationship between force, mass and acceleration
TP 2 Memahami Daya dan Gerakan I serta dapat menjelaskan kefahaman tersebut.
TP 3 Mengaplikasikan pengetahuan mengenai Daya dan Gerakan I untuk menerangkan kejadian atau fenomena alam dan melaksanakan tugasan mudah.

Lengkapkan ayat, gambar rajah, graf dan ungkapan berikut. TP 2 TP 3


Complete the following sentences, diagrams, graphs and expressions.

1 Hubungan antara pecutan, a dengan daya, F


Relationship between acceleration, a and force, F

(a) Jisim
Pecutan
Jisim bertambah
Mass Mass
Pecutan Daya lebih Acceleration
Acceleration besar increases
Daya More force
Force Halaju bertambah
Halaju bertambah
Velocity increases
dengan lebih
cepat
Velocity increases
faster

Rajah 1.1/ Diagram 1.1 Rajah 1.2/ Diagram 1.2

(b) Merujuk Rajah 1.1 dan Rajah 1.2, pecutan berkadar langsung dengan daya bersih, iaitu,
Referring to Diagrams 1.1 and 1.2, acceleration is directly proportional to the net force, that is,

a∝ F

(c) Plot graf bagi a melawan F.


Plot the graph of a against F.
a (m s–2)

0 F (N)

2 Hubungan antara pecutan, a dengan jisim, m


Relationship between acceleration, a and mass, m

(a) Jisim Jisim lebih besar


More mass
Mass Pecutan
Acceleration
Daya Daya
Force Force
Halaju bertambah
Velocity increases

Rajah 2.1/ Diagram 2.1

Jisim lebih besar


More mass
Pecutan
(i) Pecutan berkurang .
Acceleration
Acceleration
Daya decreases .
Force
Halaju bertambah perlahan
Velocity increases (ii) Halaju bertambah dengan lebih .
Velocity increases slower .

Rajah 2.2/ Diagram 2.2

50

02 Strategi A+ SPM Fizik Tg 4 (B02) 5pp.indd 50 18/11/2023 9:44 AM


(b) Merujuk Rajah 2.1 dan Rajah 2.2, pecutan berkadar songsang dengan jisim, iaitu,
Referring to Diagrams 2.1 and 2.2, acceleration is inversely proportional to the mass, that is,

1
a∝
m

1
(c) Plot graf bagi a melawan m dan graf a melawan .
m
1
Plot the graph of a against m and the graph of a against .
m
a (m s–2) a (m s–2)

1
0 m (kg) 0 (kg–1)
m

Latihan 25 Menyelesaikan masalah melibatkan daya


Solving problems involving force
TP 3 Mengaplikasikan pengetahuan mengenai Daya dan Gerakan I untuk menerangkan kejadian atau fenomena alam dan melaksanakan tugasan mudah.
TP 4 Menganalisis pengetahuan mengenai Daya dan Gerakan I dalam konteks penyelesaian masalah mengenai kejadian atau fenomena alam.

1 Kira daya paduan bagi setiap situasi berikut. TP 3


Calculate the resultant force for each of the following situations.

Situasi Daya paduan


Situation Resultant force
(a)
30 N 50 N F = 30 + (–50)
= –20 N

(b)
100 N 100 N F = 100 + (–100)
= 0 N

(c) 8N
10 N F = 10 + 5 + (–8)
5N
= 7 N

3N
(d)

F = 3 + (–5)
= –2 N

5N

20 N
(e)

F = 20 + (–10) + (–30)
= –20 N
10 N
30 N

51

02 Strategi A+ SPM Fizik Tg 4 (B02) 5pp.indd 51 18/11/2023 9:44 AM


2 Cari pecutan objek dalam setiap situasi berikut. TP 3
Find the acceleration of the object in each of the following situations.

Situasi Pecutan
Situation Acceleration
(a)

F = ma
45 = (9)(a)
45 N
a = 5 m s–2
m = 9 kg

(b)
F = ma
400
65 N 65 – 25 = � �(a)
25 N 1 000
40
a =
m = 400 g 0.4
= 100 m s–2

3 Cari jisim objek dalam setiap situasi berikut. TP 3


Find the mass of the object in each of the following situations.

Situasi Jisim
Situation Mass
(a)
a = 3 m s–2
F = ma
150 + 150 = m(3)
150 N 300 = 3m
m = 100 kg
150 N

(b)
F = ma
15 N 110 N 110 – 15 = m(5)
95 = 5m
m = 19 kg

a = 5 m s–2

(c)
a = 15 m s–2 F = ma
–200 + 35 = m(–15)
200 N 35 N –165 = –15m
m = 11 kg

52

02 Strategi A+ SPM Fizik Tg 4 (B02) 5pp.indd 52 18/11/2023 9:44 AM


4 Cari nilai daya, F dalam setiap situasi berikut. TP 3
Find the value of force, F in each of the following situations.

Situasi Daya, F
Situation Force, F
(a)

a = 4 m s−2 F = ma
F – 5 = 5(4)
5N F F = 20 + 5
= 25 N
m = 5 kg

(b)
F = ma
25 – F = 5(4)
a = 4 m s−2
–F = 20 – 25
F 25 N F =5N

(Ke kiri/ To the left)


m = 5 kg

(c)
F = ma
a = 8 m s−2 150
F – 10 – 2 = � �(–8)
1 000
2N F
F – 12 = 1.2
10 N F = 13.2 N
m = 150 g

(d)
F = ma
a = 8 m s−2 30
F – 10 – 2 = � �(–8)
1 000
F
F – 12 = –0.24
10 N 2N F = 11.76 N
m = 30 g

5 Selesaikan masalah yang berikut.


Solve the following problems.

(a) Maisarah menolak sebuah meja berjisim 10 kg dengan daya 55 N. Meja itu bergerak dengan
halaju seragam. Berapakah daya geseran yang bertindak ke atas meja itu? TP 4
Maisarah pushes a 10 kg table with a force of 55 N. The table moves with uniform velocity. What is the frictional
force acting on the table?
Halaju seragam/ Uniform velocity, a = 0
F = ma
55 – f = 10(0)
f = 55 N

53

02 Strategi A+ SPM Fizik Tg 4 (B02) 5pp.indd 53 18/11/2023 9:44 AM


(b) Berapakah daya yang diperlukan untuk menggerakkan objek 200 g dengan pecutan 3 m s–2
jika TP 4
What is the force required to move a 200 g object with an acceleration of 3 m s–2 if
(i) objek itu bergerak di atas suatu permukaan licin,
the object moves on a smooth surface,
(ii) objek itu bergerak di atas suatu permukaan dengan daya geseran 2 N.
the object moves on a surface with a frictional force of 2 N.
m = 0.2 kg, a = 3 m s–2
(i) F = ma = (0.2)(3) = 0.6 N
(ii) F – f = ma
F – 2 = 0.6
F = 2.6 N

(c) Sebuah kereta berjisim 1 200 kg bergerak dengan halaju 15 m s–1 dan berhenti setelah bergerak
sejauh 30 m. Cari TP 4
A 1 200 kg car moves with a velocity of 15 m s–1 and stops after moving at a distance of 30 m. Determine
(i) pecutan kereta itu,
the aceleration of the car,
(ii) daya purata brek kereta itu.
the average force of the brake of the car.
m = 1 200 kg, u = 15 m s–1, v = 0 m s–1, s = 30 m
(i) v2 = u2 + 2as
02 = 152 + 2a(30)
–60a = 225
a = –3.75 m s–2
(ii) F = ma
F = (1 200)(–3.75)
= –4 500 N Negatif bermaksud menentang daya enjin.
Negative means oppose the engine force.

(d) Sebuah objek berjisim 8 kg ditarik ke atas dari lantai


menggunakan tali yang melalui takal. TP 4 KBAT Mengaplikasi
An object with a mass of 8 kg is pulled upwards from the floor by using a
a rope running through a pulley. T

(i) Jika tegangan tali ialah 100 N, berapakah pecutan 100 N


8 kg
objek itu semasa dinaikkan?
If the tension in the rope is 100 N, what is the acceleration of the
object when being lifted?
W = mg
(ii) Berapakah tegangan tali yang akan menghasilkan pecutan 5 m s–2?
What is the tension in the rope that will result in an acceleration of 5 m s–2?
(iii) Berapakah tegangan tali yang akan menyebabkan objek bergerak dengan halaju seragam?
What is the tension in the rope that will cause the object to move with uniform velocity?
(i) T – mg = ma
100 – (8)(9.81) = 8(a)
21.52 = 8a
a = 2.69 m s–2
(ii) T – mg = ma
T – (8)(9.81) = (8)(5)
T – 78.48 = 40
T = 118.48 N
(iii) a = 0
T – mg = ma
T – (8)(9.81) = (8)(0)
T – 78.48 = 0
T = 78.48 N

54

02 Strategi A+ SPM Fizik Tg 4 (B02) 5pp.indd 54 18/11/2023 9:44 AM


2.7 Impuls dan Daya Impuls/ Impulse and Impulsive Force
Latihan 26 Perbandingan antara impuls dengan daya impuls
Comparison between impulse and impulsive force
TP 1 Mengingat kembali pengetahuan dan kemahiran sains mengenai Daya dan Gerakan I.
TP 2 Memahami Daya dan Gerakan I serta dapat menjelaskan kefahaman tersebut.

1 Lengkapkan peta buih berganda berikut tentang perbandingan antara impuls dengan daya
impuls. TP 1
Complete the following double bubble map about the comparison between impulse and impulsive force.
i-THINK Peta Buih Berganda

(d)
Kuantiti
terbitan
Derived
(a) (f)
quantity
Perubahan Kadar
momentum perubahan momentum
Change of Rate of change
momentum of momentum

(g)
(b) Daya Rumus:
Rumus: Impuls impuls Formula:
Formula: Impulse Impulsive mv – mu
J = Ft = mv – mu force F=
t

(c) (h)
Unit S.I.: Unit S.I.:
S.I. unit: S.I. unit:
N s atau kg m s–1 N atau kg m s–2
N s or kg m s–1 (e) N or kg m s–2
Kuantiti
vektor
Vector
quantity

2 Isi tempat kosong dengan perkataan yang betul bagi pernyataan di bawah. TP 2
Fill in the blanks with the correct words for the statements below.

(a) Impuls dapat ditingkatkan dengan menggunakan daya yang besar dan masa impak yang
panjang oleh tindakan ikut lajak .
Impulse can be increased by using a large force and a long impact time by follow through
action.
(b) Daya impuls dapat dikurangkan dengan memanjangkan masa impak.
Impulsive force can be reduced by extending the impact time.

55

02 Strategi A+ SPM Fizik Tg 4 (B02) 5pp.indd 55 18/11/2023 9:44 AM


Latihan 27 Menyelesaikan masalah melibatkan impuls dan daya impuls
Solving problems involving impulse and impulsive force
TP 3 Mengaplikasikan pengetahuan mengenai Daya dan Gerakan I untuk menerangkan kejadian atau fenomena alam dan melaksanakan tugasan mudah.
TP 4 Menganalisis pengetahuan mengenai Daya dan Gerakan I dalam konteks penyelesaian masalah mengenai kejadian atau fenomena alam.

Selesaikan masalah yang berikut.


Solve the following problems.
1 Radzuan menendang bola dengan daya 1 200 N. Masa sentuhan kasut dengan bolanya ialah 0.01 s.
Radzuan kicks a ball with a force of 1 200 N. The time of impact between the shoes and the ball is 0.01 s. TP 3
(a) Berapakah impuls yang dipindahkan kepada bola itu?
What is the impulse transferred to the ball?
(b) Jika jisim bola ialah 0.5 kg, kira halaju bola tersebut.
If the mass of the ball is 0.5 kg, calculate the velocity of the ball.
(a) Impuls/Impulse, J = Ft = 1 200 × 0.01 = 12 N s
Juga/Also,
(b) Ft = mv – mu
12 = (0.5)(v) – (0.5)(0)
12
v =
0.5
= 24 m s–1

2 Sebuah kereta berjisim 1 050 kg yang bergerak dengan


halaju 100 m s–1 melanggar sebatang pokok dan berhenti
serta-merta. TP 3
A car with a mass of 1 050 kg moving with a velocity of 100 m s–1
hits a tree and stops immediately.
(a) Berapakah impuls kereta itu?
What is the impulse of the car?
Ft = mv – mu
= (1 050)(0) – (1 050)(100)
= –105 000 N s

(b) Jika selang masa tindakan antara kereta dengan pokok ialah 0.5 s, tentukan daya impuls yang
dikenakan ke atas kereta itu.

If the time of impact between the car and the tree is 0.5 s, determine the impulsive force that acted on the car.
mv – mu
F =
t
(1 050)(0) – (1 050)(100)
=
0.5
= –210 000 N

3 Dalam suatu pertandingan badminton, seorang pemain


memukul bulu tangkis berjisim 5 g yang datang ke u
v
arahnya dengan halaju 30 m s–1. Bulu tangkis itu
bergerak dengan halaju 60 m s–1 setelah dipukul. Masa
perlanggaran antara bulu tangkis dengan raket badminton
ialah 0.02 s. TP 3
In a badminton match, a player hits a shuttlecock with a mass of
5 g coming towards him with a velocity of 30 m s–1. The shuttlecock
moves at a velocity of 60 m s–1 after being hit. The time of impact
between the shuttlecock and badminton racquet is 0.02 s.
(a) Berapakah impuls yang dialami oleh bulu tangkis?
What is the impulse experienced by the shuttlecock?
Ft = mv – mu
= (0.005)(60) – (0.005)(–30)
= 0.45 N s

56

02 Strategi A+ SPM Fizik Tg 4 (B02) 5pp.indd 56 18/11/2023 9:44 AM


(b) Berapakah daya impuls yang dikenakan ke atas bulu tangkis?

What is the impulsive force acting on the shuttlecock?
mv – mu
Ft =
t
(0.005)(60) – (0.005)(–30)
=
0.02
= 22.5 N

4 Alia yang berjisim 55 kg melompat dari tingkat satu rumahnya semasa kebakaran berlaku di
rumahnya. Halajunya semasa mencecah tanah ialah 5 m s–1.
Alia who weighs 55 kg jumps from the first floor of her house during a fire. Her velocity when hitting the ground is
5 m s–1.
(a) Hitung impuls apabila kakinya mencecah tanah. TP 3
Calculate the impulse when her feet touch the ground.
Ft = mv – mu
= (55)(0) – (55)(5)
= –275 N s

(b) Jika Alia membengkokkan kakinya ketika mendarat dan mengambil masa 0.5 s untuk berhenti,
berapakah daya impuls yang bertindak pada kaki Alia? TP 3

If Alia bends her legs while landing and takes 0.5 s to stop, what is the impulsive force acting on her legs?
mv – mu
Ft =
t
(55)(0) – (55)(5)
=
0.5
= –550 N

(c) Jika Alia tidak membengkokkan kakinya ketika mendarat dan mengambil masa 0.05 s untuk
berhenti, berapakah daya impuls yang bertindak pada kaki Alia? TP 3

If Alia does not bend her legs while landing and takes 0.05 s to stop, what is the impulsive force acting on her legs?
mv – mu
Ft =
t
(55)(0) – (55)(5)
=
0.05
= –5 500 N

(d) Jelaskan tindakan terbaik yang boleh diambil oleh Alia untuk mengelakkan kecederaan.
Explain the best action that can be taken by Alia to avoid injury. TP 4 KBAT Menganalisis

Alia perlu membengkokkan kakinya semasa mendarat untuk memanjangkan masa hentakan supaya
daya impuls dapat dikurangkan.
Alia needs to bend her legs while landing to lengthen the impact time so that the impulsive force can be reduced.

57

02 Strategi A+ SPM Fizik Tg 4 (B02) 5pp.indd 57 18/11/2023 9:44 AM


Latihan 28 Daya impuls dalam kehidupan seharian
Impulsive force in daily life
TP 3 Mengaplikasikan pengetahuan mengenai Daya dan Gerakan I untuk menerangkan kejadian atau fenomena alam dan melaksanakan tugasan mudah.

1 Lengkapkan peta pelbagai alir berikut untuk membincangkan cara-cara mengurangkan daya
impuls. TP 3
Complete the following multi-flow map to discuss the ways to reduce impulsive force.
i-THINK Peta Pelbagai Alir

(a) Pemain kriket memakai (b) Tindakan ini akan


sarung tangan memanjangkan masa

dan menghayunkan tangan hentaman dan mengurangkan


ke belakang semasa daya impuls. Hal ini dapat
menangkap bola kriket. mengelakkan kecederaan

gloves berlaku pada pemain.


A cricket player wears
and swings the hand backwards This action will lengthen the time of
when catching the ball. impact and reduce the impulsive
force. This will prevent injury to
the player.

(c) Bekas telur diperbuat daripada


(d) Bahan yang lembut akan
bahan yang lembut untuk memanjangkan masa hentaman
mengelakkan telur daripada
Cara-cara dan mengurangkan daya
retak atau pecah .
mengurangkan impuls.
soft daya impuls
Egg trays are made from Ways to reduce Soft materials will lengthen the
materials to prevent the eggs from impulsive force
time of impact and reduce the
cracking or breaking .
impulsive force.

(e) Seorang ahli payung terjun (f) Tindakannya akan


membengkokkan kakinya memanjangkan masa

apabila mendarat. hentaman dan


mengurangkan daya impuls .
A parachutist bends his legs
Hal ini dapat mengelakkan
when landing.
kecederaan berlaku pada ahli
payung terjun.

His action will lengthen the


time of impact and reduce
the impulsive force . This will
prevent injury to the parachutist.

58

02 Strategi A+ SPM Fizik Tg 4 (B02) 5pp.indd 58 18/11/2023 9:44 AM


2 Lengkapkan peta pelbagai alir berikut untuk membincangkan cara-cara menambahkan daya
impuls. TP 3
Complete the following multi-flow map to discuss the ways to increase impulsive force.
i-THINK Peta Pelbagai Alir

(b) Hal ini akan memendekkan


masa hentaman dan
(a) Kepala penukul diperbuat meningkatkan daya impuls.

daripada bahan yang keras . besar


Daya yang diperlukan

The hammer head is made from apabila memukul paku.


hard material. shorten
This will the time of impact
and increase the impulsive force.
A large force is required when
hitting the nail.

(c) Penukul cerucuk diperbuat


(d) Hal ini akan memendekkan
daripada bahan yang keras
masa hentaman dan
dan dijatuhkan dari paras yang meningkatkan daya impuls.
tinggi dengan halaju yang
Daya yang besar
tinggi .
Cara-cara diperlukan untuk memukul
meningkatkan
The pile hammer is made from daya impuls cerucuk ke dalam tanah.
hard material and is dropped Ways to increase
impulsive force It will shorten the time of impact
from a high position with
and increase the impulsive force.
a high velocity.
A large force is required to drive
the pile into the ground.

(f) Hal ni akan memendekkan


masa hentaman dan
(e) Alu dan lesung diperbuat meningkatkan daya impuls.
daripada bahan yang Daya yang besar diperlukan
sangat keras . untuk menghancurkan bahan-
Mortar and pestle are made from bahan.
very hard materials. It will shorten the time of impact
and increase the impulsive force.
A large force is needed to crush
the ingredients.

59

02 Strategi A+ SPM Fizik Tg 4 (B02) 5pp.indd 59 18/11/2023 9:44 AM


Latihan 29 Ciri-ciri keselamatan dalam kenderaan
Safety features in vehicles
TP 6 Mereka cipta menggunakan pengetahuan dan kemahiran sains mengenai Daya dan Gerakan I dalam konteks penyelesaian masalah atau membuat keputusan
atau dalam melaksanakan aktiviti/tugasan dalam situasi baharu secara kreatif dan inovatif dengan mengambil kira nilai sosial/ekonomi/budaya masyarakat.

Aktiviti PAK-21
Tujuan: Mengumpul maklumat tentang ciri-ciri keselamatan dalam kenderaan
Aim: To collect information about safety features in vehicles
1 Bahagikan kelas kepada beberapa kumpulan kecil.
Divide the class into several small groups.
2 Cari maklumat dari pusat sumber atau Internet berkaitan perkara berikut:
Find the information from the resource centre or Internet regarding the following:
(a) Ciri-ciri keselamatan dalam kenderaan/ The safety features in vehicles
(b) Fungsi bagi setiap ciri-ciri keselamatan dalam kenderaan/ The functions for each safety feature in vehicles
3 Bincang dan lakukan aktiviti berikut dalam kumpulan. Kemudian, persembahkan hasil yang diperoleh.
Discuss and conduct the following activity in the group. Then, present the results obtained.

Anda diminta mereka bentuk sebuah kereta yang dilengkapi dengan ciri-ciri keselamatan bagi mengurangkan kecederaan
kepada pemandu dan penumpang jika berlakunya kemalangan. Tulis lima ciri keselamatan dan fungsinya. KBAT Mencipta
You are asked to design a car that is equipped with safety features to reduce injuries to the driver and passengers if an accident occurs.
Write five safety features and their functions.

Cadangan jawapan/ Suggested answers:


Ciri-ciri Sel keselamatan
Fungsi penumpang
keselamatan Function Passenger safety cell Kaca keselamatan
Safety feature Safety glass
Zon remuk belakang
Beg udara • Mencegah pemandu daripada Rear crumple zone
Beg udara
Aktiviti PAK-21

Airbag menghentam stereng dan Air bags


penumpang menghentam
papan pemuka hadapan
Prevents the driver from crashing
into the steering wheel and the
passengers from crashing into the
front dashboard
Tali pinggang
• Menghentikan pemandu dan keledar
penumpang dalam masa yang Safety belts
lebih panjang (masa hentaman
panjang) Palang hentakan sisi Zon remuk
Stops the driver and passengers in Side-impact bars
hadapan
a longer time (long time of impact) Sistem brek antikunci Front crumple
• Mengurangkan kesan daya Anti-lock braking system (ABS) zone
impuls dan inersia
Reduces the effects of impulsive
force and inertia

Tali pinggang • Mencegah pemandu dan penumpang daripada terhumban ke hadapan


keledar Prevents the driver and passengers from being thrown forwards
Safety belt • Mengurangkan kesan inersia/ Reduces the effects of inertia
Zon remuk • Memanjangkan masa hentaman
hadapan Lengthens the time of impact
dan belakang • Mengurangkan kesan daya impuls
Front and rear Reduces the effects of impulsive force
crumple zones
Palang • Memanjangkan masa hentaman
hentakan sisi Lengthens the time of impact
Side-impact bar • Mengurangkan kesan daya impuls
Reduces the effects of impulsive force

Sistem brek • Mengekalkan cengkaman antara tayar kereta dengan permukaan jalan supaya kereta tidak
antikunci tergelincir
Anti-lock braking Maintains the grip between car tyres and road surface so that the car does not slip
system (ABS) • Mengelakkan roda daripada terkunci dan berhenti berputar
Prevents the wheels from locking and stops spinning

Projek Berkumpulan STEAM TP 6

60

02 Strategi A+ SPM Fizik Tg 4 (B02) 5pp.indd 60 18/11/2023 9:44 AM


2.8 Berat/ Weight
Latihan 30 Penerangan tentang berat dan perbandingan antara berat dengan jisim
Explanation of weight and comparison between weight and mass
TP 1 Mengingat kembali pengetahuan dan kemahiran sains mengenai Daya dan Gerakan I.
TP 2 Memahami Daya dan Gerakan I serta dapat menjelaskan kefahaman tersebut.

1 Isi tempat kosong dengan perkataan yang betul. TP 1 TP 2


Fill in the blanks with the correct words.

(a) Berat, W suatu objek ialah daya tindakan graviti terhadap


suatu objek.
Weight, W of an object is the force of gravity acting on an object.
(b) Berat dapat dihitung dengan rumus
Weight can be calculated by formula
Berat, W
Weight
W= mg

dengan keadaan/where
W = Berat/ Weight
m = Jisim/ Mass
g = Kekuatan medan graviti/ Gravitational field strength

2 Lengkapkan peta buih berganda berikut tentang perbandingan antara berat dengan jisim. TP 1
Complete the following double bubble map about the comparison between weight and mass.
i-THINK Peta Buih Berganda

(a)
Daya graviti (g)
yang Jumlah jirim
bertindak ke atas dalam objek
(b) suatu objek Amount of matter (h)
Simbol: Gravitational force in object Simbol:
Symbol: acting on an object Symbol:
W m

(c) (i)
Kuantiti (f) Kuantiti
terbitan dan Kuantiti asas dan
kuantiti vektor Berat fizik Jisim kuantiti skalar
Weight Mass
Derived quantity Physical Base quantity
and vector quantity and scalar
quantity quantity

(e) (k)
(d) Unit S.I.: Unit S.I.:
Berubah apabila S.I. unit: S.I. unit: (j)
kekuatan medan graviti N kg Tidak berubah
berubah walau di mana objek itu
berada (tetap)
Changes when the
strength of Does not change

gravitational field
no matter where the object is
(fixed)
changes

61

02 Strategi A+ SPM Fizik Tg 4 (B02) 5pp.indd 61 18/11/2023 9:44 AM


Latihan 31 Mengira berat dan jisim objek
Calculating weight and mass of objects
TP 3 Mengaplikasikan pengetahuan mengenai Daya dan Gerakan I untuk menerangkan kejadian atau fenomena alam dan melaksanakan tugasan mudah.

Video
1 Diberi bahawa pecutan graviti, g ialah 9.81 m s–2, Untuk tujuan pembelajaran
It is given that the gravitational acceleration, g is 9.81 m s , –2
Imbas kod QR atau layari https://
www.youtube.com/watch?v=rFdbY_
(a) kira berat objek berikut, TP 3 V7vIo untuk menonton video
calculate the weight of the following objects, tentang berat dan jisim.

(i) (ii) (iii)

Jisim/ Mass, m = 70 kg Jisim/ Mass, m = 1 200 kg Jisim/ Mass, m = 50 g

50
W = (70)(9.81) W = (1 200)(9.81) W = � �(9.81)
= 686.7 N = 11 772 N 1 000
= 0.4905 N

(b) kira jisim objek berikut. TP 3


calculate the mass of the following objects.

(i) (ii) (iii)

Berat/ Weight, W = 53.9 N Berat/ Weight, W = 19 600 N Berat/ Weight, W = 2.94 N

53.9 19 600 2.94


m = m = m =
9.81 9.81 9.81
= 5.494 kg = 1 997.961 kg = 0.3 kg

Latihan 32 Mengira berat objek di tempat berbeza


Calculating weight of object at different locations
Video

TP 3 Mengaplikasikan pengetahuan mengenai Daya dan Gerakan I untuk menerangkan kejadian atau fenomena alam dan melaksanakan tugasan mudah.

1 Lengkapkan jadual berikut bagi objek berjisim 50 kg. TP 3


Complete the following table for an object with a mass of 50 kg.

Pecutan graviti, g
Tempat/ Locations Berat/ Weight
Gravitational acceleration

(a) Bumi W = (50)(9.81)


9.81 m s–2
Earth = 490.5 N

(b) Bulan W = (50)(1.64)


1.64 m s–2
Moon = 82 N

(c) Neptun W = (50)(11.15)


11.15 m s–2
Neptune = 557.5 N

62

02 Strategi A+ SPM Fizik Tg 4 (B02) 5pp.indd 62 18/11/2023 9:44 AM


Praktis Berformat SPM
A s (m) s (m)
Kertas 1

1 Seorang pemandu memandu kereta


dari kampung ke pekan melalui jalan
lurus dengan halaju purata 70 km j–1. Jika
pemandu itu memasuki pekan dengan halaju t (s)
0 0
95 km j–1, berapakah halaju keretanya semasa s (m) s (m)
B
di kampung? Anggap pecutan
s (m) kereta adalah s (m)
seragam sepanjang perjalanan. s (m) s (m)
A driver drives a car from a village to a town through
a straight road at an average velocity of 70 km h–1. If
the driver enters the town with a velocity of 95 km h–1,
t (s)
what is the velocity of his car at the village? Assume 0 0
t (s) t (s)
the acceleration of the car is 0uniform throughout the 0
journey. 0 t (s) 0
A 15 km j–1/ km h–1 s (m) C s (m)
s (m) s (m)
B 45 km j / km h
–1 –1
s (m) s (m)
C 82.5 km j–1/ km h–1
D 165 km j–1/ km h–1
2 Sebuah bas bergerak daripada keadaan rehat
t (s) t (s)
di perhentian bas dan bergerak
0 di sepanjang 0
0
t (s)
0
jalan lurus dengan pecutan
0
seragam 2.8 m s–2 t (s) 0 t (s)
selama 40 saat. Bas itu kemudian bergerak
dengan halaju seragam.s (m)
Berapakah sesaran D s (m)
bas semasa ia memecut?
A bus starts from rest at a bus stop and moves
along a straight road with a uniform acceleration of
2.8 m s–2 for 40 seconds. The bus then moves at
a uniform velocity. What is the displacement of the bus
when it is accelerating? t (s) t (s)
0 0
A 560 m C 2 240 m
B 1 120 m D 4 480 m 4 Rajah 2 menunjukkan graf halaju-masa bagi
3 Rajah 1 menunjukkan sebuah kereta sedang gerakan sebuah van. KBAT Mengaplikasi
Diagram 2 shows the velocity-time graph for the motion
mendaki bukit. Halaju kereta itu semakin
of a van.
berkurang semasa mendaki bukit dan
kemudiannya menjadi seragam.
Diagram 1 shows a car moving up a hill. The velocity Halaju (m s–1)
Velocity
of the car decreases as it moves up the hill and then
becomes uniform. 25

Masa (s)
0 1 2 3 4 5 6 Time

–25
Rajah 1/ Diagram 1
Antara graf yang berikut, yang manakah Rajah 2/ Diagram 2
graf sesaran melawan masa yang betul bagi Berapakah sesaran kereta itu dalam masa 6 s?
pergerakan kereta tersebut? KBAT Menganalisis What is the displacement of the car in 6 s?
Which of the following graphs is the correct graph of A 0m C 62.5 m
displacement against time for the motion of the car? B 37.5 m D 87.5 m

63

02 Strategi A+ SPM Fizik Tg 4 (B02) 5pp.indd 63 18/11/2023 9:44 AM


5 Rajah 3 menunjukkan sebiji bola yang sedang 8 Antara rajah yang berikut, yang manakah
jatuh dari sebuah bangunan. menunjukkan momentum yang paling rendah
Diagram 3 shows a ball falling from a building. bagi kereta yang sama?
Which of the following diagrams shows the lowest
momentum for the same car?
A C

Rajah 3/ Diagram 3
Halaju = 120 km j–1 Halaju = 80 km j–1
Apakah kuantiti fizik yang malar semasa bola
Velocity = 120 km h–1 Velocity = 80 km h–1
itu jatuh?
What is the physical quantity that is constant while the B D
ball is falling?
A Tenaga keupayaan
Potential energy
B Tenaga kinetik Halaju = 100 km j–1 Halaju = 60 km j–1
Kinetic energy Velocity = 100 km h –1
Velocity = 60 km h–1
C Pecutan
Acceleration 9 Rajah 5 menunjukkan seorang lelaki
D Halaju melompat dari perahu P ke perahu Q.
Velocity Diagram 5 shows a man jumping from boat P to boat Q.

6 Rajah 4 menunjukkan seekor anjing sedang


mengeringkan bulunya. B
Diagram 4 shows a dog drying its fur.
Q P
D C

A
Rajah 5/ Diagram 5
Antara A, B, C dan D, yang manakah
menunjukkan arah gerakan perahu P?
Rajah 4/ Diagram 4
Among A, B, C and D, which is the direction of motion
Apakah konsep fizik yang terlibat dalam of boat P?
situasi di atas?
What is the physics concept involved in the above 10 Suatu objek dikenakan tiga daya, F yang sama
situation? magnitud tetapi arah berbeza. Situasi yang
A Momentum C Pecutan manakah menyebabkan daya paduan yang
Momentum Acceleration paling besar? KBAT Menilai
B Inersia D Impuls An object is acted upon by three forces, F which have
Inertia Impulse the same magnitude but different directions. Which
situation results in the largest resultant force?
7 Objek yang manakah mempunyai inersia A C
yang paling besar?
Which object has the greatest inertia?
F F
A C F F
F F

16 kg 97 000 kg B D
B D
F F
F F
F F

1 350 kg 6 000 kg

64

02 Strategi A+ SPM Fizik Tg 4 (B02) 5pp.indd 64 18/11/2023 9:44 AM


11 Rajah 6 menunjukkan sebuah lori mainan A Mengurangkan masa perlanggaran atlet
berjisim 2 kg yang ditolak dengan daya itu dengan tilam
6 N. Daya geseran antara lori mainan dengan To reduce the collision time between the athlete and
lantai ialah 2 N. the mattress
Diagram 6 shows a toy lorry with a mass of 2 kg being B Meningkatkan perubahan momentum
pushed by a force of 6 N. The frictional force between atlet apabila terkena tilam
the toy lorry and the floor is 2 N. To increase the change of momentum of the athlete
when hitting the mattress
C Mengurangkan perubahan momentum
atlet apabila terkena tilam
To reduce the change of momentum of the athlete
when hitting the mattress
D Mengurangkan daya impuls yang
Rajah 6/ Diagram 6 bertindak ke atas atlet apabila terkena
Berapakah pecutan lori mainan itu? tilam
What is the acceleration of the toy lorry? To reduce the impulsive force acting on the athlete
A 2 m s–2 when hitting the mattress
B 3 m s–2
C 4 m s–2 13 Rajah 8 menunjukkan seorang angkasawan
D 5 m s–2 yang mempunyai jisim 50 kg dan berat
81.75 N di permukaan Bulan.
12 Rajah 7 menunjukkan seorang atlet sedang Diagram 8 shows an astronaut with a mass of 50 kg and
melakukan lompatan dalam suatu acara a weight of 81.75 N on the surface of the Moon.
lompat galah mendarat di atas sebuah tilam.
Diagram 7 shows an athlete performing pole vault in a
pole vault event landing on a mattress.

Rajah 8/ Diagram 8
Berapakah kekuatan medan graviti di
Rajah 7/ Diagram 7 permukaan Bulan?
What is the gravitational field strength on the surface
Apakah fungsi tilam tersebut?
of the Moon?
What is the function of the mattress?
A 0.325 N kg–1 C 4.905 N kg–1
B 1.635 N kg–1 D 9.813 N kg–1

Kertas 2

Bahagian A

1 Rajah 1 menunjukkan satu keratan pita detik yang A B C D

diperoleh daripada suatu eksperimen.


Diagram 1 shows a ticker tape obtained from an experiment.
(a) Satu detik ialah masa yang diambil dari titik A ke Rajah 1/ Diagram 1
titik B .
One tick is the time taken from dot A to dot B .
[1 markah/mark]
(b) Lengkapkan ayat berikut dengan menulis perkataan yang betul pada ruangan yang disediakan.
Complete the following sentence by writing the correct words in the spaces provided.

Pita detik dapat dianalisis untuk menentukan sesaran dan halaju/ pecutan .
The ticker tape can be analysed to determine displacement and velocity/ acceleration .
[1 markah/mark]

65

02 Strategi A+ SPM Fizik Tg 4 (B02) 5pp.indd 65 18/11/2023 9:44 AM


(c) (i) Apakah jenis gerakan yang ditunjukkan oleh pita detik dalam Rajah 1?
What is the type of motion shown by the ticker tape in Diagram 1?
Laju seragam/Halaju seragam/Pecutan sifar
Uniform speed/ Uniform velocity/ Zero acceleration

[1 markah/mark]
(ii) Beri satu sebab bagi jawapan anda di 1(c)(i).
Give one reason for your answer in 1(c)(i).
Jarak di antara dua titik berturutan adalah sama./ The distances between two consecutive dots are the same.
[1 markah/mark]

2 Rajah 2.1 menunjukkan kedudukan permulaan dan


akhir bagi Fatimah ketika berjalan di sebuah taman.
Diagram 2.1 shows the starting and finishing positions of Fatimah
while walking in a park.
50 m 100 m
Rajah 2.2 menunjukkan graf sesaran-masa bagi
pergerakan Fatimah.
Diagram 2.2 shows the displacement-time graph for the movement
of Fatimah.
(a) Apakah definisi sesaran? Akhir Mula
What is the definition for displacement? Finish Start

Jarak terpendek di antara dua titik dalam arah tertentu Rajah 2.1/ Diagram 2.1

Shortest distance between two points in a specific direction Sesaran (m)


Displacement
[1 markah/mark] 100
(b) Apakah yang berlaku kepada pergerakan Fatimah
dari saat ke-80 hingga ke-120?
What happens to the motion of Fatimah from the 80th to the 50
120th seconds?
Tidak bergerak/Pegun/Berhenti/Rehat 0 Masa (s)
20 40 60 80 100 120 140 160 180 200 220 Time
Not moving/ Stationary/ Stop/ Rest

[1 markah/mark] –50
(c) Tentukan jumlah sesaran Fatimah. Rajah 2.2/ Diagram 2.2
Determine the total displacement of Fatimah.
Jumlah sesaran/ Total displacement
= 100 + 0 – 100 – 50
= –50 m
[1 markah/mark]
(d) Hitung halaju purata Fatimah untuk keseluruhan perjalanan.
Calculate the average velocity of Fatimah for the whole journey.
s –50
v= = = –0.227 m s–1
t 220
[2 markah/marks]

3 Rajah 3.1(a) dan Rajah 3.1(b) menunjukkan dua cara seorang atlet boleh mendarat dalam suatu
acara lompat jauh.
Diagrams 3.1(a) and 3.1(b) show two ways an athlete may land in a long jump event.

Rajah 3.1(a)/ Diagram 3.1(a) Rajah 3.1(b)/ Diagram 3.1(b)

66

02 Strategi A+ SPM Fizik Tg 4 (B02) 5pp.indd 66 18/11/2023 9:44 AM


(a) Namakan daya yang menyebabkan atlet tersebut jatuh kembali ke tanah dari satu ketinggian.
Name the force that causes the athlete to fall back to the ground from a height.
Daya graviti/Gravitational force
[1 markah/mark]

(b) Atlet berjisim 60 kg itu melompat dan mencecah tanah dengan halaju 5 m s–1. Masa pendaratan
yang diambil ialah 0.2 s bagi Rajah 3.1(a) dan 0.5 s bagi Rajah 3.1(b).
The athlete with a mass of 60 kg jumps and reach the ground with a velocity of 5 m s–1. The landing times are
0.2 s for Diagram 3.1(a) and 0.5 s for Diagram 3.1(b).
(i) Hitung magnitud daya impuls yang bertindak dalam Rajah 3.1(a) dan Rajah 3.1(b).
Calculate the magnitude of impulsive force acting in Diagrams 3.1(a) and 3.1(b).
Rajah 3.1(a)/Diagram 3.1(a),
mv – mu
F =
t
60(0) – 60(5)
=
0.2
= –1 500 N
Magnitud/Magnitude = 1 500 N

Rajah 3.1(b)/Diagram 3.1(b),


mv – mu
F =
t
60(0) – 60(5)
=
0.5
= –600 N
Magnitud/Magnitude = 600 N
[2 markah/marks]
(ii) Apakah kesan masa impak terhadap daya impuls?
What is the effect of time of impact on the impulsive force?
Masa impak bertambah, daya impuls berkurang atau sebaliknya/Masa impak berkadar songsang
dengan daya impuls
Time of impact increases, impulsive force decreases or vice versa/ Time of impact is inversely proportional to impulsive

force

[1 markah/mark]

(c) Rajah 3.2 menunjukkan seorang lelaki sedang bermain golf.


Diagram 3.2 shows a man playing golf.
Jadual 1 menunjukkan empat jenis kayu golf dan ketinggian, h yang
boleh dicapai semasa menghayun kayu golf. h
Table 1 shows four types of golf clubs and the height, h that can be attained when
swinging the golf club.

Pilihan Bahan kayu golf Ketinggian, h (m)


Choice Material of golf club Height, h (m)
Rajah 3.2/ Diagram 3.2
Titanium
P 2.5
Titanium
Kayu
Q 2.0
Wood
Gentian karbon
R 1.0
Carbon fibre
Jadual 1/ Table 1

67

02 Strategi A+ SPM Fizik Tg 4 (B02) 5pp.indd 67 18/11/2023 9:44 AM


Cadangkan jenis kayu golf yang sesuai untuk menambah jarak pukulan bola golf itu
berdasarkan aspek yang berikut:
Suggest the suitable type of golf club to increase the throw distance of the golf ball based on the following
aspects:
(i) Bahan kayu golf/Material of golf club
Titanium/ Titanium
[1 markah/mark]
Sebab/Reason
Kuat/Tidak mudah patah
Strong/ Not easily broken

[1 markah/mark]
(ii) Ketinggian/Height, h
Paling tinggi/Highest
[1 markah/mark]
Sebab/Reason
Lebih banyak tenaga/Bola golf bergerak lebih laju
More energy/ The golf ball moves faster

[1 markah/mark]

(d) Berdasarkan jawapan anda di 3(c), pilih kayu golf yang paling sesuai untuk
permainan golf itu.
Based on your answers in 3(c), choose the most suitable golf club for the golf game.
P
[1 markah/mark]
Bahagian B

4 (a) Rajah 4.1 menunjukkan seorang pemandu sedang memakai


tali pinggang keselamatan sebelum memulakan perjalanan.
Diagram 4.1 shows a driver wearing seat belt before starting his journey.
Tujuan memakai tali pinggang keselamatan adalah
untuk mengurangkan kecederaan pada pemandu apabila
berlakunya kemalangan.
The purpose of wearing a seat belt is to reduce injuries to the driver
when accident occurs.
(i) Apakah maksud inersia? Rajah 4.1/ Diagram 4.1
What is inertia?
[1 markah/mark]
(ii) Terangkan bagaimana memakai tali pinggang keselamatan boleh mengurangkan
kecederaan apabila berlakunya perlanggaran. KBAT Menganalisis
Explain how wearing a seat belt can reduce injuries when collision occurs. [4 markah/marks]

(b) Sebuah kereta berjisim 1 650 kg sedang bergerak dengan


halaju 110 km j–1 di atas lebuh raya ditunjukkan dalam
Rajah 4.2.
A car with a mass of 1 650 kg is moving at a velocity of 110 km j–1

on a highway is shown in Diagram 4.2.
(i) Kira halaju kereta dalam unit S.I.
Calculate the velocity of the car in S.I. unit.
[2 markah/marks]
(ii) Berapakah momentum kereta itu? Rajah 4.2/ Diagram 4.2
What is the momentum of the car?
[1 markah/mark]

68

02 Strategi A+ SPM Fizik Tg 4 (B02) 5pp.indd 68 18/11/2023 9:44 AM


(iii) Jika pemandu itu menekan pedal minyak dan kereta memecut pada 2 m s–2, berapakah
halaju akhir kereta tersebut selepas 2 saat?
If the driver steps on the accelerator and the car accelerates at 2 m s–2, what is the final velocity of the car
after 2 seconds? [4 markah/marks]

(c) Rajah 4.3 menunjukkan sebuah topi keledar litar yang digunakan dalam sebuah pertandingan
trek basikal di dalam velodrom.
Diagram 4.3 shows a track helmet used in a track cycling competition in a velodrome.

Lubang udara
Air hole
Rangka dalam
Inner frame Bahagian depan
Front end
Tali
Strap

Rajah 4.3/ Diagram 4.3


Jadual 2 menunjukkan ciri-ciri empat jenis topi keledar litar.
Table 2 shows the characteristics of four types of track helmets.

Topi
keledar
Tali Bahagian depan Lubang udara Bahan rangka dalam
litar Strap Front end Air hole Inner frame material
Track
helmet

Lebar Bulat dan pendek Ada Polisterina


W
Wide Round and short Present Polystyrene
Kecil Panjang dan tirus Tiada Polisterina
X
Narrow Long and pointed Absent Polystyrene
Kecil Panjang dan tirus Ada Getah
Y
Narrow Long and pointed Present Rubber
Lebar Bulat dan pendek Tiada Getah
Z
Wide Round and short Absent Rubber

Jadual 2/ Table 2

Terangkan kesesuaian setiap ciri topi keledar trek dan tentukan topi keledar yang paling sesuai
digunakan untuk pertandingan di dalam velodrom. KBAT Menganalisis
Explain the suitability of each characteristic of the track helmet and determine the most suitable track helmet
to be used for competition in the velodrome.
[10 markah/marks]

Jawapan/ Answers:
4 (a) (i) Inersia ialah kecenderungan suatu objek untuk kekal pegun atau jika bergerak, terus bergerak dalam
garis lurus dengan halaju seragam.
Inertia is the tendency of an object to remain at rest or if in motion, to continue its motion in a straight line at uniform

velocity.

(ii) – Pemandu mempunyai jisim dan inersia.


The driver has mass and inertia.

– [Apa] Inersia ialah kecenderungan suatu objek untuk kekal pegun atau jika bergerak, terus bergerak
dalam garis lurus dengan halaju seragam.

69

02 Strategi A+ SPM Fizik Tg 4 (B02) 5pp.indd 69 18/11/2023 9:44 AM


[What] Inertia is the tendency of an object to remain at rest or if in motion, to continue its motion in a straight line

at uniform velocity.

– [Bagaimana] Jika berlaku perlanggaran, pemandu akan terhumban ke depan kerana inersia
menghalang perubahan gerakan.
[How] In a collision, the driver will be thrown forwards because inertia prevents changes in motion.

– Tali pinggang keselamatan menghalang pemandu daripada terhentak pada stereng kenderaan.
Safety belt prevents the driver from hitting the steering wheel of the vehicle.

(b) (i) v = 110 × 1 000



1 × 60 × 60

v = 30.556 m s–1
(ii) p = mv
p = 1 650 × 30.556
= 50 417.4 kg m s–1
(iii) a = v – u

t
v – 30.556
2 =
2
v = 34.556 m s–1
(c) [Apa/ What, Mengapa/ Why]
Ciri-ciri/ Characteristic Sebab/ Reason
Tali lebar Mengurangkan tekanan
Wide strap Reduce pressure

Bahagian depan bulat dan pendek Kurangkan rintangan udara


Front end round and short Reduce air resistance

Ada lubang udara Udara mengalir ke dalam topi keledar untuk penyejukan
Air hole present Air flows into helmet for cooling

Bahan rangka dalam: Polisterina Ringan/Ketumpatan rendah


Inner frame material: Polystyrene Lighter/ Low density

Topi keledar litar W dipilih kerana mempunyai tali lebar, bahagian depan bulat dan pendek, ada lubang
udara dan bahagian rangka dalam yang diperbuat daripada polisterina.
Track helmet W is chosen because it has wide strap, front end round and short, air hole present and an inner frame made of

polystyrene.

70

02 Strategi A+ SPM Fizik Tg 4 (B02) 5pp.indd 70 18/11/2023 9:44 AM


Bab Kegravitian
3 Gravitation
Tema: Mekanik Newton


Revisi Pantas
4 Hubungan pecutan graviti, g di permukaan Bumi
3.1 Hukum Kegravitian Semesta Newton
dengan pemalar kegravitian semesta, G ialah
Newton’s Universal Law of Gravitation The relationship between gravitational acceleration, g on the
1 Kewujudan daya graviti atau daya semesta surface of the Earth with the universal gravitational constant, G is
menunjukkan bahawa dua jasad dalam alam semesta
GM
akan mengenakan daya antara satu sama lain dengan g=
magnitud yang sama. r2
The existence of gravitational force or universal force shows that
two bodies in the universe will exert force on each other with the dengan keadaan/ where
same magnitude. g = Pecutan graviti/Gravitational acceleration
2 Daya graviti boleh dibuktikan dengan Hukum G = Pemalar kegravitian semesta
Kegravitian Semesta Newton. Universal gravitational constant
Gravitational force can be proved by Newton’s Universal Law of M = Jisim Bumi/Mass of the Earth
Gravitation.
r = Jarak di antara pusat Bumi dengan pusat objek
3 Hukum Kegravitian Semesta Newton menyatakan Distance between the centre of the Earth and the centre of
bahawa daya graviti, F antara dua jasad berkadar terus the object
dengan hasil darab jisim-jisim jasad dan berkadar
songsang dengan kuasa dua jarak di antara pusat dua 5 Daya memusat ialah daya yang membolehkan suatu
jasad tersebut. objek bergerak mengikut laluan membulat. Arahnya
Newton’s Universal Law of Gravitation states that gravitational sentiasa menuju ke pusat bulatan.
force, F between two bodies is directly proportional to the product of Centripetal force is the force that enables an object to move in a
the masses of the bodies and inversely proportional to the square of circular path. Its direction is always towards the centre of the circle.
the distance between the centres of the two bodies. Rumus bagi daya memusat diberi sebagai
The formula for centripetal force is given as
Jasad pertama Jasad kedua
First body Second body mv2
F=
r

F F dengan keadaan/ where


m1 m2
F = Daya memusat/Centripetal force
m = Jisim/Mass
v = Laju linear/Linear speed
r
r = Jejari bulatan/Radius of circle
6 Jisim Bumi dan Matahari dapat ditentukan dengan
Rumus Hukum Kegravitian Semesta Newton diberi
menggunakan rumus Hukum Kegravitian Semesta
sebagai
Newton dan daya memusat:
The formula for the Newton’s Universal Law of Gravitation is
The mass of the Earth and the Sun can be determined by using
given as
Newton's Universal Law of Gravitation and centripetal force:
Gm1m2
F= 4π2r3
r2 M=
GT2
dengan keadaan/ where
dengan keadaan/ where
F = Daya graviti antara dua jasad
Gravitational force between two bodies M = Jisim planet atau Matahari yang ingin ditentukan
Mass of the planet or Sun to be determined
m1 = Jisim jasad pertama/Mass of the first body
r = Jejari orbit mana-mana satelit atau planet yang
m2 = Jisim jasad kedua/Mass of the second body mengorbit
r = Jarak di antara pusat jasad pertama dengan Radius of orbit of any orbiting satellite or planet
jasad kedua T = Tempoh peredaran mana-mana satelit atau
Distance between the centres of the first and second bodies
planet yang mengorbit
G = Pemalar kegravitian semesta Period of revolution of any orbiting satellite or planet
Universal gravitational constant
G = Pemalar kegravitian semesta
(6.67 × 10–11 N m2 kg–2)
Universal gravitational constant

71

03 Strategi A+ SPM Fizik Tg 4 (B03) 5pp.indd 71 18/11/2023 9:55 AM


3.2 Hukum Kepler
Kepler’s Laws
i-THINK Peta Pokok
Hukum Kepler
Kepler's Laws

Hukum Kepler Pertama Hukum Kepler Kedua Hukum Kepler Ketiga


Kepler's First Law Kepler's Second Law Kepler's Third Law

Orbit bagi setiap planet adalah elips dengan Matahari Garis yang Kuasa dua tempoh
berada di salah satu fokusnya. menyambungkan suatu orbit planet adalah
The orbit of each planet is elliptical with the Sun at one of its foci. planet dengan Matahari berkadar terus dengan
akan mencakupi luas kuasa tiga jejari
Planet yang sama dalam orbitnya,
selang masa yang sama The square of orbital period
Paksi minor apabila planet bergerak of a planet is proportional to
Minor axis the cube of radius of its orbit,
dalam orbitnya.
The line that connects a
T2 ∝ r3
Paksi major planet to the Sun will sweep
Matahari Major axis out equal areas in equal times
Sun when the planet moves in its
orbit.

Video Tutorial
Untuk tujuan pembelajaran
Imbas kod QR atau layari
https://www.youtube.
com/watch?v=kyR6EO_
RMKE&t=776s untuk
menonton video tutorial
tentang Hukum Kepler.

Satelit Buatan Manusia Rumus bagi laju linear satelit diberi sebagai
3.3 The formula for the linear velocity of the satellite is given as
Man-Made Satellites
GM
1 Satelit semula jadi ialah sebarang planet yang bergerak v=
r
mengelilingi planet lain yang lebih besar saiznya.
Contohnya, Bulan ialah satelit kepada Bumi, dan Bumi
ialah satelit kepada Matahari. dengan keadaan/ where
A natural satellite is any planet that moves around another planet v = Laju linear satelit/Linear speed of satellite
that is larger in size. For examples, the Moon is a satellite to the G = Pemalar kegravitian/Gravitational constant
Earth, and the Earth is a satellite to the Sun. M = Jisim Bumi/Mass of the Earth
2 Satelit buatan manusia ialah struktur binaan manusia r = Jejari orbit/Radius of orbit
yang dilancarkan ke angkasa lepas dan mengorbit 6 Terdapat dua jenis satelit yang mengorbit Bumi, iaitu
Bumi untuk melaksanakan fungsi tertentu. satelit geopegun dan satelit bukan geopegun.
Man-made satellites are man-made structures that are launched
There are two types of satellites that orbit the Earth, the
into space and orbit the Earth to carry out certain functions.
geostationary satellites and the non-geostationary satellites.
3 Satelit buatan manusia memerlukan daya tarikan graviti 7 Halaju lepas, v ialah halaju minimum yang diperlukan
untuk mengekalkannya dalam orbit mengelilingi Bumi. oleh suatu objek di permukaan Bumi untuk mengatasi
Video Tutorial

Man-made satellites need the gravitational pull to keep them in orbit


daya graviti dan terlepas ke angkasa lepas. Halaju
around the Earth.
lepas dicapai apabila tenaga kinetik minimum yang
4 Satelit buatan manusia akan bergerak dalam orbit pada diberikan kepada objek itu boleh mengatasi tenaga
ketinggian tertentu dengan laju linear tertentu. keupayaan graviti.
Man-made satellites will move in orbit at a certain height with Escape velocity, v is the minimum velocity required by an object
a certain linear speed. on the Earth's surface to overcome the gravitational force and escape
into outer space. Escape velocity is achieved when the minimum
5 Laju linear satelit boleh diterbitkan dengan
kinetic energy given to the object can overcome the gravitational
menggunakan persamaan daya memusat dan Hukum potential energy.
Kegravitian Semesta Newton. Rumus bagi halaju lepas diberi sebagai
The linear velocity of the satellite can be derived using the centripetal
The formula for escape velocity is given as
force and Newton's Universal Law of Gravitation equations.

2GM
v=
r

72

03 Strategi A+ SPM Fizik Tg 4 (B03) 5pp.indd 72 18/11/2023 9:55 AM


Praktis PBD
3.1 Hukum Kegravitian Semesta Newton
Newton’s Universal Law of Gravitation

Latihan 1 Hukum Kegravitian Semesta Newton


Newton’s Universal Law of Gravitation
TP 1 Mengingat kembali pengetahuan dan kemahiran sains mengenai Kegravitian.
TP 2 Memahami Kegravitian serta dapat menjelaskan kefahaman tersebut.
TP 3 Mengaplikasikan pengetahuan mengenai Kegravitian untuk menerangkan kejadian atau fenomena alam dan melaksanakan tugasan mudah.

Isi tempat kosong dengan perkataan yang betul. TP 1 TP 2 TP 3


Fill in the blanks with the correct words.

1 Daya graviti ialah daya yang menarik mana-mana dua objek berjisim. Hal ini bermaksud
setiap objek berjisim mengenakan daya graviti kepada objek lain.
Gravitational force is a force that attracts any two objects with masses. This means every object with mass exerts
a gravitational force on every other object.

2 Daya graviti boleh dijelaskan oleh Hukum Kegravitian Semesta Newton .


The gravitational force can be explained by Newton’s Universal Law of Gravitation .

3 Menurut Hukum Kegravitian Semesta Newton, daya graviti antara dua jasad adalah berkadar
terus dengan hasil darab jisim kedua-dua jasad dan berkadar songsang dengan
kuasa dua jarak di antara dua jasad itu .
According to Newton’s Universal Law of Gravitation, the gravitational force between two bodies is directly
proportional to the product of masses of the two bodies and inversely proportional to the
square of the distance between the two bodies .

Jasad pertama Jasad kedua


First body Second body

F F
m1 m2

4 Rumus bagi Hukum Kegravitian Semesta Newton diberi sebagai


The formula for Newton’s Universal Law of Gravitation is given as

Gm1m2
F= r2
dengan keadaan/where

F = Daya graviti antara dua jasad/ Gravitational force between two bodies
m1 = Jisim jasad pertama/Mass of the first body

m2 = Jisim jasad kedua/Mass of the second body

r = Jarak di antara pusat jasad pertama dengan jasad kedua


Distance between the centres of the first and second bodies
g
= Pemalar kegravitian semesta/Universal gravitational constant
(6.67 × 10–11 N m2 kg–2)

73

03 Strategi A+ SPM Fizik Tg 4 (B03) 5pp.indd 73 18/11/2023 9:55 AM


5 Berdasarkan rumus daya graviti, lengkapkan peta pokok berikut dengan menyatakan dua faktor
yang mempengaruhi daya graviti serta perkaitannya.
Based on the gravitational force formula, complete the following tree map by stating two factors that affect the
gravitational force and their relationship. i-THINK Peta Pokok

Faktor yang mempengaruhi daya graviti


Factors that affect the gravitational force

Jisim jasad Jarak di antara pusat dua jasad


Mass of body Distance between the centres of the two bodies

Perkaitan Perkaitan
Relationship Relationship

Semakin besar jisim jasad Semakin besar jarak di antara pusat


dua jasad , semakin kuat dua jasad , semakin lemah
daya graviti. daya graviti.
The larger the mass of the body The larger the distance between the centres
of the two bodies , the stronger the of the two bodies , the weaker the
gravitational force. gravitational force.

Latihan 2 Menyelesaikan masalah melibatkan Hukum Kegravitian Semesta Newton


Solving problems involving Newton’s Universal Law of Gravitation
TP 3 Mengaplikasikan pengetahuan mengenai Kegravitian untuk menerangkan kejadian atau fenomena alam dan melaksanakan tugasan mudah.

Selesaikan masalah yang berikut.


Solve the following problems.

1 Dua objek berjisim 2.5 kg setiap satu diletakkan pada jarak 5.0 m di antara satu sama lain. Hitung
daya graviti antara kedua-dua objek itu. TP 3
Two objects, with a mass of 2.5 kg each are placed at a distance of 5.0 m from each other. Calculate the gravitational force
between the two objects.
[G = 6.67 × 10–11 N m2 kg–2]
Gm1m2
F=
r2
(6.67 × 10–11)(2.5)(2.5)
=
52
= 1.668 × 10–11 N

2 Sebuah kereta kebal mengalami daya graviti 4.98 × 105 N. Berapakah jisim kereta kebal itu? TP 3
A tank experiences a gravitational force of 4.98 × 105 N. What is the mass of the tank?
[Jisim Bumi = 6.00 × 1024 kg, jarak di antara pusat Bumi dengan pusat kereta kebal = 6.37 × 106 m]
[Mass of the Earth = 6.00 × 1024 kg, the distance between the centre of the Earth and the centre of the tank
= 6.37 × 106 m]
Gm1m2
F =
r2
(6.67 × 10–11)(6.00 × 1024)m2
4.98 × 105 =
(6.37 × 106)2
m2 = 5.049 × 104 kg

74

03 Strategi A+ SPM Fizik Tg 4 (B03) 5pp.indd 74 18/11/2023 9:55 AM


Latihan 3 Hubung kait antara pecutan graviti, g di permukaan Bumi dengan pemalar kegravitian
semesta, G
Relating gravitational acceleration, g on the surface of the Earth with universal gravitational constant, G
TP 2 Memahami Kegravitian serta dapat menjelaskan kefahaman tersebut.

1 Lengkapkan rajah berikut untuk menerbitkan hubungan antara g dengan G. TP 2


Complete the following diagram to derive the relationship between g and G.
Bumi
Objek Earth
Object

g = Pecutan graviti/Gravitational acceleration


G = Pemalar kegravitian semesta/Universal gravitational constant
M = Jisim Bumi/Mass of the Earth
r = Jarak di antara pusat Bumi dengan pusat objek
Distance between the centre of the Earth and the centre of the object

Menurut Hukum Gerakan Newton Berdasarkan Hukum Kegravitian Semesta


Kedua, daya graviti ialah Newton, daya graviti diungkapkan sebagai
According to the Newton’s Second Law of According to the Newton’s Universal Law of
Motion, the gravitational force is Gravitation, the gravitational force is stated as

GMm
F= mg ..................... ❶ F= ..................... ❷
r2

❶=❷

GM
g=
r2

2 Graf dalam rajah di sebelah menunjukkan Di permukaan Bumi


variasi bagi nilai pecutan graviti, g dengan g On the surface of the Earth

jarak suatu objek dari pusat Bumi, r. R ialah GM


jejari Bumi. g=
R2
The graph in the diagram on the right shows the
variation in the values of gravitational acceleration, g Di dalam Bumi Di atas dari permukaan Bumi
Inside the Earth Above the surface of the Earth
with the distance of an object from the centre of the
(g ∝ r) (g ∝ 12 )
Earth, r. R is the radius of the Earth. r

Lengkapkan maklumat berikut berdasarkan 0 R


r
graf yang ditunjukkan. TP 2
Complete the following information based on the
graph shown.

(a) Nilai g berkadar terus dengan r jika r < R.


The value of g is directly proportional to r if r < R.

(b) Nilai g berkadar songsang dengan r2 jika r ù R.


The value of g is inversely proportional to r2 if r ù R.

75

03 Strategi A+ SPM Fizik Tg 4 (B03) 5pp.indd 75 18/11/2023 9:55 AM


Latihan 4 Menyelesaikan masalah melibatkan pecutan graviti
Solving problems involving gravitational acceleration
TP 3 Mengaplikasikan pengetahuan mengenai Kegravitian untuk menerangkan kejadian atau fenomena alam dan melaksanakan tugasan mudah.

Selesaikan masalah yang berikut.


Solve the following problems.

1 Sebuah satelit telekomunikasi mengorbit Bumi pada ketinggian 410 km. Berapakah nilai pecutan
graviti satelit itu? TP 3
[Jejari Bumi, R = 6.37 × 106 m, jisim Bumi = 6.00 × 1024 kg]
A telecommunication satellite orbits the Earth at a height of 410 km. What is the value of gravitational acceleration of
the satellite?
[Radius of the Earth, R = 6.37 × 106 m, mass of the Earth = 6.00 × 1024 kg]
GM
g=
(R + r)2
(6.67 × 10–11)(6.00 × 1024)
=
(6.37 × 106 + 4.10 × 105)2
= 8.706 m s–2

2 Rajah di sebelah menunjukkan sebuah satelit pada ketinggian Satelit


Satellite
6 850 km dari pusat Bumi. TP 3
The diagram on the right shows a satellite at a height of 6 850 km from
the centre of the Earth.
Hitung pecutan graviti pada kedudukan satelit itu.
Calculate the gravitational acceleration at the position of the satellite. Orbit
Bumi
Aktiviti PAK-21

[Jisim Bumi/Mass of the Earth = 5.97 × 1024 kg, Earth


pemalar kegravitian/gravitational constant = 6.67 × 10–11 N m2 kg–2]
GM
g=
r2
6.67 × 10–11 × 5.97 × 1024
=
(6.85 × 106)2
= 8.486 m s–2

Latihan 5 Kepentingan mengetahui nilai pecutan graviti


The importance of knowing the value of gravitational acceleration
TP 4 Menganalisis pengetahuan mengenai Kegravitian dalam konteks penyelesaian masalah mengenai kejadian atau fenomena alam.
TP 6 Mereka cipta menggunakan pengetahuan dan kemahiran sains mengenai Kegravitian dalam konteks penyelesaian masalah atau membuat keputusan atau dalam
melaksanakan aktiviti/tugasan dalam situasi baharu secara kreatif dan inovatif dengan mengambil kira nilai sosial/ekonomi/budaya masyarakat.

Aktiviti PAK-21
Tujuan: Mengumpul maklumat tentang kepentingan mengetahui nilai pecutan graviti
Aim: To collect information about the importance of knowing the value of gravitational acceleration
1 Bahagikan kelas kepada beberapa kumpulan kecil.
Divide the class into several small group.
2 Cari maklumat dari pusat sumber atau Internet berkaitan perkara berikut:
Find the information from the resource centre or Internet regarding the following:
(a) Perbandingan pecutan graviti yang berbeza bagi Bulan, Matahari dan planet-planet dalam Sistem Suria
Comparison of different gravitational accelerations for the Moon, Sun and planets in the Solar System
(b) Kepentingan mengetahui nilai pecutan graviti
The importance of knowing the value of gravitational acceleration
3 Bincang dan lengkapkan aktiviti berikut dalam kumpulan. Kemudian, persembahkan hasil yang diperoleh
di hadapan kelas.
Discuss and complete the following activity in the group. Then, present the results obtained in front of the class.

Projek Berkumpulan STEM TP 6

76

03 Strategi A+ SPM Fizik Tg 4 (B03) 5pp.indd 76 18/11/2023 9:55 AM


1 Terangkan kesan terhadap tumbesaran badan manusia jika manusia tinggal di Bulan. TP 4
Explain the effects on the growth of the human body if humans live on the Moon.
• Ketumpatan badan manusia akan berkurang (terasa lebih ringan).
The density of the human body will decrease (feel lighter).

• Tulang menjadi rapuh dan boleh mengalami osteoporosis.


Bones become fragile and may suffer from osteoporosis.

• Darah mengalir lebih perlahan.


Blood flows more slowly.

• Tekanan darah menjadi rendah.


Blood pressure becomes low.

Latihan 6 Daya memusat dalam pergerakan satelit dan planet


Centripetal force in the motion of satellites and planets
TP 2 Memahami Kegravitian serta dapat menjelaskan kefahaman tersebut.
TP 3 Mengaplikasikan pengetahuan mengenai Kegravitian untuk menerangkan kejadian atau fenomena alam dan melaksanakan tugasan mudah.

Isi tempat kosong dengan perkataan yang betul. TP 2 TP 3


Fill in the blanks with the correct words.

1 Daya memusat ialah daya yang diperlukan oleh suatu objek untuk mengekalkannya dalam gerakan
membulat pada suatu jarak dari pusat bulatan dengan suatu halaju.
Centripetal force is the force needed by an object to keep it in a circular motion at a distance from the
centre of a circle with a velocity.

2 Daya memusat boleh dihitung menggunakan rumus:


Centripetal force can be calculated using the formula:

mv2
F =
r

dengan keadaan/ where


F = Daya memusat/ Centripetal force
Jisim/ Mass
m =
v = Laju linear/ Linear speed
Jejari bulatan/ Radius of circle Satelit
r = Satellite

Daya memusat
3 Satelit mengorbit mengelilingi Bumi tanpa tujahan Centripetal force, F
roket kerana daya graviti Bumi ke atas satelit
bertindak sebagai daya memusat . v

Satellites orbit the Earth without rocket thrust because


r
the gravitational force of the Earth on the satellites acts as a Bumi Orbit
Arah gerakan Earth
centripetal force . Direction of
motion

4 Oleh sebab halaju satelit berubah secara berterusan dalam satu gerakan membulat
seragam, maka satelit itu memecut. Pecutan ini sentiasa ke arah pusat bulatan dan dikenali sebagai
pecutan memusat .
Since the velocity of satellite changes constantly in a uniform circular motion, then the satellite accelerates.
This acceleration is always directed towards the centre of the circle and is known as centripetal acceleration .

77

03 Strategi A+ SPM Fizik Tg 4 (B03) 5pp.indd 77 18/11/2023 9:55 AM


5 Rumus pecutan memusat boleh diterbitkan daripada rumus daya memusat. Terbitkan rumus itu.
The centripetal acceleration formula can be derived from the centripetal force formula. Derive the formula.
F = ma .................... ❶
mv2
F= .................... ❷
r
❶=❷
mv2
ma =
r
v2
a=
r
dengan keadaan/ where
v = Laju linear satelit/ Linear speed of the satellite
r = Jejari orbit satelit/ Radius of the orbit of the satellite

Latihan 7 Menyelesaikan masalah melibatkan daya memusat dan pecutan memusat


Solving problems involving centripetal force and centripetal acceleration
TP 3 Mengaplikasikan pengetahuan mengenai Kegravitian untuk menerangkan kejadian atau fenomena alam dan melaksanakan tugasan mudah.

Selesaikan masalah yang berikut.


Solve the following problems.

1 Sebuah satelit kaji cuaca mengelilingi Bumi untuk memetakan cuaca negara-negara yang berbeza.
Ketinggian satelit itu dari Bumi ialah 620 km dan laju linearnya ialah 8.00 × 103 m s–1. Berapakah
pecutan memusat satelit itu? TP 3
[Jejari Bumi, R = 6.37 × 106 m]
A weather satellite orbits the Earth to map the weather of different countries. The height of the satellite from the Earth
is 620 km and its linear speed is 8.00 × 103 m s−1. What is the centripetal acceleration of the satellite?
[Radius of the Earth, R = 6.37 × 106 m]
v2
a=
R+r
(8.00 × 103)2
=
6.37 × 106 + 6.20 × 105
= 9.156 m s–2

2 Rajah berikut menunjukkan seorang atlet sedang menghayun tukul besi berjisim 6 kg dalam gerakan
membulat semasa suatu acara sukan. TP 3
The following diagram shows an athlete swinging an iron hammer with a mass of 6 kg in circular motion during
a sports event.

Tukul
2m Hammer

Jika tukul itu bergerak dengan halaju seragam 20 m s–1, hitung daya memusat yang bertindak ke
atas tukul itu.
If the hammer moves with a uniform velocity of 20 m s−1, calculate the centripetal force acting on the hammer.
mv2
F=
r
(6)(20)2
=
2
= 1 200 N

78

03 Strategi A+ SPM Fizik Tg 4 (B03) 5pp.indd 78 18/11/2023 9:55 AM


Latihan 8 Menentukan jisim Bumi dan Matahari
Determining the masses of the Earth and the Sun
TP 2 Memahami Kegravitian serta dapat menjelaskan kefahaman tersebut.
TP 3 Mengaplikasikan pengetahuan mengenai Kegravitian untuk menerangkan kejadian atau fenomena alam dan melaksanakan tugasan mudah.

1 Lengkapkan rajah berikut untuk menentukan rumus Jisim Bulan, m


Mass of the Moon
jisim Bumi. TP 2
Jejari orbit, r
Complete the following diagram to determine the formula for the Radius of orbit
mass of the Earth.
Laju linear
Bulan, v
Linear speed
T = Tempoh peredaran Bulan mengelilingi Bumi of the Moon Jisim Bumi, M
Period of revolution of the Moon around the Earth Mass of the Earth

Rumus Hukum Kegravitian Semesta Newton, Rumus daya memusat,


The formula for Newton’s Universal Law of Gravitation, The formula for centripetal force,

GMm mv2
F= ..................... ❶ F= ..................... ❷
r2 r

❶ = ❷

GMm mv2
=
r2 r

GM
v=
r

Gantikan/ Substitute v = 2πr


T
Jisim Bumi 4π2r3
Mass of the Earth M=
GT 2

2 Selesaikan masalah yang berikut.


Solve the following problem.

Jejari orbit Bumi mengelilingi Matahari ialah 1.50 × 1011 m. Bumi mengambil masa 3.15 × 107 s untuk
melengkapkan satu orbit. Hitung jisim Matahari. TP 3
The orbital radius of the Earth around the Sun is 1.50 × 1011 m. The Earth takes 3.15 × 107 s to complete one orbit.
Calculate the mass of the Sun.
4π2r3
M=
GT2
4π2(1.50 × 1011)3
=
(6.67 × 10–11)(3.15 × 107)2
= 2.013 × 1030 kg

79

03 Strategi A+ SPM Fizik Tg 4 (B03) 5pp.indd 79 18/11/2023 9:55 AM


3.2 Hukum Kepler
Kepler’s Laws

Latihan 9 Hukum Kepler


Kepler’s Laws
TP 1 Mengingat kembali pengetahuan dan kemahiran sains mengenai Kegravitian.
TP 2 Memahami Kegravitian serta dapat menjelaskan kefahaman tersebut.

Isi tempat kosong dengan perkataan yang betul. TP 1 TP 2


Fill in the blanks with the correct words.

1 Hukum Kepler Pertama


Kepler’s First Law
menyatakan bahawa/ states that:

Semua planet dalam Sistem Suria mempunyai orbit berbentuk elips dengan Matahari
sentiasa berada di salah satu fokus elips.
All planets in the Solar System have elliptical orbits with the Sun always at one of the foci

of the ellipse.

Rajah di sebelah menunjukkan


bahawa paksi major adalah lebih
panjang daripada paksi minor. Paksi minor Planet
Minor axis
The diagram on the right shows that the
major axis is longer than the minor axis.

Paksi major
Major axis
Matahari
Sun

2 Hukum Kepler Kedua


Kepler’s Second Law
menyatakan bahawa/ states that:

Garis yang menyambungkan planet dengan Matahari akan mencakupi luas yang sama
dalam selang masa yang sama apabila planet bergerak dalam orbitnya.
The line joining a planet and the Sun will sweep out equal areas in equal intervals of time when
the planet moves in its orbit.

Dalam rajah di sebelah, luas kawasan AFB


adalah sama dengan luas kawasan CFD, Luas sama
jika masa untuk planet itu bergerak dari Same area
A ke B adalah sama dengan masa Planet
planet itu bergerak dari C ke D. Hal ini
menunjukkan bahawa planet itu bergerak B

dengan laju linear yang lebih tinggi dari A F C


ke B berbanding dengan dari C ke D.
In the diagram on the right, the area of AFB is Matahari D
the same as the area of CFD, if the time for the Sun
planet to move from A to B is the same as the time A
for the planet to move from C to D. This shows
that the planet moves at a higher linear speed
from A to B compared to from C to D.

80

03 Strategi A+ SPM Fizik Tg 4 (B03) 5pp.indd 80 18/11/2023 9:55 AM


3 Hukum Kepler Ketiga
Kepler’s Third Law
menyatakan bahawa/ states that:

Kuasa dua tempoh orbit planet adalah berkadar terus dengan kuasa tiga jejari orbitnya,
T2 ∝ r3.
The square of the orbital period of a planet is directly proportional to the cube of the radius of
its orbit, T2 ∝ r3.

Planet yang berada jauh dari Matahari mengambil masa yang lebih lama untuk mengelilingi
Matahari berbanding dengan planet yang berada dekat dengan Matahari.
Planets further from the Sun take a longer time to orbit the Sun compared to planets closer to the Sun.

Latihan 10 Merumuskan Hukum Kepler Ketiga


Express Kepler’s Third Law
Jisim planet, m
TP 1 Mengingat kembali pengetahuan dan kemahiran sains mengenai Kegravitian. Mass of planet Daya memusat, F
TP 2 Memahami Kegravitian serta dapat menjelaskan kefahaman tersebut. Centripetal force

Jejari orbit, r
1 Lengkapkan persamaan yang berikut untuk Radius of orbit
Laju linear
merumuskan Hukum Kepler Ketiga. TP 1 TP 2 planet, v
Complete the following equations to express Kepler’s Third Law. Linear speed
of planet
Jisim Matahari, M
T = Tempoh peredaran planet mengelilingi Matahari Mass of the Sun

Period of revolution of the planet around the Sun

Rumus daya graviti, Rumus daya memusat,


The formula for gravitational force, The formula for centripetal force,

GMm mv2
F= ..................... ❶ F= ..................... ❷
r2 r

❶ = ❷

GMm mv2
=
r2 r

Video Tutorial
Untuk tujuan pembelajaran
GM
= v2 Imbas kod QR atau layari
r https://www.youtube.com/
watch?v=0lJNUdTe8g8 untuk
menonton video tutorial tentang
Gantikan/ Substitute v = 2πr langkah menerbitkan rumus
T
Video Tutorial

Hukum Kepler Ketiga.

4π2 3
T2 = � �r
GM
4π2
dengan keadaan pemalar, k =
where the constant
GM
T2 = kr3
Oleh itu/ Therefore,
T2 ∝ r3
Maka/ Thus,
T12 r13
=
T22 r23

81

03 Strategi A+ SPM Fizik Tg 4 (B03) 5pp.indd 81 18/11/2023 9:55 AM


Latihan 11 Menyelesaikan masalah menggunakan rumus Hukum Kepler Ketiga
Solving problems using Kepler’s Third Law Formula
TP 3 Mengaplikasikan pengetahuan mengenai Kegravitian untuk menerangkan kejadian atau fenomena alam dan melaksanakan tugasan mudah.

Selesaikan masalah yang berikut.


Solve the following problems.

1 Bumi mengelilingi Matahari dengan jejari orbit 1.50 × 1011 m dan tempoh orbit Bumi ialah
365 hari. Sebuah planet, J mengorbit mengelilingi Matahari dengan jejari orbit 4.37 × 1011 m. Kira
tempoh planet J mengorbit mengelilingi Matahari. TP 3
The Earth orbits around the Sun at an orbital radius of 1.50 × 1011 and the orbital period of the Earth is 365 days.
A planet, J orbits around the Sun at an orbital radius of 4.37 × 1011 m. Calculate the period of planet J orbiting the Sun.
T12 r 13
=
T22 r 23
(4.37 × 1011)3 (1)2
T22 =
(1.50 × 1011)3
T = 4.973 tahun/ years

2 Bumi bergerak mengeliling Matahari dengan tempoh 3.15 × 107 s pada jarak purata 1.50 × 1011 m
dari Matahari. Hitung tempoh Musytari mengelilingi Matahari pada jarak purata 7.79 × 1011 m dari
Matahari. TP 3
The Earth moves around the Sun with a period of 3.15 × 107 s at an average distance of 1.50 × 1011 m from the Sun.
Calculate the period of Jupiter orbiting the Sun at an average distance of 7.79 × 1011 m from the Sun.
3.15 × 107
3.15 × 107 s =
24 × 3 600
= 365 hari/ days
= 1 tahun/ year
T12 r 13
=
T22 r 23
(7.79 × 1011)3 (1)2
T22 =
(1.50 × 1011)3
T = 11.835 tahun/ years

3 Planet S mengorbit mengelilingi Matahari dengan jejari orbit 2.28 × 1011 m dan tempoh peredaran
686 hari. Sebuah asteroid dilihat sedang mengorbit Matahari dengan tempoh 1.43 tahun. Kira jejari
orbit asteroid tersebut. TP 3
Planet S orbiting the Sun with an orbital radius of 2.28 × 1011 m and a period of revolution of 686 days. An asteroid
is seen orbiting the Sun with a period of 1.43 years. Calculate the orbital radius of the asteroid.
686
TS =
365
= 1.879 tahun/ years

TS2 r S3
=
Ta2 ra3
Ta2 rS3
ra3 =
TS2
(1.43)2(2.28 × 1011)3
ra3 =
(1.879)2
ra = 1.901 × 1011 m

82

03 Strategi A+ SPM Fizik Tg 4 (B03) 5pp.indd 82 18/11/2023 9:55 AM


3.3 Satelit Buatan Manusia
Man-made Satellites

Latihan 11 Laju linear


Linear speed
TP 1 Mengingat kembali pengetahuan dan kemahiran sains mengenai Kegravitian.
TP 2 Memahami Kegravitian serta dapat menjelaskan kefahaman tersebut.

Isi tempat kosong dengan perkataan yang betul. TP 1 TP 2


Fill in the blanks with the correct words.
Jisim satelit, m
Mass of satellite
1 Rumus laju linear bagi satu satelit diterbitkan dengan
menggunakan persamaan daya tarikan graviti
antara satelit dengan Bumi dan daya memusat yang F Jejari orbit satelit, r
bertindak pada satelit. Laju linear Radius of orbit of satellite
The formula for linear velocity of a satellite is derived using the satelit, v
Linear speed
equation of the gravitational pull between the satellite and the of satellite Jisim Bumi, M Orbit
Earth and the centripetal force acting on the satellite. Mass of the Earth

Daya tarikan graviti, Daya memusat,


Gravitational attraction force, Centripetal force,
GMm mv2
F1 = F2 =
r2 r

F 1 = F2
GMm mv2
=
r 2
r
GM
v2 =
r

GM
v= r

dengan keadaan/ where


M = Jisim Bumi/ Mass of the Earth
Jisim satelit/ Mass of satellite
m =
r = Jejari orbit satelit/ Radius of orbit of satellite
v = Laju linear satelit/ Linear speed of satellite
G = Pemalar kegravitian/ Gravitational constant

2 Oleh sebab GM adalah malar , laju linear satelit hanya bergantung pada jejari orbitnya .
As GM is constant , the linear speed of the satellite depends only on the radius of its orbit .

3 Satelit bergerak dengan laju linear dalam orbit membulat mengelilingi Bumi. Pecutan memusat
satelit itu adalah sama dengan pecutan graviti .
Satellites move at a linear speed in a circular orbit around the Earth. The centripetal acceleration of the satellite
is equal to the gravitational acceleration.

4 Jika laju linear satelit berkurang , satelit itu akan jatuh ke orbit yang semakin rendah sehingga
memasuki atmosfera Bumi. Seterusnya, satelit akan terbakar dan musnah.
If the linear speed of the satellite decreases , the satellite will fall into lower orbits until it enters
the Earth’s atmosphere. Then, the satellite will burn and be destroyed.

83

03 Strategi A+ SPM Fizik Tg 4 (B03) 5pp.indd 83 18/11/2023 9:55 AM


Latihan 13 Satelit geopegun dan satelit bukan geopegun
Geostationary and non-geostationary satellites
TP 2 Memahami Kegravitian serta dapat menjelaskan kefahaman tersebut.

Padankan ciri-ciri satelit berikut kepada jenis satelit, sama ada satelit geopegun atau satelit bukan
geopegun. TP 2
Match the following features of satellite to the types of satellites, whether geostationary satellite or non-geostationary satellite.

Ciri-ciri satelit Jenis satelit


Feature of satellite Type of satellite
1 Tempoh orbit, T lebih pendek atau lebih panjang daripada
24 jam •
The orbital period, T is shorter or longer than 24 hours

2 Arah gerakan tidak perlu sama dengan arah putaran Bumi


The direction of motion does not have to be the same as the direction of • Satelit
the rotation of the Earth •
geopegun
Geostationary
3 Satelit komunikasi satellite

Communication satellite
4 Arah gerakan sama dengan arah putaran Bumi
The direction of motion is the same as the direction of the rotation of •
the Earth
5 Pengimejan Bumi, GPS dan ramalan keadaan cuaca

Earth imaging, GPS and weather forecasting

6 Berada di atas kedudukan geografi yang berbeza di


permukaan Bumi • Satelit
Over different geographical locations on the surface of the Earth bukan
• geopegun
7 Tempoh orbit, T = 24 jam Non-geostationary

Orbital period, T = 24 hours satellite
8 Berada di atas kedudukan geografi yang sama di
permukaan Bumi •
Over the same geographical location on the surface of the Earth

Latihan 14 Halaju lepas


Escape velocity
TP 1 Mengingat kembali pengetahuan dan kemahiran sains mengenai Kegravitian.
TP 2 Memahami Kegravitian serta dapat menjelaskan kefahaman tersebut.
TP 4 Menganalisis pengetahuan mengenai Kegravitian dalam konteks penyelesaian masalah mengenai kejadian atau fenomena alam.

1 Isi tempat kosong dengan perkataan yang sesuai. TP 1 TP 2


Fill in the blanks with the suitable words.

(a) Halaju lepas, ve ialah halaju minimum yang diperlukan oleh objek di permukaan Bumi untuk
mengatasi daya tarikan graviti dan terlepas ke angkasa lepas.
Escape velocity, ve is the minimum velocity required by an object on the surface of the Earth to overcome
the gravitational pull and escape into outer space.

(b) Rajah di sebelah menunjukkan sebuah roket dilancarkan ve


dari Bumi ke angkasa lepas. r
The diagram on the right shows a rocket being launched from the m
Earth into space.

84

03 Strategi A+ SPM Fizik Tg 4 (B03) 5pp.indd 84 18/11/2023 9:55 AM


Jika roket berjisim m berada pada jarak r dari pusat Bumi yang berjisim M, maka tenaga
keupayaan graviti yang dimiliki oleh roket itu ialah
If the rocket with a mass of m is at a distance of r from the centre of the Earth with a mass of M, then the gravitational
potential energy possessed by the rocket is

GMm
U= –
r

(c) Halaju lepas dicapai apabila tenaga kinetik minimum yang dibekalkan kepada objek itu
dapat mengatasi tenaga keupayaan graviti .

Escape velocity is achieved when the minimum kinetic energy supplied to the object can overcome its
gravitational potential energy.

Maka/ Therefore,

Tenaga kinetik minimum + Tenaga keupayaan graviti = 0


Minimum kinetic energy + Gravitational potential energy = 0

Terbitkan rumus halaju lepas berdasarkan persamaan di atas.
Derive the formula for escape velocity based on the equation above.
1 GMm
mve2 + �– � =0
2 r
1 GMm
mve2 =
2 r
2GM
ve =
r
r=R
Maka/ Therefore,
2GM
ve =
R

(d) Halaju lepas bagi suatu objek bergantung pada jisim Bumi dan jarak objek dari
pusat Bumi . Halaju lepas tidak bergantung pada jisim objek yang dilepaskan ke angkasa
lepas.
The escape velocity of an object depends on the mass of the Earth and the distance of the object from the
centre of the Earth . Escape velocity does not depend on the mass of the object that is released into outer space.

(e) Halaju lepas Bumi adalah sangat tinggi, iaitu 11 200 m s–1. Hal ini adalah kerana Bumi
mempunyai jisim yang sangat besar .
The escape velocity of the Earth is very high, that is, 11 200 m s−1. This is because the Earth has a very large
mass.

2 Bumi mempunyai halaju lepas yang sangat tinggi. Terangkan secara ringkas manfaat halaju lepas
yang tinggi terhadap manusia. TP 4 KBAT Menganalisis
The Earth has a high escape velocity. Explain the benefits of high escape velocity to humans.
• Bumi berupaya mengekalkan lapisan atmosfera di sekelilingnya. Molekul-molekul udara tidak akan
terlepas ke angkasa lepas.
The Earth is able to maintain the layer of atmosphere around it. Air molecules will not escape into outer space.

• Kapal terbang boleh terbang pada altitud tinggi tetapi tidak boleh terlepas ke angkasa lepas.
Airplanes can fly at high altitudes but cannot escape into outer space.

85

03 Strategi A+ SPM Fizik Tg 4 (B03) 5pp.indd 85 18/11/2023 9:55 AM


Latihan 15 Menyelesaikan masalah melibatkan laju linear dan halaju lepas
Solving problems involving linear speed and escape velocity
TP 3 Mengaplikasikan pengetahuan mengenai Kegravitian untuk menerangkan kejadian atau fenomena alam dan melaksanakan tugasan mudah.

Selesaikan masalah yang berikut.


Solve the following problems.

1 Satelit MEASAT berada di orbit bulatan dengan jarak 613 km di atas permukaan Bumi. Jejari purata
Bumi ialah 6.37 × 106 m dan jisim Bumi ialah 5.98 × 1024 kg. Berapakah laju linear, v satelit itu dalam
orbitnya? TP 3
The MEASAT satellite is in a circular orbit at a distance of 613 km above the surface of the Earth. The average
radius of the Earth is 6.37 × 106 m and the mass of the Earth is 5.98 × 1024 kg. What is the linear speed, v of the satellite
in its orbit?
[G = 6.67 × 10–11 N m2 kg–2]
GM
v=
r
(6.67 × 10-11)(5.98 × 1024)
=
(6.37 × 106 + 6.13 × 105)
= 7.558 × 103 m s–1

2 Sebuah satelit bukan geopegun mengorbit mengelilingi Bumi pada ketinggian 1 620 km. Berapakah
laju linear satelit itu? TP 3
[Jisim Bumi = 5.98 × 1024 kg , jejari Bumi = 6.37 × 106 m]
A non-geostationary satellite orbits around the Earth at a height of 1 620 km. What is the linear speed of the satellite?
[Mass of the Earth = 5.98 × 1024 kg, radius of the Earth = 6.37 × 106 m]
GM
v=
r
(6.67 × 10-11)(5.98 × 1024)
=
(6.37 × 106 + 1.62 × 106)
= 7.065 × 103 m s–1

3 Sebuah kapal angkasa akan dilancarkan dari permukaan planet Marikh. Tentukan halaju lepas bagi
kapal angkasa tersebut. TP 3
[Jisim dan jejari Marikh masing-masing ialah 6.42 × 1023 kg dan 3.4 × 106 m.]
A spaceship will be launched from the surface of Mars. Determine the escape velocity of the spaceship.
[The mass and radius of the Mars are 6.42 × 1023 kg and 3.4 × 106 m respectively.]
2GM
v=
r
2(6.67 × 10-11)(6.42 × 1023)
=
3.4 × 10
6

= 5.019 × 103 m s–1

4 Diberi bahawa jejari dan jisim Bulan masing-masing ialah 1.74 × 106 m dan 7.35 × 1022 kg. Berapakah
halaju lepas yang diperlukan oleh sebuah roket untuk meninggalkan Bulan? TP 3
It is given that the radius and mass of the Moon are 1.74 × 106 m and 7.35 × 1022 kg respectively. What is the escape
velocity needed by a rocket to leave the Moon?
[G = 6.67 × 10–11 N m2 kg–2]
2GM
v=
r
2(6.67 × 10-11)(7.35 × 1022)
=
1.74 × 10
6

= 2.374 × 103 m s–1

86

03 Strategi A+ SPM Fizik Tg 4 (B03) 5pp.indd 86 18/11/2023 9:55 AM


Praktis Berformat SPM
4 Sebuah kapal terbang mempunyai berat
Kertas 1 200 000 N di permukaan Bumi.
Jika kapal terbang itu terbang pada
1 Apakah yang menyebabkan pergerakan Bumi ketinggian 1 500 km dari permukaan
pada orbitnya? Bumi, berapakah berat kapal terbang itu?
What causes the motion of the Earth in its orbit? An airplane weighs 200 000 N on the surface of the
A Daya graviti yang dikenakan oleh Earth. If the airplane is flying at a height of 1 500 km
Matahari from the surface of the Earth, what is the weight of the
The gravitational force exerted by the Sun airplane? KBAT Mengaplikasi
B Daya graviti yang dikenakan oleh planet- [Jejari Bumi/Radius of the Earth = 6 370 km,
planet g = 9.81 N kg–1]
The gravitational force exerted by the planets A 1.31 × 105 N
C Daya graviti yang dikenakan pada Bumi B 2.00 × 105 N
oleh Bulan C 2.31 × 105 N
The gravitational force exerted on the Earth by
D 7.31 × 105 N
the Moon
D Daya graviti yang dikenakan pada Bumi 5 Satu daya yang membolehkan suatu objek
oleh Matahari bergerak mengikut laluan melengkung
The gravitational force exerted on the Earth by dikenali sebagai …
the Sun A force that makes an object moves along a curved path
is known as …
2 Mengapakah sebuah satelit kaji cuaca yang
A daya graviti/gravitational force.
mengelilingi Bumi kekal dalam orbitnya dan
B daya geseran/frictional force.
tidak terlepas ke angkasa lepas?
C daya gerakan/motion force.
Why does a weather satellite moving around the Earth
remain in its orbit and not escape to outer space? D daya memusat/centripetal force.
KBAT Mengaplikasi
6 Rajah 1 menunjukkan sebuah kereta sedang
A Satelit menjadi tiada berat diuji di sebuah litar berbentuk bulat. Jisim
The satellite becomes weightless kereta tersebut ialah 1 000 kg dan diameter
B Satelit berpusing dalam vakum litar ialah 50 m.
The satellite rotates in a vacuum Diagram 1 shows a car being tested on a circular track.
C Satelit sentiasa ditarik oleh daya graviti The mass of the car is 1 000 kg and the diameter of the
Bumi track is 50 m.
The satellite is always pulled by the gravitational
force of the Earth
D Satelit bergerak dengan kelajuan yang
sangat perlahan
The satellite moves at a very low speed

3 Sebuah satelit komunikasi mengorbit


mengelilingi Bumi pada ketinggian
35 000 km. Hitung pecutan graviti pada
kedudukan satelit itu.
A communication satellite orbits around the Earth Rajah 1/ Diagram 1
at a height of 35 000 km. Calculate the gravitational
acceleration at the position of the satellite. Jika kereta itu bergerak dengan halaju
[G = 6.67 × 10 N m kg , R = 6.37 × 10 m,
–11 2 –2 6 25 km j–1, kira daya memusat yang bertindak
MBumi/Earth = 5.97 × 1024 kg] pada kereta itu.
If the car moves at a velocity of 25 km h−1, calculate the
A 0.233 m s–2
centripetal force acting on the car.
B 2.333 m s–2
A 0.965 × 103 N
C 9.89 m s–2
B 1.929 × 103 N
D 10.00 m s–2
C 5.388 × 103 N
D 9.644 × 103 N

87

03 Strategi A+ SPM Fizik Tg 4 (B03) 5pp.indd 87 18/11/2023 9:55 AM


7 Sebuah satelit telekomunikasi sentiasa berada 12 Antara yang berikut, yang manakah benar
di atas suatu tempat yang sama di permukaan tentang satelit bukan geopegun?
Bumi sepanjang tahun. Berapakah tempoh Which of the following is true about non-geostationary
orbit satelit itu? satellites?
A telecommunication satellite is always above the same I Bergerak dalam arah yang sama dengan
place on the surface of the Earth throughout the year. putaran Bumi
What is the orbital period of the satellite? It moves in the same direction as the rotation
A 12 jam/hours C 36 jam/hours of the Earth
B 24 jam/hours D 365 hari/days II Tempoh putarannya ialah 24 jam
Its period of rotation is 24 hours
8 Bumi mengelilingi Matahari dengan jejari III Orbitnya sentiasa berada di atas
orbit 1.50 × 1011 m dan tempoh orbit Bumi ialah kedudukan geografi yang sama di
1 tahun. Sebuah asteroid dijumpai mengorbit permukaan Bumi
mengelilingi Matahari dalam masa 4.97 tahun. Its orbit is always above the same geographical
Berapakah jarak asteroid dari Matahari? location on the surface of the Earth
The Earth orbits around the Sun at an orbital radius IV Arah gerakan tidak perlu sama dengan
of 1.50 × 1011 m and the orbital period of the Earth is
arah putaran Bumi
1 year. An asteroid is found to orbit around the Sun in
The direction of motion does not have to be the same
4.97 years. What is the distance of the asteroid from the
as the direction of the rotation of the Earth
Sun?
A II sahaja/only C II dan/ and IV
A 4.368 × 1010 m C 4.368 × 1011 m
B IV sahaja/only D I, II dan/ and III
B 3.251 × 1011 m D 5.151 × 1010 m
13 Diberi bahawa jejari Bulan ialah 1.74 × 106 m
9 Apakah hubungan antara tempoh orbit
dan jisim Bulan ialah 7.35 × 1022 kg. Berapakah
sebuah planet, T dengan jejari orbit, r planet
halaju lepas yang diperlukan oleh sebuah
itu mengelilingi Matahari?
roket untuk meninggalkan Bulan?
What is the relationship between the orbital period of
It is given that the radius of the Moon is 1.74 × 106 m
a planet, T and the orbital radius, r of the planet around
and the mass of the Moon is 7.35 × 1022 kg. What is the
the Sun?
escape velocity required for a rocket to leave the Moon?
A T∝r C T2 ∝ r2
[G = 6.67 × 10–11 N m2 kg–2]
B T2 ∝ r D T2 ∝ r3
A 2.374 m s–1 C 2 374 m s–1
10 Rajah 2 menunjukkan Neptun beredar dalam B 9.810 m s –1
D 4 972 m s–1
orbit elips mengelilingi Matahari.
Diagram 2 shows Neptune revolving in an elliptical 14 Sebuah satelit buatan manusia mengorbit
orbit around the Sun. mengelilingi Bumi pada ketinggian 1 620 km.
Q Berapakah laju linear satelit itu?
A man-made satellite orbits around the Earth at a height
of 1 620 km. What is the linear speed of the satellite?
O Matahari
R P [Jisim Bumi/Mass of the Earth = 5.97 × 1024 kg,
Sun
jejari Bumi/radius of the Earth = 6.37 × 106 m,
G = 6.67 × 10–11 N m2 kg–2]
S A 7.06 × 103 m s–1
Rajah 2/ Diagram 2 B 7.91 × 103 m s–1
Pada kedudukan manakah, halaju Neptun C 9.81 × 103 m s–1
adalah maksimum? D 7.91 × 104 m s–1
At what position is the velocity of Neptune at its
15 Antara yang berikut, yang manakah
maximum?
merupakan faktor yang mempengaruhi halaju
A P C R
lepas suatu objek dari suatu planet?
B Q D S
Which of the following is the factor that affects the
11 Tempoh satelit A dalam suatu orbit bulatan escape velocity of an object from a planet?
berjejari R ialah T. Apakah tempoh satelit B A Jisim objek
dalam suatu orbit bulatan berjejari 4R? Mass of object
The period of satellite A in a circular orbit of a radius of B Jarak planet dari Matahari
R is T. What is the period of satellite B in a circular orbit Distance of the planet from the Sun
of a radius of 4R? KBAT Mengaplikasi C Jejari planet
3 Radius of planet
A 8T C 2T
D Jisim planet
B T D 8T Mass of planet

88

03 Strategi A+ SPM Fizik Tg 4 (B03) 5pp.indd 88 18/11/2023 9:55 AM


Kertas 2

Bahagian A

1 Rajah 1 menunjukkan dua orang kanak-kanak yang berjisim m1


dan m2 dipisahkan oleh jarak r. Daya graviti, F yang bertindak
F F
pada kanak-kanak tersebut boleh diterangkan oleh satu hukum. m1 m2
Diagram 1 shows two children with masses of m1 and m2 separated by a r
distance of r. The gravitational force, F acting on the children can be explained
by a law.

(a) Nyatakan hukum tersebut.


Rajah 1/ Diagram 1
State the law.
[Apa] Hukum Kegravitian Semesta Newton
[What] Newton's Universal Law of Gravitation

[1 markah/mark]

(b) Tandakan (3) pada kotak yang betul.


Mark (3) in the correct boxes.
Berdasarkan hukum di 1(a), daya graviti yang bertindak antara dua objek adalah
Based on the law in 1(a), the gravitational force acting on the object is

berkadar terus dengan hasil darab jisim kedua-dua objek, m1 dan m2


3
directly proportional to the product of masses of the two objects, m1 and m2

berkadar songsang dengan hasil darab jisim kedua-dua objek, m1 dan m2


inversely proportional to the product of masses of the two objects, m1 and m2

dan/and

berkadar terus dengan kuasa dua jarak di antara kedua-dua objek, r2


directly proportional to the square of distance between the two objects, r2

berkadar songsang dengan kuasa dua jarak di antara kedua-dua objek, r2


3
inversely proportional to the square of distance between the two objects, r2

[2 markah/marks]

(c) Jika jarak r bertambah, apakah yang akan berlaku kepada daya graviti, F? KBAT Mengaplikasi
If the distance of r is increased, what will happen to the gravitational force, F?
Berkurang/ Decreases
[1 markah/mark]

2 Rajah 2 menunjukkan sebuah roket yang membawa satelit mengalami


pecutan graviti, g = 9.81 m s–2 di permukaan Bumi sebelum dilancarkan.
Diagram 2 shows a rocket carrying a satellite experiencing acceleration due to gravity,
g = 9.81 m s−2 on the surface of the Earth before being launched.
[Jejari Bumi/Radius of the Earth, R = 6.37 × 106 m, pemalar kegravitian/
gravitational constant, G = 6.67 × 10-11 N m2 kg-2]

Rajah 2/ Diagram 2

89

03 Strategi A+ SPM Fizik Tg 4 (B03) 5pp.indd 89 18/11/2023 9:55 AM


(a) Apakah maksud daya graviti?
What is the meaning of gravitational force?
Daya graviti ialah daya yang bertindak antara dua jasad dalam alam semesta.
Gravitational force is the force acting between two bodies in the universe.

[1 markah/mark]

(b) Hitung jisim Bumi.


Calculate the mass of the Earth.
GM
g =
R2
(6.67 × 10–11)M
9.81 =
(6.37 × 106)2
M = 5.968 × 1024 kg
[2 markah/marks]

(c) Jika satelit tersebut dilancarkan ke ketinggian 480 km dari permukaan Bumi, hitung pecutan
graviti, g yang dialami oleh satelit itu.
If the satellite is launched to a height of 480 km from the surface of the Earth, calculate the gravitational acceleration,
g experienced by the satellite.
GM
g =
r2
(6.67 × 10–11)(5.968 × 1024)
=
(6.37 × 106 + 4.80 × 105)2
= 8.483 m s–2
[2 markah/marks]

3 Rajah 3 menunjukkan beberapa satelit buatan manusia dari pelbagai negara.


Diagram 3 shows a few man-made satellites from various countries.

Rajah 3/ Diagram 3

(a) Apakah yang dimaksudkan dengan satelit geopegun?


What is meant by geostationary satellites?
Satelit geopegun ialah satelit yang sentiasa berada di atas kedudukan geografi yang sama di permukaan
Bumi.
Geostationary satellites are satellites that are always above the same geographical position on the surface of the Earth.

[1 markah/mark]
(b) Nyatakan dua ciri satelit geopegun.
State two characteristics of a geostationary satellite.
• [Apa] Mengorbit mengelilingi Bumi dalam arah yang sama dengan putaran Bumi
[What] Orbits around the Earth in the same direction as the rotation of the Earth

• Tempoh orbit satelit ialah 24 jam, iaitu sama dengan tempoh putaran Bumi.
The orbital period of the satellites is 24 hours, that is the same as the rotation of the Earth.

[2 markah/marks]

90

03 Strategi A+ SPM Fizik Tg 4 (B03) 5pp.indd 90 18/11/2023 9:55 AM


(c) Sebuah satelit geopegun berjisim 1 200 kg diletakkan sejauh 4.23 × 107 m dari pusat Bumi.
A geostationary satellite with a mass of 1 200 kg is placed 4.23 × 107 m from the centre of the Earth.
[Pemalar kegravitian/Gravitational constant, G = 6.67 × 10–11 N m2 kg–2,
jisim Bumi/mass of the Earth = 5.97 × 1024 kg]
Hitung
Calculate
(i) laju linear satelit itu,
the linear speed of the satellite,
GM
v =
r
(6.67 × 10–11)(5.97 × 1024)
=
4.23 × 107
= 3.068 × 103 m s–1

[2 markah/marks]
(ii) daya tarikan graviti Bumi terhadap satelit itu.
the gravitational pull of the Earth on the satellite.
mv2
F =
r
(1 200)(3.068 × 103)2
=
4.23 × 107
= 267.025 N

[2 markah/marks]

(d) Jika laju satelit adalah kurang daripada laju linearnya, apakah yang berlaku kepada satelit?
If the speed of the satellite is lower than its linear speed, what will happen to the satellite? KBAT Mengaplikasi

Satelit akan jatuh ke orbit lebih rendah dan memasuki atmosfera Bumi. Geseran udara menghasilkan
haba yang menyebabkannya terbakar.
The satellite will fall to the lower orbit and enters the atmosphere of the Earth. Friction with air produces heat, causing it to burn.

[1 markah/mark]

4 Rajah 4.1 dan Rajah 4.2 masing-masing menunjukkan seorang atlet sedang menghayun tukul besi
pada laju yang sama.
Diagrams 4.1 and 4.2 show an athlete swinging iron hammers at the same speed respectively.
6 kg 6 kg 6 kg 6 kg

1.0 m 1.0 m 1.5 m 1.5 m

Rajah 4.1/ Diagram 4.1 Rajah 4.2/ Diagram 4.2

(a) Apakah yang dimaksudkan dengan daya memusat?


What is meant by centripetal force?
Daya memusat ialah daya yang membolehkan suatu objek bergerak mengikut laluan membulat dengan
arah yang sentiasa menuju ke pusat bulatan.
Centripetal force is a force that makes an object moves in a circular path with the direction always towards the centre of the circle.

[1 markah/mark]

91

03 Strategi A+ SPM Fizik Tg 4 (B03) 5pp.indd 91 18/11/2023 9:55 AM


(b) Berdasarkan Rajah 4.1 dan Rajah 4.2,/Based on Diagrams 4.1 and 4.2,
(i) bandingkan jisim tukul besi,/compare the mass of the iron hammers,
Sama/Same
[1 markah/mark]
(ii) bandingkan panjang tali tukul besi tersebut,/compare the length of strings of the iron hammers,
Panjang tali tukul besi dalam Rajah 4.1 lebih pendek daripada dalam Rajah 4.2.
The length of the string of the iron hammer in Diagram 4.1 is shorter than that of in Diagram 4.2.

[1 markah/mark]
(iii) bandingkan daya memusat dua tukul besi tersebut,
compare the centripetal forces of the two iron hammers,
Daya memusat tukul besi dalam Rajah 4.1 lebih besar daripada dalam Rajah 4.2.
The centripetal force of the iron hammer in Diagram 4.1 is larger than that of in Diagram 4.2.

[1 markah/mark]
(iv) nyatakan hubungan antara jejari bulatan dengan daya memusat.
state the relationship between the radius of circle and the centripetal force.
[Apa] Semakin besar jejari bulatan, semakin kecil daya memusat.
[What] The greater the radius of circle, the smaller the centripetal force.

[1 markah/mark]
(c) (i) Jika tukul besi diputarkan dengan laju seragam 20 m s–1, kira daya memusat bagi tukul besi
dalam Rajah 4.1./If the iron hammer is rotated with a uniform speed of 20 m s−1, calculate the centripetal
force of the iron hammer in Diagram 4.1.
mv2
F =
r
(6)(20)2
=
1.0
= 2 400 N
[2 markah/marks]
(ii) Jika tali tukul besi dipanjangkan, apakah yang akan terjadi kepada daya memusat
tukul besi? KBAT Mengaplikasi
If the string of the iron hammer is lengthened, what will happen to the centripetal force of the iron hammer?
Berkurang/Decreases
[1 markah/mark]
(iii) Terangkan jawapan anda di 4(c)(ii)./Explain your answer in 4(c)(ii). KBAT Menganalisis

[Mengapa] Daya memusat berkadar songsang dengan jejari bulatan (panjang tali)
[Why] Centripedal force is inversely proportional to radius of circle (length of string)

[1 markah/mark]

Bahagian B

5 (a) Rajah 5 menunjukkan sebuah satelit bukan


geopegun dalam Sistem Kedudukan Global
(GPS) yang sedang mengorbit Bumi.
Diagram 5 shows a non-geostationary satellite in the
Global Positioning System (GPS) which is currently
orbiting the Earth.
(i) Apakah yang dimaksudkan dengan satelit
bukan geopegun?
What is meant by non-geostationary satellites?
[1 markah/mark] Rajah 5/ Diagram 5

92

03 Strategi A+ SPM Fizik Tg 4 (B03) 5pp.indd 92 18/11/2023 9:55 AM


(ii) Beri empat ciri satelit bukan geopegun./ Give four characteristics of a non-geostationary satellite.
[4 markah/marks]

(b) Sebuah satelit berjisim 1.55 × 103 kg dilancarkan dari permukaan Bumi ke orbit dengan jejari
3.8 × 107 m. Bandingkan daya graviti yang bertindak ke atas satelit itu sebelum pelancaran
dengan daya graviti semasa satelit itu berada dalam orbit.
A satellite with a mass of 1.55 × 103 kg is launched from the surface of the Earth into orbit with a radius of
3.8 × 107 m. Compare the gravitational force acting on the satellite before launch to the gravitational force while
the satellite is in the orbit.
[Jisim Bumi/Mass of the Earth = 5.97 × 1024 kg, jejari Bumi/Radius of the Earth = 6.37 × 106 m, pemalar
kegravitian/gravitational constant, G = 6.67 × 10–11 N m2 kg–2]
[5 markah/marks]

(c) Kerajaan Malaysia ingin menghantar sebuah satelit telekomunikasi ke orbit mengelilingi
Bumi. Anda ditugaskan untuk mengkaji ciri satelit yang sesuai untuk tujuan tersebut.
Jadual 1 menunjukkan ciri-ciri empat model satelit, P, Q, R dan S yang boleh dipertimbangkan.
The Malaysian government wants to send a telecommunication satellite into orbit around the Earth. You are
assigned to study the characteristics of a satellite that is suitable for this purpose.
Table 1 shows the characteristics of four models of satellites, P, Q, R and S that can be considered.

Kedudukan geografi
Bahan badan satelit Jisim
Satelit satelit di atas Bumi Sumber kuasa
Material of Mass
Satellite Geographical position of Power source
body of satellite (kg)
satellite above the Earth

Muatan haba tentu Sentiasa di atas


Tenaga nuklear
P rendah Malaysia 1 000
Nuclear energy
Low specific heat capacity Always above Malaysia

Lokasi berlainan pada


Muatan haba tentu
masa berlainan Kerosin
Q tinggi 200
Different locations at Kerosene
High specific heat capacity
different times

Muatan haba tentu Sentiasa di atas


Sel-sel solar
R tinggi Malaysia 300
Solar cells
High specific heat capacity Always above Malaysia

Lokasi berlainan pada Cecair oksigen


Muatan haba tentu
S
masa berlainan dan hidrogen
tinggi 600
Different locations at Liquid oxygen
High specific heat capacity
different times and hydrogen

Jadual 1/ Table 1

Terangkan kesesuaian setiap ciri satelit dan tentukan model satelit yang paling sesuai.
Justifikasikan jawapan anda. KBAT Menganalisis KBAT Menilai
Explain the suitability of each characteristic of the satellite and determine the most suitable model of satellite. Justify your
answer.
[10 markah/marks]

Jawapan/ Answers:
5 (a) (i) Satelit bukan geopegun ialah satelit yang berada di atas kedudukan geografi yang berubah-ubah di
permukaan Bumi.
Non-geostationary satellites are satellites that are located above different geographical positions on the surface of the Earth.

(ii) [Mengapa/ Why]


– Tempoh orbit satelit boleh melebihi atau kurang daripada 24 jam.

93

03 Strategi A+ SPM Fizik Tg 4 (B03) 5pp.indd 93 18/11/2023 9:55 AM


The orbital period of satellite can be more than or less than 24 hours.

– Arah putaran satelit tidak semestinya sama dengan arah putaran Bumi.
The direction of the rotation of satellite is not necessarily the same as the direction of the rotation of the Earth.

– Biasanya berada dalam orbit lebih rendah atau lebih tinggi daripada orbit geopegun Bumi.
Usually in a lower or higher orbit than the geostationary orbit of the Earth.

– Orbit satelit tidak semestinya berada di atas Khatulistiwa.


The orbit of satellite is not necessarily above the equator.

(b) Sebelum pelancaran/Before launch:


r = R = 6.37 × 106 m
Daya graviti/Gravitational force, F = Gm1m2
r2

= (6.67 × 10 )(1.55 × 10 )(5.97 × 10 )


–11 3 24

(6.37 × 106)2

= 15 210.833 N
Berada di orbit/In the orbit:
r = R + h = 6.37 × 106 + 3.8 × 107 = 4.437 × 107 m
Daya graviti/Gravitational force, F = Gm1m2
r2

= (6.67 × 10 )(1.55 × 10 )(5.97 × 10 )


–11 3 24

(4.437 × 107)2

= 313.511 N
Daya graviti sebelum pelancaran lebih besar daripada daya graviti semasa satelit berada dalam orbit.
The gravitational force before launch is greater than the gravitational force after the satellite in the orbit.

(c) [Apa/ What, Mengapa/ Why]


Ciri-ciri/ Characteristic Penerangan/ Explanation

Muatan haba tentu tinggi Kadar peningkatan suhu badan satelit adalah rendah
High specific heat capacity The rate of increase in satellite body temperature is low

Sentiasa di Malaysia Satelit telekomunikasi ialah satelit geopegun


Always above Malaysia Telecommunication satellites are geostationary satellites

Menggunakan sel-sel solar Sumber tenaga boleh diperbaharui, jimat kos dan tidak
Use solar cells perlu ditukar dengan kerap
Energy source is renewable, cost-effective and no need to be changed

frequently

Jisim kecil Tidak memerlukan banyak tenaga semasa pelancaran ke


Small mass angkasa lepas
Does not require a lot of energy when launching to outer space

Satelit R dipilih kerana mempunyai muatan haba tentu tinggi, sentiasa di atas Malaysia, menggunakan
sel-sel solar dan berjisim kecil.
Satellite R is chosen because it has a high specific heat capacity, always above Malaysia, uses solar cells and has small mass.

94

03 Strategi A+ SPM Fizik Tg 4 (B03) 5pp.indd 94 18/11/2023 9:55 AM


Bab Haba
4 Heat
Tema: Haba


Revisi Pantas
The difference in the length of column between two points,
4.1 Keseimbangan Terma L is divided into 100 equal parts. Thus, the thermometer that
Thermal Equilibrium has been calibrated can be used to measure the temperature.
1 Haba ialah satu bentuk tenaga yang mengalir dari objek (f) Suhu suatu bahan boleh ditentukan dengan
yang bersuhu tinggi ke objek yang bersuhu rendah. menggunakan rumus berikut:
Heat is a form of energy that flows from a high-temperature object to The temperature of a substance can be determined by using
a low-temperature object. the following formula:
2 Suhu ialah darjah kepanasan atau kesejukan sesuatu
lθ – l0
objek. θ= × 100 °C
Temperature is the degree of hotness or coldness of an object. l100 – l0

3 Dua objek berlainan suhu yang bersentuhan secara dengan keadaan/ where
terma mencapai keseimbangan terma apabila syarat- θ = Suhu bahan/ Temperature of substance
syarat berikut dipenuhi: lθ = Panjang turus merkuri dalam bahan
Two objects of different temperatures that are in thermal contact Length of mercury column in the substance
reach thermal equilibrium when the following conditions are met: l0 = Panjang turus merkuri dalam air ais (0 °C)
(a) Pemindahan haba bersih antara dua objek ialah Length of mercury column in ice water (0 °C)
sifar. l100 = Panjang turus merkuri dalam air mendidih
The net heat transfer between the two objects is zero. (100 °C)
(b) Kedua-dua objek mempunyai suhu yang sama. Length of mercury column in boiling water (100 °C)
Both objects have the same temperature.

4 Cara menentu ukur termometer cecair dalam kaca Muatan Haba Tentu
menggunakan dua takat tetap: 4.2
The ways to calibrate a liquid-in-glass thermometer using two fixed Specific Heat Capacity
points:
l100 1 Muatan haba, C bagi suatu objek ialah kuantiti haba
yang diperlukan untuk menaikkan suhu objek itu
sebanyak 1 °C.
Heat capacity, C of an object is the quantity of heat needed to raise
l0 the temperature of the object by 1 °C.
A B
Q
(a) Letakkan termometer di dalam bikar yang C=
dipenuhi ais dan air. Perhatikan turus merkuri ∆θ
di dalam termometer sehingga tiada perubahan dengan keadaan/ where
yang berlaku. Q = Kuantiti haba yang dibekalkan
Place the thermometer in a beaker filled with ice and water. Quantity of heat supplied
Observe the mercury column in the thermometer until there ∆θ = Perubahan suhu
is no change. Change in temperature
(b) Tandakan takat turus merkuri seperti yang
ditunjukkan dalam rajah di atas sebagai A (0 °C). Unit bagi C ialah J ºC–1
Mark the level of the mercury column as shown in the diagram Unit for C is J ºC–1.
above as A (0 °C).
2 Muatan haba tentu, c bagi suatu bahan ialah kuantiti
(c) Letakkan termometer di dalam bikar berisi air
haba yang diperlukan untuk menaikkan suhu 1 kg
suling yang sedang mendidih. Perhatikan
bahan itu sebanyak 1 °C.
turus merkuri di dalam termometer sehingga Specific heat capacity, c of a substance is the quantity of heat
tiada perubahan yang berlaku. needed to raise the temperature of 1 kg of the substance by 1 °C.
Place the thermometer in a beaker filled with boiling distilled
water. Observe the mercury column in the thermometer until Q
there is no change. c=
m∆θ
(d) Tandakan takat turus merkuri seperti yang
ditunjukkan dalam rajah di atas sebagai dengan keadaan/ where
B (100 °C). Q = Kuantiti haba yang dibekalkan
Mark the level of the mercury column as shown in the diagram Quantity of heat supplied
above as B (100 °C). ∆θ = Perubahan suhu
(e) Perbezaan panjang turus di antara dua takat, L Change in temperature
dibahagi kepada 100 bahagian yang sama. m = Jisim bahan/Mass of substance
Maka, termometer yang telah ditentu ukur ini Unit bagi c ialah J kg–1 °C–1 atau J kg–1 K–1.
boleh digunakan untuk mengukur suhu. Unit for c is J kg–1 °C–1 or J kg–1 K–1.

95

04 Strategi A+ SPM Fizik Tg 4 (B04) 5pp.indd 95 18/11/2023 10:23 AM


4.3 Haba Pendam Tentu
Specific Latent Heat

1 Haba pendam ialah tenaga haba yang diperlukan untuk mengubah keadaan suatu bahan tanpa sebarang perubahan
suhu.
Latent heat is the heat energy required to change the phase of a substance without any change in temperature.

2 Proses perubahan fasa jirim/ Changes in phases of matter:

Peleburan Pendidihan
Melting Boiling

Pembekuan Kondensasi
Freezing Condensation

Pepejal Cecair Gas


Solid Liquid

Haba yang diserap semasa peleburan dan pendidihan tanpa perubahan suhu
Heat absorbed during melting and boiling without change in temperature
Haba pendam
Latent heat
Haba yang dibebaskan semasa kondensasi dan pembekuan tanpa perubahan suhu
Heat released during condensation and freezing without change in temperature

3 Haba pendam tentu, l bagi suatu bahan ialah kuantiti haba, Q yang diserap atau dibebaskan oleh 1 kg bahan itu semasa
perubahan fasa tanpa perubahan suhu.
Specific latent heat, l of a substance is the quantity of heat, Q that is absorbed or released by 1 kg of the substance during a change of phase without
any change in temperature.
Q
l=
m
Unit bagi l ialah J kg–1.
The unit for l is J kg−1.
Haba pendam tentu pelakuran, lf bagi suatu bahan ialah kuantiti haba, Q yang
diserap semasa peleburan atau dibebaskan semasa pembekuan oleh 1 kg bahan itu
tanpa perubahan suhu.
Specific latent heat of fusion, lf of a substance is the quantity of heat, Q that is absorbed during
Haba pendam melting or released during freezing by 1 kg of the substance without any change in temperature.
tentu
Specific latent heat Haba pendam tentu pengewapan, lv bagi suatu bahan ialah kuantiti haba, Q yang
diserap semasa pendidihan atau dibebaskan semasa kondensasi oleh 1 kg bahan itu
tanpa perubahan suhu.
Specific latent heat of vaporisation, lv of a substance is the quantity of heat, Q that is absorbed during
boiling or released during condensation by 1 kg of the substance without any change in temperature.

4.4 Hukum Gas


Gas Laws

Teori Kinetik Gas i-THINK Peta Pokok
Kinetic Theory of Gases

Tekanan gas Suhu gas Isi padu gas


Pressure of gas Temperature of gas Volume of gas

• Melibatkan bilangan molekul yang banyak Apabila gas dipanaskan, • Molekul gas bergerak
Involves a large number of molecules suhu meningkat dan bebas dan memenuhi
• Molekul sentiasa bergerak secara rawak pada tenaga kinetik purata keseluruhan ruang bekas
halaju tinggi dan berlanggar antara satu sama molekul meningkat. Gas molecules move freely
lain dan dengan dinding bekas When the gas is heated, the and occupy all the space in the
Molecules constantly moving at random at high velocity, temperature rises and the container
colliding with each other and with the wall of container average kinetic energy of gas • Isi padu gas sama dengan
• Daya dikenakan ke atas dinding bekas semasa molecules increases. isi padu bekas
perlanggaran Volume of gas is the same as
Force is exerted on the wall of container during collision volume of container
• Tekanan gas ialah daya per unit luas
Pressure of gas is force per unit area

96

04 Strategi A+ SPM Fizik Tg 4 (B04) 5pp.indd 96 18/11/2023 10:23 AM


i-THINK Peta Pokok
Hukum Gas
Gas Laws

Hukum Boyle Hukum Charles Hukum Gay-Lussac


Boyle’s Law Charles’s Laws Gay-Lussac’s Laws

Tekanan berkadar songsang Isi padu berkadar terus dengan Tekanan berkadar terus dengan
dengan isi padu bagi suatu gas suhu mutlak bagi suatu gas suhu mutlak bagi suatu gas
berjisim tetap pada suhu malar. berjisim tetap pada tekanan berjisim tetap pada isi padu
Pressure is inversely proportional malar. malar.
to volume for a fixed mass of gas at Volume is directly proportional to Pressure is directly proportional to
constant temperature. absolute temperature for a fixed mass of absolute temperature for a fixed mass of
gas at constant pressure. gas at constant volume.
P V Tekanan adalah malar Isi padu adalah malar
Pressure is constant Volume is constant
T

Suhu mutlak adalah malar P V P V


Absolute temperature is constant
T T

P1V1 = P2V2
V1 V2 P1 P2
= =
dengan keadaan/ where T1 T2 T1 T2
P = Tekanan gas (Pa)
Gas pressure dengan keadaan/ where dengan keadaan/ where
V = Isi padu gas (m3) T = Suhu mutlak (K) P = Tekanan gas (Pa)
Gas volume Absolute temperature Gas pressure
V = Isi padu gas (m3) T = Suhu mutlak (K)
Gas volume Absolute temperature

Praktis PBD
4.1 Keseimbangan Terma
Thermal Equilibrium

Latihan 1 Keseimbangan terma


Thermal equilibrium
TP 1 Mengingat kembali pengetahuan dan kemahiran sains mengenai Haba.
TP 2 Memahami Haba serta dapat menjelaskan kefahaman tersebut.
TP 3 Mengaplikasikan pengetahuan mengenai Haba untuk menerangkan kejadian atau fenomena alam dan melaksanakan tugasan mudah.

Isi tempat kosong dengan perkataan yang betul. TP 1 TP 2


Fill in the blanks with the correct words.

1
Objek/ Object A Objek/ Object B
80 ºC 30 ºC
Suhu adalah tinggi. Suhu adalah rendah.
Temperature is high. Temperature is low.

Apabila dua objek dengan suhu berbeza bersentuhan secara terma , suhu objek yang panas
akan menurun dan suhu objek yang sejuk akan meningkat .
When two objects of different temperatures are in thermal contact, the temperature of the hot object will
decrease and the temperature of the cold object will increase .

97

04 Strategi A+ SPM Fizik Tg 4 (B04) 5pp.indd 97 18/11/2023 10:23 AM


2
Objek/ Object A Objek/ Object B
50 ºC 50 ºC

Apabila suhu kedua-dua objek menjadi sama , pemindahan haba bersih antara dua objek itu
ialah sifar .
When the temperature of the two objects becomes equal , the net heat transfer between the two objects is
zero .

3 Senaraikan contoh keseimbangan terma dalam kehidupan harian. TP 3


List examples of thermal equilibrium in daily life.

Penyejukan/ Cooling Pemanasan/ Heating


(a) Memasukkan kiub ais ke dalam air panas (a) Menggoreng ikan
Putting ice cubes into hot water Frying fish

(b) Mandi untuk menyejukkan badan (b) Memanaskan badan dalam sauna
Taking bath to cool down body Heating body in the sauna

(c) Sejuk bekukan ayam di dalam peti sejuk (c) Mendidihkan air
Freezing chicken in the freezer Boiling water

Latihan 2 Menentu ukur termometer cecair


Calibrating liquid thermometers
TP 1 Mengingat kembali pengetahuan dan kemahiran sains mengenai Haba.
TP 2 Memahami Haba serta dapat menjelaskan kefahaman tersebut.
TP 3 Mengaplikasikan pengetahuan mengenai Haba untuk menerangkan kejadian atau fenomena alam dan melaksanakan tugasan mudah.
TP 4 Menganalisis pengetahuan mengenai Haba dalam konteks penyelesaian masalah mengenai kejadian atau fenomena alam.

1 Hurai dan lengkapkan langkah-langkah dalam menentu ukur termometer cecair. TP 4


Describe and complete the steps in calibrating the liquid thermometer.
l100 100 °C

l0
0 °C

(a) Menanda takat tetap bawah/Marking lower fixed point:


Letakkan termometer di dalam bikar berisi ais dan sedikit air suling. Tunggu sehingga tiada perubahan
pada paras turus cecair. Tanda paras turus cecair sebagai takat lebur ais (0 °C) (takat tetap bawah).
Place the thermometer in a beaker filled with ice and a small amount of distilled water. Wait until there is no change in the level

of the liquid column. Mark the level of the liquid column as the ice melting point (0 ° C) (lower fixed point).

(b) Menanda takat tetap atas/Marking upper fixed point:


Letakkan termometer di dalam bikar berisi air suling mendidih. Tunggu sehingga tiada perubahan
pada paras turus cecair. Tanda paras turus cecair sebagai takat didih air (100 °C) (takat tetap atas).
Place the thermometer in a beaker filled with boiling distilled water. Wait until there is no change in the level of the liquid column.

Mark the level of the liquid column as the boiling point of water (100 °C) (upper fixed point).

(c) Menanda skala/Marking the scale:


Jarak di antara dua takat itu ditanda dengan 100 bahagian yang sama.
The distance between the two points is marked with 100 equal parts.

98

04 Strategi A+ SPM Fizik Tg 4 (B04) 5pp.indd 98 18/11/2023 10:23 AM


(d) Jarak di antara dua tanda pada skala termometer mewakili 1 °C.
The distance between two marks on the thermometer scale represents 1 ºC.

(e) Rumus untuk menentukan suhu suatu bahan ialah


The formula to determine the temperature of a substance is

lθ – l0
θ= × 100 °C
l100 – l0
dengan keadaan/where
θ = Suhu bahan (°C)/ Temperature of substance (ºC)
lθ = Panjang turus cecair apabila suhu ialah θ °C (cm)/ Length of the liquid column when temperature is θ ºC (cm)
l0 = Panjang turus cecair apabila suhu ialah 0 °C (cm)/ Length of the liquid column when temperature is 0 ºC (cm)
l100 = Panjang turus cecair apabila suhu ialah 100 °C (cm)/ Length of the liquid column when temperature is 100 ºC (cm)

2 Selesaikan masalah yang berikut. TP 3


Solve the following problem.

Panjang turus merkuri di dalam sebuah termometer pada suhu 0 °C ialah 2.0 cm. Panjang turus
merkuri pada suhu 100 °C ialah 22.0 cm. Berapakah suhu suatu bahan jika panjang turus merkuri ialah
15.0 cm?
The length of the mercury column in a thermometer at 0 °C is 2.0 cm. The length of the mercury column at 100 °C is 22.0 cm.
What is the temperature of a substance if the length of mercury column is 15.0 cm?
15.0 – 2.0
θ= × 100 °C
22.0 – 2.0
13.0
= × 100 °C
20.0
= 65 °C

3 Proses penentu ukuran mengaplikasikan sifat termometrik yang ada pada cecair di dalam kaca.
Lengkapkan peta buih berikut dengan ciri-ciri cecair termometrik. TP 1 TP 2
The process of calibrating applies the thermometric properties of liquid in glass. Complete the following bubble map
with the characteristics of thermometric liquids. i-THINK Peta Buih

(a) (b)
Takat didih Tidak melekat pada
tinggi dinding tiub kapilari
High boiling Does not stick on
point the wall of capillary tube

(f)
Ciri-ciri
Mengembang
cecair (c)
dan mengecut
termometrik Konduktor
dengan cepat Characteristics of haba yang
Expand and contract thermometric baik
rapidly liquids
(d) Good heat
(e) Tidak Legap conductor
mengewap dan
Does not mudah dilihat
vaporise Opaque and
easily seen

99

04 Strategi A+ SPM Fizik Tg 4 (B04) 5pp.indd 99 18/11/2023 10:23 AM


4.2 Muatan Haba Tentu
Specific Heat Capacity

Latihan 3 Muatan haba dan muatan haba tentu


Heat capacity and specific heat capacity
TP 2 Memahami Haba serta dapat menjelaskan kefahaman tersebut.

1 Lengkapkan jadual berikut tentang muatan haba dan muatan haba tentu. TP 2
Complete the following table about heat capacity and specific heat capacity.

Muatan haba Muatan haba tentu


Heat capacity Specific heat capacity

Kuantiti haba yang diperlukan untuk Kuantiti haba yang diperlukan untuk
(a) Definisi menaikkan suhu objek sebanyak 1 °C menaikkan suhu 1 kg bahan sebanyak 1 °C
Definition The amount of heat required to raise the temperature The amount of heat required to raise the temperature
of the object by 1 °C of 1 kg of a substance by 1 °C

(b) Simbol
C c
Symbol

Q Q
C= c=
Δθ mΔθ
dengan keadaan/ where dengan keadaan/ where
(c) Rumus
Q = Kuantiti haba yang dibekalkan Q = Kuantiti haba yang dibekalkan
Formula
Amount of heat supplied Amount of heat supplied
Δθ = Perubahan suhu m = Jisim/Mass
Change in temperature Δθ = Perubahan suhu
Change in temperature

(d) Unit J °C–1 J kg–1 °C–1

2 Tandakan (3) pada pernyataan yang betul dan (7) pada pernyataan yang salah tentang muatan
haba tentu. TP 2
Tick (3) for the correct statements and (7) for the incorrect statements about specific heat capacity.

Pernyataan/ Statement 3/ 7
(a) Muatan haba, C ialah kuantiti tenaga haba yang diperlukan untuk menaikkan suhu suatu
bahan sebanyak 1 °C 3
Heat capacity, C is the quantity of thermal energy required to raise the temperature of a substance by 1 °C

(b) Muatan haba suatu objek tidak bergantung pada jisim dan jenis bahan objek itu
7
The heat capacity of an object does not depend on the mass and type of material of the object

(c) Unit bagi muatan haba ialah J kg–1 °C–1 atau J kg–1 K–1
7
The unit for heat capacity is J kg–1 °C–1 or J kg–1 K–1

(d) Muatan haba tentu air ialah 4 200 J kg–1 °C–1. Hal ini bermaksud 4 200 J tenaga diperlukan
untuk menaikkan suhu 1 kg air sebanyak 1 °C. 3
The specific heat capacity of water is 4 200 J kg–1 °C–1. This means 4 200 J of energy is required to raise the
temperature of 1 kg of water by 1 °C.
(e) Kuali diperbuat daripada logam yang mempunyai muatan haba tentu yang rendah. Hal
ini menyebabkan kuali panas dengan cepat. 3
The pan is made of metal which has a low specific heat capacity. This causes the pan to heat up quickly.
(f) Bahan yang mempunyai muatan haba tentu yang tinggi lambat dipanaskan dan
disejukkan 3
Materials that have a high specific heat capacity are slow to be heated up and cooled down

100

04 Strategi A+ SPM Fizik Tg 4 (B04) 5pp.indd 100 18/11/2023 10:23 AM


Latihan 4 Aplikasi muatan haba tentu
Application of specific heat capacity
TP 3 Mengaplikasikan pengetahuan mengenai Haba untuk menerangkan kejadian atau fenomena alam dan melaksanakan tugasan mudah.
TP 4 Menganalisis pengetahuan mengenai Haba dalam konteks penyelesaian masalah mengenai kejadian atau fenomena alam.

1 Nyatakan kegunaan muatan haba tentu dalam kehidupan harian. TP 3


State the use of specific heat capacity in daily life.

Muatan haba tentu rendah Muatan haba tentu tinggi


Low specific heat capacity High specific heat capacity
(a) Badan perkakas memasak (a) Pemegang perkakas memasak
Body of cooking utensils Handle of cooking utensils

(b) Minyak untuk menggoreng (b) Air sebagai penyejuk radiator kereta
Oil for frying Water as a car radiator coolant

Video
(c) Cecair termometrik (c) Bahan binaan bangunan menjadi kurang panas
Thermometric liquid pada waktu tengah hari dan panas pada waktu
malam
Building construction materials become less hot in the
afternoon and warm at night

2 Rajah berikut menunjukkan air digunakan sebagai cecair penyejuk di dalam sebuah enjin kereta.
The following diagram shows water being used as a coolant liquid in a car engine. TP 4

Pam Air
Pump Water
Video
Untuk tujuan pembelajaran
Imbas kod QR atau layari
Bilah sirip https://www.youtube.com/
Fin blade watch?v=_QnWneQJGk0
untuk menonton video
tentang penggunaan air
sebagai cecair penyejuk di
dalam radiator.

Kipas
Fan

Terangkan peranan air dalam sistem penyejukan enjin tersebut. Kemudian, beri sebab mengapa air
digunakan sebagai bahan penyejuk. KBAT Menganalisis
Explain the role of water in the car engine cooling system. Then, give the reasons why water is used as a coolant.
– Air digunakan sebagai medium untuk menyimpan dan membawa tenaga haba.
Water is used as a medium to store and carry heat energy.

– Air membawa keluar haba daripada enjin kereta ke persekitaran.


Water transfers heat from the engine to the surroundings.

– Air digunakan kerana mempunyai muatan haba tentu yang tinggi yang membolehkannya menyerap
tenaga haba yang banyak.
Water is used because it has a high specific heat capacity that allows it to absorb a lot of thermal energy.

– Air boleh menyerap tenaga haba yang banyak tanpa mendidih.


Water can absorb a lot of heat energy without boiling.

101

04 Strategi A+ SPM Fizik Tg 4 (B04) 5pp.indd 101 18/11/2023 10:23 AM


3 Isi tempat kosong dengan perkataan yang betul tentang bayu laut dan bayu darat. TP 4
Fill in the blanks with the correct words about sea and land breezes.

(a) Bayu laut/Sea breeze

Udara
panas
Hot air

Udara sejuk
Cold air

(i) Bayu laut berlaku pada waktu siang .


Sea breeze occurs during daytime .
(ii) Darat menyerap haba lebih cepat (muatan haba tentu tanah rendah ) dan
menjadi panas .
Land absorbs heat faster
(specific heat capacity of soil is low ) and becomes hot .
(iii) Laut menyerap haba lebih lambat (muatan haba tentu air tinggi ) dan menjadi
lebih sejuk .
Sea absorbs heat slower
(specific heat capacity of water is high ) and becomes cold .
(iv) Udara panas di atas darat naik dan udara sejuk dari laut bergerak ke darat ,
menghasilkan bayu laut.
Hot air above the land rises and cold air from the sea moves towards land , resulting in sea
breeze.

(b) Bayu darat/Land breeze

Udara sejuk
Cold air
Udara panas
Hot air

(i) Bayu darat berlaku pada waktu malam .


Land breeze occurs at night .
(ii) Darat kehilangan haba lebih cepat (muatan haba tentu tanah rendah ) dan
menjadi sejuk .
Land loses heat faster
(the specific heat capacity of soil is low ) and becomes cold .
(iii) Laut kehilangan haba lebih lambat (muatan haba tentu air tinggi ) dan menjadi
lebih panas .
Sea loses heat slower
(the specific heat capacity of water is high ) and becomes hot .
(iv) Udara panas di atas laut naik dan udara sejuk dari darat bergerak ke laut ,
menghasilkan bayu darat.
Hot air above the sea rises and cold air from the land moves towards the sea , resulting in
land breeze.

102

04 Strategi A+ SPM Fizik Tg 4 (B04) 5pp.indd 102 18/11/2023 10:23 AM


Latihan 5 Menyelesaikan masalah melibatkan muatan haba tentu
Solving problems involving specific heat capacity
TP 3 Mengaplikasikan pengetahuan mengenai Haba untuk menerangkan kejadian atau fenomena alam dan melaksanakan tugasan mudah.
TP 4 Menganalisis pengetahuan mengenai Haba dalam konteks penyelesaian masalah mengenai kejadian atau fenomena alam.

Selesaikan masalah yang berikut.


Solve the following problems.

1 Satu blok logam berjisim 400 g dipanaskan daripada 18 °C hingga 70 °C. Jika blok itu mempunyai
muatan haba tentu 800 J kg–1 °C–1, kira jumlah tenaga yang diserap. TP 3
A metal block with a mass of 400 g is heated from 18 °C to 70 °C. If the block has a specific heat capacity of 800 J kg−1 °C−1,
calculate the amount of energy absorbed.
Q = mc∆θ
= (0.4)(800)(70 – 18)
= 16 640 J

2 Satu blok logam berjisim 600 g dipanaskan dengan pemanas elektrik 150 W selama 2 minit. Suhu
blok logam itu meningkat daripada 20 °C kepada 86 °C. Kira muatan haba tentu logam itu. TP 3
A metal block with a mass of 600 g is heated by an electric heater at a power of 150 W for 2 minutes. The temperature of
the metal block increases from 20 °C to 86 °C. Calculate the specific heat capacity of the metal.
Pt = mc∆θ
Pt
c =
m∆θ
(150)(60 × 2)
c =
(0.6)(86 – 20)
= 454.545 J kg–1 °C–1

3 Apabila satu blok logam panas berjisim 0.5 kg diletakkan ke dalam 4 kg minyak, suhu minyak itu
meningkat sebanyak 25 °C. Jika muatan haba tentu minyak itu ialah 2 800 J kg–1 °C–1, berapakah
jumlah tenaga haba yang dibebaskan oleh blok logam itu? Nyatakan andaian anda untuk menjawab
soalan ini. TP 3 TP 4
When a hot metal block with a mass of 0.5 kg is placed in 4 kg of oil, the temperature of oil rises by 25 °C. If the specific heat
capacity of the oil is 2 800 J kg−1 °C−1, what is the amount of heat energy released by the metal block? State your
assumptions to answer this question.
Q = mc∆θ
= (4)(2 800)(25)
= 280 000 J
Semua haba daripada blok logam diserap oleh minyak. Tiada haba yang hilang ke persekitaran.
All heat from the metal block is absorbed by the oil. No heat is lost to the surroundings.

4.3 Haba Pendam Tentu


Specific Latent Heat

Latihan 6 Haba pendam tentu


Specific latent heat
TP 1 Mengingat kembali pengetahuan dan kemahiran sains mengenai Haba.
TP 2 Memahami Haba serta dapat menjelaskan kefahaman tersebut.
TP 3 Mengaplikasikan pengetahuan mengenai Haba untuk menerangkan kejadian atau fenomena alam dan melaksanakan tugasan mudah.
TP 4 Menganalisis pengetahuan mengenai Haba dalam konteks penyelesaian masalah mengenai kejadian atau fenomena alam.
TP 5 Menilai pengetahuan mengenai Haba dalam konteks penyelesaian masalah dan membuat keputusan untuk melaksanakan satu tugasan.

1 Gariskan jawapan yang betul. TP 1 TP 2


Underline the correct answers.

(a) Haba pendam ialah haba yang (diserap / dibebaskan) oleh suatu bahan semasa peleburan dan
pendidihan tanpa sebarang perubahan suhu.
Latent heat is the heat (absorbed / released) by a substance during melting and boiling without any change in
temperature.
103

04 Strategi A+ SPM Fizik Tg 4 (B04) 5pp.indd 103 18/11/2023 10:23 AM


(b) Haba pendam ialah haba yang (diserap / dibebaskan) oleh suatu bahan semasa kondensasi dan
pembekuan tanpa sebarang perubahan suhu.
Latent heat is the heat (absorbed / released) by a substance during condensation and freezing without any change in
temperature.

(c) (Haba pendam tentu / Muatan haba tentu) ialah kuantiti haba yang diserap atau dibebaskan
oleh 1 kg bahan semasa perubahan fasa tanpa sebarang perubahan suhu.
(Specific latent heat / Specific heat capacity) is the quantity of heat absorbed or released by 1 kg of substance during
phase change without any change in temperature.

2 Lengkapkan jadual berikut tentang perbandingan antara haba pendam tentu pelakuran dengan
haba pendam tentu pengewapan. TP 3
Complete the following table about the comparison between specific latent heat of fusion and specific latent heat of
vaporisation.

Haba pendam tentu pelakuran, lf Haba pendam tentu pengewapan, lv


Specific latent heat of fusion, lf Specific latent heat of vaporisation, lv

(i) Kuantiti haba yang diserap semasa (i) Kuantiti haba yang diserap semasa
peleburan untuk mengubah 1 kg pendidihan untuk mengubah 1 kg
pepejal kepada cecair cecair kepada gas
tanpa perubahan suhu tanpa perubahan suhu
The amount of heat absorbed during The amount of heat absorbed during
melting to change 1 kg of a solid boiling to change 1 kg of liquid

into liqiud without a change in into gas without a change in


temperature (a) Definisi temperature
(ii) Kuantiti haba yang dibebaskan Definition (ii) Kuantiti haba yang dibebaskan
semasa pembekuan untuk semasa kondensasi untuk
mengubah 1 kg cecair kepada mengubah 1 kg gas kepada
pepejal tanpa perubahan suhu cecair tanpa perubahan suhu
The amount of heat released during The amount of heat released during
freezing to change 1 kg of liquid condensation to change 1 kg of gas

into a solid without a change in into liquid without a change in


temperature temperature

(b) Unit
J kg–1 J kg–1
Unit

Q Q
lf = lv =
m m

dengan keadaan/ where (c) Rumus dengan keadaan/ where


m = Jisim bahan Formula m = Jisim bahan
Mass of substance Mass of substance
Q = Kuantiti haba yang diserap/ Q = Kuantiti haba yang diserap/
dibebaskan dibebaskan
Amount of heat absorbed/ released Amount of heat absorbed/ released

104

04 Strategi A+ SPM Fizik Tg 4 (B04) 5pp.indd 104 18/11/2023 10:23 AM


3 Rajah berikut menunjukkan lengkung pemanasan bagi suatu bahan. TP 4 TP 5
The following diagram shows the heating curve of a substance.

Suhu (°C)
Temperature
F

s s
ga ga
g an
tin as
of
ea an
m

as
Pe
H

G
Pendidihan

n
sa
Boiling
Takat D E

H ce na
liq ing ir
a
didih

d f
a
m

ui o
ea c
Cecair dan
Pe
Boiling
t
n gas
point sa
a na l Peleburan Liquid and
m ej f a Melting
Pe pep ing o B C qu air gas
c
Takat t id
e
a d
He soli
C
Li

lebur Pepejal dan


Melting cecair
jal
point pe Solid and
A Pe lid liquid
oS
Masa (minit)
0 Time

Nyatakan keadaan bahan tersebut yang diwakili oleh bahagian-bahagian berlabel pada graf tersebut.
State the phase of the substance represented by the labelled parts of the graph.

Haba pendam
Keadaan Haba diserap/
diserap/
jirim dibebaskan Tenaga kinetik Suhu
State of Heat is absorbed/
dibebaskan Kinetic energy Temperature
Latent heat is
matter released
absorbed/ released

Pepejal Haba diserap Bertambah Bertambah


AB –
Solid Heat is absord Increases Increases

Pepejal dan Haba pendam


Tidak berubah Tidak berubah
BC cecair – diserap Remain unchanged Remain unchanged
Solid and liquid Latent heat is absord

Cecair Haba diserap Bertambah Bertambah


CD –
Liquid Heat is absord Increases Increases

Haba pendam
Cecair dan gas Tidak berubah Tidak berubah
DE – diserap
Liquid and gas Remain unchanged Remain unchanged
Latent heat is absord

Gas Haba diserap Bertambah Bertambah


EF –
Gas Heat is absord Increases Increases

105

04 Strategi A+ SPM Fizik Tg 4 (B04) 5pp.indd 105 18/11/2023 10:23 AM


4 Rajah berikut menunjukkan lengkung penyejukan bagi suatu bahan. TP 4 TP 5
The following diagram shows the cooling curve of a substance.

Suhu (°C)
Temperature
Pe
ny
ej
Co ga uka
P oli s n
n
ga g o
s f
G Kondensasi Pe
as Condensation ny
e
Takat Co cec juk
Q ol ai an
didih Gas dan R liq ng r
i
Boiling ui o
cecair d f
point Gas and Ce Pembekuan
Li c Pe
liquid qu air Freezing n
Takat id pe yej
C
S oo pe uka
beku Cecair T
Freezing so ling jal n
dan l
id of
point
pepejal

Pe Sol
Liquid and

pe id
ja
solid U

l
0 Masa (minit)
Time

Nyatakan keadaan bahan tersebut yang diwakili oleh bahagian-bahagian berlabel pada graf tersebut.
State the phase of the substance represented by the labelled parts of the graph.

Haba pendam
Keadaan Haba diserap/
diserap/
jirim dibebaskan Tenaga kinetik Suhu
State of Heat is absorbed/
dibebaskan Kinetic energy Temperature
Latent heat is
matter released
absorbed/ released

Gas Haba dibebaskan Berkurang Berkurang


PQ –
Gas Heat is released Decreases Decreases

Haba pendam
Gas dan cecair dibebaskan Tidak berubah Tidak berubah
QR –
Gas and liquid Latent heat is Remain unchanged Remain unchanged
released

Cecair Haba dibebaskan Berkurang Berkurang


RS –
Liquid Heat is released Decreases Decreases

Haba pendam
Cecair dan
dibebaskan Tidak berubah Tidak berubah
ST pepejal –
Latent heat is Remain unchanged Remain unchanged
Liquid and solid
released

Pepejal Haba dibebaskan Berkurang Berkurang


TU –
Solid Heat is released Decreases Decreases

106

04 Strategi A+ SPM Fizik Tg 4 (B04) 5pp.indd 106 18/11/2023 10:23 AM


5 Terangkan mengapa seseorang yang berpeluh berasa sejuk apabila berada di bawah kipas yang
berputar. TP 4 KBAT Menganalisis
Explain why a person who is sweating feels cold when under a rotating fan.
– Haba pendam tentu pengewapan air diserap daripada badan.
Specific latent heat of vaporisation of water is absorbed from the body.

– Peluh menyerap haba dan menyejat.


Sweat absorbs heat and evaporates.

– Apabila terdapat pergerakan udara, kadar penyejatan bertambah.


When there is air movement, the rate of evaporation increases.

– Badan berasa sejuk selepas haba dibebaskan.


The body feels cold after the heat is released.

Latihan 7 Menyelesaikan masalah melibatkan haba pendam tentu


Solving problems involving specific latent heat
TP 3 Mengaplikasikan pengetahuan mengenai Haba untuk menerangkan kejadian atau fenomena alam dan melaksanakan tugasan mudah.
TP 4 Menganalisis pengetahuan mengenai Haba dalam konteks penyelesaian masalah mengenai kejadian atau fenomena alam.

Selesaikan masalah yang berikut.


Solve the following problems.

1 1 500 g cecair berubah menjadi wap selepas dipanaskan dengan 3 000 J tenaga. Berapakah haba
pendam tentu pengewapan cecair itu? TP 3
A 1 500 g of liquid turns into vapour after being heated with an energy of 3 000 J. What is the specific latent heat of
vaporisation of the liquid?
Q = mlv
Q
lv =
m
3 000
=
1.5
= 2 000 J kg–1

2 500 g kiub ais dipanaskan oleh sebuah pemanas rendam elektrik 780 W selama 3 minit. Ais yang
cair dikumpulkan di dalam bikar. Hitung TP 3
A 500 g of ice cubes are heated by an electric immersion heater with a power of 780 W for 3 minutes. The melted ice is
collected in a beaker. Calculate
(a) tenaga yang diserap oleh ais,
the energy absorbed by the ice,
(b) jisim ais yang cair di dalam bikar.
the mass of melted ice in the beaker.
[Haba pendam tentu pelakuran ais = 3.3 × 105 J kg–1]
[Specific latent heat of fusion of ice = 3.3 × 105 J kg−1]
(a) Q = Pt
= (780)(60 × 3)
= (780)(180)
= 140 400 J
(b) Q = mlf
Q
m =
lf
140 400
=
3.3 × 105
= 0.425 kg

107

04 Strategi A+ SPM Fizik Tg 4 (B04) 5pp.indd 107 18/11/2023 10:23 AM


3 900 g air pada suhu 50 °C dipanaskan sehingga menjadi wap pada suhu 100 °C. Keseluruhan proses
mengambil masa 1 minit. TP 4
A 900 g of water at a temperature of 50 °C is heated to become steam at a temperature of 100 °C. The whole process
takes 1 minute.
(a) Hitung jumlah tenaga yang diserap oleh air.
Calculate the amount of energy absorbed by the water.
(b) Berapakah kuasa alat pemanas yang memanaskan air itu?
What is the power of the heater that heats the water?
(c) Nyatakan andaian yang dibuat dalam pengiraan anda.
State the assumptions made in your calculations.
[Muatan haba tentu air = 4 200 J kg–1 °C–1, haba pendam tentu pengewapan air = 2.26 × 106 J kg–1]
[Specific heat capacity of water = 4 200 J kg−1 °C−1, specific latent heat of vaporisation of water = 2.26 × 106 J kg−1]
(a) Q = mc∆θ + mlv
= (0.9)(4 200)(100 – 50) + (0.9)(2.26 × 106)
= 189 000 + 2 034 000
= 2.223 × 106 J
(b) E = Pt
E
P =
t
2.223 × 106
=
60
= 37 050 W
(c) Semua haba daripada pemanas diserap oleh air. Tiada haba yang hilang ke persekitaran.
All the heat from the heater is absorbed by water. No heat is lost to the surroundings.

4.4 Hukum Gas


Gas Laws

Latihan 8 Hukum gas


Gas laws
TP 1 Mengingat kembali pengetahuan dan kemahiran sains mengenai Haba.
TP 2 Memahami Haba serta dapat menjelaskan kefahaman tersebut.
TP 3 Mengaplikasikan pengetahuan mengenai Haba untuk menerangkan kejadian atau fenomena alam dan melaksanakan tugasan mudah.
TP 4 Menganalisis pengetahuan mengenai Haba dalam konteks penyelesaian masalah mengenai kejadian atau fenomena alam.

1 Isi tempat kosong dengan perkataan yang betul. TP 1 TP 2


Fill in the blanks with the correct words.

(a) Menurut Teori Kinetik Gas, molekul gas bergerak secara rawak dan berlanggar
dengan dinding bekas dan melantun balik.
According to the Kinetic Theory of Gases, gas molecules move randomly and collide with the wall of
the container and bounce back.
(b) Hukum Boyle menyatakan bahawa pada suhu malar , tekanan berkadar songsang

dengan isi padu bagi suatu gas berjisim tetap .


Boyle’s Law states that at a constant temperature, pressure is inversely proportional to the volume of a
fixed mass of gas.
(c) Hukum Gay-Lussac menyatakan bahawa pada isi padu malar, tekanan berkadar terus
dengan suhu mutlak bagi suatu gas berjisim tetap.
Gay-Lussac’s Law states that at a constant volume , pressure is directly proportional to
absolute temperature for a fixed mass of gas.
(d) Apabila suhu suatu gas meningkat, tenaga kinetik molekul gas meningkat . Molekul gas

bergerak dengan halaju yang lebih tinggi dan tekanan yang bertindak ke atas dinding
bekas juga meningkat. Untuk tekanan gas kekal malar , isi padu gas mestilah bertambah
supaya kadar perlanggaran molekul dengan dinding bekas adalah malar .

108

04 Strategi A+ SPM Fizik Tg 4 (B04) 5pp.indd 108 18/11/2023 10:23 AM


As the temperature of a gas increases, the kinetic energy of the gas molecules increases . Gas molecules move
with higher velocity and the pressure acting on the wall of the container also increases. In order to
maintain a constant gas pressure, the gas volume must increase so that the rate of collision of the
molecules with the wall of the container is constant .
(e) Hukum Charles menyatakan bahawa pada tekanan malar, isi padu berkadar terus
dengan suhu mutlak bagi suatu gas berjisim tetap.
Charles’ Law states that at a constant pressure , volume is directly proportional to
absolute temperature for a fixed mass of gas.

2 Padankan hukum gas dengan graf yang betul. TP 2


Match the gas laws with the correct graphs.

(a) Hukum Boyle P


Boyle’s Law

• •

T (K)
0

(b) Hukum Charles P


Charles’ Law

• •

1
0 V

(c) Hukum Gay-Lussac V


Gay-Lussac’s Law

• •

T (K)
0

3 Lengkapkan peta titi berikut tentang hukum gas dan rumus berkaitan. TP 2
Complete the following bridge map about the gas laws and related formulae. i-THINK Peta Titi

Hukum Boyle
Boyle’s Law
Hukum gas Hukum Charles Hukum Gay-Lussac
Gas law as Charles’ Law as Gay-Lussac’s Law

Rumus P1 P2
V1 V2
Formula = =
P1V1 = P2V2 T1 T2 T1 T2

Keadaan Suhu adalah Tekanan Isi padu


Condition adalah adalah
malar . malar. malar.
Temperature is Pressure is Volume is
constant .
constant. constant.

(as = sama seperti)

109

04 Strategi A+ SPM Fizik Tg 4 (B04) 5pp.indd 109 18/11/2023 10:23 AM


4 Isi tempat kosong dengan perkataan yang betul. TP 3
Fill in the blanks with the correct words.

(a) Hukum Boyle


Boyle’s Law
Apabila omboh ditolak ke bawah pada suhu malar,
When the piston is pushed down
at a constant temperature ,
Omboh
(i) isi padu gas menjadi lebih kecil , Piston

the volume of the gas becomes smaller ,


(ii) kadar perlanggaran antara molekul dengan dinding
bekas meningkat ,
the rate of collision between the molecules and the wall of the
container increases ,
(iii) tekanan meningkat .
the pressure increases .

(b) Hukum Charles


Charles’ Law
Apabila suhu meningkat pada tekanan malar,
When temperature rises at a constant pressure ,
(i) molekul gas bergerak dengan lebih cepat,
the gas molecules move faster , Omboh
(ii) tenaga kinetik molekul meningkat , Piston

the kinetic energy of the molecules increases ,


(iii) perlanggaran antara molekul dengan dinding bekas
meningkat ,

the collision between the molecules and the wall of the container increases ,
(iv) isi padu meningkat apabila molekul menolak omboh.
the volume increases when the molecules push the piston.

(c) Hukum Gay Lussac


Gay-Lussac’s Law
Apabila suhu meningkat pada isi padu malar ,
When temperature rises at a constant volume,
Omboh
(i) molekul gas bergerak dengan lebih cepat , Piston

the gas molecules move faster ,


(ii) tenaga kinetik molekul meningkat ,

the kinetic energy of the molecules increases ,


(iii) perlanggaran antara molekul dengan dinding bekas
meningkat ,

collision between the molecules and the wall of the container increases ,
(iv) tekanan meningkat .
the pressure increases .

110

04 Strategi A+ SPM Fizik Tg 4 (B04) 5pp.indd 110 18/11/2023 10:23 AM


5 Periuk tekanan digunakan untuk memasak makanan.
Apabila periuk dipanaskan, suhu dan tekanan dalam
periuk juga meningkat. TP 4
Pressure cookers are used to cook food. When the cooker is heated, the
temperature and pressure in the cooker also increase.
Terangkan bagaimana tekanan bertambah apabila suhu
bertambah. KBAT Menganalisis
Explain how the pressure increases when temperature increases.
– Pada isi padu yang tetap, bilangan molekul gas per unit isi padu adalah malar.
At a fixed volume, the number of gas molecules per unit volume is constant.

– Jika suhu gas bertambah, tenaga kinetik molekul-molekul gas turut bertambah.
If the temperature of gas increases, the kinetic energy of gas molecules also increases.

– Molekul gas bergerak dengan lebih cepat dan kadar perubahan momentum turut meningkat.
The molecules of gas move faster and the rate of change of momentum also increases.

– Hal ini akan menambahkan kekerapan perlanggaran antara molekul gas dengan dinding bekas. Maka,
tekanan gas turut bertambah.
This will increase the frequency of collision between the gas molecules and the wall of the container. Hence, the gas pressure also

increases.

Latihan 9 Menyelesaikan masalah melibatkan hukum gas


Solving problems involving gas laws
TP 3 Mengaplikasikan pengetahuan mengenai Haba untuk menerangkan kejadian atau fenomena alam dan melaksanakan tugasan mudah.

Selesaikan masalah yang berikut.


Solve the following problems.

1 Sebiji belon diisi dengan 6 m3 gas helium pada tekanan 100 kPa. Apabila belon itu naik ke udara,
tekanan atmosfera berkurang dan belon itu mengembang. Jika suhu tidak berubah, berapakah
isi padu belon itu apabila tekanan atmosfera turun kepada 40 kPa? TP 3
A balloon is filled with 6 m3 of helium gas at a pressure of 100 kPa. When the balloon rises into the air, the atmospheric
pressure decreases and the balloon expands. If the temperature does not change, what is the volume of the balloon when
the atmospheric pressure drops to 40 kPa?
V1 = 6 m3, P1 = 100 kPa, P2 = 40 kPa
P1V1 = P2V2
(100 kPa)(6) = (40 kPa)V2
V2 = 15 m3

2 Sebiji periuk tekanan yang mengandungi 1.5 kg air mempunyai tekanan 1.0 × 105 Pa pada suhu
27 °C. Apabila periuk dipanaskan, tekanan dalam periuk meningkat sehingga air mendidih pada
suhu 120 °C. Hitung tekanan dalam periuk itu apabila air mendidih. TP 3
A pressure cooker containing 1.5 kg of water has a pressure of 1.0 × 105 Pa at a temperature of 27 °C. When the cooker
is heated, the pressure in the cooker increases until the water boils at a temperature of 120 ºC. Calculate the pressure in
the cooker when the water boils.
P1 = 1.0 × 105 Pa
T1 = 27 °C = (27 + 273) K = 300 K
T2 = 120 °C = (120 + 273) K = 393 K
P1 P2
=
T1 T2
P2 1.0 × 105
=
393 300
P2 = 1.310 × 105 Pa

111

04 Strategi A+ SPM Fizik Tg 4 (B04) 5pp.indd 111 18/11/2023 10:23 AM


3 Sebuah bekas tertutup mengandungi 100 cm3 gas pada tekanan 150 kPa. Bekas tersebut dimampatkan
sehingga isi padunya berkurang menjadi 50 cm3. Cari tekanan baharu bekas tersebut. TP 3
A closed container contains 100 cm3 of gas at a pressure of 150 kPa. The container is compressed until the volume of the
container is reduced to 50 cm3. Find the new pressure of the container.
V1 = 100 cm3, P1 = 150 kPa, V2 = 50 cm3
P1V1 = P2V2
(150 kPa)(100) = P2 (50)
P2 = 300 kPa

4 Sebuah bekas tertutup mengandungi 100 cm3 gas pada suhu 50 °C. Bekas itu disejukkan supaya
isi padunya berkurang menjadi 90 cm3. Cari suhu baharu bekas itu. TP 3
A closed container contains 100 cm3 of gas at a temperature of 50 °C. The container is cooled so that the volume is reduced
to 90 cm3. Find the new temperature of the container.
V1 = 100 cm3, V2 = 90 cm3
323
T1 = 50 °C = (50 + 273) K = 323 K T2 = 90�

100
V1 V2
= = 290.7 K
T1 T2 = (290.7 – 273) °C
100 90 = 17.7 °C
=
323 T2

Praktis Berformat SPM


Pernyataan manakah adalah betul apabila
Kertas 1 keseimbangan terma dicapai?
Which statement is correct when thermal equilibrium
1 Apakah konsep yang terlibat dalam is reached?
pengukuran suhu air kopi panas menggunakan A Suhu kek lebih tinggi daripada suhu
termometer? persekitaran
What is the concept involved in the measurement of the The temperature of the cake is higher than the
temperature of hot coffee drink using a thermometer? temperature of the surroundings
A Perolakan terma B Suhu kek lebih rendah daripada suhu
Thermal convection persekitaran
B Muatan haba tentu The temperature of the cake is lower than the
Specific heat capacity temperature of the surroundings
C Haba pendam tentu C Pengaliran haba bersih adalah daripada
Specific latent heat kek ke persekitaran
D Keseimbangan terma Net heat flow is from the cake to the surroundings
Thermal equilibrium D Pengaliran haba bersih antara kek dengan
persekitaran adalah sifar
2 Rajah 1 menunjukkan sepotong kek yang Net heat flow between the cake and the surroundings
dikeluarkan dari peti sejuk diletakkan di atas is zero
sebuah meja. Suhu awal kek itu ialah 3 °C.
Diagram 1 shows a piece of cake that is taken out 3 Mengapakah merkuri digunakan sebagai
from a refrigerator being placed on a table. The initial cecair di dalam termometer?
temperature of the cake is 3 °C. Why is mercury used as a liquid in thermometers?
A Takat didih yang rendah
Low boiling point
B Ketumpatan yang rendah
Low density
C Kebolehan membasahi tiub kaca
Ability to wet the glass tube
D Kebolehan untuk mengembang secara
sekata
Rajah 1/ Diagram 1 Ability to expand uniformly

112

04 Strategi A+ SPM Fizik Tg 4 (B04) 5pp.indd 112 18/11/2023 10:23 AM


4 Panjang turus merkuri, l, sebuah termometer B 800 J haba diperlukan untuk menaikkan
diukur ketika bebuli termometer diletakkan suhu logam Q sebanyak 1 °C
di dalam air pada suhu 0 °C dan 100 °C. 800 J of heat is required to increase the temperature
Jadual 1 menunjukkan bacaan yang diperoleh. of metal Q by 1 °C
The length of mercury column, l, of a thermometer is C 800 J haba diperlukan untuk menaikkan
measured when the thermometer bulb is placed in water suhu 1 kg logam Q sebanyak 1 °C
at temperatures of 0 °C and 100 °C. Table 1 shows the 800 J of heat is required to increase the temperature
readings obtained. of 1 kg of metal Q by 1 °C
Jadual 1/ Table 1
D 800 J haba diperlukan untuk menaikkan
suhu 1 g logam Q dari 0 °C hingga 100 °C
Suhu/ Temperature Panjang/ Length, l 800 J of heat is required to increase the temperature
(°C) (cm) of 1 g of metal Q from 0 °C until 100 °C

0 2.0 8 Mengapakah air sesuai digunakan sebagai


100 26.0 agen penyejuk?
Why is water suitable to be used as a cooling agent?
Panjang turus merkuri, l, meningkat secara A Air mempunyai ketumpatan yang tinggi
seragam dengan suhu. Jika bebuli diletakkan Water has a high density
di dalam air pada suhu 50 °C, berapakah B Air mempunyai muatan haba yang tinggi
nilai l? Water has a high heat capacity
The length of mercury column, l, increases uniformly C Air mempunyai haba pendam yang tinggi
with temperature. If the bulb is placed in water at a Water has a high latent heat
temperature of 50 °C, what is the value of l? D Air mempunyai daya lekatan antara
A 11.0 cm C 14.0 cm molekul yang tinggi
B 13.0 cm D 16.0 cm Water has a high adhesive force between molecules

5 30 000 J haba digunakan untuk meningkatkan 9 Rajah 2 menunjukkan lengkung pemanasan


suhu sebuah bongkah logam 2.0 kg daripada bagi suatu pepejal.
20 °C hingga 50 °C. Berapakah muatan haba Diagram 2 shows the heating curve of a solid.
tentu bongkah logam itu? Suhu (°C)
30 000 J of heat is used to raise the temperature of Temperature

a 2.0 kg metal block from 20 °C until 50 °C. What is the


specific heat capacity of the metal block?
A 375 J kg–1 °C–1 C 750 J kg–1 °C–1
Takat lebur
B 500 J kg–1 °C–1 D 1 200 J kg–1 °C–1 Melting point

6 Sebuah pemanas dengan spesifikasi Masa (s)


240 V, 3 000 W mengambil masa 30 saat 0 4 7 9 Time
untuk meningkatkan suhu suatu cecair yang Rajah 2/ Diagram 2
berjisim 1.5 kg sebanyak 25 °C. Berapakah
Berapakah masa yang diambil untuk semua
muatan haba tentu cecair itu?
pepejal itu melebur?
A heater with a specification of 240 V, 3 000 W takes
What is the time taken for all the solid to melt?
30 seconds to raise the temperature of a liquid with
A 2s
a mass of 1.5 kg by 25 °C. What is the specific heat
B 4s
capacity of the liquid?
C 7s
A 1.92 J kg–1 °C–1 C 240 J kg–1 °C–1
D 9s
B 192 J kg–1 °C–1 D 2 400 J kg–1 °C–1
10 Rajah 3 menunjukkan
7 0.50 kg air bersuhu
Muatan haba tentu bagi logam Q ialah
800 J kg–1 °C–1. 70 °C dituang ke dalam
The specific heat capacity of metal Q is segelas kiub ais bersuhu
800 J kg−1 °C−1. 0 °C.
Diagram 3 shows a 0.50 kg of
Apakah maksud pernyataan di atas? water at a temperature of 70 °C
What does the above statement mean? being poured into a glass of ice Rajah 3/ Diagram 3
A 800 J haba diperlukan untuk meleburkan cubes at a temperature of 0 °C.
1 kg logam Q
Berapakah jisim ais yang melebur?
800 J of heat is required to melt 1 kg of metal Q
What is the mass of the melting ice cubes?

113

04 Strategi A+ SPM Fizik Tg 4 (B04) 5pp.indd 113 18/11/2023 10:23 AM


0 Suhu (°C)
Temperature

[Haba pendam tentu pelakuran ais/ D Tekanan (kPa)


Specific latent heat of fusion of ice = Pressure
3.36 × 105 J kg–1, muatan haba tentu air
specific heat capacity of water = 4 200 J kg–1 °C−1]
A 0.04 kg C 0.4375 kg
B 0.08 kg D 0.5714 kg
0 Suhu (°C)
11 Rajah 4 menunjukkan suatu proses perubahan Temperature
fasa jirim.
13 Rajah 5 menunjukkan sebiji bola pingpong
Diagram 4 shows a process of changes in the phase of
yang kemek dengan isi padu 40.00 cm3. Bola
matter.
itu kemudiannya dimasukkan ke dalam
Pepejal/ Solid Cecair/ Liquid air mendidih untuk mengembalikannya ke
0 °C bentuk asal dengan isi padu 50.00 cm3.
Rajah 4/ Diagram 4 Diagram 5 shows a dented ping-pong ball with a
volume of 40.00 cm3. The ball then is placed in boiling
Apakah nama haba yang diserap semasa
water to return it to its original shape with a volume of
proses ini?
50.00 cm3.
What is the name of the heat absorbed during this
process? Bola
A Muatan haba tentu cecair pingpong Bola pingpong
Ping-pong ball
Specific heat capacity of liquid kemek
Dented
B Muatan haba tentu pepejal ping-pong Air panas
ball Boiling water
Specific heat capacity of solid
C Haba pendam tentu pelakuran Rajah 5/ Diagram 5
Specific latent heat of fusion Berapakah suhu udara dalam bola pingpong
D Haba pendam tentu pengewapan sebelum dimasukkan ke dalam air mendidih?
Specific latent heat of vaporisation What is the temperature of the air inside the ping-pong
ball before being placed in the boiling water?
12 Satu eksperimen dijalankan untuk menyiasat
KBAT Menganalisis
perubahan tekanan dengan suhu bagi suatu
A 25.4 °C C 31.2 °C
gas berjisim tetap dalam sebuah kelalang kon.
B 27.0 °C D 38.9 °C
Graf yang manakah menunjukkan hubungan
tekanan dengan suhu yang betul? 14 Suatu gas berjisim tetap pada tekanan malar
An experiment is carried out to investigate the change mempunyai isi padu 3V pada 45 °C. Jika gas
in pressure with temperature for a fixed mass of gas in a mengembang menjadi 6V, berapakah suhu
conical flask. Which graph shows the correct relationship gas?
Tekanan
between (kPa) and temperature?
pressure
Tekanan
Pressure(kPa) A fixed mass of gas at a constant pressure has a volume
A Pressure
Tekanan (kPa) of 3V at 45 °C. If the gas expands to 6V, what is the
Pressure
temperature?
A –123.0 °C C 327.0 °C
B 13.5 °C D 363.0 °C
0 Suhu (°C)
Suhu (°C)
0 Temperature 15
0
Temperature
Suhu (°C) Selepas suatu perjalanan yang jauh,
Temperature
tekanan udara dalam tayar sebuah kereta
Tekanan (kPa)
B Tekanan
Pressure (kPa) bertambah.
Pressure
Tekanan (kPa) After a long journey, the air pressure in the tyres
Pressure
of a car increases.

Apakah hukum gas yang menerangkan situasi


di atas?
0 Suhu (°C)
0 Suhu (°C)
Temperature What is the gas law that explains the situation above?
Temperature
0 Suhu (°C) A Hukum Gay-Lussac
C Tekanan (kPa) Temperature
Gay-Lussac’s Law
Tekanan
Pressure (kPa)
Pressure
Tekanan (kPa) B Hukum Boyle
Pressure Boyle’s Law
C Hukum Charles
Charles’ Law
D Hukum Snell
0 Suhu (°C)
0 Suhu (°C)
Temperature Snell’s Law
Temperature
Suhu (°C)
0
Temperature 114
Tekanan (kPa)
Tekanan
Pressure (kPa)
Pressure
Tekanan (kPa)
Pressure

04 Strategi A+ SPM Fizik Tg 4 (B04) 5pp.indd 114 18/11/2023 10:23 AM


Kertas 2

Bahagian A

1 Rajah 1 menunjukkan termometer klinik yang diletakkan di bawah lidah seorang budak lelaki yang
mengalami demam panas. Termometer klinik itu berfungsi berdasarkan keseimbangan terma.
Diagram 1 shows a clinical thermometer that is placed under the tongue of a boy who has fever. The clinical thermometer
works based on thermal equilibrium.

Pencerut/ Constriction

35 36 37 38 39 40 41 42

Rajah 1/ Diagram 1

(a) Apakah maksud suhu?


What is the meaning of temperature?
Suhu ialah darjah kepanasan suatu jasad, diukur dalam darjah Celsius.
Temperature is the degree of hotness of a body, measured in degree Celsius.

[1 markah/mark]

(b) Terangkan tujuan pencerut pada termometer klinik itu.


Explain the purpose of the constriction in the clinical thermometer.
• [Mengapa] Mengelakkan merkuri daripada mengalir balik ke bebuli semasa mengambil bacaan suhu
badan pesakit.
[Why] To prevent the mercury from flowing back into the bulb when taking the reading of a patient's body temperature.

• Oleh itu, bacaan suhu badan pesakit dapat diambil dengan lebih tepat.
Therefore, the reading of the patient's body temperature can be taken more accurately.

[2 markah/marks]

(c) Merkuri ialah cecair yang paling biasa digunakan dalam termometer cecair-dalam-kaca.
Mengapa? KBAT Mengaplikasi
Mercury is the most common liquid used in the liquid-in-glass thermometer. Why?
Tidak melekat pada dinding tiub kapilari/Legap dan mudah dilihat/Takat didih tinggi/Mengembang
dan mengecut dengan cepat/Tidak mengewap/Konduktor haba yang baik
Does not stick on the wall of capillary tube/ Opaque and easy to see/ High boiling point/ Expands and contracts rapidly/
Does not vaporise/ Good heat conductor

(Mana-mana 1 jawapan/Any 1 answer) [1 markah/mark]

2 Rajah 2.1 menunjukkan dua mangkuk sup yang dibiarkan di atas sebuah meja.
Diagram 2.1 shows two bowls of soup that are left on a table.

Mangkuk X: 400 g sup panas Mangkuk Y: 400 g sup sejuk


Bowl X: 400 g of hot soup Bowl Y: 400 g of cold soup

Rajah 2.1/ Diagram 2.1

115

04 Strategi A+ SPM Fizik Tg 4 (B04) 5pp.indd 115 18/11/2023 10:23 AM


Rajah 2.2 menunjukkan graf suhu, θ melawan masa, t bagi sup di dalam mangkuk X dan Y.
Diagram 2.2 shows the graphs of temperature, θ against time, t for the soup in bowls X and Y.
θ (°C) θ (°C)

80

28

Mangkuk X Mangkuk Y
Bowl X Bowl Y

28 5

0 t (min) 0 t (min)
5 10 15 20 25 5 10 15 20 25

Rajah 2.2/ Diagram 2.2

(a) Tandakan (3) pada kotak dengan frasa yang betul untuk melengkapkan ayat berikut.
Tick (3) in the box with the correct phrase to complete the following sentence.
Apabila suhu bertambah,
When the temperature increases,
tenaga kinetik molekul adalah tetap.
the kinetic energy of molecules is constant.

tenaga kinetik molekul bertambah.


3
the kinetic energy of molecules increases.

[1 markah/mark]

(b) Berdasarkan graf-graf dalam Rajah 2.2,


Based on the graphs in Diagram 2.2,
(i) bandingkan suhu awal sup di dalam mangkuk X dengan mangkuk Y.
compare the initial temperatures of the soup in bowl X and bowl Y.
Suhu awal sup di dalam mangkuk X lebih tinggi daripada mangkuk Y
The initial temperature of soup in bowl X is higher than that in bowl Y

[1 markah/mark]

(ii) bandingkan bagaimana suhu sup di dalam mangkuk X dan mangkuk Y berubah
dengan masa.
compare how the temperatures of the soup in bowl X and bowl Y change with time.
Suhu sup di dalam mangkuk X berkurang dengan masa. Suhu sup di dalam mangkuk Y bertambah
dengan masa.
The temperature of soup in bowl X decreases with time. The temperature of soup in bowl Y increases with time.

[1 markah/mark]

(iii) bandingkan pemindahan tenaga haba antara persekitaran dengan sup di dalam
mangkuk X dan mangkuk Y.
compare the transfer of heat energy between the surroundings and the soup in bowl X and bowl Y.
Sup di dalam mangkuk X membebaskan haba ke persekitaran. Sup di dalam mangkuk Y menyerap
haba daripada persekitaran.
The soup in bowl X releases heat to the surroundings. The soup in bowl Y absorbs heat from the surroundings.

[1 markah/mark]

116

04 Strategi A+ SPM Fizik Tg 4 (B04) 5pp.indd 116 18/11/2023 10:23 AM


(iv) hubung kait perubahan suhu sup dengan pemindahan haba yang berlaku.
relate the change in temperature of soup to the heat transfer that occurs.
[Bagaimana] Apabila suhu suatu objek berkurang, objek itu membebaskan haba. Apabila suhu
suatu objek bertambah, objek itu menyerap haba.
[How] When the temperature of an object increases, it absorbs heat. When the temperature of an object decreases, it
releases heat.

[2 markah/marks]

(c) Namakan konsep fizik untuk keadaan suhu sup di dalam mangkuk X dan mangkuk Y yang
menjadi malar pada 28 °C.
Name the physics concept for the condition where the temperatures of soup in bowl X and bowl Y become constant
at 28 °C.
[Apa/ What] Keseimbangan terma/ Thermal equilibrium
[1 markah/mark]

(d) Sup di dalam mangkuk X digantikan dengan minyak masak yang mempunyai jisim dan suhu
awal yang sama.
The soup in bowl X is replaced with cooking oil of the same mass and initial temperature.
(i) Nyatakan sama ada minyak masak mengambil masa yang lebih lama atau singkat untuk
menyejuk pada suhu 28 °C.
State whether the cooking oil takes a shorter or longer time to cool down to the temperature of 28 °C.
Masa lebih singkat/ Shorter time
[1 markah/mark]
(ii) Beri satu sebab bagi jawapan anda di 2(d)(i).
Give one reason for your answer in 2(d)(i).
[Mengapa] Minyak masak mempunyai muatan haba tentu yang lebih rendah.
[Why] Cooking oil has lower specific heat capacity.

[1 markah/mark]

3 Rajah 3.1 menunjukkan satu omboh ditolak dengan perlahan sehingga isi padu udara di dalam
silinder berkurang seperti yang ditunjukkan dalam Rajah 3.2. Udara di dalam silinder itu kekal
pada suhu malar.
Diagram 3.1 shows a piston being pushed slowly until the volume of air in the cylinder decreases as shown in
Diagram 3.2. The air in the cylinder remains at a constant temperature.

Udara
Air
Udara
Air

Ditolak
Pushed

Rajah 3.1/ Diagram 3.1 Rajah 3.2/ Diagram 3.2

117

04 Strategi A+ SPM Fizik Tg 4 (B04) 5pp.indd 117 18/11/2023 10:23 AM


(a) Berdasarkan Rajah 3.1 dan Rajah 3.2, nyatakan dua pemerhatian tentang isi padu udara dan
kadar perlanggaran antara molekul udara.
Using Diagrams 3.1 and 3.2, state two observations about the volume of air and the rate of collision between air
molecules.
Isi padu udara dalam Rajah 3.2 lebih kecil daripada dalam Rajah 3.1. Kadar perlanggaran antara
molekul udara dalam Rajah 3.2 lebih tinggi daripada dalam Rajah 3.1.
The volume of air in Diagram 3.2 is smaller than that of in Diagram 3.1. The rate of collision between air molecules in

Diagram 3.2 is higher than that of in Diagram 3.1.

[2 markah/marks]
(b) (i) Bandingkan tekanan udara dalam Rajah 3.1 dengan Rajah 3.2.
Compare the pressure of air in Diagrams 3.1 and 3.2.
Tekanan udara dalam Rajah 3.2 lebih tinggi daripada dalam Rajah 3.1.
The pressure of air in Diagram 3.2 is higher than that of in Diagram 3.1.

[1 markah/mark]
(ii) Nyatakan hubungan antara tekanan udara dengan isi padu udara.
State the relationship between the air pressure and the volume of air.
[Bagaimana] Apabila isi padu udara berkurang, tekanan udara bertambah.
[How] When the volume of air decreases, the pressure of air increases.

[1 markah/mark]
(iii) Berdasarkan jawapan anda di 3(b)(ii), namakan hukum gas yang terlibat.
Based on your answer in 3(b)(ii), name the gas law involved.
[Apa/ What] Hukum Boyle/Boyle’s Law
[1 markah/mark]
(iv) Apakah kuantiti fizik yang tetap dalam Rajah 3.1 dan Rajah 3.2?
What is the physical quantity that fixed in Diagrams 3.1 and 3.2?
Jisim gas/Mass of the gas
[1 markah/mark]

(c) Sebiji belon mengandungi 10 m3 gas helium pada tekanan 150 kPa. Apabila belon itu naik
ke langit, tekanan udara akan berkurang dan belon akan mengembang. Jika suhu tidak berubah,
berapakah isi padu belon itu apabila tekanan berkurang kepada 30 kPa?
A balloon contains 10 m3 of helium gas at a pressure of 150 kPa. As the balloon rises into the sky, the air pressure
will decrease and the balloon will expand. If the temperature does not change, what is the volume of the balloon
when the pressure decreases to 30 kPa?
P1V1 = P2V2
(150 × 10 )(10) = (30 × 103)V2
3

V2 = 50 m3
[3 markah/marks]

Bahagian C

4 Jadual 1 menunjukkan dua periuk yang sama saiz tetapi berlainan jenis dipanaskan di atas plat
pemanas elektrik berkuasa 800 W selama lima minit.
Table 1 shows two pots of the same size but different types being heated on electric hot plates with a power of 800 W
for five minutes.

118

04 Strategi A+ SPM Fizik Tg 4 (B04) 5pp.indd 118 18/11/2023 10:23 AM


Periuk
Pot

Periuk aluminium
Periuk aluminium Periuk kaca
Periuk kaca
Aluminium
Aluminium
pot pot Glass potGlass pot

Jisim
3 kg 3 kg
Mass
Suhu awal
30 °C 30 °C
Initial temperature
Suhu akhir
45 °C 37 °C
Final temperature
Muatan haba tentu
398 J kg–1 °C–1 980 J kg–1 °C–1
Specific heat capacity

Jadual 1/ Table 1

(a) Apakah maksud muatan haba tentu?/ What is the meaning of specific heat capacity?
[1 markah/mark]

(b) Berdasarkan Jadual 1, bandingkan jisim, perubahan suhu, jumlah haba yang dibekalkan oleh
plat pemanas dan muatan haba tentu periuk. Nyatakan hubungan antara muatan haba tentu
bahan dengan perubahan suhu bahan.
Based on Table 1, compare the mass, the change in temperature, the amount of heat supplied by the hot plate and
the specific heat capacity of the pot. State the relationship between the specific heat capacity of the substance
and the change in temperature of the substance.
[5 markah/marks]

(c) Rajah 4.1 menunjukkan sebuah termometer direndam


di dalam air sejuk.
Diagram 4.1 shows a thermometer immersed in cold water. Termometer
Terangkan bagaimana termometer itu mengukur suhu Thermometer

air sejuk. KBAT Mengaplikasi


Explain how the thermometer measures the temperature of cold
water.
[4 markah/marks]
Air sejuk
Cold water
(d) Rajah 4.2 menunjukkan sebuah belon udara panas.
Diagram 4.2 shows a hot-air balloon.
Anda dikehendaki mengemukakan beberapa Rajah 4.1/ Diagram 4.1
cadangan dalam reka bentuk sebuah belon udara
panas yang boleh membawa penumpang ke altitud
yang lebih tinggi dalam masa yang lebih singkat.
You are required to present a few suggestions in the design of Badan
a hot-air balloon which is able to carry passengers to a higher belon
Envelope
altitude in a shorter time. KBAT Menganalisis KBAT Menilai
Terangkan cadangan anda berdasarkan aspek yang
berikut:
Explain your suggestions based on the following aspects: Penunu gas
• Saiz badan belon/ Size of envelope Gas burner
Bakul
• Bilangan penunu/ Number of burners Basket
• Jenis bahan fabrik untuk badan belon Rajah 4.2/ Diagram 4.2
Type of fabric material for the envelope
• Jenis bahan untuk bakul/ Type of material for the basket
• Suhu nyalaan penunu gas/ Temperature of the flame of the gas burner
[10 markah/marks]
119

04 Strategi A+ SPM Fizik Tg 4 (B04) 5pp.indd 119 18/11/2023 10:23 AM


Jawapan/ Answers:
4 (a) Muatan haba tentu ialah kuantiti haba yang diperlukan untuk meningkatkan suhu 1 kg bahan sebanyak
1°C.
Specific heat capacity is the amount of heat needed to raise the temperature of 1 kg of substance by 1 °C.

(b) – Jisim periuk aluminium adalah sama dengan jisim periuk kaca
The mass of the aluminium pot is the same as the mass of the glass pot

– Perubahan suhu bagi periuk aluminium lebih besar daripada periuk kaca
The change in temperature of the aluminium pot is greater than that of the glass pot

– Jumlah haba yang dibekalkan kepada periuk aluminium adalah sama dengan jumlah haba yang
dibekalkan kepada periuk kaca
The amount of heat supplied to the aluminium pot is equal to the amount of heat supplied to the glass pot

– Muatan haba tentu bagi periuk aluminium kurang daripada periuk kaca
The specific heat capacity of the aluminium pot is lower than that of the glass pot

– Semakin rendah muatan haba tentu bahan, semakin tinggi perubahan suhu bahan.
The lower the specific heat capacity of the substance, the higher the change in temperature of the substance.

(c) – Suhu air sejuk lebih rendah daripada suhu termometer.


The temperature of the cold water is lower than the temperature of the thermometer.

– Haba mengalir daripada termometer ke air sejuk sehingga keseimbangan terma dicapai.
Heat flows from thermometer to cold water until thermal equilibrium is reached.

– Apabila haba mengalir daripada termometer, cecair termometer akan mengecut dan turus cecair akan
menurun.
When heat flows from thermometer, the thermometer liquid will contract and the liquid column will drop.

– Apabila keseimbangan terma dicapai, tiada lagi pemindahan haba berlaku. Turus cecair akan kekal
tidak berubah dan suhu air sejuk dapat dibaca.
When thermal equilibrium is reached, there is no more heat transfer. The liquid column will remain unchanged and the

temperature of cold water can be read.

(d) Ciri-ciri/ Characteristic Penerangan/ Explanation


Badan belon besar Menghasilkan tujahan ke atas yang besar/ Meningkatkan isi
Large envelope padu udara yang tersesar
Produces large upthrust/ Increases the volume of air displaced

Dua atau lebih banyak penunu Membekalkan lebih banyak haba/ Memanaskan gas dalam
Two or more burners badan belon dengan lebih cepat/ Untuk keselamatan
Supplies more heat/ Heats up the gas in the envelope faster/ For safety

Badan belon diperbuat daripada nilon Kuat, ringan dan tahan lama/ Kedap udara untuk
sintetik memerangkap udara panas
Envelope is made of synthetic nylon Strong, light and durable/ Airtight to trap hot air

Bakul rotan/ Rattan basket Ringan dan kuat/ Light and strong
Suhu nyalaan penunu gas yang tinggi Mengurangkan ketumpatan udara dalam badan belon
High temperature of the flame of the gas burner dengan lebih cepat
Reduces the density of air in the envelope more quickly

120

04 Strategi A+ SPM Fizik Tg 4 (B04) 5pp.indd 120 18/11/2023 10:23 AM


Bab Gelombang
5 Waves
Tema: Gelombang, Cahaya dan Optik

Revisi Pantas
A stationary wave is a wave in which the wave profile does not
5.1 Asas Gelombang
propagate with time. There is no transfer of energy in the medium.
Fundamentals of Waves An example of a stationary wave is the vibration of a guitar
string when it is being plucked.
1 Gelombang dihasilkan apabila satu sistem berayun
atau bergetar dalam satu medium. Video
Waves are produced when a system oscillates or vibrates in a
Untuk tujuan pembelajaran
medium.
Imbas kod QR atau layari https://www.youtube.
com/watch?v=6sgI7S_G-XI untuk menonton
2 Perambatan gelombang memindahkan tenaga dari video tentang getaran tali gitar.
satu titik ke titik yang lain tanpa memindahkan jirim

Video
medium itu.
The propagation of waves transfer energy from one point to another 6 Gelombang juga boleh dikelaskan kepada gelombang
without transferring the matter of the medium. mekanikal dan gelombang elektromagnetik.
Waves can also be classified as mechanical waves or electromagnetic
3 Gelombang boleh diklasifikasikan berdasarkan waves.
i-THINK Peta Pokok
perambatan profil gelombang.
Waves can be classified according to the propagation of wave Jenis gelombang
profile. Type of waves

Gelombang
pegun
Stationary
Gelombang mekanik Gelombang elektromagnet
Mechanical waves Electromagnetic waves
waves Gelombang
Gelombang melintang
Waves Transverse
Gelombang Contoh/ Examples: Contoh/ Examples:
waves • Udara/ Air • Cahaya/ Light
progresif
Progressive • Bunyi/ Sound • Radio/ Radio
waves Gelombang • Gelombang mikro
Microwave
membujur
Longitudinal • Sinar-X/ X-ray
waves
7 Istilah berkaitan dengan gelombang:
Terms related to waves:
4 Gelombang progresif ialah gelombang yang
mempunyai profil gelombang yang merambat Istilah Definisi
dengan masa dalam arah perambatan gelombang. Terms Definition
Perambatan profil gelombang progresif menunjukkan Amplitud, A Sesaran maksimum suatu zarah
pemindahan tenaga. Contoh gelombang progresif Amplitude dari kedudukan keseimbangan
ialah getaran pada permukaan air apabila sebiji batu The maximum displacement of a particle
dijatuhkan ke dalam sebuah tasik. from its equilibrium position
A progressive wave is a wave in which the wave profile propagate Tempoh, T Masa yang diambil oleh suatu
with time in the direction of propagation of the wave. The Period zarah untuk membuat satu ayunan
propagation of the wave profile indicates a transfer of energy. An lengkap
example of a progressive wave is the vibration on the water surface The time taken by a particle to make one
when a stone is dropped into a lake. complete oscillation
Frekuensi, f Bilangan ayunan lengkap suatu
Video
Frequency zarah dalam masa satu saat
Untuk tujuan pembelajaran
The number of complete oscillation by a
Imbas kod QR atau layari https://www.youtube.
particle in one second
com/watch?v=Yi3LW5riHfc untuk menonton
video tentang getaran pada permukaan air. Panjang Jarak di antara dua titik sefasa yang
gelombang, λ berturutan
Wavelength The distance between two consecutive points
5 Gelombang pegun ialah gelombang yang mempunyai in phase
profil gelombang yang tidak merambat dengan masa.
Laju Jarak yang dilalui sesaat oleh profil
Tiada pemindahan tenaga dalam medium. Contoh
gelombang, v gelombang
gelombang pegun ialah getaran tali gitar apabila ia Wave speed The distance travelled in one second by the
dipetik. wave

121

Strategi A+ SPM Fizik Tg4_05_updated.indd 121 18/11/2023 10:22:09 AM


8 Rumus yang berkaitan dengan gelombang:
Formulae related to waves:

1
v = fλ f=
T
9 Graf-graf gelombang:
Wave graphs:
(a) Graf sesaran-masa
Displacement-time graph

Sesaran (m)
Displacement

Puncak
Tempoh, T Crest
Period

Amplitud, A
Amplitude
Masa (s)
Time
Amplitud, A
Amplitude
Kedudukan
keseimbangan
Tempoh, T Equilibrium
Period position Lembangan
Trough

Graf sesaran melawan masa memberi maklumat tentang


The graph of displacement against time gives information on the
(i) amplitud/amplitude,
(ii) tempoh/ period,
(iii) frekuensi/ frequency
suatu gelombang.
of a wave.

(b) Graf sesaran-jarak


Displacement-distance graph

Sesaran (m)
Displacement

Panjang Puncak
gelombang, λ Crest
Wavelength
Amplitud, A
Amplitude
Jarak (m)
Distance
Amplitud, A
Amplitude
Kedudukan
keseimbangan
Panjang Equilibrium
gelombang, λ position
Wavelength Lembangan
Trough

Graf sesaran melawan jarak memberi maklumat tentang


The graph of displacement against distance gives information on the
(i) amplitud/ amplitude,
(ii) panjang gelombang/ wavelength
suatu gelombang.
of a wave.
The amplitude of the oscillating system becomes progressively
5.2 Pelembapan dan Resonans smaller because energy is being lost. The energy is lost due to friction
Damping and Resonance between the system and the air particles.

3 Amplitud yang berkurangan bagi suatu sistem ayunan


1 Frekuensi asli ialah frekuensi bagi suatu sistem dengan disebabkan oleh kehilangan tenaga dikenali sebagai
keadaan sistem berayun apabila tiada daya luar pelembapan.
dikenakan ke atasnya. The decrease in amplitude of the oscillating system due to energy
The natural frequency is the frequency of a system at which the loss is known as damping.
system oscillates when there is no external force acting on it.
4 Terdapat dua jenis pelembapan:
2 Amplitud bagi sistem ayunan menjadi semakin There are two types of damping:
kecil kerana tenaga semakin hilang. Tenaga hilang (a) Pelembapan dalaman
disebabkan berlakunya geseran antara sistem dengan Internal damping
zarah-zarah udara. (b) Pelembapan luaran
External damping

122

Strategi A+ SPM Fizik Tg4_05_updated.indd 122 18/11/2023 10:22:09 AM


5 Apabila suatu sistem ayunan dikenakan daya luar 2 Laju, panjang gelombang dan arah perambatan
yang sama frekuensi dengan frekuensi aslinya, sistem gelombang berubah apabila ia dibiaskan, manakala
dikatakan berayun dengan amplitud maksimum. frekuensi gelombang kekal tidak berubah.
Fenomena ini disebut sebagai resonans. The speed, wavelength and direction of propagation of the waves
When an oscillating system is applied to an external force of the same change when they are refracted, while the frequency of the wave
frequency as its natural frequency, the system is said to oscillate with remains unchanged.
maximum amplitude. This phenomenon is called resonance.
3 Perubahan arah perambatan gelombang adalah
5.3 Pantulan Gelombang disebabkan oleh perubahan halaju gelombang semasa
Reflection of Waves gelombang bergerak melalui medium yang berlainan
sifat.
1 Pantulan gelombang berlaku apabila gelombang tuju The change in the direction of propagation of waves is caused by the
melanggar pemantul dan terpantul semula. change in the speed of the waves while they travel through mediums
Reflection of waves occurs when incident waves hit a reflector and of different properties.
bounce back.
4 Pembiasan gelombang air bergantung pada
2 Arah perambatan gelombang berubah apabila ia kedalaman air.
dipantulkan. Panjang gelombang, frekuensi dan laju The refraction of water waves depends on the depth of water.
gelombang kekal tidak berubah.
The direction of propagation of the waves changes when it is reflected. 5 Pembiasan gelombang bunyi bergantung pada
The wavelength, frequency and speed of waves remain unchanged. ketumpatan udara.
3 Pantulan gelombang mematuhi hukum pantulan The refraction of sound waves depends on the density of air.
gelombang, iaitu
Reflection of waves obeys the law of reflection of waves, that is 6 Pembiasan gelombang cahaya bergantung pada
(a) sudut tuju adalah sama dengan sudut pantulan, ketumpatan optik medium.
the angle of incidence is equal to the angle of reflection, The refraction of light waves depends on the optical density of
(b) gelombang tuju, gelombang pantulan dan normal medium.
semuanya terletak pada satah yang sama.
the incident wave, the reflected wave and the normal are all lie Pembelauan Gelombang
on the same plane.
5.5
Diffraction of Waves
Garis normal
Normal 1 Pembelauan gelombang ialah penyebaran gelombang
Gelombang tuju λ λ Gelombang pantulan apabila ia bergerak melalui satu bukaan atau satu
Incident wave Reflected wave
halangan kecil.
Diffraction of waves is the spreading of waves when they travel
through an opening or a small obstacle.

i r
2 Panjang gelombang, frekuensi dan laju gelombang tidak
berubah, tetapi amplitud gelombang berkurang selepas
pembelauan gelombang. Arah perambatan gelombang
Pemantul satah
Plane reflector
berubah dari satu arah ke banyak arah.
The wavelength, frequency and speed of waves do not change, but
the amplitude of waves decreases after diffraction of waves. The
Definisi/ Definition
direction of propagation of waves changes from one direction to many
Sudut tuju, i : Sudut di antara arah
directions.
Angle of incidence perambatan gelombang tuju
dengan garis normal 3 Kesan pembelauan bergantung pada saiz celah dan
The angle between the direction of panjang gelombang.
propagation of the incident wave and The effect of the diffraction depends on the size of slit and the
the normal wavelength.
Sudut pantulan, r : Sudut di antara arah
Angle of reflection perambatan gelombang pantulan Interferens Gelombang
dengan garis normal 5.6
Interference of Waves
The angle between the direction of
propagation of the reflected wave and
the normal 1 Prinsip superposisi menyatakan bahawa sesaran paduan
Garis normal : Garis yang berserenjang dengan dua gelombang yang bersuperposisi ialah hasil tambah
Normal pemantul satah sesaran individu bagi gelombang-gelombang itu.
A line perpendicular to a plane reflector Principle of superposition states that the resultant displacement of
two waves in superposition is the sum of the individual displacements
of the waves.

5.4 Pembiasan Gelombang 2 Interferens gelombang ialah kesan superposisi bagi


Refraction of Waves dua gelombang koheren.
Interference of waves is the effect of the superposition of two
1 Pembiasan gelombang ialah perubahan arah coherent waves.
perambatan gelombang yang disebabkan oleh
3 Gelombang koheren merujuk kepada gelombang-
perubahan halaju gelombang semasa gelombang
gelombang yang mempunyai frekuensi yang sama dan
merambat dari satu medium ke medium yang lain.
Refraction of waves is the change in the direction of propagation
beza fasa adalah tetap.
Coherent waves refer to waves that have the same frequency and the
of waves caused by the change in the velocity of waves when they
phase difference is constant.
propagate from one medium to another.

123

Strategi A+ SPM Fizik Tg4_05_updated.indd 123 18/11/2023 10:22:09 AM


4 Terdapat dua jenis interferens gelombang: Electromagnetic waves consist of both electric fields and magnetic
There are two types of interference of waves: fields that oscillate perpendicularly to each other and to the direction
(a) Interferens membina of the propagation of waves.
Constructive interference
2 Ciri-ciri gelombang elektromagnet:
(b) Interferens memusnah Characteristics of electromagnetic waves:
Destructive interference
(a) Gelombang melintang
5 Interferens membina berlaku apabila dua puncak Transverse waves
atau dua lembangan bersuperposisi dan menghasilkan (b) Terdiri daripada medan elektrik dan medan
gelombang dengan puncak atau lembangan dengan magnet yang berayun secara berserenjang antara
amplitud maksimum. satu sama lain
Constructive interference occurs when two crests or two troughs Consist of electric field and magnetic field that oscillate
are in a superposition and produce a wave with a crest or trough of perpendicular to each other
maximum amplitude. (c) Tidak memerlukan medium untuk merambat
Do not need medium to propagate
6 Interferens memusnah berlaku apabila satu puncak (d) Boleh memindahkan tenaga dari satu tempat ke
dan satu lembangan bersuperposisi menghasilkan tempat lain
amplitud sifar. Can transfer energy from one place to another
Destructive interference occurs when a crest and a trough are in a
(e) Bergerak pada kelajuan cahaya dalam vakum,
superposition, resulting in zero amplitude.
c = 3.0 × 108 m s-1
Moves at the speed of light in vacuum, c = 3.0 × 108 m s-1
5.7 Gelombang Elektromagnet (f) Mematuhi persamaan gelombang, v = fλ
Electromagnetic Waves Obey wave equation, v = f λ
(g) Boleh merambat melalui vakum
1 Gelombang elektromagnet terdiri daripada kedua- Can propagate through vacuum
dua medan elektrik dan medan magnet yang berayun (h) Menunjukkan sifat-sifat gelombang iaitu pantulan,
secara berserenjang antara satu sama lain pada arah pembiasan, pembelauan dan interferens
perambatan gelombang. Show the properties of waves which are reflection, refraction,
diffraction and interference

Praktis PBD
5.1 Asas Gelombang
Fundamentals of Waves
Latihan 1 Penerangan tentang gelombang
Description of waves
TP 1 Mengingat kembali pengetahuan dan kemahiran sains mengenai Gelombang.
TP 2 Memahami Gelombang serta dapat menjelaskan kefahaman tersebut.
TP 3 Mengaplikasikan pengetahuan mengenai Gelombang untuk menerangkan kejadian atau fenomena alam dan melaksanakan tugasan mudah.

1 Lengkapkan definisi gelombang berikut. TP 1


Complete the following definition of waves.
Gelombang ialah gangguan yang memindahkan tenaga antara dua titik melalui getaran
zarah tanpa pemindahan jirim .
A wave is a disturbance that transfers energy between two points through the vibration of particles without
the transfer of matter .

2 Lengkapkan perbincangan dan kesimpulan bagi aktiviti berikut. TP 2 TP 3


Complete the discussion and conclusion for the following activity.
Aktiviti: Gelombang memindahkan tenaga tanpa memindahkan jirim
Activity: Waves transfer energy without transferring matter
Hujung B
Hujung A ditetapkan
digerakkan Reben Spring slinki End B
End A is moved Ribbon Slinky spring is fixed

Meja
Table

124

Strategi A+ SPM Fizik Tg4_05_updated.indd 124 18/11/2023 10:22:10 AM


Prosedur/ Procedure:
1. Reben diikat pada spring slinki.
The ribbon is tied to the slinky spring.
2. Hujung A dan B dipegang oleh dua orang murid.
The ends A and B are held by two students.
3. Hujung A digerakkan ke atas dan ke bawah, manakala hujung B ditetapkan pegun.
End A is moved up and down, while end B is held stationary.
4. Gerakan gelombang yang dibentuk sepanjang spring slinki dan reben diperhatikan.
The movements of the waves formed along the slinky spring and the ribbon are observed.
Perbincangan/ Discussion:
(a) Gelombang terhasil apabila spring slinki digerakkan.
Wavesare produced when the slinky spring is moved.
(b) Murid di hujung B dapat merasakan getaran spring slinki .
Student at end B can feel the vibration of slinky spring .
(c) Arah pemindahan tenaga adalah dari kiri ke kanan .
The direction of energy transfer is from left to right .
(d) Reben yang mewakili jirim tidak dipindahkan ke B.
The ribbon that represents matter is not transferred to B.
Kesimpulan/ Conclusion:
Gelombang memindahkan tenaga tanpa memindahkan jirim .
Waves transfer energy without transferring matter .

Latihan 2 Jenis gelombang


Types of waves
TP 1 Mengingat kembali pengetahuan dan kemahiran sains mengenai Gelombang.
TP 2 Memahami Gelombang serta dapat menjelaskan kefahaman tersebut.
TP 3 Mengaplikasikan pengetahuan mengenai Gelombang untuk menerangkan kejadian atau fenomena alam dan melaksanakan tugasan mudah.

1 Lengkapkan jenis gelombang yang berikut berdasarkan aspek yang dinyatakan. TP 1 TP 2


Complete the following types of waves based on the aspects stated.
i-THINK Peta Pokok
(a) Aspek: Perambatan profil gelombang
Aspect: Propagation of the wave profile

Gelombang progresif Gelombang pegun


Progressive waves Stationary waves

Definisi/ Definition: Definisi/ Definition:

Profil gelombang merambat dengan masa Profil gelombang tidak merambat


sepanjang arah perambatan gelombang . dengan masa .
The wave profile propagates with time along The wave profile does not propagate
the direction of wave propagation . with time .

Merambat melalui suatu medium sebagai


Propagate through a medium as

Gelombang melintang Gelombang membujur


Transverse waves Longitudinal waves

125

Strategi A+ SPM Fizik Tg4_05_updated.indd 125 18/11/2023 10:22:10 AM


(b) Aspek: Medium
Aspect: Medium

Gelombang mekanik Gelombang elektromagnet


Mechanical waves Electromagnetic waves

(i) Memerlukan medium untuk (i) Tidak memerlukan medium


memindahkan tenaga dari satu untuk memindahkan tenaga
titik ke titik yang lain. dari satu titik ke titik yang lain.
Requires a medium to transfer Does not require a medium to transfer
energy
from one point to another. energy from one point to another.
(ii) Terdiri daripada getaran zarah-zarah
(ii) Terdiri daripada ayunan medan elektrik
medium.
particles dan medan magnet yang berserenjang
Consists of the vibration of medium .
antara satu sama lain.
(iii) Contoh/ Example: electric
Consists of field and magnetic
Gelombang air, gelombang bunyi
field that oscillate perpendicular to each other.
Water waves, sound waves
(iii) Contoh/ Example:
Gelombang radio, gelombang cahaya
Radio waves, light waves

2 Rajah di bawah menunjukkan perambatan gelombang melintang dan gelombang membujur.


The diagrams below show the propagation of transverse wave and longitudinal wave.
Label rajah tersebut dengan perkataan yang betul. TP 2
Label the diagrams with the correct words.

Mampatan/ Compression Lembangan/ Trough Renggangan/ Rarefaction Puncak/ Crest

(a) Gelombang melintang


(i) Puncak Transverse waves (ii) Lembangan
Crest Trough

(iii) Puncak
Crest

Arah getaran Arah perambatan


zarah Direction of
Direction of propagation of the waves
vibration of
particles

(iv) Lembangan
Trough

(b) Gelombang membujur


Arah getaran zarah Arah perambatan
Direction of vibration of particles Longitudinal waves Direction of propagation of the waves

(i) Renggangan (ii) Mampatan (iii) Renggangan (iv) Mampatan (v) Renggangan
Rarefaction Compression Rarefaction Compression Rarefaction

126

Strategi A+ SPM Fizik Tg4_05_updated.indd 126 18/11/2023 10:22:11 AM


3 Lengkapkan peta buih berganda berikut tentang perbandingan antara gelombang melintang dengan
gelombang membujur.
Complete the following double bubble map about the comparison between transverse waves and longitudinal waves. TP 3

i-THINK Peta Buih Berganda

(a)
Zarah-zarah (e)
medium bergetar dalam Zarah-zarah
arah yang berserenjang medium bergetar dalam
arah yang selari
dengan arah perambatan
(d) dengan arah perambatan
gelombang.
Gelombang
gelombang.
The particles of the medium vibrate progresif
in a direction perpendicular The particles of the medium vibrate
Progressive
in a direction parallel to
to the direction of wave waves

Laman Web
propagation. the direction of wave
propagation.

(b)
(f)
Terdiri daripada
Gelombang Gelombang Terdiri daripada
puncak dan
melintang membujur mampatan dan
lembangan .
Transverse Longitudinal renggangan .
Consists of crest waves waves
Consists of compression
and trough .
and rarefaction .

(c) (g)

Laman Web
Contoh/ Example: Contoh/ Example:
Gelombang radio, Gelombang bunyi
gelombang cahaya, gelombang Sound waves Imbas kod QR atau
layari https://www.
air/ Radio waves, acs.psu.edu/drussell/

Untuk tujuan pembelajaran


Demos/waves/
light waves, water waves wavemotion.html
untuk mempelajari
tentang perambatan
gelombang melintang
dan gelombang
membujur.
Latihan 3 Ciri-ciri gelombang
Characteristics of waves
TP 1 Mengingat kembali pengetahuan dan kemahiran sains mengenai Gelombang.
TP 2 Memahami Gelombang serta dapat menjelaskan kefahaman tersebut.
TP 3 Mengaplikasikan pengetahuan mengenai Gelombang untuk menerangkan kejadian atau fenomena alam dan melaksanakan tugasan mudah.

1 Berdasarkan rajah, lengkapkan jadual berikut. TP 1 TP 2


Based on the diagrams, complete the following table.
(a) Gelombang melintang (b) Gelombang membujur
Transverse waves Longitudinal waves
Puncak R M R M R
Crest
Panjang gelombang
Wavelength
Kedudukan keseimbangan Panjang gelombang
Amplitud Equilibrium position Wavelength
Amplitude
Panjang gelombang
Wavelength

Panjang gelombang M = Mampatan/ Compression


Wavelength R = Renggangan/ Rarefaction
Lembangan
Trough

127

Strategi A+ SPM Fizik Tg4_05_updated.indd 127 18/11/2023 10:22:11 AM


Istilah/ Rumus Simbol Unit S.I. Definisi
Term/ Formula Symbol S.I. unit Definition

(a) Amplitud Sesaran maksimum suatu zarah dari kedudukan


Amplitude meter keseimbangan .
A metre
The maximum displacement of particle from equilibrium
(m)
position.

(b) Tempoh
Period
Rumus: Masa yang diambil oleh suatu zarah untuk membuat satu
Formula: saat ayunan lengkap
T second .
1 (s) The time taken by a particle to make one complete oscillation .
T=
f

(c) Frekuensi Bilangan ayunan lengkap yang dilakukan oleh


Frequency suatu zarah atau bilangan gelombang yang dihasilkan oleh
Rumus: suatu sumber gelombang dalam satu saat.
Formula: hertz, The number of complete oscillation performed by a particle or
f
Hz/s–1 the number of waves produced by a wave source in one second.
1
f=
T

(d) Panjang Jarak di antara dua titik berturutan yang sama


gelombang fasa.
Wavelength The distance between two consecutive points with the same
phase.
(i) Bagi gelombang melintang:
For transverse wave:
Jarak di antara dua puncak berturutan atau dua
lembangan berturutan bagi suatu gelombang.
meter consecutive crests
The distance between two or two
λ metre
consecutive troughs of a wave.
(m)

(ii) Bagi gelombang membujur:


For longitudinal wave:
Jarak di antara titik tengah dua mampatan berturutan
atau dua renggangan berturutan bagi suatu gelombang.
The distance between the midpoints of two consecutive compressions
or two consecutive rarefactions of a wave.

(e) Laju
gelombang
Wave speed meter
per saat Jarak suatu gelombang merambat dalam satu saat.
Rumus:
v metre per
one
Formula: The distance of a wave propagates in second.
second
(m s )
–1
v=fλ

128

Strategi A+ SPM Fizik Tg4_05_updated.indd 128 18/11/2023 10:22:12 AM


2 Selesaikan masalah yang berikut.
Solve the following problems.

(a) Suatu gelombang mempunyai panjang gelombang 20.0 cm dan tempoh 0.2 saat.
Hitung laju gelombang itu. TP 3
A wave has a wavelength of 20.0 cm and a period of 0.2 second. Calculate the speed of the wave.
1 1
f= = = 5 Hz
T 0.2
v = fλ = 5 × 20 = 100 cm s–1

(b) Suatu gelombang mempunyai panjang gelombang 36.0 cm dan laju gelombang 60 cm s–1.
Hitung tempoh gelombang itu. TP 3
A wave has a wavelength of 36.0 cm and a wave speed of 60 cm s−1. Calculate the period of the wave.
v 60
f= = Hz
λ 36
1 36
T= = = 0.6 s
f 60

Latihan 4 Graf gelombang


Graphs of waves
TP 1 Mengingat kembali pengetahuan dan kemahiran sains mengenai Gelombang.
TP 2 Memahami Gelombang serta dapat menjelaskan kefahaman tersebut.
TP 3 Mengaplikasikan pengetahuan mengenai Gelombang untuk menerangkan kejadian atau fenomena alam dan melaksanakan tugasan mudah.

1 Lengkapkan peta pokok berikut. TP 1 TP 2


Complete the following tree map. i-THINK Peta Pokok

Graf gelombang
Graphs of waves

Graf sesaran melawan masa Graf sesaran melawan jarak


Graph of displacement against time Graph of displacement against distance

Sesaran Sesaran
Displacement Displacement

T λ

A A
Masa Jarak
O Time O Distance
T λ

(a) Maklumat yang boleh diperoleh


(a) Maklumat yang boleh diperoleh
daripada graf:
daripada graf:
Information that can be obtained from the graph:
Information that can be obtained from the graph:
(i) Amplitud/ Amplitude, A
(i) Amplitud/ Amplitude, A
(ii) Tempoh/ Period, T
(ii) Panjang gelombang/ Wavelength, λ
(b) Label amplitud sebagai A dan tempoh
(b) Label amplitud sebagai A dan panjang
sebagai T pada graf.
Label amplitude as A and period as T on the
gelombang sebagai λ pada graf.
Label amplitude as A and wavelength as λ on the
graph.
graph.

129

Strategi A+ SPM Fizik Tg4_05_updated.indd 129 18/11/2023 10:22:12 AM


2 Selesaikan masalah yang berikut.
Solve the following problems.

(a) Rajah di sebelah menunjukkan graf sesaran Sesaran (cm)


Displacement
melawan masa dan graf sesaran melawan jarak
bagi satu gelombang. TP 3 5
The diagrams on the right show a graph of displacement
Masa (s)
against time and a graph of displacement against distance for 0
2 4 6 8 Time
a wave.
–5
Hitung/ Calculate
(i) amplitud/ amplitude, A Sesaran (cm)
(ii) tempoh/ period, T Displacement

(iii) frekuensi/ frequency, f 5


(iv) panjang gelombang/ wavelength, λ
Jarak (cm)
(v) laju gelombang/ wave speed, v 0
10 20 30 40 Distance
(i) 5.0 cm
–5
(ii) 4 saat/ seconds
1 1
(iii) f = = = 0.25 Hz
T 4
(iv) λ = 20.0 cm
(v) v = fλ = (0.25)(20.0) = 5 cm s–1

(b) Rajah di sebelah menunjukkan satu graf sesaran


melawan jarak bagi satu gelombang dengan
Sesaran (cm)
frekuensi 50 Hz. TP 3 Displacement
The diagram on the right shows a graph of displacement 20
against distance for a wave with a frequency of 50 Hz.
Berapakah amplitud, panjang gelombang dan 0 Jarak (cm)
5 10 15 Distance
laju gelombang itu?
What is the amplitude, wavelength and speed of the wave? –20
Amplitud/ Amplitude = 20 cm
Panjang gelombang/ Wavelength = 10 cm
Laju gelombang/ Wave speed = v = fλ = (50)(10) = 500 cm s–1

Latihan 5 Mengkaji gelombang air


Investigating water waves
TP 3 Mengaplikasikan pengetahuan mengenai Gelombang untuk menerangkan kejadian atau fenomena alam dan melaksanakan tugasan mudah.

Lengkapkan perbincangan bagi aktiviti berikut. TP 3


Complete the discussion for the following activity.
Aktiviti: Tangki riak untuk mengkaji jenis gelombang
Activity: Ripple tank to investigate type of wave
Lampu
Lamp

Ke bekalan kuasa
Motor penggetar To power supply
Vibrator motor
Gelang getah
Rubber band
Rod penggetar
Air
Vibrating rod
Water

Stroboskop Span
Stroboscope Sponge

Kertas putih (skrin)


White paper (screen)

130

Strategi A+ SPM Fizik Tg4_05_updated.indd 130 18/11/2023 10:22:13 AM


Prosedur:
Procedure:
1. Susunan radas disediakan seperti yang ditunjukkan dalam rajah di atas.
The apparatus is set up as shown in the diagram above.
2. Penggetar satah direndahkan sehingga penggetar menyentuh permukaan air.
The plane vibrator is lowered until it touches the surface of water.
3. Gelombang yang terhasil diperhatikan menerusi stroboskop.
The resulted wavefront is observed through a stroboscope.
4. Aktiviti diulang dengan menggantikan penggetar satah kepada penggetar sfera.
The activity is repeated by replacing the plane vibrator with the spherical vibrator.
Perbincangan:
Discussion:
(a) Penggetar menggetarkan permukaan air untuk menghasilkan gelombang .
The vibrator vibrates the water surface to produce waves .
(b) Terdapat dua jenis penggetar yang menghasilkan dua jenis gelombang:
There are two types of vibrators that produce two types of waves:

(i) Penggetar satah menghasilkan (ii) Penggetar sfera menghasilkan


gelombang satah . gelombang membulat .
A plane vibrator produces plane A spherical vibrator produces circular
wavefronts. wavefronts.
λ
λ

5.2 Pelembapan dan Resonans


Damping and Resonance

Latihan 6 Pelembapan
Damping
TP 1 Mengingat kembali pengetahuan dan kemahiran sains mengenai Gelombang.
TP 2 Memahami Gelombang serta dapat menjelaskan kefahaman tersebut.
TP 3 Mengaplikasikan pengetahuan mengenai Gelombang untuk menerangkan kejadian atau fenomena alam dan melaksanakan tugasan mudah.
TP 4 Menganalisis pengetahuan mengenai Gelombang dalam konteks penyelesaian masalah mengenai kejadian atau fenomena alam.

1 Isi tempat kosong dengan perkataan yang betul. TP 1 TP 2


Fill in the blanks with the correct words.
(a) Terdapat dua jenis pelembapan:
There are two types of damping:
(i) Pelembapan luaran/ External damping:
Sistem ayunan kehilangan tenaga bagi mengatasi daya geseran atau rintangan udara
yang bertindak ke atasnya.
An oscillating system loses energy to overcome frictional force or air resistance that act on it.
(ii) Pelembapan dalaman/ Internal damping:
Sistem ayunan kehilangan tenaga kerana renggangan dan mampatan molekul-
molekul dalam sistem tersebut.
An oscillating system loses energy due to the rarefaction and compression of the molecules in the
system.

131

Strategi A+ SPM Fizik Tg4_05_updated.indd 131 18/11/2023 10:22:13 AM


(b) Semasa pelembapan:
During damping:
(i) amplitud ayunan berkurang .
the amplitude of oscillation decreases .
(ii) frekuensi ayunan tidak berubah/ tetap .
the frequency of oscillation does not change/ is fixed .
(c) Cara mengatasi pelembapan:
Way to overcome damping:
Mengenakan daya luar secara berkala untuk memindahkan tenaga kepada sistem
ayunan (ayunan paksa).
Apply periodic external force to transfer energy to the oscillating system (forced oscillation).

2 Lengkapkan perbincangan bagi aktiviti berikut. TP 3 TP 4


Complete the discussion for the following activity.

Aktiviti: Ayunkan bandul dengan menarik ladung bandul ke arah P dan melepaskannya.
Perhatikan ayunan bandul tersebut.
Activity: Swing the pendulum by pulling the pendulum bob towards P and release it. O
Observe the oscillation of the pendulum.
Perbincangan:
Discussion:
(a) Amplitud awal ayunan bandul ialah jarak OP .
P Q
The initial amplitude of oscillation of the pendulum is distance OP . R
O

(b) Selepas beberapa kali berayun, didapati bandul berayun hanya sehingga R.
After several oscillations, it is found that the pendulum swings only up to R.

(c) Jarak OR lebih pendek berbanding dengan jarak OP. Hal ini menunjukkan bahawa
amplitud semakin berkurang .
Length of OR is shorter compared to length of OP. This shows that the amplitude is decreasing .

(d) Rajah berikut menunjukkan graf sesaran melawan masa bagi ayunan bandul itu.
The following diagram shows the graph of displacement against time for the oscillation of the pendulum.

Sesaran (cm)
Displacement

A1 A2 A3
Masa (s)
Time

Tempoh, T Tempoh, T Tempoh, T


Period Period Period

(i) Didapati A3 < A2 < A1.

132

Strategi A+ SPM Fizik Tg4_05_updated.indd 132 18/11/2023 10:22:14 AM


(ii) Amplitud
ayunan bandul semakin berkurang, manakala tempoh dan frekuensi
adalah tetap.
The amplitude of oscillation of the pendulum is decreasing, while the period and frequency are
constant.
(iii) Ayunan dengan amplitud berkurang dengan masa menunjukkan bahawa sistem
yang sedang berayun mengalami kehilangan tenaga .
The oscillations with decreasing amplitude with time show that the oscillating system experiences
energy loss.
(iv) Akhirnya, ayunan akan berhenti .
Finally, the swing will stop .
(v) Fenomena ini dikenali sebagai pelembapan .
This phenomenon is known as damping .

Latihan 7 Resonans
Resonance
TP 1 Mengingat kembali pengetahuan dan kemahiran sains mengenai Gelombang.
TP 2 Memahami Gelombang serta dapat menjelaskan kefahaman tersebut.
TP 3 Mengaplikasikan pengetahuan mengenai Gelombang untuk menerangkan kejadian atau fenomena alam dan melaksanakan tugasan mudah.

1 Isi tempat kosong dengan perkataan yang betul. TP 1 TP 2


Fill in the blanks with the correct words.

(a) Resonans berlaku apabila suatu sistem ayunan dikenakan daya luar yang mempunyai
frekuensi yang sama dengan frekuensi asli sistem ayunan tersebut.
Resonance occurs when an oscillating system is subjected to an external force that has the same frequency
as the natural frequency of the oscillating system.

(b) Semasa resonans,


During resonance,
(i) sistem berayun dengan frekuensi aslinya .
the system oscillates with its natural
frequency.
(ii) sistem berayun pada amplitud maksimum .
the system oscillates at maximum amplitude.

(c) Kesan resonans:


Effects of resonance:
(i) Siaran televisyen dan radio diterima apabila talaan gelombang radio mengalami
resonans . Apabila televisyen atau radio dilaras kepada frekuensi yang sama
dengan frekuensi stesen penyiaran , isyarat elektrik yang kuat terhasil.
Television and radio broadcasts are received when the radio wave tuning undergoes resonance . When the
television or radio is tuned to the same frequency as the frequency of the broadcasting station ,
strong electrical signals are produced.
(ii) Jambatan gantung boleh runtuh apabila tiupan angin yang kuat memaksa tali dan lantai
jambatan berayun pada frekuensi aslinya . Jambatan akan berayun pada amplitud
maksimum disebabkan oleh resonans dan runtuh seperti yang terjadi kepada
Tacoma Narrows Bridge di Amerika Syarikat.
A suspension bridge can collapse when strong winds force the ropes and bridge floor to oscillate at their
natural frequency. The bridge will oscillate at maximum amplitude due to resonance and
collapse as happened to the Tacoma Narrows Bridge in the United States.

133

Strategi A+ SPM Fizik Tg4_05_updated.indd 133 18/11/2023 10:22:14 AM


(iii) Seorang penyanyi boleh menggunakan suaranya pada frekuensi yang sama dengan
frekuensi asli dinding sebiji gelas untuk memecahkan gelas tersebut. Dinding
gelas akan bergetar pada amplitud maksimum dan pecah apabila resonans berlaku.
A singer can use his voice at the same frequency as the natural frequency of the wall of a glass
to break the glass. The wall of the glass will vibrate at maximum amplitude and break when resonance
occurs.

2 Lengkapkan pemerhatian bagi aktiviti berikut. TP 3


Complete the observation for the following activity.

Aktiviti: Mengkaji resonans dalam sistem berayun


Activity: Investigating resonance in an oscillating system

L L

T
S
X R
Q

Prosedur:
Procedure:

1. Satu bandul Barton disediakan seperti yang ditunjukkan dalam rajah di atas.
A Barton’s pendulum is set up as shown in the diagram above.
2. Bandul X dan R diletakkan pada paras mengufuk yang sama dengan panjang yang sama, L cm.
Pendulums X and R are placed at the same horizontal level with the same length of L cm.
3. Bandul X disesarkan dan dilepaskan.
Pendulum X is displaced and released.
4. Ayunan setiap bandul diperhatikan.
The oscillation of every pendulum is observed.

Pemerhatian:
Observation:
(a) Apabila bandul X berayun, semua bandul yang lain dipaksa berayun kerana tenaga
dipindahkan dari X ke bandul-bandul yang lain.
When pendulum X oscillates, all other pendulums are forced to oscillate because energy is transferred
from X to the other pendulums.
(b) Bandul R akan berayun dengan amplitud maksimum kerana panjang bandul R
adalah sama dengan panjang bandul X.
Pendulum R will oscillate at maximum frequency because the length of pendulum R is the same

as the length of pendulum X.


(c) Bandul R dikatakan mengalami resonans dan amplitudnya adalah maksimum .
Pendulum R is said to experience resonance and its amplitude is the maximum .

134

Strategi A+ SPM Fizik Tg4_05_updated.indd 134 18/11/2023 10:22:14 AM


5.3 Pantulan Gelombang
Reflection of Waves
Latihan 8 Muka gelombang
Wavefront
TP 1 Mengingat kembali pengetahuan dan kemahiran sains mengenai Gelombang.
TP 2 Memahami Gelombang serta dapat menjelaskan kefahaman tersebut.
TP 3 Mengaplikasikan pengetahuan mengenai Gelombang untuk menerangkan kejadian atau fenomena alam dan melaksanakan tugasan mudah.

1 Lengkapkan definisi muka gelombang berikut. TP 1


Complete the following definition of wavefront.

Muka gelombang ialah garisan atau permukaan yang menyambungkan titik-titik sefasa
dalam suatu gelombang.
Wavefront is a line or surface that connects points of the same phase in a wave.

2 Rajah berikut menunjukkan dua jenis muka gelombang yang berlainan. TP 2


The following diagrams show two different types of wavefronts.
(a) Labelkan panjang gelombang (λ) dan arah perambatan gelombang (➝) pada rajah berikut.
Seterusnya, namakan jenis muka gelombang.
Label the wavelength (λ) and the direction of wave propagation (➝) on the following diagrams. Then, name the type
of the wavefronts.

Jenis muka gelombang: Jenis muka gelombang:


Type of wavefront: Type of wavefront:
Muka gelombang satah/ Plane wavefront Muka gelombang membulat/ Circular wavefront

λ
λ

(b) Isi tempat kosong dengan perkataan yang betul.


Fill in the blanks with the correct words.
(i) Arah perambatan gelombang berserenjang dengan muka gelombang.
The direction of wave propagation is perpendicular
to the wavefront.
(ii) Panjang gelombang, λ ialah jarak di antara dua muka gelombang yang berturutan .
Wavelength, λ is the distance between two successive wavefronts.

3 Lukis dan labelkan jalur


cerah dan gelap yang
dibentuk oleh sinar cahaya
yang merambat melalui air
di dalam sebuah tangki riak.
Draw and label the bright and
dark fringes formed by light rays
Tapak tangki riak
propagating through water in Base of ripple tank
a ripple tank. TP 3
Skrin/ Screen
Jalur Jalur Jalur Jalur Jalur Jalur Jalur
cerah gelap cerah gelap cerah gelap cerah
Bright Dark Bright Dark Bright Dark Bright
fringe fringe fringe fringe fringe fringe fringe

135

Strategi A+ SPM Fizik Tg4_05_updated.indd 135 18/11/2023 10:22:15 AM


Latihan 9 Pantulan gelombang
Reflection of waves
TP 1 Mengingat kembali pengetahuan dan kemahiran sains mengenai Gelombang.
TP 2 Memahami Gelombang serta dapat menjelaskan kefahaman tersebut.
TP 3 Mengaplikasikan pengetahuan mengenai Gelombang untuk menerangkan kejadian atau fenomena alam dan melaksanakan tugasan mudah.

1 Lengkapkan ayat dan persamaan berikut. TP 1


Complete the following sentences and equation.
(a) Pantulan gelombang ialah fenomena sebahagian atau keseluruhan gelombang dibalikkan
setelah terkena suatu penghalang/ pemantul .
Wave reflection is the phenomenon of a part or entire wave is reflected after hitting an obstacle/ reflector .
(b) Hukum pantulan:
Law of reflection:
Sudut tuju, i = Sudut pantulan, r

Angle of incidence, i = Angle of reflection, r

Gelombang tuju , gelombang pantulan dan garis normal berada pada


satah yang sama.
Incident wave, reflected wave and normal are on the same plane.

(c) Fenomena pantulan gelombang dapat diperhatikan menggunakan tangki riak .


The phenomenon of wave reflection can be observed using a ripple tank.

2 Gariskan jawapan yang betul tentang kesan pantulan ke atas ciri-ciri gelombang. TP 2
Underline the correct answers about the effect of the reflection on the characteristics of wave.

Ciri-ciri gelombang Selepas pantulan


Characteristic of wave After reflection

(a) Panjang gelombang Lebih panjang / Lebih pendek / Tidak berubah


Wavelength Longer / Shorter / No change

(b) Frekuensi Lebih tinggi / Lebih rendah / Tidak berubah


Frequency Higher / Lower / No change

(c) Laju gelombang Lebih tinggi / Lebih rendah / Tidak berubah


Wave speed Higher / Lower / No change

(d) Arah perambatan Berubah / Tidak berubah


Direction of propagation Changes / No change

(e) Sudut Sudut tuju sama dengan sudut pantulan /


Angle Sudut tuju lebih besar daripada sudut pantulan /
Sudut pantulan lebih besar daripada sudut tuju
Angle of incidence is equal to angle of reflection /
Angle of incidence is more than angle of reflection /
Angle of reflection is more than angle of incidence

136

Strategi A+ SPM Fizik Tg4_05_updated.indd 136 18/11/2023 10:22:15 AM


3 Lengkapkan gambar rajah pantulan gelombang berikut dengan melukis gelombang pantulan yang
terhasil. TP 3
Complete the following diagrams of reflection of waves by sketching the reflected waves produced.

(a) Pantulan pada suatu sudut (b) Pantulan pada sudut tegak
Reflection at an angle Reflection at right angles

Gelombang tuju Gelombang pantulan Gelombang tuju


Incident wave Reflected wave Incident wave

Pemantul
Gelombang satah
i r
pantulan Plane
Reflected reflector
wave
Pemantul satah
Plane reflector

(c) Pantulan pada sudut tegak Pemantul


Reflection at right angles satah
Plane
reflector

r i Gelombang
tuju
Incident
wave

Gelombang pantulan
Reflected wave

Latihan 10 Aplikasi pantulan gelombang dalam kehidupan seharian


Applications of reflection of waves in daily life
TP 3 Mengaplikasikan pengetahuan mengenai Gelombang untuk menerangkan kejadian atau fenomena alam dan melaksanakan tugasan mudah.
TP 4 Menganalisis pengetahuan mengenai Gelombang dalam konteks penyelesaian masalah mengenai kejadian atau fenomena alam.

Isi tempat kosong dengan perkataan yang betul tentang aplikasi pantulan gelombang dalam
kehidupan harian. TP 3 TP 4
Fill in the blanks with the correct words about the applications of reflection of waves in daily life.

1 Gema (a) Gema terhasil apabila gelombang bunyi yang


Echo terpantul permukaan keras
dari suatu kembali
Bunyi asal
Original sound kepada pendengar.
Echoes are produced when sound waves are reflected from
a hard surface return to the listener.
Permukaan
keras
Hard surface (b) Bunyi yang dipantulkan ini menyerupai bunyi asal
Arah gema
Direction of echo tetapi mengambil sedikit masa untuk sampai
ke telinga pendengar.
This reflected sound is the same as the original sound but takes
some time to reach the listener’s ears.

137

Strategi A+ SPM Fizik Tg4_05_updated.indd 137 18/11/2023 10:22:15 AM


2 Gelombang ultrasonik dalam (a) Transduser memancarkan gelombang ultrasonik
perubatan yang dipantulkan oleh organ dalaman atau fetus.
Ultrasonic waves in medicine
The transducer emits ultrasonic waves which are reflected
Transduser ultrasonik
Ultrasonic transducer
by internal organs or foetus.

(b) Gelombang yang dipantulkan akan diterima kembali


dan dianalisis oleh komputer untuk membentuk
imej di skrin.
The reflected waves will be received and analysed by the
computer to form an image on the screen.

3 Sonar dalam perikanan (a) Sonar digunakan untuk menentukan kedudukan


Sonar in fisheries kawanan ikan.
Sonar is used to determine the location of schools of fish.

(b) Transduser memancarkan gelombang ultrasonik ke


dalam laut.
Transduser
Tranduser
Transducer
Tranducer The transducer emits ultrasonic waves into the sea.

(c) Gelombang itu dipantulkan oleh ikan dan kembali ke


transduser.
The wave is reflected by the fish and returns to the
transducer.
(d) Dengan mengetahui sela masa pancaran dan
penerimaan semula gelombang, kedudukan kawanan
ikan dapat ditentukan.
By knowing the time interval between the transmission and
receiving of the waves, the location of the schools of fish can be
determined.

4 Kedalaman laut ultrasonik


Depth of sea (a) Pemancar memancar gelombang ke
dasar laut .
The transmitter emits ultrasonic waves to the
seabed .
(b) Gelombang dipantulkan oleh dasar laut dan dikesan
oleh penerima.
Pemancar Penerima reflected
Transmitter Receiver
The waves are by the seabed and detected by the
receiver.
(c) Sela masa, t gelombang yang dipancar dan diterima
Dasar laut semula diukur. Maka, kedalaman laut boleh dihitung.
Seabed
Time interval, t for the waves to be transmitted and received is
measured. Thus, the depth of the sea can be calculated.

vt
d=
2
dengan keadaan/ where
d = Kedalaman laut/ Depth of sea
v = Laju gelombang / Speed of wave
t = Sela masa/ Time interval

138

Strategi A+ SPM Fizik Tg4_05_updated.indd 138 18/11/2023 10:22:16 AM


Latihan 11 Menyelesaikan masalah melibatkan pantulan gelombang
Solving problems involving reflection of waves
TP 3 Mengaplikasikan pengetahuan mengenai Gelombang untuk menerangkan kejadian atau fenomena alam dan melaksanakan tugasan mudah.
TP 4 Menganalisis pengetahuan mengenai Gelombang dalam konteks penyelesaian masalah mengenai kejadian atau fenomena alam.

Selesaikan masalah yang berikut.


Solve the following problems.

1 Radzuan menjerit di hadapan dinding di dalam sebuah bilik tertutup. Dia mendengar gema
suaranya selepas 0.2 saat. Diberi bahawa laju bunyi dalam udara ialah 340 m s–1, hitung jarak di
antara Radzuan dengan dinding tersebut. TP 3
Radzuan shouts in front of a wall in a closed room. He hears the echo of his voice after 0.2 seconds. It is given that the
speed of sound in air is 340 m s−1, calculate the distance between Radzuan and the wall.
Masa 0.2 saat adalah untuk suara Radzuan pergi ke dinding dan dipantul semula. Oleh itu, jarak yang diambil
kira ialah jarak pergi dan balik.
Time of 0.2 seconds is for Radzuan’s voice to travel to the wall and is reflected back. Therefore, the distance that is taken into account
is the to-and-fro distance.
Jumlah jarak = Laju × Masa
Total distance = Speed × Time
2d = vt
= 340 × 0.2
= 68 m
68
d= = 34 m
2
2 Rajah berikut menunjukkan sebuah bot nelayan menggunakan sonar untuk menentukan
kedudukan kawanan ikan. TP 3
The following diagram shows a fishing boat using sonar to determine the location of a school of fish.

Bot nelayan
Fishing boat

Laut
Sea

Gelombang bunyi
Kawanan ikan Sound waves
School of fish

Dasar laut
Seabed

Jika gelombang tersebut mempunyai laju 1 500 m s–1 di dalam laut dan sela masa untuk gelombang
diterima kembali ialah 0.22 s, hitung kedalaman kawanan ikan tersebut dari permukaan laut.
If the waves have a speed of 1 500 m s−1 in the sea and the time interval for the wave to be received back is 0.22 s,
calculate the depth of the school of fish from the surface of the sea.
Jumlah jarak = Laju × Masa
Total distance = Speed × Time
2d = vt
= 1 500 × 0.22
= 330 m
330
d= = 165 m
2

139

Strategi A+ SPM Fizik Tg4_05_updated.indd 139 18/11/2023 10:22:16 AM


3 Sebuah kapal memancarkan gelombang ultrasonik berfrekuensi 25 kHz ke dasar laut untuk
menentukan kedalaman laut. Gelombang bergerak dalam air laut dengan laju 1 500 m s–1. Gelombang
yang telah dipantulkan diterima selepas 150 ms. Tentukan kedalaman laut dan panjang gelombang
bagi gelombang tersebut. TP 4
A ship emits ultrasonic waves with a frequency of 25 kHz to the seabed to determine the depth of the sea. The waves travel
in seawater at a speed of 1 500 m s−1. The reflected waves are received after 150 ms. Determine the depth of the sea and the
wavelength of the waves.
Sela masa/ Time interval = 150 ms = 150 × 10–3 s = 0.15 s

Jumlah jarak = Laju × Masa


Total distance = Speed × Time
2d = vt
= 1 500 × 0.15
= 225 m
225
d= = 112.5 m
2

f = 25 kHz = 25 × 103 Hz

v = fλ

v 1 500
λ=
= = 0.06 m
f 25 × 103

5.4 Pembiasan Gelombang


Refraction of Waves

Latihan 12 Pembiasan gelombang


Refraction of waves
TP 1 Mengingat kembali pengetahuan dan kemahiran sains mengenai Gelombang.
TP 2 Memahami Gelombang serta dapat menjelaskan kefahaman tersebut.
TP 3 Mengaplikasikan pengetahuan mengenai Gelombang untuk menerangkan kejadian atau fenomena alam dan melaksanakan tugasan mudah.

1 Isi tempat kosong dengan perkataan yang sesuai. TP 1


Fill in the blanks with the suitable words.

(a) Pembiasan gelombang ialah perubahan arah gelombang yang disebabkan oleh perubahan
halaju gelombang apabila gelombang merambat dari satu medium ke medium yang lain.
Refraction of wave is the change in direction of wave due to the change in wave velocity when the
wave propagates from one medium to another.

(b) Fenomena pembiasan gelombang air dapat diperhatikan menggunakan tangki riak. Kesan
pembiasan bergantung pada kedalaman air .
The phenomenon of refraction of water waves can be observed using a ripple tank. The refraction effect
depends on the depth of water .

2 Perhatikan rajah di sebelah dan isi jadual dengan


Rod penggetar
perkataan yang betul tentang kesan pembiasan ke Vibrating rod
Kepingan kaca
Glass plate
atas ciri-ciri gelombang di kawasan air dalam dan air
cetek. TP 2 Tangki riak
Observe the diagram on the right and fill in the table with the d1 d2 Ripple tank
correct words regarding the effect of reflection on the characteristics
of waves in the deep region and shallow region. Kawasan Kawasan
dalam cetek
Deep region Shallow
region

140

Strategi A+ SPM Fizik Tg4_05_updated.indd 140 18/11/2023 10:22:16 AM


Ciri-ciri gelombang Kawasan air dalam Kawasan air cetek
Characteristic of waves Deep region Shallow region

(a) Panjang gelombang Panjang Pendek


Wavelength Long Short

(b) Frekuensi Sama Sama


Frequency Same Same

(c) Laju gelombang Tinggi Rendah


Wave speed High Low

(d) Arah perambatan Menjauhi garis normal Mendekati garis normal


Direction of propagation Away from normal Towards normal

(e) Sudut biasan Besar Kecil


Angle of refraction Big Small

3 Lengkapkan gambar rajah pembiasan gelombang berikut dengan melukis gelombang biasan yang
terhasil. TP 3
Complete the following diagrams of refraction of waves by sketching the refracted waves produced.
(a) Rajah 1/ Diagram 1
Gelombang tuju
Incident wave

Kawasan cetek
Shallow region

Kawasan dalam
Deep region Gelombang biasan
Refracted wave

(b) Rajah 2/ Diagram 2 Bongkah kaca Gelombang biasan


Glass block Refracted wave

Gelombang tuju
Incident wave

Air dalam Air cetek Air dalam


Deep water Shallow water Deep water

(c) Rajah 3/ Diagram 3 Air dalam Air cetek Air dalam


Deep water Shallow water Deep water

Gelombang tuju
Incident wave Gelombang biasan
Refracted wave

141

Strategi A+ SPM Fizik Tg4_05_updated.indd 141 18/11/2023 2:58:24 PM


Latihan 13 Fenomena pembiasan dalam kehidupan seharian
Phenomena of refraction in daily life
TP 3 Mengaplikasikan pengetahuan mengenai Gelombang untuk menerangkan kejadian atau fenomena alam dan melaksanakan tugasan mudah.
TP 4 Menganalisis pengetahuan mengenai Gelombang dalam konteks penyelesaian masalah mengenai kejadian atau fenomena alam.

1 Isi tempat kosong dengan perkataan yang betul tentang pembiasan gelombang bunyi. TP 3 TP 4
Fill in the blanks with the correct words about the refraction of sound waves.
(a) Pembiasan gelombang bunyi bergantung pada ketumpatan udara .
The refraction of sound waves depends on the density of air .

(b) Bunyi dalam udara panas merambat dengan lebih laju berbanding dengan dalam udara sejuk
kerana udara panas kurang tumpat.
Sound in hot air travels faster than in cold air because hot air is less dense .
(c) Pada waktu siang:
During day time:
(i) Udara yang hampir dengan tanah lebih
panas .
Air closer to the ground is warmer . Udara sejuk (lebih tumpat)
Cold air (more dense)
(ii) Laju gelombang bunyi berkurang ketika
Udara panas
merambat dari udara panas ke udara sejuk. (kurang tumpat)
The speed of sound waves decreases when travelling
Hot air (less dense)

from hot to cold air.


(iii) Pembiasan menyebabkan lintasan perambatan
gelombang dibiaskan menjauhi permukaan
Bumi.
Refraction causes the path of wave propagation to be refracted away from the surface of the Earth.
(iv) Maka, kenyaringan berkurang untuk pemerhati yang berada di atas tanah.
Thus, the loudness decreases for an observer on the ground.

(d) Pada waktu malam:


During the night:
(i) Udara yang hampir dengan tanah lebih
sejuk .
Air closer to the ground is cooler . Udara panas (kurang tumpat)
Hot air (less dense)
(ii) Laju gelombang bunyi bertambah ketika
Udara sejuk
merambat dari udara sejuk ke udara panas. (lebih tumpat)
Cold air
The speed of sound waves increases when travelling (more dense)

from cold to hot air.


(iii) Pembiasan menyebabkan lintasan perambatan
gelombang dibiaskan mendekati permukaan
Bumi.
Refraction causes the path of wave propagation to be refracted towards the surface of the Earth.
(iv) Maka, kenyaringan bertambah untuk pemerhati yang berada di atas tanah.
Thus, the loudness increases for an observer on the ground.

142

Strategi A+ SPM Fizik Tg4_05_updated.indd 142 18/11/2023 10:22:17 AM


2 Isi tempat kosong dengan perkataan yang betul tentang pembiasan gelombang air. TP 3 TP 4
Fill in the blanks with the correct words about the refraction of water waves.
Rajah di sebelah menunjukkan pembiasan air
laut apabila menghampiri kawasan pantai. Gelombang
The diagram on the right shows the refraction of hampir lurus
Tanjung
seawater when approaching the beach. Near-straight
Cape
waves

(a) Tanjung merupakan kawasan yang cetek . Pantai


Beach
The cape is a shallow region. Teluk
Bay
(b) Teluk merupakan kawasan yang dalam .
The bay is a deep region.
(c) Muka gelombang hampir lurus sebelum menghampiri pantai kerana kelajuan yang seragam .
The wavefront is almost straight before approaching the shore due to uniform speed .
(d) Jika gelombang merambat ke tanjung, laju gelombang air akan berkurang dan
panjang gelombang akan berkurang .
and the wavelength will decrease .
If the waves propagate to the cape, the speed of water waves will decrease

(e) Jika gelombang merambat ke teluk, laju gelombang air akan bertambah dan panjang
gelombang akan bertambah .
If the waves propagate to the bay, the speed of water waves will increase and the wavelength will increase .
(f) Muka gelombang akan membengkok dan mengikut bentuk garis pantai .
The wavefront will bend and follow the shape of the shoreline .

Latihan 14 Menyelesaikan masalah melibatkan pembiasan gelombang


Solving problems involving refraction of waves
TP 3 Mengaplikasikan pengetahuan mengenai Gelombang untuk menerangkan kejadian atau fenomena alam dan melaksanakan tugasan mudah.
TP 4 Menganalisis pengetahuan mengenai Gelombang dalam konteks penyelesaian masalah mengenai kejadian atau fenomena alam.
TP 5 Menilai pengetahuan mengenai Gelombang dalam konteks penyelesaian masalah dan membuat keputusan untuk melaksanakan satu tugasan.

1 Berdasarkan rajah yang diberi, isi petak kosong


dengan rumus yang berkaitan dengan pembiasan Frekuensi/ Frequency = f
gelombang. TP 3 v2 v1
Base on the diagram given, fill in the empty boxes with the λ1 λ2 λ1
formula related to the refraction of waves.
Rajah di sebelah menunjukkan perambatan Kawasan cetek
Shallow region
gelombang air dari kawasan air dalam ke kawasan
air cetek. Kawasan Kawasan
dalam dalam
The diagram on the right shows the propagation of water Deep region Deep region
waves from the deep region to the shallow region.

Rumus laju gelombang/ Formula for wave speed: v = fλ

Kawasan dalam Kawasan cetek


Deep region Shallow region

v1 = f λ1 v2 = f λ2

Frekuensi tidak berubah.


Frequency does not change.
v1 v2
Maka/ Therefore, =
λ1 λ2

143

Strategi A+ SPM Fizik Tg4_05_updated.indd 143 18/11/2023 10:22:18 AM


2 Selesaikan masalah yang berikut.
Solve the following problems.

(a) Gelombang air dengan panjang gelombang 1.2 m merambat dari laut ke pantai dengan laju
3.6 m s–1. Laju gelombang itu berkurang kepada 2.4 m s–1 apabila menghampiri pantai.
Hitung frekuensi gelombang itu dan panjang gelombangnya apabila menghampiri pantai. TP 4
A water wave with a wavelength of 1.2 m propagates from the sea to the shore with a velocity of 3.6 m s–1. The
wave speed decreases to 2.4 m s–1 when approaching the shore. Calculate the frequency of the wave and its wavelength
when approaching the shore.
Diberi/Given
kawasan dalam/deep region: λ1 =1.2 m, v1= 3.6 m s–1
kawasan cetek/shallow region: v2 = 2.4 m s–1
v = fλ
3.6 = f(1.2)
3.6
f = = 3 Hz
1.2
Apabila menghampiri pantai,
When approaching the shore,
v1 v2
=
λ1 λ2
3.6 2.4
=
1.2 λ2
2.4(1.2)
λ2 = = 0.8 m
3.6
(b) Rajah berikut menunjukkan gelombang air merambat dari kawasan A ke kawasan B dengan
kedalaman yang berbeza. TP 5
The following diagram shows water waves propagating from region A to region B at different depths.

A B

8 cm 8 cm

Jika laju gelombang air di kawasan A ialah 20 cm s–1, tentukan laju gelombang air
di kawasan B.
If the speed of the water waves in region A is 20 cm s–1, determine the speed of the water waves in region B.
Diberi/Given
kawasan A/region A:
8
λ1 = = 2 cm, v1 = 20 cm s–1
4
kawasan B/region B:
8
λ2 = = 1 cm
8
v1 v2
=
λ1 λ2
20 v2
=
2 1
20(1)
v2 = = 10 cm s–1
2

144

Strategi A+ SPM Fizik Tg4_05_updated.indd 144 18/11/2023 10:22:18 AM


5.5 Pembelauan Gelombang
Diffraction of Waves

Latihan 15 Pembelauan gelombang


Diffraction of waves
TP 1 Mengingat kembali pengetahuan dan kemahiran sains mengenai Gelombang.
TP 2 Memahami Gelombang serta dapat menjelaskan kefahaman tersebut.
TP 3 Mengaplikasikan pengetahuan mengenai Gelombang untuk menerangkan kejadian atau fenomena alam dan melaksanakan tugasan mudah.
TP 4 Menganalisis pengetahuan mengenai Gelombang dalam konteks penyelesaian masalah mengenai kejadian atau fenomena alam.

1 Lengkapkan definisi pembelauan gelombang berikut. TP 1


Complete the following definition of diffraction of waves.

Pembelauan ialah penyebaran gelombang yang berlaku apabila gelombang merambat melalui
suatu celah atau mengelilingi tepi suatu penghalang.
Diffraction is the dispersion of waves that occurs when waves propagate through a gap or around the
edge of a barrier.

2 Gariskan jawapan yang betul tentang kesan pembelauan ke atas ciri-ciri gelombang. TP 2
Underline the correct answers about the effect of the diffraction on the characteristics of wave.

Ciri-ciri gelombang Selepas pembelauan


Characteristic of wave After diffraction
(a) Panjang gelombang Lebih panjang / Lebih pendek / Tidak berubah
Wavelength Longer / Shorter / No change

(b) Frekuensi Lebih tinggi / Lebih rendah / Tidak berubah


Frequency Higher / Lower / No change

(c) Laju gelombang Lebih tinggi / Lebih rendah / Tidak berubah


Wave speed Higher / Lower / No change

(d) Arah perambatan Satu arah / Banyak arah


Direction of propagation One direction / Many direction

(e) Amplitud Lebih tinggi / Lebih rendah / Tidak berubah


Amplitude Higher / Lower / No change

3 Lengkapkan pemerhatian dan kesimpulan bagi aktiviti berikut. TP 3 TP 4


Complete the observation and conclusion for the following activity.
Aktiviti: Faktor yang mempengaruhi pembelauan gelombang air
Activity: Factors affecting the diffraction of water waves

Prosedur: Lampu
Procedure: Lamp

A Saiz celah
Size of slit
1. Saiz celah dilaraskan supaya lebih besar
daripada panjang gelombang. Celah
Slit
The size of the slit is adjusted to be larger than the
wavelength. Tangki riak
Ripple tank
2. Gelombang terbelau diperhatikan dan
coraknya dilukis.
The diffracted wave is observed and the pattern is
drawn.
3. Saiz celah dilaraskan supaya hampir sama Skrin
Screen
dengan panjang gelombang.
The size of the slit is adjusted to be almost the same as the
wavelength.

145

Strategi A+ SPM Fizik Tg4_05_updated.indd 145 18/11/2023 10:22:18 AM


B Panjang gelombang
Wavelength
1. Frekuensi penjana gelombang dilaraskan untuk menghasilkan panjang gelombang yang
lebih kecil daripada saiz celah.
The frequency of the wave generator is adjusted to produce a wavelength smaller than the size of the slit.
2. Gelombang terbelau diperhatikan dan coraknya dilukis.
The diffracted wave is observed and the pattern is drawn.
3. Frekuensi penjana gelombang dilaraskan untuk menghasilkan panjang gelombang yang
hampir sama dengan saiz celah.
The frequency of the wave generator is adjusted to produce a wavelength that is almost equal to the size of
the slit.
Pemerhatian:
Observation:
Lakarkan corak pembelauan yang terhasil.
Sketch the diffraction patterns obtained.

Pemboleh ubah dimalarkan Pemboleh ubah dimanipulasikan Lakaran corak pembelauan


Constant variable Manipulated variable Sketch of diffraction pattern

Saiz celah: lebar


Slit size: wide

(a) Saiz celah


Size of slit

Saiz celah: sempit


Slit size: narrow

λ lebih pendek
Shorter λ

(b) Panjang gelombang


Wavelength

λ lebih panjang
Longer λ

146

Strategi A+ SPM Fizik Tg4_05_updated.indd 146 18/11/2023 10:22:19 AM


Kesimpulan:
Conclusion:
Berdasarkan pemerhatian yang diperoleh dalam aktiviti ini, lengkapkan kesimpulan dalam peta
pokok berikut.
Based on the observation obtained in this activity, complete the conclusion in the following tree map. i-THINK Peta Pokok

Faktor-faktor yang mempengaruhi


pembelauan gelombang
Factors that affect the diffraction of waves

Saiz celah Panjang gelombang, λ


Size of slit Wavelength, λ

Panjang gelombang adalah tetap. Saiz celah adalah tetap.


Wavelength is constant. Size of slit is constant.

Celah lebar Celah sempit λ lebih pendek λ lebih panjang


Wide slit Narrow slit Shorter λ Longer λ

Kesimpulan: Kesimpulan: Kesimpulan: Kesimpulan:


Conclusion: Conclusion: Conclusion: Conclusion:
Kesan Kesan Kesan Kesan
penyebaran penyebaran penyebaran penyebaran
gelombang gelombang gelombang gelombang
kurang ketara. lebih ketara. kurang ketara. lebih ketara.
The effect of The effect of The effect of The effect of
dispersion is dispersion is dispersion is dispersion is
less obvious. more obvious. less obvious. more obvious.

4 Jawab soalan berikut. TP 3


Answer the following questions.

Rajah di sebelah menunjukkan satu penghalang yang diletakkan


merentangi laluan suatu gelombang air. Lebar penghalang tersebut λ
adalah lebih besar daripada panjang gelombang bagi gelombang air
itu. Penghalang
Obstacle
The diagram on the right shows an obstacle that is placed in the path of a water wave.
The width of the obstacle is greater than the wavelength of the water wave.

(a) Lukis corak gelombang di sekeliling penghalang itu.


Draw the wave pattern around the obstacle.
Jawapan/Answer:

λ λ

Kesan penyebaran gelombang adalah kurang


ketara.
The effect of dispersion is less obvious.

147

Strategi A+ SPM Fizik Tg4_05_updated.indd 147 18/11/2023 10:22:19 AM


(b) Lebar penghalang dijadikan kurang daripada panjang gelombang bagi gelombang air itu. Lukis
corak gelombang yang baharu.
The width of the obstacle is made less than the wavelength of the water wave. Draw the new wave pattern.
Jawapan/Answer:

λ λ

Kesan penyebaran gelombang adalah lebih


ketara.
The effect of dispersion is more obvious.

5 Lengkapkan pemerhatian bagi aktiviti berikut. TP 3 TP 4


Complete the observation for the following activity.
Aktiviti: Corak pembelauan gelombang cahaya
Activity: Diffraction pattern of light waves

Kaki retort
Retort stand
Skrin
Screen

Pen laser
Laser pen Celah tunggal
Single slit

Prosedur:
Procedure:
1. Alur cahaya laser ditujukan melalui celah tunggal yang lebar.
The laser beam is directed through a wide single slit.
2. Corak yang terbentuk pada skrin diperhatikan dan coraknya dilukis.
The pattern formed on the screen is observed and the pattern is drawn.
3. Aktiviti diulang dengan celah yang sempit, lubang jarum bersaiz besar dan lubang jarum
bersaiz kecil.
The activity is repeated with a narrow slit, large size pinhole and small size pinhole.
Pemerhatian:
Observation:
Lakarkan corak pembelauan yang terhasil.
Sketch the diffraction pattern obtained.

Pemboleh ubah dimanipulasikan Lebar/ Besar Sempit/ Kecil


Manipulated variable Wide/ Large Narrow/ Small

Saiz celah
Size of slit

Saiz lubang jarum


Size of pinhole

148

Strategi A+ SPM Fizik Tg4_05_updated.indd 148 18/11/2023 10:22:20 AM


6 Lengkapkan keputusan bagi aktiviti berikut. TP 3 TP 4
Complete the result for the following activity.
Aktiviti: Pembelauan gelombang bunyi
Activity: Diffraction of sound waves
X Y Z

Pembesar suara
Loudspeaker

Prosedur/ Procedure:
1. Bunyi berfrekuensi 5 000 Hz dipancarkan.
A sound with a frequency of 5 000 Hz is transmitted.
2. Tiga orang murid ditanya sama ada mendengar bunyi tersebut di kedudukan masing-masing
dan keputusannya dicatatkan dalam jadual berikut.
Three students are asked if they hear the sound at their respective positions and the results are recorded in the following table.
3. Aktiviti diulang dengan menggunakan bunyi berfrekuensi 500 Hz.
The activity is repeated with a sound with a frequency of 500 Hz.
Keputusan/ Result:
Lengkapkan jadual berikut.
Complete the following table.
Frekuensi/ Frequency (Hz) X Y Z

Boleh didengari Boleh didengari Boleh didengari


500 Hz Can be heard Can be heard Can be heard

Tidak boleh didengari Boleh didengari Boleh didengari


5 000 Hz Cannot be heard Can be heard Can be heard

Latihan 16 Pembelauan dalam kehidupan harian


Diffraction in daily life
TP 3 Mengaplikasikan pengetahuan mengenai Gelombang untuk menerangkan kejadian atau fenomena alam dan melaksanakan tugasan mudah.
TP 4 Menganalisis pengetahuan mengenai Gelombang dalam konteks penyelesaian masalah mengenai kejadian atau fenomena alam.

Isi tempat kosong dengan perkataan yang sesuai tentang situasi yang melibatkan pembelauan.
Fill in the blanks with the suitable words about the situations involving diffraction. TP 3 TP 4

1 Benteng penahan (a) Apabila ombak memasuki celah , ia akan dibelau .


pelabuhan When the waves enter the gap , it will be diffracted .
Embankment barrier in the
port (b) Amplitud ombak berkurang. Hal ini menunjukkan bahawa
tenaga gelombang ombak juga berkurang.
The amplitude of the waves decreases. This shows that the energy of
the waves also decreases.
(c) Kawasan air menjadi tenang . Hal ini menyebabkan
kawasan pelabuhan sesuai dijadikan tempat persinggahan
Celah
Gap kapal, dapat mengurangkan kerosakan kapal dan mengelakkan
hakisan pantai.
The water area becomes calm . This makes the port area suitable for
berthing for ship, reducing ship damage and preventing erosion

of the beach.

149

Strategi A+ SPM Fizik Tg4_05_updated.indd 149 18/11/2023 2:58:24 PM


2 Hologram
(a) Hologram terhasil daripada kesan pembelauan gelombang.
Hologram is the result of the effect of wave diffraction .
(b) Hologram boleh digunakan sebagai tanda keselamatan pada
kad bank , kad pengenalan dan sebagainya.
Hologram can be used as security marks on bank cards,
identity cards and so on.

3 Gelombang bunyi (a) Gelombang infrasonik dengan frekuensi kurang daripada


infrasonik haiwan 20 Hz dan panjang gelombang yang panjang
Animal infrasonic sound dibelau
waves menyebabkan ia mudah .
Infrasonic waves with frequencies less than 20 Hz and long

wavelength causes them to be easily diffracted .


(b) Gelombang infrasonik memudahkan komunikasi antara
sesetengah spesies haiwan pada jarak jauh.
Infrasonic waves facilitate communication among some species of animals at
long distances.

5.6 Interferens Gelombang


Interference of Waves

Latihan 17 Interferens gelombang


Interference of waves
TP 1 Mengingat kembali pengetahuan dan kemahiran sains mengenai Gelombang.
TP 2 Memahami Gelombang serta dapat menjelaskan kefahaman tersebut.

1 Isi tempat kosong dengan perkataan yang sesuai tentang interferens gelombang. TP 1
Fill in the blanks with the suitable words about interference of waves.
(a) Prinsip superposisi menyatakan bahawa:
Principle of superposition states that:
Apabila dua gelombang bersuperposisi atau bertindih, sesaran paduan ialah hasil tambah
sesaran setiap gelombang tersebut.
When two waves are in superposition or overlapping, the resultant displacement is the sum

of the displacement of each wave.


(b) Interferens gelombang ialah superposisi dua gelombang dari dua sumber gelombang yang
koheran .
Interference of wave is the superposition of two waves from two wave sources which are coherent .
(c) Sumber-sumber koheren merujuk dua gelombang yang mempunyai frekuensi yang
sama dan beza fasa adalah tetap .
Coherent sources refer to two waves with the same frequency and the phase difference is constant .

2 Terdapat dua jenis interferens, iaitu interferens membina dan interferens memusnah. Lengkapkan
jadual berikut dengan melakar gelombang yang terhasil daripada interferens gelombang.
Kemudian, tulis ungkapan yang menunjukkan sesaran paduan gelombang itu. TP 2
There are two types of interference, constructive interference and destructive interference. Complete the following tables
by sketching the waves resulted from the interference of waves. Then, write the expressions that show the resultant
displacements of the waves.

150

Strategi A+ SPM Fizik Tg4_05_updated.indd 150 18/11/2023 10:22:21 AM


(a) Interferens membina:
Constructive interference:

(i) Superposisi dua puncak untuk menghasilkan satu puncak dengan amplitud
maksimum .
The superpositioning of two crests to produce a crest of maximum amplitude.

Sebelum superposisi Semasa superposisi


Before superposition During superposition

2A

A A

Sesaran paduan/ Resultant displacement


= A + A = 2A

(ii) Superposisi dua lembangan untuk menghasilkan satu lembangan dengan amplitud
maksimum .
The superpositioning of two troughs to produce a trough of maximum amplitude.

Sebelum superposisi Semasa superposisi


Before superposition During superposition

−2A

−A −A

Sesaran paduan/ Resultant displacement


= –A + (–A) = –2A

(b) Interferens memusnah:


Destructive interference:

Superposisi puncak dan lembangan untuk membatal sesama sendiri dan menghasilkan
sesaran paduan sifar .
The superpositioning of a crest and a trough to cancel each other, resulting in zero

resultant displacement.

Sebelum superposisi Semasa superposisi


Before superposition During superposition

−A Sesaran paduan/ Resultant displacement


= A + (–A) = 0

151

Strategi A+ SPM Fizik Tg4_05_updated.indd 151 18/11/2023 10:22:22 AM


Latihan 18 Corak interferens gelombang
Interference pattern of waves
TP 3 Mengaplikasikan pengetahuan mengenai Gelombang untuk menerangkan kejadian atau fenomena alam dan melaksanakan tugasan mudah.
TP 4 Menganalisis pengetahuan mengenai Gelombang dalam konteks penyelesaian masalah mengenai kejadian atau fenomena alam.

Lengkapkan perbincangan bagi aktiviti yang berikut. TP 3 TP 4


Complete the discussion for the following activities.
Aktiviti 1: Corak interferens gelombang air
Activity 1: Interference pattern of water waves
Lampu
Lamp

Ke bekalan kuasa
To power supply
Stroboskop
Air Stroboscope
Gelang getah Water
Rubber band

Pencelup sfera
Spherical dipper Span
Sponge

Skrin putih
White screen

Prosedur:
Procedure:
1. Penjana gelombang dihidupkan untuk menghasilkan dua gelombang yang bertindih.
The wave generator is switched on to produce two overlapping waves.
2. Corak gelombang yang terhasil diperhatikan.
The resulted wave pattern is observed.
Perbincangan:
Discussion:
Berdasarkan corak yang diperoleh, laksanakan tugasan berikut.
Based on the pattern obtained, carry out the following tasks.
(a) Labelkan kawasan cerah, kawasan gelap dan kawasan kelabu pada rajah berikut.
Label the bright, dark and grey areas in the following diagram.

(i) Kawasan kelabu


Grey area

S1

(ii) Kawasan gelap


Dark area

S2

(iii) Kawasan cerah


Bright area

152

Strategi A+ SPM Fizik Tg4_05_updated.indd 152 18/11/2023 10:22:23 AM


(b) Interferens membina berlaku di kawasan cerah dan kawasan gelap , manakala
interferens memusnah berlaku di kawasan kelabu .
Constructive interference occurs in bright areas and dark areas, while destructive interference occurs
in grey areas.

(c) Apakah yang berlaku di


What happens in the
(i) kawasan cerah/ bright areas?
Dua puncak bersuperposisi/ Two crests superimposed

(ii) kawasan gelap/ dark areas?


Dua lembangan bersuperposisi/ Two troughs superimposed
(iii) kawasan kelabu/ grey areas?
Puncak dan lembangan bersuperposisi/ A crest and a trough superimposed

Aktiviti 2: Corak interferens gelombang cahaya (Eksperimen Dwicelah Young)


Activity 2: Interference pattern of light waves (Young’s Double-slit experiment)

Dwicelah Skrin
Double-slit Screen

Pen laser
Laser pen

Prosedur:
Procedure:
1. Alur cahaya laser ditujukan melalui dwicelah.
The laser beam is directed through the double-slit.
2. Corak gelombang yang terhasil pada skrin diperhatikan.
The resulted wave pattern on the screen is observed.
3. Aktiviti diulang dengan jarak pemisahan dwicelah yang lebih besar.
The activity is repeated with a larger double-slit separation distance.

Perbincangan/ Discussion:
Berdasarkan corak yang diperoleh, laksanakan tugasan berikut.
Based on the pattern obtained, carry out the following tasks.

(a) Isi tempat kosong dengan perkataan yang sesuai. Kemudian, labelkan pinggir cerah dan pinggir
gelap pada rajah berikut.
Fill in the blanks with the correct words. Then, label the bright fringe and dark fringe in the following diagrams.

(i) Jarak pemisahan celah adalah lebih kecil . (ii) Jarak pemisahan celah adalah lebih besar .
The slit separation distance is smaller . The slit separation distance is bigger .

1. Pinggir cerah 2. Pinggir gelap 1. Pinggir gelap 2. Pinggir cerah


Bright fringe Dark fringe Dark fringe Bright fringe

Diperhatikan bahawa kedudukan pinggir cerah dan pinggir gelap adalah berselang-seli .
It is observed that the positions of the bright fringe and dark fringe are alternated .

153

Strategi A+ SPM Fizik Tg4_05_updated.indd 153 18/11/2023 10:22:24 AM


(b) Pinggir cerah terhasil disebabkan oleh interferens membina , manakala pinggir gelap terhasil
disebabkan oleh interferens memusnah .
Bright fringes are produced due to constructive interference , while dark fringes are produced due to
destructive interference .

(c) Labelkan semua kawasan interferens membina dan memusnah pada rajah berikut berdasarkan
superposisi gelombang yang ditunjukkan.
Label the constructive and destructive interference regions in the following diagram based on the superpositions of waves
shown.

(i) Interferens membina


Constructive interference

(ii) Interferens memusnah


Destructive interference

Sumber cahaya
monokromatik
(iii) Interferens membina
Monochromatic Constructive interference
light source

Sisip (iv) Interferens memusnah


dwicelah Destructive interference
Double-
slit plate

Skrin (v) Interferens membina


Screen
Constructive interference

Aktiviti 3: Corak interferens gelombang bunyi


Activity 3: Interference pattern of sound waves

S1
Penjana
isyarat
audio
Audio
signal
generator

S2

Prosedur:
Procedure:
1. Garis PQ berada 1.0 meter dari penjana isyarat audio. Penjana isyarat radio dilaraskan kepada
1 000 Hz.
The line PQ is located 1.0 metres from the audio signal generator. The audio signal generator is adjusted to 1 000 Hz.
2. Seorang murid berjalan perlahan-lahan sepanjang garis PQ.
A student walks slowly along line PQ.
3. Kenyaringan bunyi yang didengar oleh murid itu sepanjang PQ diambil kira.
The loudness of the sound heard by the student along PQ is considered.

154

Strategi A+ SPM Fizik Tg4_05_updated.indd 154 18/11/2023 10:22:24 AM


Perbincangan:
Discussion:
Berdasarkan kenyaringan bunyi yang didengari, laksanakan tugasan berikut.
Based on the loudness of sound heard, carry out the following tasks.
(a) Bunyi kuat dan perlahan didengari secara berselang-seli.
Loud and soft sounds are heard alternately.
(b) Jika murid itu memegang mikrofon yang disambung kepada osiloskop sinar katod (O.S.K) semasa
bergerak sepanjang PQ, paparan pada skrin O.S.K adalah seperti yang ditunjukkan dalam rajah
berikut.
If the student holds a microphone connected to a cathode ray oscilloscope (C.R.O) while moving along PQ, the display on
the C.R.O screen will be as shown in the following diagram.
Isi petak kosong dalam rajah menggunakan istilah yang diberi.
Fill in the empty boxes in the diagram using the terms given.

Interferens memusnah (bunyi perlahan) Interferens membina (bunyi kuat)


Destructive interference (soft sound) Constructive interference (loud sound)

P
S1 Skrin O.S.K.
C.R.O. screen
Penjana (i) Interferens memusnah (bunyi perlahan)
isyarat
Destructive interference (soft sound)
audio
Audio
signal
generator (ii) Interferens membina (bunyi kuat)
S2 Constructive interference (loud sound)

Latihan 19 Menghuraikan corak interferens gelombang


Describing the interference pattern of waves
TP 2 Memahami Gelombang serta dapat menjelaskan kefahaman tersebut.
TP 3 Mengaplikasikan pengetahuan mengenai Gelombang untuk menerangkan kejadian atau fenomena alam dan melaksanakan tugasan mudah.

Rajah berikut menunjukkan corak interferens gelombang. Jawab soalan yang berikut berdasarkan rajah.
The following diagram shows the interference pattern of waves. Answer the following questions based on the diagram.
1 Labelkan garis antinod dan garis nod dalam rajah berikut. TP 2
Label the antinodal and nodal lines in the following diagram.
(c) Garis antinod
Antinodal line
(b) Garis antinod (d) Garis antinod
Antinodal line Antinodal line

(a) Garis nod (e) Garis nod


Nodal line Z Nodal line

D Y

C
X

B
W
A

S1 S2
Kunci/ Key:
Puncak/ Crest
Lembangan/ Trough

155

Strategi A+ SPM Fizik Tg4_05_updated.indd 155 18/11/2023 10:22:24 AM


2 Namakan titik-titik TP 3
Name the points
(a) A, B, C dan/ and D
Titik-titik antinod/ Antinode points

(b) W, X, Y dan/ and Z


Titik-titik nod/ Node points

3 Apakah yang berlaku pada titik-titik TP 3


What happens at the points

(a) A, B, C dan/ and D?


Interferens membina/ Constructive interference

(b) W, X, Y dan/ and Z?


Interferens memusnah/ Destructive interference

4 Lukis rajah superposisi di titik C, D dan Z pada ruangan berikut. TP 3


Draw the superposition diagrams at points C, D and Z in the following spaces.

(a) Titik C/ Point C

+ =

(b) Titik D/ Point D

+ =

(c) Titik Z/ Point Z

+ =

156

Strategi A+ SPM Fizik Tg4_05_updated.indd 156 18/11/2023 10:22:25 AM


Latihan 20 Menghubung kait pemboleh ubah dalam corak interference gelombang
Relating variables in the interference pattern of waves
TP 1 Mengingat kembali pengetahuan dan kemahiran sains mengenai Gelombang.
TP 2 Memahami Gelombang serta dapat menjelaskan kefahaman tersebut.

1 Tulis pemboleh ubah dalam corak interferens gelombang yang diwakili oleh simbol berikut.
Kemudian, labelkan corak interferens gelombang yang diberi dengan simbol tersebut pada rajah di
bawah. TP 1
Write the variables in the interference pattern of waves that represented by the following symbols. Then, label the
interference pattern of waves given with the symbols on the diagram below.

Simbol/ Symbol Pemboleh ubah/ Variable


Panjang gelombang
λ Wavelength

Jarak pemisahan di antara dua sumber koheren


a The separation distance between two coherent sources

Jarak pemisahan di antara dua garis antinod atau nod yang bersebelahan
x The separation distance between two adjacent antinodal or nodal lines

Jarak tegak dari sumber koheren ke kedudukan x yang diukur


D The perpendicular distance from the coherent source to the position where x is measured

a
S1 S2

2 Jawab soalan berkaitan dengan


hubungan antara λ, a, x dan D
dalam corak interferens gelombangS1 S1 S1 S1
yang ditunjukkan dalam rajah
berikut. TP 2
Answer the questions related to the
a a x x
relationship between λ, a, x and D in the a a x x
interference pattern of waves shown in the
following diagrams.
Perhatikan Rajah 1(a) dan Rajah 1(b).S2 S2 S2 S2
Observe Diagrams 1(a) and 1(b).

D D D D

Rajah 1(a)/ Diagram 1(a) Rajah 1(b)/ Diagram 1(b)

157

Strategi A+ SPM Fizik Tg4_05_updated.indd 157 18/11/2023 10:22:25 AM


(a) Nyatakan
State
(i) pemboleh ubah dimanipulasikan
the manipulated variable
Panjang gelombang/ Wavelength, λ

(ii) pemboleh ubah bergerak balas


the responding variables
Jarak pemisahan di antara dua garis antinod atau nod yang bersebelahan, x
The separation distance between two adjacent antinodal or nodal lines, x

(iii) pemboleh ubah dimalarkan


the constant variables
Jarak pemisahan di antara dua sumber koheren, a dan jarak tegak dari sumber koheren ke
kedudukan x yang diukur, D
The separation distance between two coherent sources, a and the perpendicular distance from coherent sources to the

position where x is measured, D

(b) Nyatakan hubungan antara panjang gelombang, λ dan jarak pemisahan di antara dua garis
antinod atau nod yang bersebelahan, x.
State the relationship between the wavelength, λ and the separation distance between two antinodal or
nodal lines, x.
Semakin bertambah panjang gelombang, λ, semakin bertambah jarak pemisahan di antara dua garis
antinod atau nod yang bersebelahan, x.
The longer the wavelength, λ, the more the separation distance between two adjacent antinodal or nodal lines, x.

(λ ∝ x)

Perhatikan Rajah 2(a) dan Rajah 2(b).


Observe Diagrams 2(a) and 2(b).

S1

S1

a x a x

S2
S2

D D

Rajah 2(a)/ Diagram 2(a) Rajah 2(b)/ Diagram 2(b)

(c) Nyatakan
State
(i) pemboleh ubah dimanipulasikan
the manipulated variable
Jarak pemisahan di antara dua sumber koheren, a
The separation distance between two coherent sources, a

158

Strategi A+ SPM Fizik Tg4_05_updated.indd 158 18/11/2023 10:22:25 AM


(ii) pemboleh ubah bergerak balas
the responding variable
Jarak pemisahan di antara dua garis antinod atau nod yang bersebelahan, x
The separation distance between two adjacent antinodal or nodal lines, x

(iii) pemboleh ubah dimalarkan


the constant variables
Panjang gelombang, λ dan jarak tegak dari sumber koheren ke kedudukan x yang diukur, D
Wavelength, λ and the perpendicular distance from the coherent sources to the position x is measured, D

(d) Nyatakan hubungan antara jarak pemisahan di antara dua sumber koheren, a dan jarak
pemisahan di antara dua garis antinod atau nod yang bersebelahan, x.
State the relationship between the separation distance between two coherent sources, a and the separation distance
between two antinodal or nodal lines, x.
Semakin bertambah jarak pemisahan di antara dua sumber koheren, a, semakin berkurang jarak
pemisahan di antara dua garis antinod atau nod yang bersebelahan, x.
The more the separation distance between two coherent sources, a, the less the separation distance between two adjacent antinodal

or nodal lines, x.
1
(x ∝ a )

(e) Berdasarkan semua rajah, apakah yang berlaku kepada x apabila D bertambah?
Based on all diagrams, what happens to x when D increases?
Apabila D bertambah, x bertambah./ When D increases, x increases.
(D ∝ x)

(f) Berdasarkan jawapan di 2(b), (d) dan (e), nyatakan hubungan antara λ, a, x dan D.
Based on the answers in 2(b), (d) and (e), state the relationship between λ, a, x and D.
ax
λ=
D

Latihan 21 Menyelesaikan masalah melibatkan interferens gelombang


Solving problems involving interference of waves
TP 3 Mengaplikasikan pengetahuan mengenai Gelombang untuk menerangkan kejadian atau fenomena alam dan melaksanakan tugasan mudah.
TP 4 Menganalisis pengetahuan mengenai Gelombang dalam konteks penyelesaian masalah mengenai kejadian atau fenomena alam.
TP 5 Menilai pengetahuan mengenai Gelombang dalam konteks penyelesaian masalah dan membuat keputusan untuk melaksanakan satu tugasan.

Selesaikan masalah yang berikut.


Solve the following problems.

1 Hitung panjang gelombang, λ dalam setiap corak interferens gelombang berikut. TP 3


Calculate the wavelength, λ in each of the following interference patterns of waves.

(a)
5m x = 5 m, a = 4 m, D = 10 m
ax
λ=
D
10 m 4(5)
=
10
= 2 m
4m

159

Strategi A+ SPM Fizik Tg4_05_updated.indd 159 18/11/2023 10:22:26 AM


(b) 2m x = 2 × 2 = 4 cm, a = 3 cm, D = 2 × 100 = 200 cm
ax
λ=
D
A 3(4)
=
3 cm 200
2 cm
B = 0.06 cm

2 Dalam satu eksperimen dwicelah Young, jarak di antara dua celah yang berada sejauh 1.50
m dari skrin ialah 0.1 mm. Cahaya dari sumber yang jauh dengan panjang gelombang 500 nm
merambat melalui celah. Hitung jarak di antara dua pinggir cerah berturutan yang terbentuk pada
skrin. TP 4
In a Young's double-slit experiment, the distance between the two slits which are 1.50 m from the screen is 0.1 mm.
Light from a distant source with a wavelength of 500 nm propagates through the slits. Calculate the distance between two
consecutive bright fringes formed on the screen.
a = 0.1 mm = 0.1 × 10–3 m, D = 1.50 m, λ = 500 nm = 500 × 10–9 m
ax
λ =
D
(0.1 × 10–3)x
500 × 10–9 =
1.50
(500 × 10–9)(1.50)
x = = 0.0075 m = 7.5 mm
0.1 × 10–3

3 Gelombang bunyi yang koheren dengan frekuensi 500 Hz dipancarkan dari dua pembesar suara.
Seorang lelaki yang berjalan 30 m di hadapan pembesar suara boleh mendengar bunyi kuat dan
bunyi lemah secara berselang-seli. TP 5
Coherent sound waves with the frequency of 500 Hz are emitted from two loudspeakers. A man walking 30 m in front of
the loudspeakers can hear alternating loud and soft sounds.

30 m
Penjana isyarat
Signal generator

4m Laluan yang dilalui lelaki itu


The route the man walked

Pembesar suara
Loudspeakers

Jika laju gelombang bunyi itu ialah 330 m s–1, berapakah jarak pemisahan di antara dua titik
berturutan bagi bunyi kuat yang didengari lelaki itu? KBAT Menilai
If the speed of sound waves is 330 m s–1, what is the separation distance between two adjacent points for loud sounds
heard by the man?
v = 330 m s–1, a = 4 m, D = 30 m, f = 500 Hz
Laju/ Speed, v = fλ
330 = 500λ
330
λ = = 0.66 m
500
ax
dan/ and, λ =
D
4x
0.66 =
30
(30)(0.66)
x =
4
= 4.95 m

160

Strategi A+ SPM Fizik Tg4_05_updated.indd 160 18/11/2023 10:22:26 AM


Latihan 22 Aplikasi interferens dalam kehidupan seharian
Applications of interference in daily life
TP 3 Mengaplikasikan pengetahuan mengenai Gelombang untuk menerangkan kejadian atau fenomena alam dan melaksanakan tugasan mudah.
TP 4 Menganalisis pengetahuan mengenai Gelombang dalam konteks penyelesaian masalah mengenai kejadian atau fenomena alam.

Isi tempat kosong dalam jadual berikut tentang aplikasi interferens dalam kehidupan seharian.
Fill in the blanks in the following table about the applications of interference in daily life. TP 3 TP 4

1 Luan bebuli kapal depan


Bulbous bow of ship (a) Luan kapal ialah bahagian badan kapal yang direka
untuk mengurangkan tekanan ketika kapal membelah air.
The bow of ship is the front part of the hull of the ship which is designed
to reduce resistance when the ship breaks the water.

(b) Luan bebuli menjana gelombang air yang membentuk


interferens memusnah dengan gelombang air yang dijana
oleh luan kapal .
A bulbous bow generates water waves that form destructive interference
Bebuli
Bulb with water waves generated by the bow of ship .

(c) Hal ini menjadikan air di sekitar kapal lebih tenang dan
dapat mengurangkan seretan .
This makes the water around the ship calmer and can reduce drag

2 Kanta anti-pantulan salutan


Anti-reflective lens (a) Terdapat pada kanta anti-pantulan agar cahaya
terpantul akan membentuk interferens memusnah .
There is a coating on anti-reflective lenses so that the reflected light will
form destructive interference.

(b) Penglihatan menjadi lebih jelas dan imej tidak


terbentuk pada kanta cermin mata.
Vision becomes clearer and images are not formed on the
Salutan/ Coating spectacle lenses.
Kanta/ Lens

5.7 Gelombang Elektromagnet


Electromagnetic Waves
Latihan 23 Ciri-ciri gelombang elektromagnet
Characteristics of electromagnetic waves
TP 1 Mengingat kembali pengetahuan dan kemahiran sains mengenai Gelombang.
TP 2 Memahami Gelombang serta dapat menjelaskan kefahaman tersebut.

1 Lengkapkan pernyataan di bawah tentang gelombang elektromagnet. TP 1


Complete the statement below about electromagnetic waves.

Gelombang elektromagnet terdiri daripada medan elektrik dan medan magnet yang
berayun secara berserenjang antara satu sama lain .
Electromagnetic waves consist of electric field and magnetic field that oscillate
perpendicular to each other .

161

Strategi A+ SPM Fizik Tg4_05_updated.indd 161 18/11/2023 10:22:27 AM


2 Lukis medan-medan yang
menerangkan gelombang
elektromagnet pada paksi Medan elektrik
Electric field
dalam rajah di sebelah. TP 2
Draw the fields that describe Medan magnet
Magnetic field
electromagnetic waves on the axis
in the diagram on the right.

Arah perambatan
Direction of propagation

3 Rajah berikut menunjukkan spektrum elektromagnet. Lengkapkan pernyataan dalam anak panah
dan label jenis gelombang elektromagnet dalam rajah. TP 2
The following diagram shows electromagnetic spectrum. Complete the statements in the arrows and label the types of
electromagnetic waves in the diagram.

(i) Frekuensi semakin bertambah.


Frequency increases.

(ii) Panjang gelombang semakin bertambah.


Wavelength increases.

Panjang
10–11 10–10 10–9 10–8 10–7 10–6 10–5 10–4 10–3 10–2 10–1 1 101 102 103 104 gelombang (m)
Wavelength

1020 1019 1018 1017 1016 1015 1014 1013 1012 1011 1010 109 108 107 106 105 104 Frekuensi (Hz)
Frequency

(d)
Panjang gelombang (nm)
400 500 600 700 750 Wavelength

(a) Sinar gama/ Gamma ray (e) Sinaran inframerah/ Infrared ray
(b) Sinar-X/ X-ray (f) Gelombang mikro/ Microwave
(c) Sinaran ultraungu/ Ultraviolet ray (g) Gelombang radio/ Radio wave
(d) Cahaya nampak/ Visible light

162

Strategi A+ SPM Fizik Tg4_05_updated.indd 162 18/11/2023 10:22:28 AM


4 Lengkapkan peta buih berikut tentang ciri-ciri gelombang elektromagnet. TP 2
Complete the following bubble map about the characteristics of electromagnetic waves.
i-THINK Peta Buih

(b)
(a) Tidak memerlukan
Gelombang medium untuk
melintang merambat
Transverse wave No need medium
to propagate

(c)
(e) Ciri-ciri Bergerak pada
Boleh merambat gelombang kelajuan cahaya dalam
elektromagnet vakum , c
melalui vakum
Characteristics
Can propagate through Moves at the speed of light in
of electromagnetic
vacuum vacuum ,c
waves
c= 3.0 × 108 m s–1

(d)
Menunjukkan sifat-sifat gelombang, iaitu
pantulan , pembiasan , pembelauan dan interferens
Show the properties of waves which are reflection ,
refraction , diffraction and interference

Latihan 24 Aplikasi spektrum elektromagnet dalam kehidupan seharian


Applications of electromagnetic spectrum in daily life
TP 3 Mengaplikasikan pengetahuan mengenai Gelombang untuk menerangkan kejadian atau fenomena alam dan melaksanakan tugasan mudah.
TP 4 Menganalisis pengetahuan mengenai Gelombang dalam konteks penyelesaian masalah mengenai kejadian atau fenomena alam.

Lengkapkan jadual berikut tentang aplikasi spektrum elektromagnet dalam kehidupan seharian.
Complete the following table about the applications of electromagnetic spectrum in daily life. TP 3 TP 4

Jenis
Kegunaan
gelombang Application
Type of wave

1 Sinar gama sel kanser


Gamma ray (a) Membunuh dalam radioterapi
Kill cancer cells in radiotherapy

(b) Pensterilan peralatan pembedahan dan perubatan (secara pukal)


Sterilisation of surgical and medical equipment (in large quantities)
(c) Digunakan dalam industri pemprosesan makanan supaya
makanan tahan lebih lama
Used in the food processing industry so that food lasts longer

163

Strategi A+ SPM Fizik Tg4_05_updated.indd 163 18/11/2023 10:22:28 AM


Jenis
Kegunaan
gelombang Application
Type of wave

2 Sinar-X (a) Imej sinar-X membantu doktor mengesan retakan atau patah pada
X-ray
tulang dan memeriksa organ dalaman
X-ray images help doctors to detect cracks or fractures in bones and examine
internal organs

(b) Pemeriksaan sambungan kimpalan


Inspection of welding joints

(c) Pengimbas bagasi di lapangan terbang


Baggage scanner at airport

(d) Mengimbas keretakan dalam struktur bangunan


Scanning for cracks in building structures

(e) Mengesan pemalsuan hasil seni


Detecting art forgery

3 Sinaran (a) Rawatan penyakit kuning pada bayi


ultraungu Treatment for jaundice in babies
Ultraviolet
ray (b) Mengeraskan bahan tampalan gigi
Hardens tooth filling material

(c) Menentukan kesahihan wang kertas atau mengenal pasti wang palsu
Determining the authenticity of banknotes or identifying counterfeit money

(d) Penghasilan vitamin D pada kulit


Production of vitamin D in the skin

(e) Pensterilan alat perubatan dan air


Sterilisation of medical devices and water

(f) Alat perangkap serangga


Insect trap

4 Cahaya (a) Membolehkan benda hidup untuk melihat


nampak Allows living things to see
Visible light
(b) Fotografi
Photography

(c) Fotosintesis dalam tumbuhan hijau


Photosynthesis in green plants

(d) Cahaya laser digunakan dalam pemotongan logam, ukur tanah dan
penghantaran maklumat melalui gentian optik
Laser light is used in metal cutting , land survey and transmitting information
through optical fibres .

164

Strategi A+ SPM Fizik Tg4_05_updated.indd 164 18/11/2023 10:22:29 AM


Jenis
Kegunaan
gelombang Application
Type of wave

5 Sinaran (a) Rawatan sakit otot


inframerah Treatment of muscle pain
Infrared ray
(b) Alat kawalan jauh untuk televisyen dan pemain DVD
Remote control for televisions and DVD players
(c) Untuk memasak (ketuhar, pemanggang dan pembakar)
For cooking (oven, grill and toaster)

(d) Untuk melihat dalam keadaan gelap ( kamera dan binokular


inframerah)
To see in the dark (infrared cameras and binoculars )

(e) Mengeringkan cat pada kereta


Drying paint on cars

6 Gelombang (a) Komunikasi antarabangsa melalui satelit


mikro International communication through satellites
Microwave
(b) Komunikasi antara alat elektronik
Communication between electronic devices

(c) Memasak (ketuhar gelombang mikro )


Cooking ( microwave oven)

(d) Radar dan perangkap laju


Radar and speed trap

7 Gelombang (a) Penyiaran radio dan televisyen


radio Radio and television broadcast
Radio wave
(b) Komunikasi tanpa wayar
Wireless communication

(c) Pengimbas gelombang-millimeter untuk mengimbas badan penumpang


di lapangan terbang
Millimeter-wave scanner to scan the body of passengers at airports

Praktis Berformat SPM


A Gelombang pegun/ Stationary waves
Kertas 1 B Gelombang progresif/ Progressive waves
C Gelombang melintang/ Transverse waves
1 Sejenis gelombang dengan keadaan zarah- D Gelombang membujur/ Longitudinal waves
zarah medium bergetar secara berserenjang
dengan arah perambatan gelombang 2 Pilih pernyataan yang betul.
A type of wave in which the particles of the medium vibrate Choose the correct statement.
in perpendicular to the direction of wave propagation A Frekuensi ialah bilangan ayunan lengkap
Frequency is the number of complete oscillations
Apakah jenis gelombang yang dimaksudkan
dalam pernyataan di atas?
What is the type of wave meant in the statement above?

165

Strategi A+ SPM Fizik Tg4_05_updated.indd 165 18/11/2023 10:22:29 AM


B Frekuensi berkadar terus dengan tempoh 5 Rajah 3 menunjukkan graf sesaran-jarak bagi
Frequency is directly proportional to period suatu gelombang dengan frekuensi 10 Hz.
C Tempoh ialah masa yang diambil bagi satu Diagram 3 shows the displacement-distance graph of
ayunan lengkap a wave with a frequency of 10 Hz.
Period is the time taken for one complete oscillation Sesaran (cm)
D Amplitud merupakan sesaran minimum Displacement
suatu zarah dari kedudukan asalnya
15
Amplitude is the minimum displacement of a
particle from its original position 0 Jarak (cm)
10 20 30 40 Distance
–15
3 Rajah 1 menunjukkan corak muka gelombang
yang terhasil daripada satu penggetar sfera
yang bergetar dengan frekuensi 12 Hz di Rajah 3/ Diagram 3
dalam sebuah tangki riak. Berapakah laju gelombang itu?
Diagram 1 shows a wavefront pattern produced by a What is the speed of the wave?
spherical dipper that vibrates at a frequency of 12 Hz in A 100 cm s–1
a ripple tank. B 200 cm s–1
Muka gelombang C 300 cm s–1
Wavefronts D 400 cm s–1
6 Rajah 4 menunjukkan muka gelombang suatu
6 cm gelombang satah yang ditujukan ke suatu
pemantul satah.
Diagram 4 shows the wavefronts of a plane wave
incident on a plane reflector.

Muka gelombang
Rajah 1/ Diagram 1 Wavefronts

Berapakah laju gelombang tersebut?


What is the speed of the wave?
A 12 cm s–1
B 24 cm s–1
C 36 cm s–1 Pemantul satah
D 72 cm s–1 Plane reflector

Rajah 4/ Diagram 4
4 Rajah 2 menunjukkan graf sesaran-masa bagi
Pernyataan yang manakah adalah betul
satu sistem bergetar yang mengalami suatu
tentang gelombang tuju dan gelombang
fenomena.
pantulan?
Diagram 2 shows the displacement-time graph of
Which statement is correct about the incident and
a vibrating system that experiences a phenomenon.
reflected waves?
Sesaran A Laju gelombang pantulan adalah sama
Displacement
dengan laju gelombang tuju
The speed of reflected wave is the same as the speed
A1 of incident wave
A2 A3 A4 Masa
0
T 2T 3T Time B Frekuensi gelombang pantulan lebih
tinggi daripada frekuensi gelombang tuju
The frequency of reflected wave is higher than that
Rajah 2/ Diagram 2 of the incident wave
Apakah fenomena yang dialami sistem itu? C Arah gelombang pantulan sentiasa
What is the phenomenon experienced by the system? bersudut tegak dengan arah gelombang
A Resonans tuju
The direction of reflected wave is always at right
Resonance
angle to the incident wave
B Rektifikasi
D Panjang gelombang bagi gelombang
Rectification
pantulan lebih besar daripada panjang
C Amplifikasi
gelombang bagi gelombang tuju
Amplification
The wavelength of reflected wave is greater than
D Pelembapan
that of the incident wave
Damping

166

Strategi A+ SPM Fizik Tg4_05_updated.indd 166 18/11/2023 10:22:30 AM


7 Rajah 5 menunjukkan sebuah kapal selam
memancarkan gelombang ultrasonik ke arah
batu besar di dasar laut. Selepas 50 milisaat,
Titisan air
kapal selam itu mengesan gelombang yang Water droplet
dipantulkan. Besen
Diagram 5 shows a submarine transmitting ultrasonic Basin
waves at a boulder on the seabed. After 50 milliseconds,
Rajah 6/ Diagram 6
the submarine detects the reflected waves.
Corak muka gelombang yang manakah akan
Gelombang pantulan
Reflected wave
diperhatikan dalam besen itu?
Which pattern of the wavefronts will be observed in the
basin?
Gelombang tuju A C
Original wave
Batu besar
Boulder
Dasar laut
Seabed

Rajah 5/ Diagram 5
Berapakah jarak kapal selam dari batu besar B D
itu?
What is the distance of the submarine from the boulder?
[Halaju gelombang ultrasonik/Ultrasonic wave
velocity = 1 500 m s–1]
A 37.5 m
10 Rajah 7 menunjukkan gelombang air laut
B 75.0 m
merambat melalui celah di antara dua batu
C 37 500 m
besar.
D 75 000 m
Diagram 7 shows the sea waves propagating through
8 Pilih padanan yang betul bagi ciri-ciri the gap between two big rocks.
gelombang air yang terlibat dalam fenomena
pembiasan di tepi pantai.
Choose the correct match for the characteristics of water
waves involved in the phenomenon of refraction at the
seaside.
Panjang
Frekuensi Laju
gelombang Frequency Speed
Wavelength
A Tidak Tidak Tidak A
berubah berubah berubah
No change No change No change
B Tidak Tidak
Berubah
berubah berubah
Change
No change No change
C Tidak Batu
Berubah Berubah
berubah Rocks
Change Change
No change Rajah 7/ Diagram 7
D Tidak Apakah fenomena yang dialami oleh
Berubah Berubah
berubah gelombang air di A?
Change Change
No change What is the phenomenon experienced by the waves
at A?
9 Rajah 6 menunjukkan sebuah besen berisi A Pantulan C Pembelauan
air diletakkan secara condong di bawah pili Reflection Diffraction
air. Air daripada pili itu menitis pada kadar B Pembiasan D Interferens
sekata ke dalam besen itu. Refraction Interference
Diagram 6 shows a basin filled with water placed tilted
under a tap. Water from the tap drips at an even rate
into the basin.

167

Strategi A+ SPM Fizik Tg4_05_updated.indd 167 18/11/2023 10:22:30 AM


11 Rajah 8 menunjukkan corak gelombang yang 6.5 × 10–7 m. Jarak pemisahan dua celah ialah
terbentuk apabila gelombang daripada dua 4.0 × 10–4 m dan jarak di antara dwicelah
sumber yang koheren, X dan Y bertindih. dengan skrin ialah 2.0 m.
Diagram 8 shows the wave pattern formed when waves Diagram 9 shows the apparatus set-up for a Young’s
from two coherent sources, X and Y overlap. double-slit experiment. The wavelength of the
B C monochromatic light is 6.5 × 10–7 m. The distance of
A
separation between the two slits is 4.0 × 10–4 m and the
E D
distance between the double-slit and the screen is 2.0 m.
Ke bekalan Skrin
kuasa Dwicelah Screen Corak
To power Double-slit interferens
X Y supply Interference
pattern
Kekunci/ Key:
Puncak/ Crest
Lembangan/ Trough

Rajah 8/ Diagram 8 Sumber cahaya monokromatik


Monochromatic light source
Pada kedudukan manakah interferens
Rajah 9/ Diagram 9
membina berlaku?
At which positions does constructive interference Berapakah jarak di antara dua pinggir gelap
occur? berturutan?
A A dan B C A dan D What is the distance between two consecutive dark
A and B A and D fringes?
B B dan C D D dan E A 1.30 × 10–10 m C 3.25 × 10–3 m
B and C D and E B 2.60 × 10–10 m D 6.50 × 10–3 m

12 Rajah yang manakah akan menghasilkan 14 Pernyataan yang manakah adalah tidak benar
interferens memusnah apabila gelombang- tentang ciri-ciri gelombang elektromagnet?
gelombang itu bertemu? Which statement is not correct about the characteristics
Which of the following diagrams will produce of electromagnetic waves?
destructive interference when the waves meet? A Tidak memerlukan medium untuk merambat
A + Does not require a medium to propagate
B Bergerak dengan laju cahaya,
c = 3.0 × 108 m s–1
Moves at the speed of light, c = 3.0 × 108 m s–1
C Gelombang elektromagnet ialah
gelombang melintang
B Electromagnetic waves are transverse waves
D Bergerak dengan laju melebihi laju cahaya,
c = 3.0 × 108 m s-1 dalam suatu medium
Moves at the speed more than the speed of light,
+ c = 3.0 × 108 m s–1 in a medium

15 Susunan manakah yang betul bagi gelombang


elektromagnet dalam turutan frekuensi yang
menurun?
Which arrangement of electromagnetic waves is correct
C in the order of decreasing frequency?
A Sinar-X, sinar gama, gelombang mikro,
sinaran inframerah
X-ray, gamma ray, microwave, infrared ray
B Sinar gama, sinar-X, sinaran inframerah,
+ gelombang mikro
Gamma ray, X-ray, infrared ray, microwave
D C Sinaran inframerah, gelombang mikro,
+ sinar gama, sinar-X
Infrared ray, microwave, gamma ray, X-ray
13 Rajah 9 menunjukkan susunan radas untuk D Gelombang mikro, sinaran inframerah,
suatu eksperimen dwicelah Young. Panjang sinar-X, sinar gama
gelombang cahaya monokromatik ialah Microwave, infrared ray, X-ray, gamma ray

168

Strategi A+ SPM Fizik Tg4_05_updated.indd 168 18/11/2023 10:22:31 AM


Kertas 2

Bahagian A

1 Rajah 1.1 menunjukkan seorang budak berayun di atas buaian setelah ianya ditolak. Selepas
beberapa minit, buaian menjadi perlahan dan akhirnya berhenti.
Diagram 1.1 shows a boy swinging on a swing after it is pushed. After a few minutes, the swing slows down and finally
stops.

Rajah 1.1/ Diagram 1.1

(a) Apakah fenomena yang berlaku?


What is the phenomenon that occurs?
Pelembapan/ Damping
[1 markah/mark]
(b) Beri satu sebab mengapa fenomena yang dinyatakan di 1(a) berlaku.
Give one reason why the phenomenon stated in 1(a) occurs.
Kehilangan tenaga/Rintangan udara
Energy loss/ Air resistance

[1 markah/mark]
(c) Lakarkan graf sesaran-masa yang mewakili fenomena di atas dalam Rajah 1.2.
Sketch a displacement-time graph that represents the above phenomenon in Diagram 1.2.

Sesaran (cm)
Displacement

Masa (s)
0 Time

Rajah 1.2/ Diagram 1.2


[2 markah/marks]

2 Jadual 1 menunjukkan empat jenis gelombang elektromagnet.


Table 1 shows four types of electromagnetic wave.

Jenis gelombang elektromagnet


Types of electromagnetic wave
Sinar gama/Gamma ray
Cahaya nampak/Visible light
Sinaran inframerah/Infrared ray
Gelombang radio/Radio wave
Jadual 1/ Table 1

169

Strategi A+ SPM Fizik Tg4_05_updated.indd 169 18/11/2023 10:22:31 AM


(a) Susun gelombang elektromagnet mengikut tertib panjang gelombang menurun.
Arrange the electromagnetic waves in the order of decreasing wavelength.
Gelombang radio, sinaran inframerah, cahaya nampak, sinar gama
Radio wave, infrared ray, visible light, gamma ray

[1 markah/mark]
(b) Berdasarkan Jadual 1, namakan satu jenis gelombang yang berasal dari Matahari.
Based on Table 1, name one type of wave that originates from the Sun.
Cahaya nampak/ Visible light
[1 markah/mark]
(c) Nyatakan satu kegunaan gelombang radio.
State one use of radio waves.
Komunikasi radio jarak jauh/Komunikasi tanpa wayar/Penyiaran radio dan televisyen tempatan
Long distance radio communication/ Wireless communication/ Local radio and television broadcasting

[1 markah/mark]
(d) Jika panjang gelombang bagi suatu gelombang mikro ialah 5.50 × 10–2 m, hitung frekuensi
gelombang itu.
If the wavelength of a microwave is 5.50 × 10–2 m, calculate the frequency of the wave.
[Laju cahaya dalam vakum/Speed of light in vacuum = 3.0 × 108 m s–1]
v = fλ
3.0 × 108 = f(5.50 × 10–2)
3.0 × 108
f=
5.50 × 10–2
= 5.455 × 109 Hz
[2 markah/marks]

3 Rajah 2.1 dan Rajah 2.2 menunjukkan corak pinggir cerah dan gelap yang dihasilkan apabila cahaya
monokromatik yang sama dipancarkan melalui dwicelah. Cahaya merah digunakan dalam aktiviti
ini.
Diagrams 2.1 and 2.2 show bright and dark fringes that are formed when identical monochromatic light passes through
the double-slits. Red light is used in this activity.

Skrin Skrin
Screen Screen

5m 5m

Dwicelah Dwicelah
Double-slit Double-slit

Rajah 2.1/ Diagram 2.1 Rajah 2.2/ Diagram 2.2

(a) Apakah maksud cahaya monokromatik?


What is the meaning of monochromatic light?
Cahaya yang mempunyai satu warna atau satu panjang gelombang.
Light that has one colour or one wavelength.

[1 markah/mark]

170

Strategi A+ SPM Fizik Tg4_05_updated.indd 170 18/11/2023 10:22:31 AM


(b) Terangkan bagaimana pinggir-pinggir cerah dan gelap terbentuk pada skrin seperti yang
ditunjukkan pada rajah. KBAT Menganalisis
Explain how the bright and dark fringes are formed on the screen as shown in the diagrams.
Pinggir-pinggir cerah terbentuk apabila berlakunya interferens membina. Pinggir-pinggir gelap terbentuk
apabila berlakunya interferens memusnah.
Bright fringes are formed when constructive interference occurs. Dark fringes are formed when destructive interference occurs.

[2 markah/marks]
(c) Berdasarkan Rajah 2.1 dan Rajah 2.2, bandingkan
Based on Diagrams 2.1 and 2.2, compare
(i) panjang gelombang,
the wavelengths,
Panjang gelombang adalah sama./The wavelengths are the same.
[1 markah/mark]
(ii) jarak di antara dua celah, a,
the distance between the two slits, a,
Jarak di antara dua celah dalam Rajah 2.1 kurang daripada dalam Rajah 2.2.
The distance between the two slits in Diagram 2.1 is less than that in Diagram 2.2.

[1 markah/mark]
(iii) jarak di antara dwicelah dan skrin, D,
the distance between the double-slit and screen, D,
Jarak di antara dwicelah dan skrin adalah sama./ The distances between the double-slits and the screens are the same.
[1 markah/mark]
(iv) jarak di antara dua pinggir gelap berturutan, x.
the distance between two consecutive dark fringes, x.
Jarak di antara dua pinggir gelap berturutan dalam Rajah 2.1 adalah lebih besar daripada dalam
Rajah 2.2./ The distance between two consecutive dark fringes in Diagram 2.1 is greater than that in Diagram 2.2.
[1 markah/mark]
(d) Nyatakan hubungan antara x dengan a.
State the relationship between x and a.
Semakin berkurang a, semakin bertambah x./ The more a decreases, the more x increases.
[1 markah/mark]
(e) Apakah yang berlaku kepada jarak di antara dua pinggir gelap berturutan jika cahaya merah
diganti dengan cahaya biru?
What will happen to the distance between two consecutive dark fringes if red light is replaced by blue light?
Berkurang/ Decreases
[1 markah/mark]

Bahagian B
4 (a) Rajah 3.1 menunjukkan corak gelombang air semasa menghampiri pantai.
Diagram 3.1 shows the pattern of water waves when approaching the shore.
Pantai
Laut Beach
Sea

Rajah 3.1/ Diagram 3.1

171

Strategi A+ SPM Fizik Tg4_05_updated.indd 171 18/11/2023 10:22:32 AM


(i) Namakan fenomena gelombang yang ditunjukkan dalam rajah.
Name the wave phenomenon shown in the diagram.
[1 markah/mark]
(ii) Terangkan fenomena gelombang yang menyebabkan gelombang air mengikut bentuk
garis pantai apabila ia menghampiri pantai. KBAT Menganalisis
Explain the phenomenon of waves that causes water waves to follow the shape of the shoreline as it approaches
the beach.
[4 markah/marks]

(b) Jadual 2 menunjukkan ciri-ciri empat tempat yang dipertimbangkan untuk digunakan sebagai
tempat bot nelayan berlabuh di pantai.
Table 2 shows the characteristics of places that are considered for use as the place for fishing boats to anchor at the beach.

Bahan pembinaan
Ketinggian
benteng penahan Saiz celah benteng
Tempat Lokasi benteng
ombak Size of slit of the
Place Location Height of the
Material of the barrier
barrier
embankment barrier

P Teluk/ Bay Kayu/ Wooden Rendah/ Short Besar/ Large

Q Tanjung/ Cape Konkrit/ Concrete Rendah/ Short Kecil/ Small

R Tanjung/ Cape Kayu/ Wooden Tinggi/ High Besar/ Large

S Teluk/ Bay Konkrit/ Concrete Tinggi/ High Kecil/ Small

Jadual 2/ Table 2

Kaji setiap ciri tempat tersebut dan terangkan kesesuaian setiap ciri. Tentukan tempat yang paling
sesuai untuk bot-bot nelayan berlabuh. KBAT Menilai
Study each characteristic of the places and explain the suitability of each characteristic. Determine the most suitable
place for fishing boats to anchor.
[10 markah/marks]

(c) Rajah 3.2 menunjukkan muka gelombang air dengan laju 8 cm s–1 merambat dari kawasan
P ke Q. Laju gelombang berkurang menjadi 5 cm s–1 apabila memasuki kawasan Q. Panjang
gelombang di kawasan Q ialah 2 cm.
Diagram 3.2 shows the wavefronts of a water wave with a speed of 8 m s–1 propagating from regions P to Q. The wave
speed decreases to 5 m s–1 when entering region Q. The wavelength in region Q is 2 cm.

Kawasan dalam
Deep region
P

Kawasan cetek
Shallow region

Rajah 3.2/ Diagram 3.2

Hitung/ Calculate
(i) frekuensi gelombang itu,
the frequency of the wave,
(ii) panjang gelombang di kawasan P.
the wavelength in region P.
[5 markah/marks]

172

Strategi A+ SPM Fizik Tg4_05_updated.indd 172 18/11/2023 10:22:32 AM


Jawapan/ Answers:
4 (a) (i) [Apa/ What] Pembiasan/ Refraction
(ii) [Bagaimana/ How]
– Gelombang air merambat dari kawasan dalam ke kawasan cetek.
Water waves propagate from deep region to shallow region.

– Panjang gelombang akan berkurang.


The wavelength of wave will decrease.

– Laju gelombang turut berkurang.


The wave speed also decreases.

– Arah perambatan gelombang dibias ke arah normal.


The direction of wave propagation is refracted towards the normal.

(b) [Apa/ What, mengapa/ Why]


Ciri/ Characteristics Penerangan/ Explanation
Lokasi: Teluk Gelombang lebih tenang

Location: Bay Waves are calmer.


Bahan pembinaan benteng: Konkrit Kuat

Material of the embankment barrier: Concrete Strong


Ketinggian benteng: Tinggi Menghalang ombak yang tinggi

Height of the barrier: High Prevents high waves


Saiz celah benteng: Kecil Pembelauan ketara/Tenaga gelombang lebih

Size of slit of the barrier: Small rendah

Apparent diffraction/ Low wave energy

Tempat Z dipilih kerana terletak di teluk, mempunyai benteng konkrit yang tinggi dan saiz celah benteng
yang kecil.
Z is chosen because it is located in the bay area, has a high concrete embankment barrier and a small slit of the barrier.

(c) (i) v = fλ
5 = f(2)
5 = 2.5 Hz
f =
2

(ii) λ = v
f

8
=
2.5

= 3.2 cm

173

Strategi A+ SPM Fizik Tg4_05_updated.indd 173 18/11/2023 10:22:32 AM


Bab Cahaya dan Optik
6 Light and Optics
Tema: Gelombang, Cahaya dan Optik


Revisi Pantas
Hukum Snell/ Snell’s Law
6.1 Pembiasan Cahaya n1 sin θ1 = n2 sin θ2
Refraction of Light
n2 sin θ1
=
1 Pembiasan cahaya ialah perubahan arah atau n1 sin θ2
pembengkokan cahaya apabila ia merambat dari suatu
n1 = Indeks biasan medium 1/ Refractive index of medium 1
medium ke medium yang berbeza ketumpatan optik.
Refraction of light is the change in direction or bending of light n2 = Indeks biasan medium 2/ Refractive index of medium 2
when it propagates from a medium to a medium of different optical θ1 = Sudut tuju dalam medium 1
densities. Angle of incidence in medium 1
θ2 = Sudut biasan dalam medium 2
2 Perubahan arah perambatan cahaya ini disebabkan Angle of refraction in medium 2
oleh perubahan halaju cahaya apabila merambat
melalui medium yang mempunyai ketumpatan optik 6 Hubungan antara indeks biasan, n suatu medium,
yang berbeza. dalam nyata, H dan dalam ketara, h diberi sebagai
This change in direction of light propagation is due to the change in The relationship between the refractive index, n of a medium,
velocity of light when propagating through mediums with different real depth, H and apparent depth, h is given by
optical densities.
H
n=
3 Ketumpatan optik tidak sama dengan ketumpatan h
fizikal, 𝜌.
Optical density is not the same as physical density, 𝜌.
6.2 Pantulan Dalam Penuh
4 Indeks biasan bagi suatu medium boleh didefinisikan Total Internal Reflection
sebagai nisbah laju cahaya dalam vakum kepada laju
cahaya dalam medium. 1 Pantulan dalam penuh hanya berlaku apabila cahaya
The refractive index of a medium can be defined as the ratio of the merambat dari medium berketumpatan optik tinggi ke
speed of light in vacuum to the speed of light in medium. medium berketumpatan optik rendah.
Total internal reflection occurs only when light travels from a
Laju cahaya dalam vakum medium of higher optical density to a medium of lower optical
Speed of light in vacuum density.
Indeks biasan, n =
Refractive index Laju cahaya dalam medium 2 Apabila sudut biasan dalam medium yang
Speed of light in medium berketumpatan optik rendah ialah 90º, sudut tuju
n = c dalam medium yang berketumpatan optik tinggi
v disebut sebagai sudut genting, c.
c = 3.0 × 108 m s–1 When the angle of refraction in the medium of lower optical density
is 90º, the angle of incidence in the medium of higher optical density
5 Hukum pembiasan menyatakan bahawa is called critical angle, c.
Law of refraction states that
3 Hubungan antara sudut genting, c dengan indeks
(a) sinar tuju, sinar biasan, dan garis normal bertemu biasan, n suatu medium diberi
pada satu titik dan berada dalam satah yang sama, The relationship between the critical angle, c and the refractive
the incident ray, the refracted ray, and the normal meet at one index, n for a medium is given by
point and all lie in the same plane,
1
n=
(b) nisbah sin sudut tuju kepada sin sudut biasan sin c
adalah malar.
the ratio of the sine of the angle of incidence to the sine of the
angle of refraction is constant. 6.3 Pembentukan Imej oleh Kanta
sin i Image Formation by Lenses
Hukum Snell
= malar/ constant
sin r Snell’s Law
1 Terdapat dua jenis kanta/ There are two types of lenses:
(a) Kanta cembung/ Convex lens
(b) Kanta cekung/ Concave lens

θ1
n1
n2
θ2

Kanta cembung Kanta cekung


Convex lens Concave lens

174

06 Strategi A+ SPM Fizik Tg 4 (B06) 5pp.indd 174 18/11/2023 10:17 AM


2 Perbezaan antara kanta cembung dengan kanta cekung:
The differences between convex lens and concave lens:

Kanta cembung Kanta cekung


Convex lens Concave lens

Titik fokus
Titik fokus
Focal point
Paksi Focal point Paksi utama
utama O F F O Principal axis
Principal
axis

Sinar-sinar tuju yang selari menumpu di titik fokus, F Sinar-sinar tuju yang selari mencapah dari titik fokus, F
Parallel incident rays converge on the focal point, F Parallel incident rays diverge from the focal point, F

Titik fokus kanta cembung dikatakan nyata kerana sinar Titik fokus kanta cekung dikatakan maya kerana sinar
cahaya yang dibias menumpu pada titik itu biasan seolah-olah mencapah dari titik itu
Focal point of convex lens is said to be real as the refracted light rays Focal point of concave lens is said to be virtual as the refracted light rays
converge at this point appear to diverge from this point

Panjang fokus, f diambil sebagai positif Panjang fokus, f diambil sebagai negatif
Focal length, f is taken as positive Focal length, f is taken as negative

* Panjang fokus, f suatu kanta ialah jarak titik fokus, F dari pusat optik, O.
The focal length, f of a lens is the distance of the focal point, F from the optical centre, O.

3 Pembesaran linear yang dihasilkan oleh kanta 2 Peraturan tanda bagi formula kanta nipis:
didefinisikan sebagai nisbah ketinggian imej kepada The sign convention for thin lens formula:
ketinggian objek. Selain itu, pembesaran linear juga
berkait dengan jarak objek dan jarak imej. Positif/ Positive Negatif/ Negative
The linear magnification produced by a lens is defined as the ratio (+) (–)
of the height of the image to the height of the object. Besides that,
the linear magnification is also related to the object distance and Kanta cembung Kanta cekung
image distance. Panjang atau kanta atau kanta
fokus, f penumpu pencapah
Ketinggian imej, hi Focal length Convex lens or Concave lens or
Image height converging lens diverging lens
Pembesaran linear, m =
Linear magnification Ketinggian objek, ho • Imej nyata • Imej maya
Object height Real image Virtual image

• Di belakang • Di sebelah
Jarak imej, v
Image distance Jarak imej, v kanta kanta yang
= Image distance Behind the lens sama dengan
Jarak objek, u objek
Object distance
On the same side of
lens as object

6.4 Formula Kanta Nipis


Thin Lens Formula 6.5 Peralatan Optik
Optical Instruments
1 Formula kanta nipis berkaitan dengan jarak objek,
Mikroskop majmuk/ Compound microscope
u, jarak imej, v, dan panjang fokus, f bagi suatu kanta
diberi sebagai 1 Rajah berikut menunjukkan gambar rajah sinar bagi
Thin lens formula relates to object distance, u, image distance, v sebuah mikroskop majmuk.
and the focal length, f of a lens is given by The following diagram shows a ray diagram for a compound
microscope.
Kanta mata, L2
1 1 1
= + Eyepiece lens
f u v
Kanta objek, L1
dengan keadaan/ where Objective lens
Lo
u = Jarak objek/ Object distance
O
v = Jarak imej/ Image distance Fo Fe
f = Panjang fokus/ Focal length Fo Fe
I1

I2

175

06 Strategi A+ SPM Fizik Tg 4 (B06) 5pp.indd 175 18/11/2023 10:17 AM


2 Bagi keseluruhan mikroskop majmuk, pembesaran Pembentukan Imej oleh Cermin Sfera
linear
6.6
Image Formation by Spherical Mirrors
For the whole compound microscope, linear magnification,

M = mo × me 1 Terdapat dua jenis cermin sfera:


There are two types of spherical mirrors:
dengan keadaan/ where
(a) Cermin cekung / Concave mirror
mo = Pembesaran bagi kanta objek
(b) Cermin cembung / Convex mirror
Magnification of objective lens
me = Pembesaran bagi kanta mata Kanta cekung Kanta cembung
Magnification of eyepiece lens Concave lens Convex lens

3 Bagi panjang fokus, Permukaan yang memantulkan cahaya


For focal length, Surface that reflects light

fo < fe

4 Jarak di antara kanta objek dengan kanta mata, L


The distance between the objective lens and the eyepiece lens, L

L > fo + fe

Teleskop/ Telescope
Permukaan dalam Permukaan luar
1 Rajah berikut menunjukkan gambar rajah sinar bagi
bahagian terpotong bahagian terpotong
sebuah teleskop.
The following diagram shows a ray diagram for a telescope. cermin sfera yang cermin sfera yang
memantulkan cahaya memantulkan cahaya
Kanta objek, L1 Inner surface of cut-off part of Outer surface of cut-off part of
Objective lens Kanta mata, L2
Eyepiece lens spherical mirror that reflects spherical mirror that reflects
light light
fo fe
2 Pantulan cahaya berlaku di permukaan dalam
melengkung bagi sebuah cermin cekung dan di
Fo, Fe permukaan luar melengkung bagi sebuah kanta
I1
cembung.
Reflection of light takes place at the curved inward surface for
a concave mirror and at the curved outward surface for a convex
mirror.

2 Pembesaran teleskop pada pelarasan normal,


Magnification of telescope at normal adjustment, Paksi utama
C F P Principal axis
fo
M=
fe
f
3 Bagi panjang fokus,
For focal length, r
fo > fe (a) Untuk cermin cekung: Sinar cahaya yang selari
dengan paksi utama akan tertumpu pada
4 Jarak di antara kanta objek dengan kanta mata, L
titik fokus, F
The distance between the objective lens and the eyepiece lens, L
For concave lens: Light rays parallel to the principal axis will
L = fo + fe converge at the focal point, F

Kanta pembesar/ Magnifying glass


1 Rajah berikut menunjukkan gambar rajah sinar bagi
sebuah kanta pembesar.
The following diagram shows a ray diagram for a magnifying glass.
I Paksi utama
P F C Principal axis

O
F f
F r

2 Pembesaran, M bagi kanta pembesar diberi sebagai (b) Untuk cermin cembung: Sinar cahaya yang selari
Magnification, M of magnifying glass is given by dengan paksi utama dilihat seolah-olah tercapah
dari titik fokus, F
Tinggi imej/ Image height, hi v For convex lens: Light rays parallel to the principal axis is
M = =
Tinggi objek/ Object height, ho u seen as diverging from the focal point, F

176

06 Strategi A+ SPM Fizik Tg 4 (B06) 5pp.indd 176 18/11/2023 10:17 AM


3 Istilah yang berkaitan dengan cermin sfera:
The list of terms related to spherical mirrors:

Istilah optik Definisi


Optical term Definition

Pusat kelengkungan, C Pusat geometri sfera yang juga pusat yang membentuk cermin cekung atau cermin
Centre of curvature cembung
The geometric centre of sphere which is also the centre forming the concave or convex mirror

Kutub cermin, P Titik pusat cermin sfera


Pole of mirror The centre point of spherical mirror

Jejari kelengkungan, r Jarak dari kutub cermin, P ke pusat kelengkungan, C


Radius of curvature The distance from the pole of mirror, P to the centre of curvature, C

Paksi utama Garis lurus yang menerusi pusat kelengkungan, C dan kutub cermin sfera, P
Principal axis A straight line that passes through the centre of curvature, C and the pole of the spherical mirror, P

Satu titik yang terletak pada paksi utama tempat semua sinar cahaya yang selari dengan
paksi utama menumpu pada titik ini, atau muncul mencapah dari titik ini selepas
Titik fokus, F
Focal point dipantulkan oleh cermin
A point on the principal axis at which rays parallel to the principal axis converge to, or appear to diverge from,
after reflection by the mirror

Panjang fokus, f Jarak dari titik fokus, F ke kutub cermin sfera, P


Focal length The distance from the focal point, F to the pole of spherical mirror, P

Jarak objek, u Jarak dari objek ke kutub cermin sfera, P


Object distance The distance from the object to the pole of spherical mirror, P

Jarak imej, v Jarak dari imej ke kutub cermin sfera, P


Image distance The distance from the image to the pole of spherical mirror, P

Praktis PBD
6.1 Pembiasan Cahaya
Refraction of Light

Latihan 1 Pembiasan cahaya


Refraction of light
TP 1 Mengingat kembali pengetahuan dan kemahiran sains mengenai Cahaya dan Optik.
TP 2 Memahami Cahaya dan Optik serta dapat menjelaskan kefahaman tersebut.
TP 3 Mengaplikasikan pengetahuan mengenai Cahaya dan Optik untuk menerangkan kejadian atau fenomena alam dan melaksanakan tugasan mudah.

1 Isi tempat kosong dengan perkataan yang betul. TP 1 TP 2


Fill in the blanks with the correct words.

(a) Semakin besar indeks biasan suatu medium, semakin rendah laju cahaya dalam medium itu.
The larger the refractive index of a medium, the lower the speed of light in the medium.

(b) Kelajuan cahaya adalah paling tinggi dalam vakum , iaitu 3.00 × 108 m s–1.
The speed of light is the highest in vacuum , that is 3.00 × 108 m s–1.

(c) Pemesongan cahaya apabila ia bergerak dari udara ke air menyebabkan anggaran kedalaman
air yang salah oleh pemerhati. Sinar cahaya itu seolah-olah berasal daripada satu titik
di atas titik asal. Hal ini menyebabkan kedalaman air kelihatan lebih cetek daripada
kedalaman sebenar.
The distortion of light when it moves from air to water causes an incorrect estimate of the depth of water
by the observer. The rays of light seem to originate from a point above the point of origin. This causes
the water depth to appear shallower than the actual depth.

177

06 Strategi A+ SPM Fizik Tg 4 (B06) 5pp.indd 177 18/11/2023 10:17 AM


(d) Sebatang pensel yang direndam separuh ke dalam air kelihatan membengkok . Hal ini adalah
disebabkan oleh pembiasan cahaya.
A pencil partially submerged in water looks bent . This is due to the refraction of light.

2 Rajah di sebelah menunjukkan perambatan sinar cahaya dari udara ke air. Labelkan rajah itu.
The diagram on the right shows the propagation of light ray from air to water.
Label the diagram. TP 2

A : Garis normal/ Normal A


Udara
B
B : Sinar tuju/ Incident ray C Air

C : Sudut tuju/ Angle of incidence


D : Sudut biasan/ Angle of refraction Air
Water
D
E : Sinar biasan/ Refracted ray
E

3 Isi tempat kosong dengan perkataan yang betul tentang cahaya yang merambat melalui
dua medium yang berlainan ketumpatan optik. TP 3
Fill in the blanks with the correct words about light propagating through two mediums of different optical densities.

(a) (i) Apabila cahaya merambat dari satu


medium yang mempunyai ketumpatan
i Garis normal optik rendah (udara) ke medium yang
Udara Normal
Air mempunyai ketumpatan optik tinggi
(blok kaca), sinar cahaya membengkok
r ke arah garis normal.
Blok Membengkok
kaca ke arah

Glass normal When light travels from a medium of low optical
block Bends towards
the normal density (air) into a medium of high optical density
(glass block), light ray bends towards the
normal.

(ii) Sudut biasan lebih kecil daripada


sudut tuju.
The angle of refraction is smaller than the
angle of incidence.

(b) (i) Apabila cahaya merambat dari medium


yang berketumpatan optik tinggi (blok
Blok kaca) ke medium berketumpatan optik
kaca
Glass i rendah (udara), sinar cahaya membengkok
block menjauhi garis normal.


Udara When light travels from a medium of high optical
Air r
density (glass block) into a medium of low optical
Garis normal
Normal density (air), light ray bends away from
the normal.
Membengkok
menjauhi normal (ii) Sudut biasan lebih besar daripada
Bends away from
the normal sudut tuju.
The angle of refraction is greater than the
angle of incidence.

178

06 Strategi A+ SPM Fizik Tg 4 (B06) 5pp.indd 178 18/11/2023 10:17 AM


Latihan 2 Indeks biasan
Refractive index
TP 1 Mengingat kembali pengetahuan dan kemahiran sains mengenai Cahaya dan Optik.
TP 2 Memahami Cahaya dan Optik serta dapat menjelaskan kefahaman tersebut.

1 Isi tempat kosong dengan perkataan yang betul. TP 1 TP 2


Fill in the blanks with the correct words.

(a) Darjah pembengkokan sinar cahaya bergantung pada indeks biasan medium tempat cahaya
merambat.
The degree of bending of the light ray depends on the refractive index of a medium through which the light
travels.
(b) Indeks biasan bagi suatu medium didefinisikan sebagai nisbah laju cahaya dalam vakum
kepada laju cahaya dalam medium .
The refractive index of a medium is defined as the ratio of the speed of light in vacuum to the speed of light in
medium .

(c) Rumus indeks biasan/ Formula of refractive index:

Laju cahaya dalam vakum/ Speed of light in vacuum, c


Indeks biasan, n =
Laju cahaya dalam medium/ Speed of light in medium, v
Refractive index

(d) Jika indeks biasan bertambah, laju cahaya dalam medium berkurang .
If the refractive index increases, the speed of light in the medium decreases .

(e) Oleh itu, cahaya merambat lebih perlahan melalui medium yang mempunyai indeks biasan
yang lebih tinggi .
Therefore, light travels slower through a medium of higher refractive index.

(f) Apabila ketumpatan optik suatu medium bertambah , indek biasan medium juga bertambah .
When the optical density of a medium increases , the refractive index of the medium also increases .

2 Lengkapkan peta buih berikut dengan menyatakan hukum pembiasan. TP 2


Complete the following bubble map by stating the law of refraction. i-THINK Peta Buih

(a) (b)

Hukum Snell
Snell's law:
Sinar tuju, sinar biasan,
dan garis normal bertemu pada Hukum n1 sin θ1 = n2 sin θ2
satu titik dan berada pada satah pembiasan
yang sama. Law of n2 sin θ1
=
The incident ray, the refracted ray, and the refraction n1 sin θ2
normal meet at one point and all lie on
the same plane.

179

06 Strategi A+ SPM Fizik Tg 4 (B06) 5pp.indd 179 18/11/2023 10:17 AM


3 Isi tempat kosong dengan perkataan yang betul Pemerhati
tentang dalam nyata dan dalam ketara. TP 2 Observer

Fill in the blanks with the correct words about real depth and Udara
apparent depth. Air
Air Dalam ketara
(a) Kesan pembiasan cahaya menyebabkan imej Water Apparent depth
lantai sebuah kolam renang kelihatan lebih cetek Lantai kolam Dalam nyata
daripada kedalaman sebenar. Rajah di sebelah kelihatan di sini Real depth
The bottom of the pool
menunjukkan dalam nyata dan dalam ketara bagi appears to be here
kolam renang.
The effect of refraction of light causes the image of the bottom of a swimming pool shallower than its real depth.
The diagram on the right shows the real depth and apparent depth of the swimming pool.
Berdasarkan rajah di atas/ Based on the diagram above,
(i) dalam nyata ialah jarak sebenar di antara permukaan air dengan lantai kolam renang,
the real depth is the actual distance between the surface of water and the bottom of the swimming pool,
(ii) dalam ketara ialah jarak di antara imej maya lantai kolam renang dengan permukaan air.
the apparent depth is the distance between the virtual image of the bottom of the swimming pool and the
surface of water.

(b) Hubungan antara indeks biasan suatu medium, dalam nyata, dan dalam ketara diberi sebagai
The relationship between the refractive index of a medium, real depth and apparent depth is given by

H
n=
h

dengan keadaan/ where
n = Indeks biasan medium/ Refractive index of medium
H = Dalam nyata/ Real depth
h = Dalam ketara/ Apparent depth

Latihan 3 Menyelesaikan masalah melibatkan pembiasan cahaya


Solving problems involving refraction of light
TP 3 Mengaplikasikan pengetahuan mengenai Cahaya dan Optik untuk menerangkan kejadian atau fenomena alam dan melaksanakan tugasan mudah.
TP 4 Menganalisis pengetahuan mengenai Cahaya dan Optik dalam konteks penyelesaian masalah mengenai kejadian atau fenomena alam.

Selesaikan masalah yang berikut.


Solve the following problems.
1 Berikut ialah laju cahaya dalam beberapa jenis medium yang berlainan. TP 3
The following are the speeds of light in several different types of medium.
(a) Air/ Water = 2.26 × 108 m s–1
(b) Kaca/ Glass = 1.97 × 108 m s–1
(c) Intan/ Diamond = 1.24 × 108 m s–1
Hitung indeks biasan bagi medium tersebut.
Calculate the refractive index of the mediums.
[Kelajuan cahaya dalam vakum/ Speed of light in vacuum = 3.0 × 108 m s–1]
c 3.0 × 108
(a) nair/ water = = = 1.327
v 2.26 × 108
c 3.0 × 108
(b) nkaca/ glass = = = 1.523
v 1.97 × 108
c 3.0 × 108
(c) nintan/ diamond = = = 2.419
v 1.24 × 108

180

06 Strategi A+ SPM Fizik Tg 4 (B06) 5pp.indd 180 18/11/2023 10:17 AM


2 Diberi bahawa indeks biasan bagi minyak zaitun ialah 1.46,
hitung sudut biasan bagi sinar cahaya dalam minyak zaitun. Udara
It is given that the refractive index of olive oil is 1.46, calculate the angle of 45° Air
refraction of light ray in olive oil. TP 3
sin i
n = Minyak zaitun
sin r Olive oil
r
sin 45°
1.46 =
sin r
r = 28.97°

3 Satu sinar dipancarkan dari air ke permukaan air. TP 3


A ray is emitted from water to the surface of water. Udara
40° Air
Hitung indeks biasan air itu.
Calculate the refractive index of water.
sin i Air
n = Water
sin r 28°
sin 40°
n =
sin 28°
= 1.369

4 Satu sinar cahaya ditujukan ke satu bongkah kaca. Sinar itu


kemudian merambat dari bongkah kaca ke dalam air. TP 3
Bongkah kaca 30°
A light ray is directed to a glass block. The ray then propagates from the glass Glass block
block into water.

Hitung sudut biasan sinar cahaya tersebut. Air


Water
Calculate the angle of refraction of the light ray. r
[nkaca/ glass = 1.50, nair/ water = 1.33]

n1 sin θ1 = n2 sin θ2

1.50 sin 30° = 1.33 sin r


1.50 sin 30°
sin r =
1.33
r = 34.33°

5 Aziz cuba mengambil duit syilingnya yang terjatuh ke dasar kolam ikan tetapi gagal.
Aziz tries to pick up his coin that fell to the bottom of a fish pond but fails.

0.25 m
20
16

Imej syiling
Image of the coin
20
16

Syiling
Coin

(a) Mengapakah Aziz gagal mengambil duit syiling tersebut? TP 4 KBAT Menganalisis
Why did Aziz fail to pick up the coin?
– Indeks biasan air lebih besar daripada indeks biasan udara.
The refractive index of water is greater than the refractive index of air.

181

06 Strategi A+ SPM Fizik Tg 4 (B06) 5pp.indd 181 18/11/2023 10:17 AM


– Apabila cahaya dari duit syiling bergerak ke udara, ia terpesong menjauhi garis normal
pada permukaan air.
When light from the coin moves to the air, it bends away from the normal at the surface of water.

– Kesan pembiasan cahaya ini menyebabkan pemerhati melihat imej duit syiling lebih dekat
dengan permukaan air.
This effect of refraction of light causes the observer to see the image of the coin closer to the surface of water.

(b) Jika kedalaman imej duit syiling tersebut ialah 0.25 m, hitung kedalaman kolam ikan itu. TP 3
If the depth of image of the coin is 0.25 m, calculate the depth of the fish pond.
[nair/water = 1.33]
Dalam nyata/ Real depth, H
n =
Dalam ketara/ Apparent depth, h
H
1.33 =
0.25
H = 0.3325 m

6.2 Pantulan Dalam Penuh


Total Internal Reflection

Latihan 4 Pantulan dalam penuh


Total internal reflection
TP 1 Mengingat kembali pengetahuan dan kemahiran sains mengenai Cahaya dan Optik.
TP 2 Memahami Cahaya dan Optik serta dapat menjelaskan kefahaman tersebut.
TP 3 Mengaplikasikan pengetahuan mengenai Cahaya dan Optik untuk menerangkan kejadian atau fenomena alam dan melaksanakan tugasan mudah.

1 Isi tempat kosong dengan perkataan yang betul. TP 1 TP 2


Fill in the blanks with the correct words.

(a) Apabila sudut biasan dalam medium berketumpatan optik rendah sama dengan 90° ,

sudut tuju dalam medium berketumpatan optik tinggi disebut sebagai sudut genting .
When the angle of refraction in the medium of low optical density is equal to 90° , the angle of incidence

in the medium of high optical density is called critical angle .

(b) Pantulan dalam penuh berlaku hanya apabila cahaya merambat dari medium yang
berketumpatan optik tinggi ke medium berketumpatan optik rendah . Sudut tuju dalam

medium berketumpatan optik tinggi mestilah lebih besar daripada sudut genting.
Total internal reflection occurs when light propagates from a medium of high optical density to a medium
of low optical density. The angle of incidence in the medium of high optical density must be greater

than the critical angle.

(c) Sudut genting bergantung pada ketumpatan optik medium itu. Semakin tinggi
indeks biasan medium, semakin kecil sudut genting medium tersebut.
The critical angle depends on the optical density of the medium. The higher the refractive index of the
medium, the smaller the critical angle of the medium.

182

06 Strategi A+ SPM Fizik Tg 4 (B06) 5pp.indd 182 18/11/2023 10:17 AM


2 Alatan berikut menggunakan pantulan dalam penuh sebagai prinsip kerjanya. Namakan alatan
tersebut. TP 3
The following equipments use total internal reflection as their working principle. Name the equipments.

(a) (b) (c)

Gentian optik Binokular prisma Periskop prisma


Optical fibre Prism binoculars Prism periscope

3 Hubungan antara sudut genting, c dengan indeks biasan, n suatu medium diberi
The relationship between the critical angle, c and the refractive index, n for a medium is given by

1
n=
sin c

Latihan 5 Menyelesaikan masalah melibatkan pantulan dalam penuh


Solving problems involving total internal reflection
TP 3 Mengaplikasikan pengetahuan mengenai Cahaya dan Optik untuk menerangkan kejadian atau fenomena alam dan melaksanakan tugasan mudah.
TP 4 Menganalisis pengetahuan mengenai Cahaya dan Optik dalam konteks penyelesaian masalah mengenai kejadian atau fenomena alam.

Selesaikan masalah yang berikut.


Solve the following problems.
1 Diberi bahawa sudut genting air ialah 48.75°. Hitung indeks biasan air itu. TP 3
It is given that the critical angle of water is 48.75°. Calculate the refractive index of the water.
c = 48.75°
1
n =
sin c
1
n =
sin 48.75°
n = 1.33

2 Rajah di sebelah menunjukkan sinar cahaya merambat dari


Udara
sebuah prisma bahan x ke udara. TP 3 Air
The diagram on the right shows a light ray propagating from a prism of
material x to air.
16°
Tentukan sudut genting bahan x.
Determine the critical angle of material x.
38°
sin i Bahan x
n = Material x
sin r
sin (90° – 16°)
=
sin 38°
= 1.561
1
n =
sin c
1
sin c =
1.561
c = 39.84°

183

06 Strategi A+ SPM Fizik Tg 4 (B06) 5pp.indd 183 18/11/2023 10:17 AM


3 Rajah di sebelah menunjukkan satu sinar cahaya yang merambat 38°
memasuki satu prisma yang diperbuat daripada bahan A.
The diagram on the right shows a ray of light propagating into a prism
made of material A. TP 4 KBAT Mengaplikasi Bahan A
Material A
54°
(a) Cari sudut genting bagi bahan A.
Find the critical angle of material A. Udara
Air
(b) Berapakah sudut biasan, r apabila sinar cahaya memasuki prisma?
What is the angle of refraction, r when the light ray enters the prism?
(c) Cari indeks biasan bagi bahan A.
Find the refractive index of material A.
(a) Sudut genting ialah sudut tuju yang menghasilkan sudut biasan 90° dari garis normal.
A critical angle is an angle of incidence that produces a deflection angle of 90° from the normal.
Oleh itu, sudut genting
Therefore, the critical angle
= 90° – 38°
= 52°
Video

(b) 38°

r

Sudut biasan, r boleh ditentukan daripada konsep sudut berselang-seli.
The angle of refraction, r can be determined from the concept of alternate angles.
Maka, r ialah 38°.
Hence, r is 38°.
1
(c) n =
sin c
1
n =
sin 52°

n = 1.269

Latihan 6 Fenomena semula jadi pantulan dalam penuh


Natural phenomena of total internal reflection
TP 4 Menganalisis pengetahuan mengenai Cahaya dan Optik dalam konteks penyelesaian masalah mengenai kejadian atau fenomena alam.

Isi tempat kosong untuk menerangkan tentang fenomena semula jadi berikut. TP 4
Fill in the blanks to explain the following natural phenomenon.
Video
1 Logamaya Awan Untuk tujuan pembelajaran
Cloud
Mirage Imbas kod QR atau layari https://www.
youtube.com/watch?v=lUjvmgDaKCs
untuk menonton video tentang
pembentukan pelangi.

Pemerhati
Observer

Tanah
Ground

Imej awan
Image of cloud

184

06 Strategi A+ SPM Fizik Tg 4 (B06) 5pp.indd 184 18/11/2023 10:17 AM


(a) Udara di atas tanah mempunyai beberapa lapisan. Ketika cuaca panas terik, udara berdekatan
tanah mempunyai suhu yang lebih tinggi daripada udara di lapisan yang lebih tinggi. Maka,
indeks biasan udara itu lebih rendah daripada udara di lapisan atas.
The air above the ground has several layers. On a hot day, the air near the ground has a higher temperature
than the air at a higher level. Therefore, the refractive index of the air is lower than the air in the upper layers.
(b) Sinar cahaya dari langit terbias menjauhi normal apabila cahaya merambat dari udara yang
lebih tumpat ke udara yang kurang tumpat.
Light rays from the sky are refracted away from the normal as light propagates from denser to less dense air.
(c) Sudut tuju bertambah sehingga ia menjadi lebih besar daripada sudut genting.
The angle of incidence increases until it becomes larger than the critical angle.
(d) Pantulan dalam penuh berlaku dan cahaya dari langit membengkok ke atas ke arah mata
pemerhati.
Total internal reflection occurs and light from the sky bends upwards towards the eyes of observer.
(e) Pemerhati melihat imej awan seperti lopak air di atas tanah.

Video
The observer sees the image of cloud as a puddle of water on the ground.

Latihan 7 Aplikasi pantulan dalam penuh


Applications of total internal reflection
TP 4 Menganalisis pengetahuan mengenai Cahaya dan Optik dalam konteks penyelesaian masalah mengenai kejadian atau fenomena alam.

Lengkapkan jadual berikut tentang aplikasi pantulan dalam penuh. TP 4


Complete the following table about applications of total internal reflection.

1 Gentian optik/ Optical fibre Gentian optik


(a) digunakan dalam bidang
Sinar cahaya
Light ray telekomunikasi dan perubatan.
Penyalut
Cladding
Optical fibre is used in telecommunication and medical
Teras dalam fields.
Inner core
(b) Gentian optik diperbuat daripada gentian plastik
atau kaca tulen .
plastic
or pure glass fibres.
Optical fibre is made of
(c) Dinding kaca di sebelah luar kurang tumpat
berbanding dengan dinding kaca di sebelah dalam.
The glass wall on the outside is less dense than the glass wall
on the inside.
Video
Untuk tujuan pembelajaran (d) Indeks biasan teras dalam adalah lebih tinggi
Imbas kod QR atau layari https://www.
youtube.com/watch?v=Lic3gCS_bKo
daripada indeks biasan lapisan penyalut.
untuk menonton video tentang aplikasi The refractive index of the inner core is higher than the
pantulan dalam penuh dalam gentian
optik. refractive index of the cladding.
(e) Cahaya merambat dari medium lebih tumpat ke
medium kurang tumpat.
Light travels from a denser medium to a less dense
medium.
(f) Cahaya yang masuk dari hujung gentian optik
mengalami pantulan dalam penuh kerana sudut
tujunya sentiasa melebihi sudut genting.
Light entering from the end of the optical fibre experiences
total internal reflection because the angle of incidence always
exceeds the critical angle.

185

06 Strategi A+ SPM Fizik Tg 4 (B06) 5pp.indd 185 18/11/2023 10:17 AM


2 Periskop/ Periscope (a) Periskop digunakan untuk melihat objek di belakang
halangan .
Prisma atas A periscope is used to see objects behind obstacles .
Sinar P Upper prism
cahaya (b) Sinar cahaya yang berserenjang dari objek memasuki
dari objek prisma di bahagian atas tanpa pembiasan.
Light ray
from object A perpendicular light ray from an object enters the upper
prism without refraction.
(c) Pantulan dalam penuh berlaku di P dan sinar cahaya
dipantulkan ke bawah.
Total internal reflection occurs at P and the light ray is
R Pemerhati
Q Observer
reflected downwards.
Prisma bawah
Lower prism (d) Sinar cahaya yang dipantulkan merambat ke bawah
dan memasuki prisma di bahagian bawah.
The reflected light ray propagates downwards and
enters the lower prism.
(e) Pantulan dalam penuh kedua berlaku di Q dan
sinar cahaya dipantulkan ke R.
Second total internal reflection occurs at Q and the light ray
is reflected to R.
(f) Sinar cahaya yang dipantulkan muncul tanpa
pembiasan dan memasuki mata pemerhati.
The reflected light ray emerges without refraction and
enters the eyes of observer.
(g) Imej yang terbentuk adalah tegak dan sama saiz
dengan objek.
The image formed is upright and the same size as the
object.

6.3 Pembentukan Imej oleh Kanta


Image Formation by Lenses

Latihan 8 Asas kanta


Basic of lens
TP 1 Mengingat kembali pengetahuan dan kemahiran sains mengenai Cahaya dan Optik.

Tandakan (3) pada pernyataan yang betul dan (7) pada pernyataan yang salah. TP 1
Mark (3) for the correct statements and (7) for the wrong statements.

1 Kanta cembung juga dikenali sebagai kanta pencapah


7
Convex lenses are also known as divergent lenses
2 Kanta cekung juga dikenali sebagai kanta penumpu
7
Concave lenses are also known as converging lenses
3 Sinar cahaya yang merambat melalui kanta cekung akan ditumpukan pada titik fokus
7
Light rays propagating through a concave lens will be focused on the focal point
4 Titik fokus kanta cekung adalah maya kerana cahaya terbias mencapah dari titik itu
3
The focal point of a concave lens is said to be virtual because the refracted light diverges from that point
5 Panjang fokus, f bagi kanta cembung adalah negatif
7
The focal length, f of a convex lens is negative
6 Kanta cembung lebih tebal di bahagian tengah daripada pinggirnya
3
Convex lens is thicker in the middle than at the edges

186

06 Strategi A+ SPM Fizik Tg 4 (B06) 5pp.indd 186 18/11/2023 10:17 AM


Latihan 9 Gambar rajah sinar
Ray diagrams
TP 1 Mengingat kembali pengetahuan dan kemahiran sains mengenai Cahaya dan Optik.
TP 2 Memahami Cahaya dan Optik serta dapat menjelaskan kefahaman tersebut.
TP 3 Mengaplikasikan pengetahuan mengenai Cahaya dan Optik untuk menerangkan kejadian atau fenomena alam dan melaksanakan tugasan mudah.

1 Lengkapkan lintasan sinar bagi kanta yang berikut. TP 1 TP 2


Complete the ray trajectory for the following lenses.

Kanta cembung/Convex lens Kanta cekung/Concave lens

(a) Sinar cahaya menuju pusat optik, O (a) Sinar cahaya menuju pusat optik, O
merambat dalam garis lurus melalui pusat merambat dalam garis lurus melalui pusat
optik tanpa dibiaskan optik tanpa dibiaskan
Light ray towards the optical centre, O propagates Light ray towards the optical centre, O propagates
in a straight line through the optical centre without in a straight line through the optical centre without
being refracted being refracted

Paksi utama Paksi utama


Principal axis O Principal axis
O

(b) Sinar yang selari dengan paksi utama (b) Sinar yang selari dengan paksi utama terbias
terbias dan menumpu ke titik fokus, F dan mencapah, seolah-olah berpunca dari
Rays parallel to the principal axis are refracted titik fokus, F
and converge towards the focal point, F Rays parallel to the principal axis are refracted
and diverge, as if originating from the focal point, F

Paksi utama Paksi utama


Principal axis Principal axis
O
F O F
F F

2 Lengkapkan gambar rajah sinar dan jadual berikut. Lukis dan nyatakan ciri-ciri imej yang terbentuk.
Complete the following ray diagrams and tables. Draw and state the characteristics of the images formed. TP 3

• Jarak objek/ Object distance, u:


Jarak di antara objek dengan pusat optik kanta, O
The distance between the object and the optical centre of lens, O
• Jarak imej/ Image distance, v:
Jarak di antara imej dengan pusat optik kanta, O
The distance between the image and the optical centre of lens, O
• Pusat optik/ Optical centre, O:
Titik di pusat kanta. Sinar cahaya yang melalui pusat optik tidak dibiaskan.
The point at the centre of lens. Light rays passing through the optical centre are not refracted.

187

06 Strategi A+ SPM Fizik Tg 4 (B06) 5pp.indd 187 18/11/2023 10:17 AM


(a) Kanta cembung
Convex lens

Kedudukan Ciri-ciri imej


Kedudukan objek Gambar rajah sinar
imej Image
Object position Ray diagram
Image position characteristic

(i) Objek di antara I v>u • Maya


F dengan kanta Virtual
Di hadapan kanta
Object between • Tegak
In front of lens
F and lens O Upright
(u < f ) F
• Diperbesar
Magnified
F

(ii) Objek di F Di infiniti • Maya


Object at F At infinity Virtual
(u = f ) • Tegak
O Di belakang Upright
F 2F kanta • Diperbesar
Behind lens Magnified
2F F

(iii) Objek di antara v > 2f • Nyata


F dengan 2F O Real
Di belakang
Object between • Songsang
F 2F kanta Inverted
F and 2F Behind lens
2F F • Diperbesar
(f < u < 2f ) Magnified

(iv) Objek di 2F v = 2f • Nyata


Object at 2F Real
O Di belakang
(u = 2f ) • Songsang
kanta Inverted
F 2F Behind lens
• Sama saiz
2F F
dengan objek
Same size as object
I

(v) Objek lebih f < v < 2f • Nyata


jauh dari 2F Real
O Di belakang
Object beyond 2F • Songsang
kanta Inverted
(u > 2f ) F 2F Behind lens
• Diperkecil
2F F Diminished
I

(vi) Objek di infiniti v=f • Nyata


Object at infinity Real
Imej di F
• Songsang
Image at F
Inverted
F
Di belakang • Diperkecil
F
kanta Diminished
Behind lens
I

188

06 Strategi A+ SPM Fizik Tg 4 (B06) 5pp.indd 188 18/11/2023 10:17 AM


(b) Kanta cekung/ Concave lens

Kedudukan Ciri imej


Kedudukan objek Gambar rajah sinar
imej Image
Object position Ray diagram
Image position characteristic
(i) Objek lebih jauh v<f • Maya
dari 2F Virtual
O Di antara objek
Object beyond 2F • Tegak
dengan kanta Upright
(u > 2f) I
Between object and
• Diperkecil
2F F lens
Diminished

(ii) Objek di antara v<f • Maya


F dengan O Virtual
O Di antara objek
Object between • Tegak
dengan kanta Upright
F and O Between object and
• Diperkecil
(u < f ) 2F F I lens
Diminished

6.4 Formula Kanta Nipis


Thin Lens Formula

Latihan 10 Menggunakan formula kanta nipis


Using thin lens formula
TP 3 Mengaplikasikan pengetahuan mengenai Cahaya dan Optik untuk menerangkan kejadian atau fenomena alam dan melaksanakan tugasan mudah.
TP 4 Menganalisis pengetahuan mengenai Cahaya dan Optik dalam konteks penyelesaian masalah mengenai kejadian atau fenomena alam.

Selesaikan masalah yang berikut.


Solve the following problems.
Formula kanta nipis/ Thin lens formula:
1 1 1
= +
f u v
dengan keadaan/ where
u = Jarak objek/ Object distance
v = Jarak imej/ Image distance
f = Panjang fokus/ Focal length

1 Sekeping kanta cembung nipis mempunyai panjang fokus 10 cm. Jika jarak objek ialah 15 cm,
tentukan kedudukan dan ciri-ciri imej. TP 3
A thin convex lens has a focal length of 10 cm. If the object distance is 15 cm, determine the position and characteristics
of the image.
u = +15 cm
f = +10 cm
1 1 1
= +
f u v
1 1 1
= +
10 15 v
1 1 1
= –
v 10 15
v = 30 cm
v > 2f
Imej adalah nyata, songsang, diperbesar, dan berada 30 cm di belakang kanta.
The image is real, inverted, magnified, and located 30 cm behind the lens.

189

06 Strategi A+ SPM Fizik Tg 4 (B06) 5pp.indd 189 18/11/2023 10:17 AM


2 Suatu objek setinggi 9 cm diletakkan pada jarak 30 cm dari kanta cekung dengan panjang fokus
15 cm. Tentukan tinggi imej yang terbentuk. TP 3
An object with a height of 9 cm is placed at a distance of 30 cm from a concave lens with a focal length of 15 cm.
Determine the height of the image formed.
u = 30 cm
f = –15 cm
1 1 1
= +
f u v
1 1 1
= +
–15 30 v
1 1 1
= – –
v 15 30
v = –10 cm
hi v
=
ho u
hi 10
=
9 30
hi = 3 cm

3 Sebuah kanta cembung nipis mempunyai panjang fokus 15 cm. Jika jarak imej ialah 20 cm, tentukan
A thin convex lens has a focal length of 15 cm. If the image distance is 20 cm, determine TP 4
(a) kedudukan objek,
the object position,
(b) ciri-ciri imej.
the characteristics of the image.
(a) f = 15 cm
v = 20 cm
1 1 1
= +
f u v
1 1 1
= +
15 u 20
1 1 1
= –
u 15 20
u = 60 cm
di hadapan kanta
in front of lens
(b) Nyata, songsang, diperkecil
Real, inverted, diminished

4 Tentukan ciri-ciri imej bagi kanta cembung berdasarkan jarak objek, u berikut. TP 4
Determine the characteristics of images of a convex lens based on the following object distances, u.
(a) u = f (d) u = infiniti/infinity
(b) u < f (e) u = 2f
(c) u > 2f (f) f < u < 2f
(a) Maya, tegak, diperbesar (d) Nyata, songsang, diperkecil
(di infiniti) Real, inverted, diminished
Virtual, upright, magnified (e) Nyata, songsang, sama saiz dengan objek
(at infinity) Real, inverted, same size as object
(b) Maya, tegak, diperbesar (f) Nyata, songsang, diperbesar
Virtual, upright, magnified Real, inverted, magnified
(c) Nyata, songsang, diperkecil
Real, inverted, diminished

190

06 Strategi A+ SPM Fizik Tg 4 (B06) 5pp.indd 190 18/11/2023 10:17 AM


6.5 Peralatan Optik
Optical Instruments

Latihan 11 Asas peralatan optik


Basic of optical instruments
TP 1 Mengingat kembali pengetahuan dan kemahiran sains mengenai Cahaya dan Optik.
TP 2 Memahami Cahaya dan Optik serta dapat menjelaskan kefahaman tersebut.
TP 4 Menganalisis pengetahuan mengenai Cahaya dan Optik dalam konteks penyelesaian masalah mengenai kejadian atau fenomena alam.
TP 5 Menilai pengetahuan mengenai Cahaya dan Optik dalam konteks penyelesaian masalah dan membuat keputusan untuk melaksanakan satu tugasan.

1 Tandakan (3) bagi pernyataan yang betul dan (7) pada pernyataan yang salah. TP 1 TP 2
Mark (3) for the correct statements and (7) for the wrong statements.

(a) Mikroskop menggunakan dua kanta cembung berkuasa tinggi


3
The microscope uses two high-powered convex lenses
(b) Teleskop mempunyai kanta-kanta dengan keadaan panjang fokus kanta objektif, fo
lebih kecil daripada panjang fokus kanta mata, fe
7
The telescope has lenses with the condition of the focal length of objective lens, fo is less than the
focal length of eyepiece lens, fe
(c) Dalam sebuah teleskop, kedudukan imej pertama yang terbentuk adalah di antara titik
fokus kanta objek dengan pusat optik kanta mata
7
In a telescope, the position of the first image formed is between the focal point of the objective lens and
the optical centre of the eyepiece lens
(d) Ciri-ciri imej pertama (kanta objek) untuk mikroskop adalah nyata, songsang dan
diperbesarkan 3
The characteristics of the first image (objective lens) of a microscope is real, inverted and magnified
(e) Pelarasan normal bagi sebuah mikroskop majmuk = fo + fe
7
Normal adjustment for a compound microscope = fo + fe
(f) Ciri-ciri imej yang terbentuk apabila menggunakan kanta pembesar ialah maya, tegak
dan diperbesarkan 3
The characteristics of the image formed when using a magnifying glass is virtual, upright and magnified
(g) Teleskop astronomi digunakan untuk memerhati objek jauh seperti bulan dan bintang
3
Astronomical telescopes are used to observe distant objects such as the Moon and the stars
(h) Seorang ahli mikrobiologi menggunakan mikroskop untuk memerhati pelbagai
mikroorganisma 3
A microbiologist uses a microscope to observe various microorganisms

2 Anda dikehendaki memilih satu daripada dua jenis kanta cembung dengan panjang fokus dan
saiz berlainan untuk dijadikan kanta objek bagi sebuah teleskop buatan sendiri. Terangkan sebab
untuk pilihan anda. TP 4 KBAT Menganalisis
You are required to choose one of two types of convex lens with different focal lengths and sizes to be the objective lens of
a homemade telescope. Explain the reasons for your choice.

Kanta A/ Lens A Kanta B/ Lens B

f = 20 cm f = 10 cm
Diameter = 10 cm Diameter = 6 cm

191

06 Strategi A+ SPM Fizik Tg 4 (B06) 5pp.indd 191 18/11/2023 10:17 AM


• Kanta berdiameter besar akan menerima lebih banyak cahaya.
Lens with a large diameter will receive more light.

• Panjang fokus yang besar akan menghasilkan imej pertama yang nyata, songsang dan diperkecil.
A large focal length will produce the first image that is real, inverted and diminished.

• Panjang fokus yang besar akan menyebabkan imej akhir menjadi lebih besar
f
(pembesaran linear = o lebih besar).
fe f o
A large focal length will cause the final image to be large (linear magnification = larger).
fe
• Kanta A dipilih kerana ia mempunyai diameter, panjang fokus dan pembesaran linear yang besar.
Lens A is chosen because it has large diameter, focal length and linear magnification.

Aktiviti PAK-21
Tujuan: Mengumpul maklumat tentang prinsip kerja kamera di dalam sebuah telefon pintar
Aim: Collect information about the working principles of the camera in a smartphone
1 Bahagikan kelas kepada beberapa kumpulan kecil.
Divide the class into several small groups.
2 Cari maklumat dari pusat sumber atau Internet tentang prinsip kerja kamera di dalam sebuah telefon pintar.
Find the information from the resource centre or Internet about the working principles of the camera in a smartphone.
3 Bincang dan lengkapkan aktiviti berikut dalam kumpulan. Kemudian, persembahkan hasil yang diperoleh di hadapan
kelas.
Discuss and complete the following activity in the group. Then, present the results obtained in front of the class.

Anda dikehendaki untuk mencadangkan kanta untuk digunakan di dalam kamera sebuah telefon pintar. Terangkan
cadangan anda.
You are required to suggest a lens used in a smartphone camera. Explain your suggestions. TP 5 KBAT Menilai
Aktiviti PAK-21

Cadangan jawapan/ Suggested answer:

• Kanta cembung yang bersaiz kecil dipilih untuk digunakan sebagai kamera bagi telefon pintar yang nipis.
A small convex lens is chosen to be used as a camera for a thin smartphone.

• Kanta yang lebih kecil dan lebar memerlukan pergerakan kecil untuk memfokus.
Smaller and wider lenses require small movements to focus.

• Kanta itu boleh membentuk suatu imej yang nyata, songsang dan diperkecil pada sensor.

The lens can form a real, inverted and diminished image on the sensor.

• Jarak minimum di antara sensor dengan pusat kanta haruslah sama dengan panjang fokus kanta kamera.
The minimum distance between the sensor and the centre of the lens should be equal to the focal length of the camera lens.

• Panjang fokus kanta tidak boleh bernilai sifar. Oleh itu, ketebalan keseluruhan suatu telefon pintar terhad kepada
panjang fokus kanta kamera.
The focal length of the lens cannot be zero. Therefore, the overall thickness of a smartphone is limited to the focal length of the

camera lens.

Kanta
Lens

Medan Saiz
penglihatan sensor
Field of
Size of
vision
sensor

Jarak objek Panjang fokus


Object distance Focal length

Projek Berkumpulan STEM TP 5

192

06 Strategi A+ SPM Fizik Tg 4 (B06) 5pp.indd 192 18/11/2023 10:17 AM


6.6 Pembentukan Imej oleh Cermin Sfera
Image Formation by Spherical Mirrors

Latihan 12 Gambar rajah sinar


Ray diagrams
TP 1 Mengingat kembali pengetahuan dan kemahiran sains mengenai Cahaya dan Optik.
TP 2 Memahami Cahaya dan Optik serta dapat menjelaskan kefahaman tersebut.
TP 3 Mengaplikasikan pengetahuan mengenai Cahaya dan Optik untuk menerangkan kejadian atau fenomena alam dan melaksanakan tugasan mudah.

1 Lengkapkan lintasan sinar bagi cermin sfera yang berikut. TP 1 TP 2


Complete the ray trajectory for the following spherical mirrors.

• Pusat kelengkungan/ Centre of curvature, C:


Pusat geometri sfera yang juga pusat yang membentuk cermin cekung atau cermin cembung
The geometric centre of sphere which is also the centre forming the concave or convex mirror
• Kutub cermin/ Pole of mirror, P:
Titik pusat cermin sfera/Centre point of spherical mirror
• Titik fokus/ Focal point, F:
Satu titik yang terletak pada paksi utama tempat semua sinar cahaya yang selari dengan
paksi utama menumpu pada titik ini, atau muncul mencapah dari titik ini selepas
dipantulkan oleh cermin
A point on the principal axis at which rays parallel to the principal axis converge to, or appear to diverge from,
after reflection by the mirror

Cermin cekung/Concave mirror Cermin cembung/Convex mirror


(a) (a)

Paksi utama
Principal axis Paksi utama
Principal axis
C F P P F C

(b) (b)

Paksi utama
Principal axis Paksi utama
Principal axis
C F P
P F C

2 Lengkapkan gambar rajah sinar dan jadual berikut. Lukis dan nyatakan ciri-ciri imej yang terbentuk.
Complete the following ray diagrams and tables. Draw and state the characteristics of the images formed. TP 3

• Jarak objek/ Object distance, u:


Jarak di antara objek dengan kutub cermin sfera, P
The distance between the object and the pole of spherical mirror, P
• Jarak imej/ Image distance, v:
Jarak di antara imej dengan kutub cermin sfera, P
The distance between the image and the pole of spherical mirror, P
• Kutub cermin sfera/ Pole of spherical mirror, P:
Titik di pusat cermin. Sinar cahaya yang melalui kutub tidak dibiaskan.
The point at centre of mirror. Light rays passing through the pole are not refracted.

193

06 Strategi A+ SPM Fizik Tg 4 (B06) 5pp.indd 193 18/11/2023 10:17 AM


(a) Kanta cekung
Concave mirror

Kedudukan Ciri-ciri imej


Kedudukan objek Gambar rajah sinar
imej Image
Object position Ray diagram
Image position characteristic

(i) Objek di antara I v>u • Maya


F dengan P Virtual
Di belakang
Object between O • Tegak
cermin Upright
F and P Behind mirror
(u < f ) • Diperbesar
C F P Magnified

(ii) Objek di F Di infiniti • Maya


Object at F At infinity Virtual
(u = f )
O • Tegak
Di belakang
Upright
cermin
C F P • Diperbesar
Behind mirror
Magnified

(iii) Objek di antara v > 2f • Nyata


F dengan C Real
O Di hadapan
Object between • Songsang
cermin Inverted
F and C In front of mirror
C F P • Diperbesar
(f < u < 2f ) Magnified
I

(iv) Objek di C v = 2f • Nyata


Object at C Real
O Di hadapan
(u = 2f ) • Songsang
cermin Inverted
In front of mirror
C F P • Sama saiz
dengan objek
I
Same size as object

(v) Objek lebih f < v < 2f • Nyata


jauh dari C Real
O Di hadapan
Object beyond C • Songsang
cermin Inverted
(u > 2f ) F P In front of mirror
C
• Diperkecil
I Diminished

(vi) Objek di infiniti v=f • Nyata


Object at infinity Real
Di hadapan
• Songsang
cermin Inverted
C F P
In front of mirror
• Diperkecil
Diminished
I

194

06 Strategi A+ SPM Fizik Tg 4 (B06) 5pp.indd 194 18/11/2023 10:17 AM


(b) Cermin cembung/ Convex mirror

Kedudukan Ciri-ciri imej


Kedudukan objek Gambar rajah sinar
imej Image
Object position Ray diagram
Image position characteristic

(i) Objek lebih jauh v<f • Maya


dari F Virtual
Di belakang
Object beyond F O • Tegak
I
cermin Upright
(u > f) Behind mirror
• Diperkecil
F P F C
Diminished

(ii) Objek di antara v<f • Maya


F dengan P Virtual
Di belakang
Object between O
I
• Tegak
cermin Upright
F and P Behind mirror
• Diperkecil
(u < f ) F P F C
Diminished

Latihan 13 Aplikasi cermin cekung dan cembung dalam kehidupan harian


Applications of concave and convex mirrors in daily life
TP 3 Mengaplikasikan pengetahuan mengenai Cahaya dan Optik untuk menerangkan kejadian atau fenomena alam dan melaksanakan tugasan mudah.

Padankan jenis cermin sfera dengan aplikasinya. TP 3


Match the types of spherical mirror with its application.

1 Cermin keselamatan 4 Cermin pergigian


jalan raya Dental mirror
Traffic safety mirror

• •
Cermin
cekung
• •
Concave
mirror

2 Cermin keselamatan 5 Cermin titik buta


dalam bangunan Blind spot mirror
Security mirror in buildings

• •

3 Pemantul dalam lampu Cermin 6 Cermin solek


hadapan kereta cembung Cosmetic mirror
• •
Reflector in a car headlight Convex
mirror
• •

195

06 Strategi A+ SPM Fizik Tg 4 (B06) 5pp.indd 195 18/11/2023 10:18 AM


Praktis Berformat SPM
Berapakah sudut biasan, r bagi sinar cahaya
Kertas 1 tersebut?
What is the angle of refraction, r of the light ray?
1 Rajah 1 menunjukkan sebuah graf sin i
melawan sin r bagi dua jenis bongkah kaca A 20.35° C 31.69°
yang berlainan, A dan B. B 24.23° D 54.25°
Diagram 1 shows the graph of sin i against sin r for two 3 Rajah 3 menunjukkan satu sinar cahaya
different glass blocks, A and B. merambat dari bongkah kaca ke udara.
sin i Diagram 3 shows a ray of light propagating from a glass
block to air.
A
Bongkah kaca
Glass block
B

45°
sin r 30°
0
Udara
Rajah 1/ Diagram 1 Air

Mengapakah kecerunan graf bagi kaca A lebih


besar daripada kaca B? Rajah 3/ Diagram 3
Why is the gradient of graph for glass A greater than Berapakah indeks biasan bongkah kaca itu?
that for glass B? What is the refractive index of the glass block?
A B lebih tumpat daripada A A 0.50 C 1.41
B is denser than A B 0.71 D 1.78
B Indeks biasan A lebih besar daripada
indeks biasan B 4 Rajah 4 menunjukkan satu sinar cahaya
The refractive index of A is greater than the bergerak dari kaca ke air. Sudut genting kaca
refraction index of B ialah 43°./Diagram 4 shows a light ray travelling
C Kelajuan cahaya dalam A lebih tinggi from glass to water. The critical angle of the glass is 43°.
daripada B A
The speed of light in A is higher than that of in B
Udara
D Frekuensi cahaya dalam A lebih tinggi Air B
daripada B C
The frequency of light in A is higher than that of Kaca
Glass 55°
in B D

2 Rajah 2 menunjukkan sinar cahaya merambat Rajah 4/ Diagram 4


melalui satu bongkah kaca. Indeks biasan
Antara lintasan A, B, C dan D, yang manakah
kaca itu ialah 1.50.
ialah lintasan sinar cahaya yang betul?
Diagram 2 shows a light ray propagating through
Among the paths, A, B, C and D, which is the correct
a glass block. The refractive index of the glass is 1.50.
path of the light ray?
Kotak sinar
Ray box 5 Rajah 5 menunjukkan satu sinar cahaya
Bongkah kaca ditujukan ke sebuah bongkah kaca dengan
Glass block indeks biasan 1.50.
Diagram 5 shows a light ray incident on a glass block
38° with a refractive index of 1.50.
Sinar cahaya
r Light ray

Bongkah
kaca
Rajah 2/ Diagram 2 Glass block

Rajah 5/ Diagram 5

196

06 Strategi A+ SPM Fizik Tg 4 (B06) 5pp.indd 196 18/11/2023 10:18 AM


Hitung sudut genting bongkah kaca itu. Berapakah jarak fokus kanta itu?
Calculate the critical angle of the glass block. What is the focal length of the lens?
A 41.81° C 60.25° A 5.01 cm C 9.22 cm
B 50.20° D 80.60° B 6.23 cm D 10.67 cm
6 Rajah 6 menunjukkan suatu objek diletakkan 9 Panjang fokus suatu kanta cekung ialah f.
di hadapan sebuah kanta cembung. Jika suatu objek diletakkan 1.5f dari kanta itu,
Diagram 6 shows an object placed in front of a convex apakah ciri-ciri imej yang terbentuk?
lens. The focal length of a concave lens is f. If an object is
placed 1.5f from the lens, what are the characteristics of
the image formed?
2F F F 2F Objek A Nyata dan diperkecil
Object
Real and diminished
B Maya dan diperbesar
Rajah 6/ Diagram 6 Virtual and magnified
Apakah ciri-ciri imej yang terbentuk? C Nyata dan songsang
What are the characteristics of the image formed? Real and inverted
A Nyata, songsang, sama saiz dengan objek D Maya dan diperkecil
Real, inverted, same size as object Virtual and diminished
B Nyata, songsang, diperbesar
10 Rajah 9 menunjukkan sebuah mikroskop
Real, inverted, magnified
majmuk yang digunakan untuk meneliti
C Maya, tegak, diperbesar
mikroorganisma.
Virtual, upright, magnified
Diagram 9 shows a compound microscope that is used
D Nyata, songsang, diperkecil
to examine microorganisms.
Real, inverted, diminished

7 Rajah 7 menunjukkan suatu objek diletakkan


di hadapan sebuah kanta cembung.
Diagram 7 shows an object placed in front of a convex
lens.
Objek
Object

2F F F 2F
Rajah 9/ Diagram 9
Ciri-ciri imej yang terbentuk yang manakah
Rajah 7/ Diagram 7 adalah betul?
Apakah ciri-ciri imej yang terbentuk? Which characteristics of the image formed are correct?
What are the characteristics of the image formed? A Nyata, songsang, diperbesar
A Nyata, songsang, diperkecil Real, inverted, magnified
Real, inverted, diminished B Maya, songsang, diperbesar
B Nyata, songsang, sama saiz dengan objek Virtual, inverted, magnified
Real, inverted, same size as object C Maya, songsang, diperkecil
C Maya, tegak, diperbesar Virtual, inverted, diminished
Virtual, upright, magnified D Nyata, songsang sisi, diperbesar
D Nyata, songsang, diperbesar Real, laterally inverted, magnified
Real, inverted, magnified
11 Rajah 10 menunjukkan seorang murid
8 Rajah 8 menunjukkan pembentukan imej bagi menggunakan sebuah kanta pembesar untuk
suatu objek oleh sebuah kanta cembung. membaca manuskrip lama.
Diagram 8 shows the formation of the image of an object Diagram 10 shows a student using a magnifying glass
by a convex lens. to read an old manuscript.

Objek A B C D
Object 32 cm
Imej 2F F
Image

48 cm
Rajah 8/ Diagram 8 Rajah 10/ Diagram 10

197

06 Strategi A+ SPM Fizik Tg 4 (B06) 5pp.indd 197 18/11/2023 10:18 AM


Antara kedudukan A, B, C dan D, yang 14 Rajah 12 menunjukkan suatu objek diletakkan
manakah merupakan kedudukan terbaik dengan jarak 15 cm di hadapan suatu cermin
untuk meletakkan manuskrip lama itu? cekung. Panjang fokus, f, cermin itu ialah
Which of the positions, A, B, C and D, is the best 7.5 cm.
position to place the old manuscript? Diagram 12 shows an object placed at a distance of
15 cm in front of a concave mirror. The focal length, f,
12 Antara golongan profesional berikut, yang of the mirror is 7.5 cm.
manakah tidak menggunakan peralatan optik
Objek Cermin cekung
dalam pekerjaan mereka? Object Concave mirror
Which of the following professionals do not use optical
instruments in their work? f
A Jurubank/Bankers 7.5 cm
F
B Ahli astronomi/Astronomers
15 cm
C Optometris/Optometrists
D Jurugambar/Photographers
Rajah 12/ Diagram 12
13 Rajah 11 menunjukkan sebuah cermin cekung.
Apakah ciri-ciri imej yang terbentuk?
Diagram 11 shows a concave mirror.
What are the characteristics of the image formed?
A Nyata, songsang, sama saiz
Real, inverted, same size
B Nyata, songsang, diperkecil
C F P
Real, inverted, diminished
C Maya, tegak, sama saiz dengan objek
Virtual, upright, same size as object
Rajah 11/ Diagram 11 D Maya, tegak, diperkecil
Jarak di antara P dengan F ialah …… Virtual, upright, diminished
The distance between P and F is ……
15 Cermin cekung digunakan sebagai ……
A jarak fokus cermin.
Concave mirrors are used as ……
the focal length of the mirror.
A cermin pandang belakang kenderaan.
B diameter sfera dari tempat cermin
vehicle rear-view mirrors.
dipotong.
B cermin keselamatan di selekoh jalan.
the diameter of sphere from which the mirror was
safety mirrors at corners of roads.
cut.
C cermin pergigian.
C garis yang menyambungkan kutub
dental mirrors.
dengan pusat lengkung.
D cermin keselamatan di kedai runcit.
the line joining the pole to the centre of curvature.
security mirrors in grocery stores.
D jejari sfera dari tempat cermin dipotong.
the radius of sphere from which the mirror was cut.

Kertas 2

Bahagian A
1 (a) Rajah 1.1 menunjukkan sebatang penyedut minuman kelihatan Penyedut minuman
membengkok di dalam air. Drinking straw
Diagram 1.1 shows a drinking straw appearing bent in water.
(i) Namakan fenomena cahaya yang terlibat.
Name the light phenomenon involved.
[Apa/ What] Pembiasan cahaya/Refraction of light Air
Water
[1 markah/mark]
(ii) Bagaimanakah fenomena cahaya di 1(a)(i) berlaku? Rajah 1.1/ Diagram 1.1
How does the light phenomenon in 1(a)(i) happen?
Halaju cahaya berubah apabila merambat melalui medium yang berlainan ketumpatan optik.
Velocity of light changes when propagating through mediums of different optical densities.

[1 markah/mark]

198

06 Strategi A+ SPM Fizik Tg 4 (B06) 5pp.indd 198 18/11/2023 10:18 AM


(b) Rajah 1.2 menunjukkan seorang lelaki cuba menangguk ikan
dari tasik. Walau bagaimanapun, tangguknya tidak mengenai
ikan tersebut.
Diagram 1.2 shows a man trying to net a fish from the lake. However, his net
misses the fish.
(i) Beri satu sebab mengapa tangguknya tidak mengenai ikan
tersebut.
Give one reason why his net fails to catch the fish.
Lelaki itu melihat ikan pada dalam ketaranya dan mensasarkan
X

imej ikan itu. Rajah 1.2/ Diagram 1.2


The man sees the fish at its apparent depth and targets the image of the fish.

[1 markah/mark]
(ii) Lukis dua sinar cahaya pada Rajah 1.2 untuk menentukan kedudukan sebenar ikan itu.
Tandakan kedudukan sebenar ikan itu dengan X.
Draw two light rays in Diagram 1.2 to determine the actual position of the fish. Mark the actual position of
the fish with X.
[3 markah/marks]
(c) Laju cahaya dalam udara dan air tasik dalam Rajah 1.2 masing-masing ialah 3.0 × 108 m s–1
dan 2.00 × 108 m s–1.
The speed of light in air and lake water in Diagram 1.2 are 3.0 × 108 m s–1 and 2.00 × 108 m s–1 respectively.
Hitung
Calculate
(i) indeks biasan air,
the refractive index of water,
c
n =
v
3.0 × 108
n =
2.0 × 108
= 1.5
[1 markah/mark]
(ii) dalam nyata ikan di 1(b) apabila dalam ketara ikan ialah 3.0 m.
the real depth of the fish in 1(b) when the apparent depth of the fish is 3.0 m.
Dalam nyata/ Real depth, H
n =
Dalam ketara/ Apparent depth, h
H
1.5 =
3.0
H = 1.5 × 3.0 = 4.5 m
[2 markah/marks]

2 (a) Rajah 2.1 dan Rajah 2.2 menunjukkan


dua kanta cekung dengan panjang
fokus, f yang berlainan. Kedua-dua
kanta itu mempunyai indeks biasan
yang sama. F f F f
Diagrams 2.1 and 2.2 show two concave
lenses of different focal lengths, f. Both
concave lenses have the same refractive
index.
Rajah 2.1/ Diagram 2.1 Rajah 2.2/ Diagram 2.2
Perhatikan Rajah 2.1 dan 2.2, kemudian
Observe Diagrams 2.1 and 2.2, then
(i) bandingkan ketebalan kanta-kanta itu,
compare the thickness of the lenses,
Kanta dalam Rajah 2.2 lebih tebal daripada dalam Rajah 2.1.
Lens in Diagram 2.2 is thicker than that of in Diagram 2.1.

[1 markah/mark]

199

06 Strategi A+ SPM Fizik Tg 4 (B06) 5pp.indd 199 18/11/2023 10:18 AM


(ii) bandingkan panjang fokus, f kanta-kanta itu,
compare the focal lengths, f of the lenses,
Panjang fokus kanta dalam Rajah 2.1 lebih besar daripada dalam Rajah 2.2.
Focal length of lens in Diagram 2.1 is greater than that of in Diagram 2.2.

[1 markah/mark]
(iii) hubung kait ketebalan kanta itu dengan panjang fokus.
relate the thickness of the lenses to their focal lengths.
[Bagaimana] Semakin nipis kanta cekung, semakin besar panjang fokus atau sebaliknya.
[How] The thinner the concave lens, the greater the focal length or vice versa.

[1 markah/mark]
(iv) namakan fenomena cahaya yang terlibat.
name the light phenomenon involved.
[Apa/ What] Pembiasan cahaya/Refraction of light
[1 markah/mark]
(b) Rajah 2.3 menunjukkan rajah sinar yang
tidak lengkap bagi suatu kanta cembung untuk O
penghasilan imej, I.
Diagram 2.3 shows the incomplete ray diagram of a convex I
lens for the formation of image, I. 2F F F 2F
(i) Pada Rajah 2.3, lukis dan lengkapkan rajah
sinar untuk menunjukkan kedudukan
objek, O.
On Diagram 2.3, draw and complete the ray Rajah 2.3/ Diagram 2.3
diagram to show the position of object, O.
[2 markah/marks]
(ii) Berdasarkan jawapan di 2(b)(i), nyatakan kedudukan objek, O.
Based on the answer in 2(b)(i), state the position of the object, O.
[Di mana] Di antara F dengan 2F/[Where] Between F and 2F
[1 markah/mark]
(iii) Nyatakan dua ciri bagi imej yang terbentuk dalam Rajah 2.3.
State two characteristics of the image formed in Diagram 2.3.
Nyata, songsang dan diperbesar/Real, inverted and magnified
[Mana-mana dua jawapan/ Any two answers] [1 markah/mark]

Bahagian C

3 (a) Rajah 3.1 menunjukkan satu pemantul cahaya (mata kucing) yang sering digunakan untuk
menanda jalan raya, terutamanya pada waktu malam.
Diagram 3.1 shows a light reflector (cat's eye) that is often used to mark the road, especially at night.
Prisma kaca
Glass prism

A
B Cahaya dari
lampu hadapan
Light from
the headlamp

Rajah 3.1/ Diagram 3.1

200

06 Strategi A+ SPM Fizik Tg 4 (B06) 5pp.indd 200 18/11/2023 10:18 AM


(i) Namakan fenomena cahaya yang terlibat.
Name the light phenomenon involved. [1 markah/mark]
(ii) Terangkan bagaimana fenomena cahaya itu berlaku. KBAT Menganalisis
Explain how the light phenomenon occurs. [4 markah/marks]
(iii) Namakan satu peralatan yang mengaplikasi fenomena cahaya ini dalam operasi kerjanya.
Name one equipment that applies this light phenomenon in its working operatiom.
[1 markah/mark]

(b) Rajah 3.2 menunjukkan satu sinar cahaya merambat dari kaca ke air dengan sudut tuju dalam
kaca θ1. Rajah 3.3 menunjukkan satu sinar cahaya merambat dari kaca ke air dengan sudut tuju
dalam kaca θ2. [Indeks biasan kaca = 1.50, indeks biasan air = 1.33]
Diagram 3.2 shows a light ray propagating from the glass into water with an incident angle in glass of θ1.
Diagram 3.3 shows a light ray propagating from the glass into water with an incident angle in glass of θ2.
[Refractive index of glass = 1.50, refractive index of water = 1.33]

Air Air
Water Water

θ1 θ2

Kaca Kaca
Glass Glass

Rajah 3.2/ Diagram 3.2 Rajah 3.3/ Diagram 3.3

Berdasarkan Rajah 3.2 dan Rajah 3.3,


Based on Diagrams 3.2 and 3.3,
(i) bandingkan ketumpatan optik kaca dengan air,
compare the optical densities of glass and water, [1 markah/mark]
(ii) bandingkan saiz sudut tuju, θ1 dengan θ2,
compare the size of the angles of incidence, θ1 and θ2, [1 markah/mark]
(iii) bandingkan sudut tuju, θ1 dan θ2, dengan sudut genting, c bagi kaca,
compare the angles of incidence, θ1 and θ2, with the critical angle, c of glass, [1 markah/mark]
(vi) nyatakan kesimpulan yang boleh dibuat.
state the conclusion that can be made. [1 markah/mark]

(c) Rajah 3.4 menunjukkan keadaan di sebuah kedai


barang runcit. Pemilik kedai ingin memasang
cermin keselamatan supaya juruwang dapat
melihat keseluruhan bahagian dalam kedai
dengan lebih jelas.
Diagram 3.4 shows the situation in a grocery store. The store
owner wants to install a security mirror so that the cashier
can see the entire interior of the store more clearly.
Sebagai rakan kongsi, cadangkan beberapa
pengubahsuaian yang patut dibuat untuk
Rajah 3.4/ Diagram 3.4
mencapai tujuan tersebut berdasarkan konsep fizik
yang berkaitan dengan aspek berikut:
As a business partner, suggest several modifications that should be made to achieve the goal based on the
physics concepts related to the following aspects: KBAT Menilai KBAT Mencipta
(i) Jenis cermin (iv) Ketebalan cermin
Type of mirror Thickness of mirror
(ii) Kedudukan cermin (v) Aspek-aspek lain
Position of mirror Other aspects
(iii) Diameter cermin
Diameter of mirror
[10 markah/marks]

201

06 Strategi A+ SPM Fizik Tg 4 (B06) 5pp.indd 201 18/11/2023 10:18 AM


Jawapan/ Answers:
3 (a) (i) Pantulan dalam penuh/ Total internal refection
(ii) – Ketumpatan optik udara lebih kecil daripada prisma.
The optical density of air is smaller than that of the prism.

– Apabila sinar cahaya merambat dari medium yang lebih tumpat ke medium yang kurang tumpat,
ia terbias menjauhi normal.
When light ray travels from a denser medium to a less dense medium, it bends away from the normal.

– Apabila sudut tuju lebih besar daripada sudut genting bahan, pantulan dalam penuh berlaku.
When the angle of incidence is greater than the critical angle of the material, total internal reflection occurs.

– Sudut genting, c ialah sudut tuju dalam medium yang berketumpatan tinggi apabila sudut biasan
dalam medium yang berketumpatan rendah ialah 90°.
Critical angle, c is the angle of incidence in the medium of high optical denity when the angle of refraction in the medium

of lower optical density is 90°.

(iii) Endoskop/ Endoscope


(b) (i) Ketumpatan optik kaca lebih besar daripada air./ Optical density of glass is more than that of water.
(ii) θ1 < θ2
(iii) θ1< c < θ2
(iv) [Apa] Pantulan dalam penuh terjadi apabila sudut tuju lebih besar daripada sudut genting.
[What] Total internal reflection occurs when the angle of incident is greater than the critical angle.

(c) [Apa/ What, Mengapa/Why]


Ciri-ciri/ Characteristic Sebab/ Reason
Cermin cembung Medan penglihatan lebih luas
Convex mirror Wider field of view

Kedudukan cermin: Tinggi Medan penglihatan tidak terhalang oleh rak


Position of mirror: High The field of view is not obstructed by shelves
Diameter cermin: Besar Imej lebih jelas
Diameter of mirror: Large The image is clearer

Cermin lebih tebal Lebih kuat/ Tahan lama


Thicker mirror Strong/ Long lasting

Cermin boleh diputar Boleh melihat kawasan yang lebih luas


Rotatable mirror Can see a wider area

202

06 Strategi A+ SPM Fizik Tg 4 (B06) 5pp.indd 202 18/11/2023 10:18 AM

You might also like